Bank PO Mains-338-678

You might also like

Download as pdf or txt
Download as pdf or txt
You are on page 1of 341

Bank Po Mains PDF Course 2024

Reasoning Day -11 (Eng)

Reasoning Aptitude
Directions (1-5): Study the following information multiple of 6, then “P” is named to the second
carefully and answer the given questions. stoppage reached by the bus.
There are two buses – X and Y started travelling Condition II: If the stoppage is either north-east
from a common depot. Both the buses travel or south-east from the depot, then the stoppage
through different stoppages. is named as Q.
Bus X started moving from depot towards south Condition III: If the stoppage is either north-west
for 8m to reach stoppage 1. Then it turns towards or south west from the depot, then the stoppage
left and travels for 12m to reach stoppage S2. is named as R.
Then from stoppage S2, it again turns towards Condition IV: If the distance between the
the left and travels for 16m to reach stoppage consecutive stoppages reached by bus is
S3. Stoppage S4 is 18m west of stoppage S3. multiple of 4, then “S” is named to the first
Then, it again takes a right turn and travels for stoppage reached by the bus.
16m to reach stoppage S5. Stoppage S6 is 24m 1) If bus A travels for 12m towards the north from
east of stoppage S5. the depot to reach stoppage 14, then the
Bus Y started moving from depot towards east distance between stoppages S14 and S9 is two
for 14m to reach stoppage S7. Then it turns more than the distance between stoppages
towards the left and travels for 12m to reach __and__.
stoppage S8. Then from stoppage S8 it takes a a) S8 and S9
right turn and travels for 12m to reach stoppage b) S5 and S6
S9. Then, it again takes a right turn and travels c) Depot and S11
for 8m to reach stoppage S10. Stoppage S11 is d) S11 and S12
8m west of stoppage S10. From stoppage S11, it e) S6 and S7
takes a left turn and travels for 18m to reach
Stoppage S12. The stoppage S13 is 6m west of 2) What is the shortest distance and direction of
stoppage S12. S11 with respect to S2?
Note: a) 5m, South-east
I. Stoppage 1 is named as S1 similarly for others. b) 3√7m, North-west
II. Depot is not considered as stoppage. c) 4√3m, North
These stoppages are assigned with different d) 6√5m, North-East
names according to the below given conditions. e) 7m, South-west
Condition I: If the distance between the
consecutive stoppages reached by bus is 3) What is the direction of S13 with respect to
Depot?

Click Here For Bundle PDF Course | support@guidely.in Page 1 of 11


Bank Po Mains PDF Course 2024
Reasoning Day -11 (Eng)

a) Northeast “P$Q” means “P is neither greater than nor equal


b) West to Q”.
c) Southeast “P#Q” means “P is neither less than nor equal to
d) Southwest Q”.
e) None of these “P&Q” means “P is neither less than nor greater
than Q”.
4) Stoppages S6 and S10 are assigned with __ 6) Statements:
and __ names respectively. P#L%M#X, L@H$T, H#W@S
a) PQ and QS Conclusion:
b) RS and PQS I. S$P
c) QS and RS II. M@H
d) Q and SQ III. T#X
e) RPQ and PRS a) Both conclusions I and III are true
b) Only conclusion II is true
5) P is assigned to which of the following c) Both conclusions I and II are true
stoppages? d) Only conclusion III is true
I. S2, S6, S13 e) None is true
II. S6, S4, S7
III. S8, S9, S12 7) Statements:
a) Only III W#U%F$T, F$M&V%X, U@G$S@B
b) Both II and III Conclusion:
c) Both I and III I. X@U
d) Only I II. T#G
e) All I, II and III III. V$W
a) Both conclusions I and III are true
Directions (6-10): In the given questions, certain b) Only conclusion III is true
symbols have been used to indicate the c) Both conclusions I and II are true
relationships between different elements in the d) Only conclusion II is true
statements followed by some conclusions as e) None is true
follows. Find the conclusion(s) which is/are
definitely true. 8) Statements:
“P@Q” means “P is not less than Q”. G$T%S#M@Z, M$W%Q#P, Q&R$L
“P%Q” means “P is not greater than Q”. Conclusion:
I. Q#G

Click Here For Bundle PDF Course | support@guidely.in Page 2 of 11


Bank Po Mains PDF Course 2024
Reasoning Day -11 (Eng)

II. Z$R Directions (11-15): Study the following


III. G@R information carefully and answer the given
a) Either conclusion I or III is true questions.
b) Only conclusion III is true Ten persons – B, D, F, G, I, L, M, P, T, and Y are
c) Either conclusion I or III and conclusion II is sitting in two circular tables, such that one
true circular table is inscribed in another circular
d) Only conclusion II is true table. Five persons are sitting in the outer circular
e) All conclusions I, II and III are true table and facing towards the centre whereas five
persons are sitting in the inner circular table and
9) Statements: facing away from the centre. There is an equal
Q%B$W, K$W#U&T$R, T@Z#M%S distance between the adjacent persons and each
Conclusion: person in the inner circular table faces exactly
I. T#Q one person in the outer circular table.
II. U%S Note: If X faces Y, then both are sitting at
III. R#K different circular tables.
a) Both conclusions II and III are true M sits second to the left of the one who faces D,
b) Only conclusion III is true who sits second to the right of Y. F sits third to
c) Both conclusions I and III are true the left of T. Y does not face T. F sits adjacent to
d) All conclusions I, II, and III are true neither Y nor D. The number of persons sitting
e) None is true between F and B is one less than the number of
persons sitting between P and D, when counted
10) Statements: from the left of both P and F. B and M are not
M#S%K$H, Z#H@L$F%G, F$T%E#Q immediate neighbours. One person sits between
Conclusion: I and L, who faces away from the centre. The
I. G#Q one who faces G sits immediate left of L.
II. S$Z Final arrangement:
III. E@L Now, the persons changed their position with
a) Only conclusion II is true respect to the closest previous alphabetical
b) Only conclusion III is true name of the person. The vacant seat at the last
c) Both conclusions I and III are true will be occupied by B.
d) All conclusions I, II, and III are true 11) What is the position of B with respect to M in
e) None is true the final arrangement?
a) Third to the left
b) Immediate right

Click Here For Bundle PDF Course | support@guidely.in Page 3 of 11


Bank Po Mains PDF Course 2024
Reasoning Day -11 (Eng)

c) Second to the right 15) If Y is related to T and F is related to G, then


d) Immediate left who among the following person is related to B in
e) Fourth to the left the final arrangement?
a) L
12) __ sits __ to the right of __ in the final b) The one who sits second to the left of D
arrangement. c) M
a) P, Third, G d) The one who faces I
b) F, second, L e) Either a or d
c) T, Second, F
d) Y, third, I Directions (16-20): Study the following
e) None of these information carefully and answer the given
questions.
13) Who among the following persons are sitting Twelve persons are sitting in a linear row and
between M and T when, counted from the left of facing north. Each person has some number of
T in the final arrangement? coins which is shown with the help of inequality.
I. L T=P≥L>R<M=U≤B<F≥Q=S≤Z<G
II. G Note: If it is given that A has definitely either
III. F equal or more number of coins that B, then it
a) Only III should be consider as A≥B.
b) Both II and III T sits fourth to the left of the one who has
c) Both I and III definitely less number of coins than T. At-most
d) All I, II, and III one person sits to the right of R. Only one person
e) Only I sits between R and S. The one who has
definitely either equal or more number of coins
14) Who among the following person sits second than S sits third to the left of S. Only three
to the right of G in the final arrangement? persons sit between M and Q, who is an
a) The one who faces T immediate neighbour of S. Only one person sits
b) Y between Q and the one who has definitely more
c) The one who sits second to the right of Y number of coins than Q. The one who has either
d) L equal or more number of coins than M sits
e) M second to the left of M. P and F don’t sits
adjacent to each other. As many persons sit
between B and P as between R and Q. Only one
person sits between U and the one who has

Click Here For Bundle PDF Course | support@guidely.in Page 4 of 11


Bank Po Mains PDF Course 2024
Reasoning Day -11 (Eng)

definitely more number of coins than U. At least b) UTM


four persons sit between L and F, who doesn’t sit c) GBZ
at the end. d) RST
16) What is the position of B with respect to P? e) FPM
a) Third to the right
b) Second to the right 19) What is the sum of the number of persons sit
c) Immediate left to the right of B and the number of persons
d) Fourth to the left sitting between M and U?
e) Third to the left a) 20
b) 9
17) How many persons are sitting between L and c) 12
Q? d) 15
a) One e) 10
b) Three
c) Four 20) The number of persons sitting between Z
d) Two and S is __the number of persons sitting
e) None of these between R and P.
a) Three less than
18) In which of the following option, third person b) same as
is not sitting exactly between the first and second c) Two more than
persons? d) One more than
a) PST e) Two less than
Click Here to Get the Detailed Video Solution for the above given Questions
Or Scan the QR Code to Get the Detailed Video Solutions

Answer Key with Explanation


Directions (1-5): 1) Answer: B

Click Here For Bundle PDF Course | support@guidely.in Page 5 of 11


Bank Po Mains PDF Course 2024
Reasoning Day -11 (Eng)

2) Answer: D 8) Answer: C
3) Answer: C G<T≤S>M≥Z; M<W≤Q>P; Q=R<L
4) Answer: A I. Q > G (R = Q ≥ W > M < S ≥ T > G)  False
5) Answer: C II. Z < R (Z ≤ M < W ≤ Q = R)  True
Final arrangement III. G ≥ R (R = Q ≥ W > M < S ≥ T > G)  False
9) Answer: E
Q≤B<W; K<W>U=T<R; T≥Z>M≤S
I. T > Q (T=U<W>B≥Q)  False
II. U ≤ S (U=T≥Z>M≤S)  False
III. R > K (R>T=U<W>K)  False

10) Answer: A
M>S≤K<H; Z>H≥L<F≤G; F<T≤E>Q
I. G > Q (G≥F<T≤E>Q)  False
II. S < Z (S≤K<H<Z)  True
III. E ≥ L (E≥T>F>L)  False

Directions (6-10): Directions (11-15):


11) Answer: D
12) Answer: C
13) Answer: C
6) Answer: C 14) Answer: A
P>L≤M>X; L≥H<T; H>W≥S 15) Answer: B
I. S < P (S≤W<H≤L<P)  True Final arrangement
II. M ≥ H (M≥L≥H)  True
III. T > X (T>H≤L≤M>X)  False

7) Answer: D
W>U≤F<T; F<M=V≤X; U≥G<S≥B
I. X ≥ U (X≥V=M>F≥U)  False
II. T > G (T>F≥U≥G)  True
III. V < W (V=M>F≥U<W)  False

Click Here For Bundle PDF Course | support@guidely.in Page 6 of 11


Bank Po Mains PDF Course 2024
Reasoning Day -11 (Eng)

We have:
 M sits second to the left of the one who
faces D, who sits second to the right of Y.
From the above conditions, we have two
possibilities.

Again, we have
 F sits third to the left of T.
 Y does not face T.
 F sits adjacent to neither Y nor D.
 The number of persons sitting between F
and B is one less than the number of
persons sitting between P and D, when
counted from the left of both P and F.
 B and M are not immediate neighbours.
After applying the above conditions extra two
possibilities are added for case-1 and case-2

Click Here For Bundle PDF Course | support@guidely.in Page 7 of 11


Bank Po Mains PDF Course 2024
Reasoning Day -11 (Eng)

Again, we have
 One person sits between I and L, who
faces away from the centre.
 The one who faces G sits immediate left
of L.
After applying the above conditions case-2 and
case-2a get eliminated because there is no
possibility to place I and L in inner circle (facing
away from the centre) and case 1 gets

Click Here For Bundle PDF Course | support@guidely.in Page 8 of 11


Bank Po Mains PDF Course 2024
Reasoning Day -11 (Eng)

eliminated, because the one who faces G sits


immediate left of P is not possible.

Now, the persons changed their position with


respect to the closest previous alphabetical
name of the person. The vacant seat at the last
will be occupied by B.
This shows the final arrangement

Click Here For Bundle PDF Course | support@guidely.in Page 9 of 11


Bank Po Mains PDF Course 2024
Reasoning Day -11 (Eng)

Thus, two persons F & Z have definitely either


equal or more coins than S.
From the above conditions, there are four
possibilities

Directions (16-20):
16) Answer: E
17) Answer: B
18) Answer: D Again, we have
19) Answer: C  Only three persons sit between M and Q,
20) Answer: A who is an immediate neighbour of S.
Final arrangement  One person sits between Q and the one
who has definitely more number of coins
than Q.
G has definitely more number of
coins than Q.
We have,
 The one who has either equal or more
T=P≥L>R<M=U≤B<F≥Q=S≤Z<G
number of coins than M sits second to the
 T sits fourth to the left of the one who has
left of M.
definitely lesser number of coins than T.
Only B has either equal or more
Only R has definitely lesser number of coins
number of coins than M.
than T.
 P and F don’t sits adjacent to each other.
 At-most one person sits to the right of R.
 As many persons sit between B and P as
 Only one person sits between R and S.
between R and Q.
 The one who has definitely either equal or
more number of coins than S sits third to
the left of S.

Click Here For Bundle PDF Course | support@guidely.in Page 10 of 11


Bank Po Mains PDF Course 2024
Reasoning Day -11 (Eng)

After applying the above conditions case-1a and  Only one person sits between U and the
case-2a get eliminated because P and F sit one who has definitely more number of
adjacent to each other. coins than U.
Only F has more number of coins
than U.
 At least four persons sit between L and F,
who doesn’t sit at the end.
After applying the above conditions case-1b gets
eliminated because there is no possibility to
place L, hence case-2b shows the final
arrangement.

Again, we have

Click Here For Bundle PDF Course | support@guidely.in Page 11 of 11


Bank Po Mains PDF Course 2024
Quantitative Aptitude Day -11 (Eng)

Quantitative Aptitude

1) In a mixture of milk and water, the quantity of efficiencies of A, B and C are respectively 20%,
milk is 20 L more than that of water and the cost 30% and 50% of the efficiency of D, then find the
of pure milk is ₹ 48 per L. If the cost of pure milk value which can be filled in the blank?
were 12.5% less, then the cost of the mixture a) Can’t be determined
was ₹ 4.5 per L less than the original cost of the b) 5 days
mixture. When 8 L quantity of this mixture is c)3 days
replaced with the same quantity of milk, then d) 8 days
what will be the cost of the new mixture? e) 6 days
a) ₹ 36.4 per L
b) ₹ 40.4 per L 4) A boat can cover ___ m downstream in
c) ₹ 38.4 per L 25seconds and ___ m upstream in 35 seconds.
d) ₹ 42.4 per L The time taken by the boat to go 540 m
e) ₹ 34.4 per L downstream is 20% more than that taken by the
boat to go 300 m upstream. If the speed of the
2) Ratio of the difference between compound boat in still water were 20% more, then its
interest and simple interest on ₹ X at R% rate downstream speed was 21 m/s.
after 2 years to the difference between Quantity I: Find the value which will be filled in
compound interest and simple interest on ₹ X at the 1st blank?
R% rate after 3 years is 25: 76. If the difference Quantity II: Find the value which will be filled in
between compound interest and simple interest the 2nd blank?
on ₹ (X – 7000) at (R + 1)% rate after 2 years is a) Quantity I > Quantity II
₹ 45, then find the simple interest received on b) Quantity I = Quantity II or the relation can’t be
at (R + 4)% rate after 2 years? established
a) ₹ 1250 c)Quantity I ≤ Quantity II
b) ₹ 2500 d) Quantity I > Quantity II
c)₹ 1750 e) Quantity I ≥ Quantity II
d) ₹ 1500
e) ₹ 2000 5) Average of present ages of A, B and C is 23
years and A’s present age is ___% of D's present
3) Workers A, B and C together can complete a age. After 6 years, the ratio of A’s age to B’s age
work in 9 days and workers B, C and D together will be 2: 3 and C’s present age is 70% of B’s
can complete the same work in ___ days. If the

Click Here For Bundle PDF Course | support@guidely.in Page 1 of 16


Bank Po Mains PDF Course 2024
Quantitative Aptitude Day -11 (Eng)

present age. The difference between the seconds and (t + 2) seconds respectively. The
present ages of B and D is 10 years. speed of the car is 50% more than that of the
Quantity I: Find the value which will be filled in bike and also 4 m/s more than that of the bus.
the blank? Quantity I: If the speed of the bus were ___% of
Quantity II:A’s present age is what per cent of its original speed, it would have crossed the
B’s present age? bridge in 16 seconds.
a) Quantity I > Quantity II Quantity II:If the speed of the bike were ___% of
b) Quantity I = Quantity II or the relation can’t be its original speed, it would have crossed the
established bridge in 20 seconds.
c)Quantity I ≤ Quantity II a) Quantity I > Quantity II
d) Quantity I > Quantity II b) Quantity I ≤ Quantity II
e) Quantity I ≥ Quantity II c)Quantity I > Quantity II
d) Quantity I = Quantity II or the relation can’t be
6)Time taken by a car, a bike and a bus to cross established
a 240 m long bridge is ‘t’ seconds, (t + 5) e) Quantity I ≥ Quantity II

Direction (7-10): Study the following data carefully and answer the questions:
There are three tanks P, Q and R of different capacities and each tank has an inlet pipe and an outlet
pipe connected to it.
Inlet pipe A and outlet pipe M are connected to tank P.
Inlet pipe B and outlet pipe N are connected to tank Q.
Inlet pipe C and outlet pipe O are connected to tank R.
The table given below shows the following data:

Note:
1) The ratio of the efficiency of pipe M to that of pipe N is 3: 2 and the ratio of the efficiency of the pipe M
to that of pipe O is 4: 3.

Click Here For Bundle PDF Course | support@guidely.in Page 2 of 16


Bank Po Mains PDF Course 2024
Quantitative Aptitude Day -11 (Eng)

7) The flowing rate of water from pipe M is 4.5 L 9) If the ratio between capacities of tanks P, Q
per minute, which is 90% of the flowing rate of and R is 7: 6: 8 respectively, the difference
water from pipe N. If the flowing rate of water between the capacities of tanks P and Q is 30 L
from pipe A is ‘x’ L per minute and that from pipe and only pipes A, B and C are connected to tank
B is ‘y’ L per minute, then find the value of ? R, then in what time tank R will be filled?
a) 5.4 a) 8 minutes
b) 6.8 b) 12 minutes
c)5.6 c)10 minutes
d) 7.5 d) 6 minutes
e) 4.9 e) 15 minutes

8) If pipes A and M work with 125% and of 10) Which of the following is/are true?

their respective efficiencies and pipes B and N X: The average time, in which pipe A alone, pipe

work with 125% and 93.75% of their respective B alone and pipe C alone can fill tanks P, Q and

efficiencies, then find that the time, in which R respectively, is 22 minutes.

pipes A and M together can fill tank P with their Y: If the capacity of tank R is 210 L, then the

new efficiencies, will be what per cent of the flowing rate of water from pipe O is 5.75 L.

time, in which pipes B and N together can fill Z: If the difference between the flowing rates of

tank Q with their new efficiencies? water from pipes B and N is 3 L per minute, then

a) 33.33% the capacity of tank Q is 300 L.

b) 40% a) Both X and Y

c)30% b) Both X and Z

d) 37.5% c) None is true

e) 44.44% d) Both Y and Z


e) All are true

Direction (11-14): Study the following data carefully and answer the questions:
The data given below is related to the total number of persons (male + female) who visited a museum
on 4 different days Mon, Tue, Wed and Thu.
The pie chart given below shows the percentage distribution of the total number of persons who visited
the museum on each day.

Click Here For Bundle PDF Course | support@guidely.in Page 3 of 16


Bank Po Mains PDF Course 2024
Quantitative Aptitude Day -11 (Eng)

The Pie chart given below shows the percentage distribution of the number of males, who visited the
museum on each day.

Note:
1) The number of females, who visited the museum on Tue, is 110, which is 175% more than the
females , who visited the museum on Mon.
11) The Cost of ticket for adults is ₹ 50 per adult child. If on Mon, 45% of the total persons were
and the cost of the ticket for children is ₹ 20 per adults and on Wed, 65% of the total persons

Click Here For Bundle PDF Course | support@guidely.in Page 4 of 16


Bank Po Mains PDF Course 2024
Quantitative Aptitude Day -11 (Eng)

were adults, then find the difference between the a) Only B


total amount collected by the museum by selling b) Only B and C
adult’s tickets on Mon and Wed together and that c) Only A and B
collected by selling children’s tickets on Mon and d) Only A and C
Wed together? e) Only C
a) ₹ 9100
b) ₹ 9900 14) If 40% of total males, who visited the
c) ₹ 9500 museum on Mon, were children, 20% of total
d) ₹ 9000 males, who visited the museum on Tue, were
e) ₹ 9300 children, 30% of total males, who visited the
museum on Wed, were children and 45% of total
12)If 74% of the total males, who visited the males, who visited the museum on Thu, were
museum on Tue, were Indian and the remaining children, then find the average of the number of
were foreigners and 90% of the total females, male children, who visited the museum on each
who visited the museum on Tue, were Indian and of the given days?
the remaining were foreigners, then find what a) 40
per cent of the total number of persons, who b) 55
visited the museum on Tue, were foreigners? c) 45
a) 12.5% d) 50
b) 25% e) 60
c) 20%
d) 16.67% Direction (15-18): Study the following data
e) 15% carefully and answer the questions:
The data given below is related to the number of
13) If the number of females, who visited the persons, who took membership in three different
museum on each day, is described in a circle sports (Tennis, Swimming, and Golf) in a sports
and the degree distribution of the number of club. Some took membership in only one sport,
females, who visited the museum on Wed and some took membership in only two sports and
Thu are M° and N° respectively, then find that the remaining took membership in all three
which of the following is/are true? sports.
A: Both M and N are multiple of 24. The number of people, who took membership in
B: The Difference of M and N is a multiple of 12. swimming only, is 80% of those, who took
C:The Value of is 2.5. membership in Tennis only and also 2 more than
those, who took membership in Golf only. The

Click Here For Bundle PDF Course | support@guidely.in Page 5 of 16


Bank Po Mains PDF Course 2024
Quantitative Aptitude Day -11 (Eng)

ratio of the number of persons, who took a) ₹ 2290


membership in both Tennis and Swimming but b) ₹ 2490
not Golf to those, who took membership in both c) ₹ 2590
Tennis and Golf but not Swimming, is 2: 3 and d) ₹ 2190
the number of persons, who took membership in e) ₹ 2390
both Swimming and Golf but not Tennis, is 3
more than those, who took membership of both 17) If the total number of persons, who took
Tennis and Golf but not Swimming. The number membership of only one sports, is M% of the
of persons, who took membership of both total number of persons in the sports club and
Swimming and Golf but not Tennis, is 60% of the total number of persons, who took
those, who took membership of Golf only. The membership of only two sports, is N% of the total
number of persons, who took membership in all number of persons in the sports club, then find
the three sports, is 5, which is 12.5% of those, thatthe value of (2M – 3N) is divisible by which of
who took membership in Tennis only. the following?
15) If the ratio of males to females, who took a) 10
membership of Tennis only, is 11: 9, the ratio of b) 5
males to females, who took membership of c) 15
Swimming only, is 5: 3 and the ratio of males to d) Both (a) and (b)
female, who took membership of Golf only, is 1: e) Both (b) and (c)
2, then find the ratio of total males to total
females, who took membership of only one 18) If 56% of the total number of persons in the
sport? sports club are male, out of which, took
a) 52: 51 membership of at-least two sports, then find that
b) 4: 3 what per cent of total females took membership
c) 104: 103 of only one sport?
d) 13: 12 a) 63.63%
e) 26: 25 b) 57.5%
c) 66.67%
16)If the cost of a Swimming membership is d) 53.33%
₹ 150 per person and the cost of a Golf e) 60%
membership is ₹ 180 per person, then find the
difference between the amount collected by the Direction (19-20): Study the following data
sports club by giving Swimming membership and carefully and answer the questions:
that collected by giving Golf membership?

Click Here For Bundle PDF Course | support@guidely.in Page 6 of 16


Bank Po Mains PDF Course 2024
Quantitative Aptitude Day -11 (Eng)

I: e) Only A and B
II:
III: 20) Find the roots of the equation: px2 + qx + (r –

IV: b 3) = 0, where:

19) If the sum of bigger roots of all the four p = Difference between smaller roots of

equations together is ‘P’, then which of the equations I and III.

following regarding P is TRUE? q = Sum of smaller roots of equations II and IV.

A: ‘P’ is a prime number. r = Smaller root of equation III.

B: ‘P + 2’ is a prime number. a) 4 and 2

C: ‘P – 2’ is a prime number. b) -4 and -2

a) Only A c) -4 and 2

b) Only A and C d) 4 and -2

c) Only C e) None of these

d) Only B and C
Click Here to Get the Detailed Video Solution for the above given Questions
Or Scan the QR Code to Get the Detailed Video Solutions

Answer Key with Explanation


1) Answer: C The new cost of the mixture will also be
Let the initial quantity of water = ‘x’ L 12.5%less than the original cost of the mixture.
So, the initial quantity of milk = (x + 20) L So, the original cost of the mixture = y =
Let the initial cost of the mixture = ₹ ‘y’ per L = ₹ 36 per L
So, From equation (1):

If the cost of pure milk were 12.5% less: 4x + 80 = 6x + 60


x = 10

Click Here For Bundle PDF Course | support@guidely.in Page 7 of 16


Bank Po Mains PDF Course 2024
Quantitative Aptitude Day -11 (Eng)

Initial quantity of water = 10 L Now, the SI received on at (R + 4)% rate


Initial quantity of milk = (10 + 20) = 30 L after 2 years:
Ratio of milk to water in the mixture = 30: 10 = 3:
1
When 8 L quantity of the mixture is replaced with
the same quantity of milk: 3) Answer: B
Let the time, in which D alone can complete the
The new quantity of milk in the mixture =
work is ‘3t’ days.
= 32 L
So, the time, in which A alone can complete the
And the new quantity of water in the mixture = 40
work = = ‘15t’ days
– 32 = 8 L
So, the new cost of the mixture = = ₹ 38.4 The time, in which B alone can complete the
work = = ‘10t’ days
per L
And the time, in which C alone can complete the

2) Answer: E work = = ‘6t’ days

Since, the difference between CI and SI after 2 Since, A, B and C together can complete the

years: work in 9 days.


So,

And the difference between CI and SI after 3


years:
t=3
Now, the time, in which B alone can complete
So,
the work = 10 * 3 = 30 days
The time, in which C alone can complete the
work = 6 * 3 = 18 days
And the time, in which D alone can complete the

R = 4% work = 3 * 3 = 9 days

Since, the difference between compound interest Since, the part of work completed by B, C and D

and simple interest on ₹ (X – 7000) at (R + 1)% together in 1 day:

rate after 2 years is ₹ 45.


So, So, the time, in which B, C and D together can
complete the work = 5 days

X = 25000
4) Answer: A

Click Here For Bundle PDF Course | support@guidely.in Page 8 of 16


Bank Po Mains PDF Course 2024
Quantitative Aptitude Day -11 (Eng)

Let the speed of the boat in still water is ‘5x’ m/s Since, the average of present ages of A, B and
and the speed of the stream is ‘y’ m/s. C is 23 years.
Since, the time taken by the boat to go 540 m So,
downstream is 20% more than that taken by the A + B + C = 69 -------------(1)
boat to go 300 m upstream. Since, the ratio of A’s age to B’s age after 6
So, years will be 2: 3.
So,

3A + 18 = 2B + 12
15x – 3y = 10x + 2y
2B – 3A = 6 -------------(2)
x = y ------------(1)
By equation (1) * 3 + equation (2):
If the speed of the boat in still water were 20%
3A + 3B + 3C + 2B – 3A = 207 + 6
more:
5B + 3C = 213 -------------(3)
The new speed of boat in still water = 120% of
Since, C’s present age is 70% of B’s present
5x = ‘6x’ m/s
age.
And
So,
6x + y = 21
From equation (1):
6x + x = 21 From equation (1):

x = 3, y = 3
The speed of the boat in still water = 5 * 3 = 15 B = 30
m/s From equation (2):
The speed of the stream = 3 m/s A = 18
Quantity I: From equation (1):
The downstream distance covered by the boat in C = 21
25 seconds: Since, the difference between B’s present age
(15 + 3) * 25 = 450 m and D’s present age is 10 years.
Quantity II: So, D’s present age will be either 30 + 10 = 40
The upstream distance covered by the boat in 35 years or 30 – 10 = 20 years.
seconds: Quantity I:
(15 – 3) * 35 = 420 m A’s present age = 18 years
Hence Quantity I > Quantity II If D’s present age is 40 years:
So, the value, which will be filled in the blank =
5) Answer: B = 45%

Click Here For Bundle PDF Course | support@guidely.in Page 9 of 16


Bank Po Mains PDF Course 2024
Quantitative Aptitude Day -11 (Eng)

If D’s present age is 20 years: So, the new speed of the bus = = 15 m/s
So, the value, which will be filled in the blank = Since, the original speed of the bus = 20 m/s
= 90% So, the value which will be filled in the blank =
Quantity II: = 75%
A’s present age = 18 years Quantity II:
B’s present age = 30 years The new time, in which the bike will cross the
Required percentage = = 60% bridge = 20 seconds
Hence, the relation can’t be established. So, the new speed of the bike = = 12 m/s
Since, the original speed of the bike = 16 m/s
6) Answer: D So, the value which will be filled in the blank =
Let the speed of the bike = ‘2x’ m/s = 75%
So, the speed of the car = 150 of ‘2x’ = ‘3x’ m/s Hence, Quantity I = Quantity II
And the speed of the Bus = (3x – 4) m/s
Since, the time taken by the car to cross the Direction (7-10):
bridge is ‘t’ seconds. Let the time, in which pipe M alone can empty
So, tank P is ‘6m’ minutes.
So, the time, in which pipe N alone can empty
Since, the time taken by the bike to cross the tank Q = = ‘9m’ minutes
bridge is (t + 5) seconds. And the time, in which pipe O alone can empty
So, tank R = = ‘8m’ minutes
Since, the time, in which pipes A and M together

From equations (1) and (2): can fill tank P is 30 minutes.


So,

2t + 10 = 3t
t = 10 Since, the time, in which pipes B and N together

From equation (1): can fill tank Q, is 90 minutes.

x=8 So,

Speed of the bike = 2 * 8 = 16 m/s


Speed of the car = 3 * 8 = 24 m/s From equations (1) and (2):
Speed of the bus = 24 – 4 = 20 m/s
Quantity I:
The new time, in which the bus will cross the
bridge = 16 seconds

Click Here For Bundle PDF Course | support@guidely.in Page 10 of 16


Bank Po Mains PDF Course 2024
Quantitative Aptitude Day -11 (Eng)

m=5 8) Answer: D
From equation (1): The time, in which pipe A alone can fill tank P
with 125% of its original efficiency = =

t = 15 12 minutes
The time, in which pipe M alone can empty tank
P with of its original efficiency = =
36 minutes
Now, the part of tank P filled by pipes A and M
together in 1 minute:

So, the time, in which pipes A and M together

7) Answer: B will fill tank P = 18 minutes

Since, the flowing rate of water from pipe M is The time, in which pipe B alone can fill tank Q

4.5 L per minute. with 125% of its original efficiency = =

And the time, in which pipe M alone can empty 24 minutes

tank P = 30 minutes The time, in which pipe N alone can empty tank

So, the capacity of tank P = 4.5 * 30 = 135 L Q with 93.75% of its original efficiency =

Since, the time, in which pipe A alone can fill = 48 minutes

tank P = 15 minutes Now, the part of tank Q filled by pipes B and N


So, the flowing rate of water from pipe A = x = together in 1 minute:
= 9 L per minute
Since, the flowing rate of water from pipe M = So, the time, in which pipes B and N together will
= 5 L per minute fill tank Q = 48 minutes
And the time, in which pipe N alone can empty Required percentage = = 37.5%
tank Q = 45 minutes
So, the capacity of tank Q = 5 * 45 = 225 L 9) Answer: A
Since, the time, in which pipe B alone can fill Since, the difference between the capacities of
tank Q = 30 minutes tanks P and Q is 30 L.
So, the flowing rate of water from pipe B = y = So, the capacity of tank P = = 210 L
= 7.5 L per minute The capacity of tank Q = = 180 L
So, the value of = = 6.8 And the capacity of tank R = = 240 L

Click Here For Bundle PDF Course | support@guidely.in Page 11 of 16


Bank Po Mains PDF Course 2024
Quantitative Aptitude Day -11 (Eng)

Now, the flowing rate of water from pipe A = Direction (11-14):


= 14 L per minute From the 1st pie chart:
The flowing rate of water from pipe B = =6L X + (Y + 20) = 100 – 30 – 15
per minute X + Y = 35 -------------(1)
And the flowing rate of water from pipe C = = From the 2nd pie chart:

10 L per minute
So, the time, in which pipes A, B and C together X + 4Y = 80 -------------(2)
will fill tank R: From equations (1) and (2):
35 – Y = 80 – 4Y
Y = 15

10) Answer: C From equation (1):

From X: X = 20

Average of time, in which pipe A alone, pipe B


alone and pipe C alone can fill tanks P, Q and R
respectively:

So, X is not true.


From Y:
If the capacity of tank R is 210 L:
Let the total number of persons, who visited the
And the time, in which pipe O alone can empty
museum on Mon, Tue, Wed and Thu are 30t,
tank R = 40 minutes
20t, 35t and 15t respectively.
So, the flowing rate of water from pipe O = =
Also, let the number of males, who visited the
5.25 L per minute
museum on Mon, Tue, Wed and Thu are 40m,
So, Y is not true.
10m, 30m and 20m respectively.
From Z:
Since, the number of females, who visited the
If the difference between the flowing rates of
museum on Tue, is 110.
water from pipes B and N is 3 L per minute:
So,
And pipes B and N together can fill tank Q in 90
20t – 10m = 110
minutes.
2t – m = 11 ----------------(3)
So, the capacity of tank Q = 3 * 90 = 270 L
Since, the number of females, who visited the
So, Z is not true.
museum on Tue, is 175% more than the
Hence, none is true.
females, who visited the museum on Mon.

Click Here For Bundle PDF Course | support@guidely.in Page 12 of 16


Bank Po Mains PDF Course 2024
Quantitative Aptitude Day -11 (Eng)

So, Total females, who visited the museum on Tue =


110

3t – 4m = 4 ---------------- (4) So, the number of female foreigners, who visited

From equations (3) and (4): the museum on Tue = 10% of 110 = 11

t = 8, m = 5 Total number of foreigners, who visited the


museum on Tue = 13 + 11 = 24
Total number of persons, who visited the
museum on Tue = 160
Required percentage = = 15%

13) Answer: A
11) Answer: B Total number of females, who visited the
Total number of persons, who visited the museum in all the 4 days together = 40 + 110 +
museum on Mon = 240 130 + 20 = 300
So, the number of adults in the museum on Mon So, M = = 156°
= 45% of 240 = 108 And, N = = 24°
And the number of children in the museum on From A:
Mon = 240 – 108 = 132 Factors of M = 156 = 12 * 13
Total number of persons, who visited the Factors of N = 24 = 12 * 2
museum on Wed = 280 Since, both M and N are multiple of 12, but not
So, the number of adults in the museum on Wed 24.
= 65% of 280 = 182 So, A is not true.
And the number of children in the museum on From B:
Wed = 280 – 182 = 98 Difference of M and N = 156 – 24 = 132
Required difference = (108 + 182) * 50 – (132 + Since, 132 is a multiple of 12.
98) * 20 = ₹ 9900 So, B is true.
From C:
12) Answer: E Value of = = 1.5
Total males, who visited the museum on Tue = So, C is not true.
50 Hence, only B is true.
So, the number of male foreigners, who visited
the museum on Tue = 26% of 50 = 13 14) Answer: C

Click Here For Bundle PDF Course | support@guidely.in Page 13 of 16


Bank Po Mains PDF Course 2024
Quantitative Aptitude Day -11 (Eng)

The number of male children, who visited the


museum on Mon = 40% of 200 = 80
The number of male children, who visited the
museum on Tue = 20% of 50 = 10
The number of male children, who visited the
museum on Wed = 30% of 150 = 45
The number of male children, who visited the
museum on Thu = 45% of 100 = 45 15) Answer: E
Required average = = 45 Number of males, who took membership in
Tennis only = = 22
Direction (15-18): Number of females, who took membership in
Number of persons, who took membership in all Tennis only = 40 – 22 = 18
the three sports = 5 Number of males, who took membership in
Number of persons, who took membership in Swimming only = = 20
Tennis only = = 40
Number of females, who took membership in
Number of persons, who took membership in Swimming only = 32 – 20 = 12
Swimming only = 80% of 40 = 32 Number of males, who took membership in Golf
Number of persons, who took membership in only = = 10
Golf only = 32 – 2 = 30
Number of females, who took membership in
Number of persons, who took membership in
Golf only = 30 – 10 = 20
both Swimming and Golf, but not Tennis = 60%
Required ratio = (22 + 20 + 10): (18 + 12 + 20) =
of 30 = 18
52: 50 = 26: 25
Number of persons, who took membership in
both Tennis and Golf, but not Swimming = 18 – 3
16) Answer: B
= 15
Total number of persons, who took Swimming
Number of persons, who took membership in
membership:
both Tennis and Swimming, but not Golf =
32 + 10 + 18 + 5 = 65
= 10
So, the total amount collected by the sports club
by giving Swimming membership = 65 * 150 =
₹ 9750
Total number of persons, who took Golf
membership:
30 + 15 + 18 + 5 = 68

Click Here For Bundle PDF Course | support@guidely.in Page 14 of 16


Bank Po Mains PDF Course 2024
Quantitative Aptitude Day -11 (Eng)

So, the total amount collected by the sports club So, the number of females, who took
by giving Golf membership = 68 * 180 = ₹ 12240 membership in only one sport = 102 – 60 = 42
Required difference = 12240 – 9750 = ₹ 2490 Required percentage = = 63.63%

17) Answer: D Direction (19-20):


Total number of persons in the sports club: I:
40 + 32 + 30 + 10 + 18 + 15 + 5 = 150
Total number of persons, who took membership
a2 + 5a – 66=0
of only one sport:
a2 + 11a – 6a – 66 0
40 + 32 + 30 = 102
a (a + 11) – 6 (a + 11) = 0
So, M = = 68%
(a + 11) (a – 6) = 0
Total number of persons, who took membership
a = -11 and 6
of only two sports:
II:
10 + 18 + 15 = 43
So, N =
So, the value of (2M – 3N) = (136 – 86) = 50 2b2 – 3b – 9 = 0
[Which is divisible by both 10 and 5 but not by 2b2 – 6b + 3b – 9 = 0
15]. 2b (b – 3) + 3 (b – 3) = 0
(b – 3) (2b + 3) = 0
18) Answer: A b = 3 and -3/2
Total number of members in the sports club: III:
40 + 32 + 30 + 10 + 18 + 15 + 5 = 150
So, the total number of males in the sports club 3c2 – 4c = 221 + 2c2
= 56% of 150 = 84 c2 – 4c – 221 = 0
And the total number of females in the sports c2 – 17c + 13c – 221 = 0
club = 150 – 84 = 66 c (c – 17) + 13 (c – 17) = 0
Total number of males, who took membership in (c – 17) (c + 13) = 0
at-least two sports = of 84 = 24 c = 17 and -13
So, the number of males, who took membership IV:
in only one sport = 84 – 24 = 60 3d2 – d2 = d + 15
Total number of persons, who took membership 2d2 – d – 15 = 0
in only one sport = 40 + 32 + 30 = 102 2d2 – 6d + 5d – 15 = 0
2d (d – 3) + 5 (d – 3) = 0

Click Here For Bundle PDF Course | support@guidely.in Page 15 of 16


Bank Po Mains PDF Course 2024
Quantitative Aptitude Day -11 (Eng)

(d – 3) (2d + 5) = 0 p = Difference between smaller roots of


d = 3 and -5/2 equations I and III
p = -11 – (-13)
p=2
q = Sum of smaller roots of equations II and IV
q = (-3/2) + (-5/2) = (-3 – 5)/2 =
q = -4
r = Smaller root of equation III
r = -13
Now equation:
19) Answer: E
px2 + qx + (r – 3) = 0
Bigger root of equation I = 6
2x2 + (-4)x + (-13 – 3) = 0
Bigger root of equation II = 3
2x2– 4x – 16 = 0
Bigger root of equation III = 17
x2– 2x – 8 = 0
Bigger root of equation IV = 3
x2– 4x + 2x – 8 = 0
Required sum = P = 6 + 3 + 17 + 3 = 29
x (x – 4) + 2 (x – 4) = 0
P + 2 = 31 and P – 2 = 27
(x – 4) (x + 2) = 0
We can clearly see that, P and ‘P + 2’ are prime
x = 4 and -2
numbers while ‘P – 2’ is not a prime number.

20) Answer: D

Click Here For Bundle PDF Course | support@guidely.in Page 16 of 16


Bank Po Mains PDF Course 2024
English Day - 11

English Language
Directions (1-5): Given below are a few 1. Russia said a. while the Kremlin
questions with a table given with two columns Tuesday it had demanded the military
with highlighted connectors to be matched in deployed jets and prevent any repeat
order to make sentences contextually correct as artillery to destroy an attack.
per the rules of connectors. If none of the options armed group that
are correct then choose option E as your answer. penetrated the border
1. from Ukraine,
Column 1 Column 2 2. Moscow reported b. which has previously
1. The new outbreak a. since China Russian forces killed faced shelling attacks
could be the largest scrapped its zero- more than 70 that have killed dozens
wave of infections Covid policy in Ukrainian fighters and of people since
December 2022. destroyed four Moscow launched its
2. Aleksandr b. who the SBU armored vehicles, offensive last year.
Skachkov, arrested in 2020 during 3. Belgorod's c. but AFP was unable
a raid on people selling Governor Vyacheslav to independently verify
translated versions of Gladkov said civilians the claims.
the Christchurch were evacuated from
Shooter’s Manifesto. nine border villages in
the region,
3. Iran has expanded c. despite opposition A. 1-a, 2-b, 3-c
its missile programme, from the United States B. 1-a, 2-c, 3-b
particularly its ballistic and expressions of C. 1-c, 2-b, 3-a
missiles, concern by European D. 1-b, 2-c, 3-a
countries. E. None of the above

A. 1-a, 2-b, 3-c 3.


B. 1-a, 2-c, 3-b Column 1 Column 2
C. 1-c, 2-b, 3-a 1. The annual a. but Ireland's
D. 1-b, 2-c, 3-a assembly in Geneva regulations are intended
E. None of the above decided not to to be more
extend Taiwan an comprehensive.
2. invitation to the
Column 1 Column 2 event,

Click Here For Bundle PDF Course | support@guidely.in Page 1 of 11


Bank Po Mains PDF Course 2024
English Day - 11

2. Ireland will b. which runs from May usually not released as


introduce mandatory 21-30. legal tender,
health labelling of 3. The Battlegrounds c. However, users will
alcoholic drinks, Mobile India, which only be able to play
warning of the links has been developed the game from May
to cancer, liver by Kraft, can now be 29.
disease and the risk preloaded by all
of drinking Android users starting
3. Other countries c. while pregnant. from today, May 27.
include warnings on A. 1-a, 2-b, 3-c
alcohol products, B. 1-a, 2-c, 3-b
A. 1-a, 2-b, 3-c C. 1-c, 2-b, 3-a
B. 1-a, 2-c, 3-b D. 1-b, 2-c, 3-a
C. 1-c, 2-b, 3-a E. None of the above
D. 1-b, 2-c, 3-a
E. None of the above 5.
Column 1 Column 2
4. 1. The previous version a. which generally
Column 1 Column 2 of the game, PUBG lowers global
1. The question for a. especially given Mobile, continues to be temperatures slightly,
investors is whether to mounting concern banned in India for the world will
jump on the AI train amongst regulators similar security experience a return to
now, or exercise about the technology's reasons and its El Nino, the warmer
caution, potentially disruptive associations with counterpart, later this
impact. China. year.
2. Higher b. On the other hand, 2. Further research on b. In addition, certain
denomination lower denomination X-rays from lawmakers expressed
commemorative coins, commemorative coins supernovae is valuable their opposition to the
which have precious usually stay in not just for availability of the
metals like gold and circulation for a limited understanding the life game in India, citing
silver in them, are time. cycle of stars, concerns about its
usually treated as impact on children.
collectibles and are

Click Here For Bundle PDF Course | support@guidely.in Page 2 of 11


Bank Po Mains PDF Course 2024
English Day - 11

3. Climate models c. but also has manufacturing, utility, transportation,


suggest that after three implications for fields construction, maritime, government, information
years of the La Nina like astrobiology, technology, and education.
weather pattern in the paleontology, and the A. Only a
Pacific Ocean, earth and planetary B. Only b
sciences. C. Only c
A. 1-a, 2-b, 3-c D. Both A&B
B. 1-a, 2-c, 3-b E. No error
C. 1-c, 2-b, 3-a
D. 1-b, 2-c, 3-a 7.
E. None of the above a. Under the new Twitter Blue program rolled out
last month, individuals can pay $8 a month for a
Directions (6-10): Given below are a few blue check mark.
questions with three sentences in each of them b. Under its new owner, Elon Musk, Twitter has
which may or may not contain errors in them. allowed paid subscriptions for anyone to obtain a
You have to find the sentence which has or have verified account in exchange for a monthly
errors in them and mark your answers charge.
accordingly. If none of the sentence contain c. The finance ministry said the new Rs 75 coin
errors then choose option E as your answer. will bear the inscription of the Parliament
6. complex and have the image of the new
a. Gripped by economic turmoil and suffering a Parliament building.
balance of payments crisis, Pakistan is trying to A. Only a
reach agreement with the International Monetary B. Only b
Fund (IMF) to disburse the stalled final $1.1 C. Only c
billion from a $6.5 billion bailout agreed in 2019. D. Both A&C
b. As China has stepped up military and E. No error
diplomatic pressure in its claim to democratically
governed Taiwan, US President Joe Biden has 8.
said he would be willing to use force to defend a. Life on Earth in itself is fragile and was a result
Taiwan. of millions of coincidences coming together on
c. Microsoft said the Chinese hacking group has the planet for us to emerge and survive.
been active since at least 2021 and was targeted b. A team of explorers announced it found a
several industries including communications, sunken Japanese ship that was transporting
Allied prisoners of war when it was torpedoed off

Click Here For Bundle PDF Course | support@guidely.in Page 3 of 11


Bank Po Mains PDF Course 2024
English Day - 11

the coast of the Philippines in 1942, resulting in 10.


Australia’s largest maritime wartime loss with a a. The world's hot year on record so far was
total of 1,080 lives. 2016, coinciding with a strong El Nino - although
c. The launch pad on the Satish Dhawan Space climate change has fuelled extreme
Centre is set to be in a busy mode for the next temperatures even in years without the
few months. The Indian Space Research phenomenon.
Organisation (Isro) is all set to conduct several b. Europe experienced its hottest summer on
big launches in a year it has decided to not just record in 2022, while climate change-fuelled
speed up its operations but also its turnaround extreme rain caused disastrous flooding in
time for rockets. Pakistan, and in February, Antarctic sea ice
A. Only a levels hit a record low.
B. Only b c. Despite most of the world's major emitters
C. Only c pledging to eventually slash their net emissions
D. Both A&B to zero, global CO2 emissions last year
E. No error continued to rise.
A. Only a
9. B. Only b
a. Isro said that it is gearing up for several bug C. Only c
missions in mid-2023. The missions will not only D. Both A&B
explore deep space but also the Sun and the E. No error
Moon.
b. ISRO will launch the much-awaited Directions (11-15): Given below are a few
Chandrayaan-3 mission to the Moon with the question with two columns given and both
next couple of months. contains parts of sentences to be matched in
c. The world could breach a new average order to make the sentence contextually correct.
temperature record in 2023 or 2024, fuelled by If none of the options are correct then choose
climate change and the anticipated return of the option E as your answer.
El Nino weather phenomenon, climate scientists 11.
say. Column 1 Column 2
A. Only a 1. Drugs are often a. while in Europe,
B. Only b identified with a symbol captured leaders
C. Only c representing stood trial for crimes
D. Both A&B against humanity.
E. No error

Click Here For Bundle PDF Course | support@guidely.in Page 4 of 11


Bank Po Mains PDF Course 2024
English Day - 11

2. Peru and Colombia b. the group which 3. For about a year, c. due to recurring pain
are the largest makes or moves the the pope has had to in his knee that he has
producers of coca leaf product. rely on a wheelchair said cannot be treated
through surgery.
3. After World War II, c. and cocaine in the
thousands of high- world, according to A. 1-a, 2-b, 3-c
ranking Nazi officers the United Nations. B. 1-a, 2-c, 3-b
sought refuge in South C. 1-c, 2-b, 3-a
America, D. 1-b, 2-c, 3-a
A. 1-a, 2-b, 3-c E. None of the above
B. 1-a, 2-c, 3-b
C. 1-c, 2-b, 3-a 13.
D. 1-b, 2-c, 3-a Column 1 Column 2
E. None of the above 1. Suffering a massive a. of nationalism that
economic crisis and has impacted the fate
12. surrounded by of its biggest election of
Column 1 Column 2 countries at war, modern times.
1. The pope's morning a. heads of states, Turkey is witnessing a
audiences are usually associations and resurgence
reserved for clerics, while his 2. Often referred to as b. illustrating the
afternoons are devoted the ‘world’s most continued demand
to work and private notorious’ spyware, among governments
meetings. and the lack of joint
international efforts to
2. Francis, who has b. for a decade, has limit the use of such
been the leader of the suffered increasing tools.
world's 1.3 billion health issues over the
Catholics past year, from 3. It was found to c. Pegasus often found
persistent pain in his have been used in its traces in the circles
right knee to sciatica to another dozen of journalists,
his recent hospital stay countries since 2021, oppositions and critics.
for bronchitis.

Click Here For Bundle PDF Course | support@guidely.in Page 5 of 11


Bank Po Mains PDF Course 2024
English Day - 11

A. 1-a, 2-b, 3-c B. 1-a, 2-c, 3-b


B. 1-a, 2-c, 3-b C. 1-c, 2-b, 3-a
C. 1-c, 2-b, 3-a D. 1-b, 2-c, 3-a
D. 1-b, 2-c, 3-a E. None of the above
E. None of the above
15.
14. Column 1 Column 2
Column 1 Column 2 1. WhatsApp, which is a. to defend itself, and
1. Sometime in mid- a. the Pegasus Project owned by Meta, is he has cast the
2021, Anna initiative investigated among a number of Ukraine war as a battle
Naghdalyan who is the the use of spyware by for the survival of
spokesperson governments on Russia against an
journalists, opposition aggressive West.
politicians, activists
and business people. 2. Russian President b. tech companies
Vladimir Putin says the pursuing legal action
2. Citizen Lab, a b. suspected Pegasus United States and its against the NSO group
Toronto-based operators in allies since 2019.
research organisation Azerbaijan that they
identified at least two call “BOZBASH” and 3. Putin has repeatedly c. are fighting an
“YANAR” by their warned that Russia, expanding proxy war
ongoing internet which has more against Russia after
scanning and DNS nuclear weapons than the Kremlin chief sent
cache probing. any other country, will troops into Ukraine 15
use all means months ago.
3. In a collaborative c. for Armenia’s
investigation by 17 foreign affairs agency, A. 1-a, 2-b, 3-c
media agencies was alerted by Apple B. 1-a, 2-c, 3-b
including Forbidden that her iPhone was C. 1-c, 2-b, 3-a
Stories, under attack by a D. 1-b, 2-c, 3-a
foreign government. E. None of the above

A. 1-a, 2-b, 3-c

Click Here For Bundle PDF Course | support@guidely.in Page 6 of 11


Bank Po Mains PDF Course 2024
English Day - 11

Directions (16-20): Given below are a few 18. For years, Armenia and Azerbaijan has been
questions with a highlighted phrase in each of at loggerheads over the dispute Nagorno-
them which may or may not require replacement. Karabakh region. While it’s internationally
If replacement is required then choose a proper recognised as part of Azerbaijan, many of its
replacement from the options. If no replacement residents are Armenian nationals.
is required then choose option E as your answer. A. Armenia or Azerbaijan have been at
16. NSO Group’s infamous Pegasus spyware, loggerheads over the disputed
which has been earlier in the news for its misuse B. Armenia and Azerbaijan has been at
against political and civil rights individuals, has loggerheads over the disputed
found its use in a war zone. C. Armenia and Azerbaijan have been at
A. which had been earlier in the news for its loggerheads over the disputed
misuse against D. Armenia and Azerbaijan had been at
B. which have been earlier in the news for its loggerheads over the disputed
misuse against E. No replacement is required
C. who has been earlier in the news for its
misuse against 19. Russia had been an huge numerical
D. who had been earlier in the news for its superiority over the United States and the NATO
misuse against military alliance when it comes to tactical nuclear
E. No replacement is required weapons: the United States believes Russia has
around 2,000 such working tactical warheads.
17. Global Intelligence and cyberwarfare in the A. Russia have a huge numerical superiority over
digital age altered forever when an Israeli the United States and
company, the NSO Group introduce its military- B. Russia has an huge numerical superiority over
grade spyware Pegasus in 2011 to the global the United States and
market. C. Russia had a huge numerical superiority over
A. the NSO Group introduced his military-grade the United States and
spyware D. Russia has a huge numerical superiority over
B. the NSO Group introduced her military-grade the United States and
spyware E. No replacement is required
C. the NSO Group introduce their military-grade
spyware 20. Vietnam and China have long been
D. the NSO Group introduced its military-grade embroiled in a territorial dispute over a potentially
spyware energy-rich stretch in South China Sea, a
E. No replacement is required

Click Here For Bundle PDF Course | support@guidely.in Page 7 of 11


Bank Po Mains PDF Course 2024
English Day - 11

strategic waterway through which more than $3 C. in a territorial dispute over a potentially
trillion of commerce passes annually. energy-rich stretch in the South China Sea
A. in a territorial dispute over a potentially D. in a territorial dispute over an potentially
energy-rich stretch in a South China Sea energy-rich stretch in the South China Sea
B. in a territorial dispute over a potentially E. No replacement is required
energy-rich stretch at the South China Sea
Click Here to Get the Detailed Video Solution for the above given Questions
Or Scan the QR Code to Get the Detailed Video Solutions

Answer Key with Explanation


1. Answer: A 3. Answer: D
Here, the first part of column 1 will be matched Here, the first part of column 1 will be matched
wthe ith first part of column 2 as it is connected with the second part of column 2 as it is
by since as we use it to give a reason for connected by which, the second part of column 1
something and here the same can be seen, the will be connected with the third part of column 2
second part of column 1 will be matched with and the final part of the first column will be
second part of column 2 and finally the leftover matched with the first part of column 1. So,
will be matched. So, option A is the correct option D is the correct answer.
answer.
4. Answer: A
2. Answer: B Here, all of the parts of column 1 will be matched
Here, the first part of column 1 will be matched with the parts which are just in opposite of them
with a and second part with c and the final means with the same parts of column 2. So,
leftover part will be matched with b. So, option B option A is the correct answer.
is the correct answer. 5. Answer: D

Click Here For Bundle PDF Course | support@guidely.in Page 8 of 11


Bank Po Mains PDF Course 2024
English Day - 11

Here, the first part of column 1 will be matched Here, the error lies in sentence b as use of with
with the second part of column 2 as both are before the next is inappropriate as within should
connected by in addition to as the same context have been used as within is used in the context
with further information is given, the second part of time as it means under certain limitations of
of column 1 will be matched with the third part of time given and here the same can be seen. So,
column 2 as both are connected with but also option B is the correct answer.
which is used to add emphasis when writing
about two related things and finally the third part 10. Answer: A
of column 1 will be matched with the first part of Here, the error lies in sentence a as use of hot is
column 2 which is connected by which. So, inappropriate because as per the context of the
option D is the correct answer. sentence the superlative degree should have
been used which is hottest. The superlative
6. Answer: C degrees are used when the noun in a particular
Here, the error lies in part c as use of was before context has the greatest or least degree of the
targeted is inappropriate as here sentence is in quality and here greatest quality is there. So,
perfect continuous tense from starting and option A is the correct answer.
second part is in present perfect tense and
therefore has should have been used here due 11. Answer: D
to perfect tense and singular subject. So, option Here, the first part of column 1 which is about
C is the correct answer. drugs should be matched with second part of
column 2 and second part of column 1 which is
7. Answer: E about the peru and Colombia being largest
Here, none of the sentence contains error. So, producer should be matched with third part of
option E is the correct answer. column 2 and finally the third part of column one
will be matched with first part of column 2. So,
8. Answer: A option D is the correct answer.
Here, the error lies in part a as use of was before
a result is inappropriate as sentence is in 12. Answer: A
present tense and therefore is should have been Here, all of the parts given in first column will be
used. So, option A is the correct answer. matched with the part given same in the column
2. So, option A is the correct answer.
9. Answer: B
13. Answer: B

Click Here For Bundle PDF Course | support@guidely.in Page 9 of 11


Bank Po Mains PDF Course 2024
English Day - 11

Here, the first part of column 1 which is about Here, the phrase highlighted is grammatically
turkey experiencing economic crisis should be correct and no replacement is required. So,
matched with first part of column 2, the second option E is the correct answer.
part of column 1 which is about notorious
spyware should be matched with the third part of 17. Answer: D
column 2 and finally the third part of column 3 Here, there should be introduced in place of
which is about the use of something by countries introduce as the sentence is in past tense and so
should be matched with the second part of the second form of verb will be used in case of
column 2. So, option B is the correct answer. past simple tense. So, option D is the correct
answer.
14. Answer: C
Here, the first part of column 1 which is about 18. Answer: C
some spokesperson should be matched with the Here, the use of has been is inappropriate as
third part of column 2, the second part of column subject is plural and therefore have been will be
1 which is about some research being done used and also the dispute should be changes
should be matched with the second part of into disputed as the second part of the sentence
column 2 and finally the third part of column 1 is in past simple tense and therefore it will use
which is about some investigations should be second form of the verb. So, option C is the
matched with the first part of column 2. So, correct answer.
option C is the correct answer.
19. Answer: D
15. Answer: D Here, the use of had been is inappropriate as
Here, the first part of column 1 which is about has should have been used as sentence is in
whatsapp should be matched with the second present perfect continuous tense and also the
part of column 2, the second part of column 1 use of an is also inappropriate as huge doesn’t
which is about putting and his statement about sound like a vowel and also h is not a vowel and
US should be matched with the third part of therefore, a should have been used. So, option
column 2 and finally the third part of column 1 D is the correct answer.
which is about warning given by Putin should be
matched with the first part of column 2. So, 20. Answer: C
option D is the correct answer. Here, there should be use of ‘the’ in between in
and South China sea as the article the is used
16. Answer: E before seas and here the sea is mentioned and

Click Here For Bundle PDF Course | support@guidely.in Page 10 of 11


Bank Po Mains PDF Course 2024
English Day - 11

therefore, the is required as an article. "The" is oceans and seas. So, option C is the correct
used in these case before the names of deserts, answer.

Click Here For Bundle PDF Course | support@guidely.in Page 11 of 11


Bank Po Mains PDF Course 2024
Reasoning Day -12 (Eng)

Reasoning Aptitude
Directions (1-5) Study the following information designated senior to N, who is neither working
carefully and answer the given questions. with O nor R.
Twenty one persons are working in an All the persons except senior to Manager are
organization at different designations viz.- transferred to three different departments viz.-
Director, General Manager (GM), Manager, Production, Marketing, and Sales as per below
Assistant Manager (AM), Probationary Officer conditions:
(PO), and Clerk. The hierarchy of designations Condition I: All persons whose name comes
are given in decreasing order such that before ‘M’ in alphabetical series and even
Chairman is the highest designation whereas number of persons working along with them are
clerk is the lowest designation. transferred to Production department.
Note: I. The number of persons working at each Condition II: All persons whose name comes
designation is one less than the number of after ‘M’ in alphabetical series and odd number of
persons working its immediate lower designation. persons working along with them are transferred
II. If said that A works with B or A and B are to Marketing department.
working together, then both A and B are working Condition III: All the remaining persons are
at the same designation. transferred to Sales departement.
B, I, and T are working together but are not The persons senior to Manager are appointed as
designated as Assistant managers. Odd number supervisors of all three departments viz.-
of persons are designated between T and F, who Production, Marketing, and Sales in alphabetical
is immediately junior to H. Both H and D are order respectively.
neither designated as probationary officer nor the 1) Which among the following pair of persons are
General Manager. D and O are working together not supervised by H?
and designated junior to F. Odd number of I. TQD
persons are working along with O. The number II. BIL
of persons senior to G is the same as the III. ORP
number of persons junior to Q, who is two a) Only I and III
designations junior to M. Neither M nor A working b) Only II and III
with H. Odd number of persons are designated c) Only III
between A and R, who is designated immediately d) Only I
senior to L. Both A and J are designated senior e) All I, II and III
to assistant manager. P, S and K are working
together and designated junior to L. Both E and 2) _____ persons works with _____ who works
U are not senior to assistant manager. C is as _______

Click Here For Bundle PDF Course | support@guidely.in Page 1 of 12


Bank Po Mains PDF Course 2024
Reasoning Day -12 (Eng)

a) Four, T, PO Eight persons – Rimii, Kimmi, Simmi, Timmy,


b) Three, C, Manager Pummy, Summy, Gummy and Rummy were born
c) Three, G, AM in different years. The age of persons is
d) Both a and c calculated on the base year 2023. Each person
e) Both a and b watched different OTT platforms viz.-Hotstar,
Arre, TVF, Zee5, Amazon, Netflix, Voot and Sony
3) _______ persons are working as senior to Liv.
_____ Note: I. If it is given A’s age is equal to the last
a) Three, Q two digits of B’s birth year, then it may be in any
b) Ten, E order. For example, if B was born in 1965, then
c) Six, R A’s age is either 65 or 56.
d) Three, A II. None of the persons were born after 2012 and
e) One, J born before 1950.
Rummy was born in 2008. Only two persons
4) Which of the following pair of persons are not were born between Rummy and the one who
working together? watched Amazon. The difference between the
I. GDR ages of Rummy and Simmi is 15 years. The one
II. BIJ who watched Amazon was born immediately
III. PEN before Simmi but not after 1986. The age of
a) Only I and III Timmy, who watched Hotstar, is the last two
b) Only II and III digits of the birth year of Simmi. The difference
c) Only III between the ages of Timmy and the one who
d) Only I watched Zee5 is 28 years. The difference
e) Only II between the ages of Pummy and the one who
5) Which of the following statements is/are not watched Zee5 is 17 years. Only one person was
true? born between Pummy and the one who was born
a) J and C transferred to production in 1986. The difference between the ages of
b) O and S are supervised by H Pummy and Timmy is the same as the difference
c) E, P and U are working together between the ages of Timmy and Kimmi. Rimmi’s
d) Only three persons are designated senior to C age is equal to the last two digits of the birth year
e) All the above statements are true of Kimmi, who was born immediately after the
one who watched Netflix. Rimmi was neither
Directions (6-10) Study the following information born in 1964 nor watched Netflix. The number of
carefully and answer the given questions. persons born after the one who watched Voot is

Click Here For Bundle PDF Course | support@guidely.in Page 2 of 12


Bank Po Mains PDF Course 2024
Reasoning Day -12 (Eng)

one more than the number of persons born 9) Who among the following person watched
before the one who watched Tvf. No one was Voot?
born between the one who watched Tvf and a) The one who was born immediately after
Summy. The difference between the ages of Kimmi
Summy and the one who was born immediately b) Timmy
before the one who watched Sony Liv is 18 c) The one who was born in 1993
years. d) Pummy
6) Who among the following person watched e) None of these
Tvf?
I. The one who was born in 1973 10) Which of the following statements is/are not
II. The one who was born immediately after true as per the given arrangement?
Summy a) Gummy was born in 2012
III. Kimmi b) Simmi Watched Sony Liv
a) Only I and III c) Only two persons were born before Timmy
b) Only II and III d) The one who watched Amazon was born in
c) Only III 1984
d) Only I e) All the above statements are true
e) All I, II, and III
Directions (11-15) Study the following
7) _____ watched ____ and was born _____ information carefully and answer the given
persons after Timmy questions.
a) Gummy, Zee5, Two In a certain code language following statements
b) Kimmi, Hotstar, Two are coded as follows:
c) Pummy, Voot, Three “Perfect Driver Safety Release Counter” is coded
d) Summy, Tvf, Two as “%8G %12G @10E &8V $12U”
e) None of these “Clint Party Manual Define Campaign” is coded
as “@4P *9C $9P #15I &4R”
8) The one who watched ______ was born in “Poverty Crown Mobile Several Bought” is coded
_____ as “&10V #4Y __(A)___ *12C @8J”.
a) Hotstar, 1984 “Action Polar Count Flight Service” is coded as
b) Netflix, 1973 “%6C @6P #9Q __(B)__ $12E”.
c) Amazon, 1985 11) What comes in place of ___(A)____?
d) Voot, 1993 a) &12U
e) Both b and d b) #9V

Click Here For Bundle PDF Course | support@guidely.in Page 3 of 12


Bank Po Mains PDF Course 2024
Reasoning Day -12 (Eng)

c) $9N
d) @4T Directions (16-20) Study the following
e) None of these information carefully and answer the given
questions.
12) Code of which of the following word is Seven different items viz.-Cookies, Cake,
incorrect in the bold highlighted codes? Biscuits, Chocolate, Cream Roll, Pastry, and Ice
a) Clint - @4P Cream are scheduled to be delivered one by one
b) Define - $9P in different apartments viz.- A, B, C, D, E, F, G
c) Party - &4R and H. No item was delivered in one of the
d) Campaign - #15I apartments due to cancelation of order. Atmost
e) None of these one item was delivered in each apartment.
Note: The apartment in which no item was
13) What comes in place of ___(B)___? delivered is scheduled to be delivered neither
a) @5J first nor last.
b) $6M Only three items were delivered between Cake
c) @7F and the item which was delivered in apartment
d) &5U G, which was delivered after Cake. Only one
e) None of these item was delivered between the item which was
delivered in apartment G and Cream Roll. Cream
Roll was delivered in apartment A and delivered
14) What is the code of “Safety Release”? before the item which was delivered in apartment
a) &8V $12U G. Chocolate was delivered immediately before
b) %12G &8R the item which was delivered in apartment A.
c) $12Q @4L Only two items were delivered between
d) @4L $12Q Chocolate and the item which was delivered in
e) None of these apartment F, which was delivered before
Chocolate. Neither Pastry nor Biscuit delivered in
15) Which of the following combination of codes apartment F. The number of items delivered
are not true? before Biscuit is one more than the number of
a) Perfect - @10E items delivered after Cookies. Cookies was
b) Action - #9Q delivered immediately after Pastry. Neither
c) Define - $9P Biscuit nor the item which was delivered in
d) Several - &10R apartment H was delivered immediately before or
e) Bought - @8J immediately after the apartment in which no item

Click Here For Bundle PDF Course | support@guidely.in Page 4 of 12


Bank Po Mains PDF Course 2024
Reasoning Day -12 (Eng)

was delivered. Neither Cake nor Pastry was a) Three


delivered in both apartments E and H. The item b) One
which was delivered in apartment D was c) Four
delivered immediately after the item which was d) Two
delivered in apartment B. Chocolate was neither e) None
delivered in apartment C nor D. At least one item
was delivered in apartments H and C. 19) Which among the following item was
16) Biscuit was delivered in which of the delivered after Cream Roll?
following apartment? I. Pastry
a) F II. Cookies
b) B III. Cake
c) A a) Only II
d) E b) Only II and III
e) None of these c) Only I and II
d) Only III
17) Which of the following combinations of e) Only I
apartment and item delivered is true?
a) G – Pastry 20) In which of the following apartment no item
b) D – Cake was delivered?
c) B – Ice cream a) C
d) H – Biscuit b) A
e) E – Cookies c) E
d) G
18) How many items were delivered before the e) H
item which was delivered in apartment D?

Click Here For Bundle PDF Course | support@guidely.in Page 5 of 12


Bank Po Mains PDF Course 2024
Reasoning Day -12 (Eng)

Click Here to Get the Detailed Video Solution for the above given Questions
Or Scan the QR Code to Get the Detailed Video Solutions

Answer Key with Explanation


Directions (1-5):  Odd number of persons are designated
1. Answer: B between T and F, who is immediately
2. Answer: D junior to H.
3. Answer: C  Both H and D are neither designated as
4. Answer: E probationary officer nor the General
5. Answer: B Manager.
That means, in case (1) B, I and T are
working as probationary officer, in case
(2) B, I, and T are working as clerk.
Based on the above given information we have:

After rearrangement:

Again, we have:
 D and O are working together and
We have: designated junior to F.
 B, I, and T are working together but are  Odd number of persons are working along
not designated as Assistant manager. with O.

Click Here For Bundle PDF Course | support@guidely.in Page 6 of 12


Bank Po Mains PDF Course 2024
Reasoning Day -12 (Eng)

Since, odd number of persons working


along with O, then even number of
persons are working on O’s designation.
That means, in case (1) O working as an
assistant manager, in case (1a) & case
(2) O working as clerk.
Based on the above given information we have: Case (2) is not valid as PSK working together
and junior to L but no such position available.
Again, we have:
 Both E and U are not senior to assistant
manager.
 C is designated senior to N, who is
neither working with O nor R.

Again, we have: That means, in case (1) E and U working

 The number of persons senior to G is the as clerk, case (1a) is not valid.

same as the number of persons junior to Based on the above given information we have:

Q, who is two designations junior to M.


 Neither M nor A working with H.
That means, in case (1) and in case (1a)
G is designated as an assistant manager
& in case (2) G is designated as clerk.
 Odd number of persons designated
between A and R, who is designated
All the persons except senior to Manager are
immediately senior to L.
transferred to three different departments viz.-
 Both A and J are designated senior to
Production, Marketing, and Sales as per below
assistant manager.
conditions:
 P, S and K are working together and
Condition I: All persons whose name comes
designated junior to L.
before ‘M’ in alphabetical series and even
That means, in case (1) & case (1a) A is
number of persons working along with them are
working with F, and case (2) is not valid.
transferred to Production department.
Based on the above given information we have:
Condition II: All persons whose name comes
after ‘M’ in alphabetical series and odd number

Click Here For Bundle PDF Course | support@guidely.in Page 7 of 12


Bank Po Mains PDF Course 2024
Reasoning Day -12 (Eng)

of persons working along with them are We have:


transferred to Marketing department.  Rummy was born in 2008.
Condition III: All the remaining persons are  Only two persons were born between
transferred to Sales department. Rummy and the one who watched
The persons senior to Manager are appointed as Amazon.
supervisors of all three departments viz.-  The difference between the ages of
Production, Marketing, and Sales in alphabetical Rummy and Simmi is 15 years.
order respectively.  The one who watched Amazon was born
Based on the above given information we have: immediately before Simmi but not after
1986.
Thus, Simmi was born in 1993.
 The age of Timmy, who watched Hotstar,
is the last two digits of the birth year of
Simmi.
Since, Simmi was born in 1993, and no
Directions (6-10): one was born before 1950, thus age of
6. Answer: E Timmy must be 39 years.
7. Answer: C Based on the above given information we have:
8. Answer: A
9. Answer: D
10. Answer: D

Again, we have:
 The difference between the ages of
Timmy and the one who watched Zee5 is
28 years.
Since, Timmy was born in 1984, thus the
one who watched Zee5 must born in 1956
or 2012.

Click Here For Bundle PDF Course | support@guidely.in Page 8 of 12


Bank Po Mains PDF Course 2024
Reasoning Day -12 (Eng)

That means, in case (1) the one who Case (1) is not valid as Rimmi was not
watched Zee5 was born in 1956, in case born in 1964.
(2) the one who watched Zee5 was born Based on the above given information we have:
in 2012.
 The difference between the ages of
Pummy and the one who watched Zee5 is
17 years.
 Only one person was born between
Pummy and the one who was born in
1986.
Based on the above given information we have:

Case (1) is not valid as Rimmi was not born in


1964.
Again, we have:
 The number of persons born after the one
who watched Voot is one more than the
number of persons born before the one
who watched Tvf.
Again, we have:  No one was born between the one who
 The difference between the ages of watches Tvf and Summy.
Pummy and Timmy is the same as the  The difference between the ages of
difference between the ages of Timmy Summy and the one who was born
and Kimmi. immediately before the one who watched
That means, in case (1) Kimmi was born Sony Liv is 18 years.
in 1995, in (2) Kimmi was born in 1973. Since, Summy was born before 1973,
 Rimmi’s age is equal to the last two digits thus only possible person who watched
of the birth year of Kimmi, who was born Sony Liv is Simmy, thus Summy must
immediately after the one who watched born in 1968.
Netflix. Based on the above given information we have:
 Rimmi was neither born in 1964 nor
watched Netflix.

Click Here For Bundle PDF Course | support@guidely.in Page 9 of 12


Bank Po Mains PDF Course 2024
Reasoning Day -12 (Eng)

Directions (16-20):
16. Answer: B
17. Answer: B
18. Answer: D
19. Answer: C
20. Answer: C
Directions (11-15):
11. Answer: C
12. Answer: C
13. Answer: A
14. Answer: A
15. Answer: D
We have:
For symbols: last letters of the word represents
symbol such as:
T  @, N  #, R  %, Y &, E  $, L  *
For number: product of the number of vowels
and consonants in the words are taken.
For letters: Second succeeding letter of the
second last letter of the word in alphabetical We have:
order are taken.  Only three items were delivered between
For example: Cake and the item which was delivered
Perfect: @10E in apartment G, which was delivered
Based on the rule followed we have: after Cake.

Click Here For Bundle PDF Course | support@guidely.in Page 10 of 12


Bank Po Mains PDF Course 2024
Reasoning Day -12 (Eng)

 Only one item was delivered between the  Neither Pastry nor Biscuit delivered in
item which was delivered in apartment G apartment F.
and Cream Roll.  The number of items delivered before
 Cream Roll was delivered in apartment A Biscuit is one more than the number of
and delivered before the item which was items delivered after Cookies.
delivered in apartment G.  Cookies was delivered immediately after
That means, in case (1) & case (3) Pastry.
Cream roll was delivered two  Neither Biscuit nor the item which was
apartments before G, in case (2) Cream delivered in apartment H was delivered
roll was delivered three apartments immediately before or immediately after
before G. the apartment in which no item was
Based on the above given information we have: delivered.
That means, in case (2) & case (3) Biscuit
was delivered immediately after item
which was delivered in apartment F, case
(1) is not valid.
Based on the above given information we have:

Again, we have:
 Chocolate was delivered immediately
before the item which was delivered in
apartment A.
 Only two items were delivered between
Chocolate and the item which was Case (1) is not valid as no place available for
delivered in apartment F, which was Biscuit.
delivered before Chocolate. Again, we have:
Since, the apartment in which no item is  Neither Cake nor Pastry delivered in both
delivered is between any two apartments. apartments E and H.
Thus, item in apartment F is delivered  The item which was delivered in
first. apartment D was delivered immediately

Click Here For Bundle PDF Course | support@guidely.in Page 11 of 12


Bank Po Mains PDF Course 2024
Reasoning Day -12 (Eng)

after item which was delivered in


apartment B.
 Chocolate was neither delivered in
apartments C nor D.
That means, Biscuit was delivered in
apartment B.
 At least one item was delivered in
apartments H and C.
That means, in case (2) Chocolate was
delivered in apartment H, case (3) is not
valid.
Case (3) is not valid as Neither Biscuit nor the
Based on the above given information we have:
item which was delivered in apartment H was
delivered immediately before or immediately
after the apartment in which no item was
delivered.

Click Here For Bundle PDF Course | support@guidely.in Page 12 of 12


Bank Po Mains PDF Course 2024
Quantitative Aptitude Day -12 (Eng)

Quantitative Aptitude

Directions (1 - 4): Study the following information carefully and answer the questions given below.
Three types of elections are conducted in panchayat [Gram panchayat (village level), Panchayat samite
(block level) and District Panchayat (District level)] in a single day. The given table shows the number of
blocks in four different states and, the average number of registered voters in each village. Only party X,
Y and Z contests in the election.

Note – I. There are two villages in each block. Total registered Voters of each block = Total registered
voters in all villages in the block. Total registered voters in each district = Total registered voters in all
villages together. Each registered voter who comes to give vote should vote in all three elections [i.e., the
Total cast vote should be equal in all three types of the election] and their vote may differ in each
election, [ i.e., If a voter gives the vote to party X in village election he or she may or may not be give vote
to party X in others two types of election].
II. In each village only one candidate for each party contests the election, in each block only one
candidate for each party contests the election, and for each district only one candidate for each party
contests the election.
1) In the District panchayat election of state A, c) 2800
Party Z got 25% of registered voters and got 3 rd d) 2500
position, and Party Y got 2900 votes which is e) 2900
100 votes less than Party X. In the block election
of the same state, party X got 30% of vote of 2) In each village of B, 10% of registered voters
total casted votes and party Y and Z got equal did not cast their vote and 10% of casted votes
numbers of votes. The sum of votes of party X were declared as invalid. In the village election
at village level and block level is 4400 and for party X got 7530 votes and the rest of the votes
party Y is 6000. Find the number of votes got by were casted to Y and Z equally. Block Parties X,
party Z in village elections? [All cast vote is valid] Y and Z got votes in the ratio of 3:2:4. In total,
a) 2240 Party Y got 30% of votes in all the formats
b) 2600 together. Find the difference between the total

Click Here For Bundle PDF Course | support@guidely.in Page 1 of 12


Bank Po Mains PDF Course 2024
Quantitative Aptitude Day -12 (Eng)

vote of party Z in all the three types and total d) 2800


vote of party Y in all the three types of election? e) None of these
a) 2410
b) 3890 4) In District D, 80% of people are registered
c) 2530 voters. The ratio of males and females out of the
d) 3020 total population in D is 3:2. 70% of female voters
e) CND are registered voters and all the male voters cast
their vote in favor of party X in district and village
3) 25% of registered voters did not cast their elections and in favor of party Y in block
votes in District C and 400, 1600 and 1200 votes elections. Females cast their vote in favor of
of village, block and district elections were Party Z in all elections. Find the difference
declared invalid. The ratio of the total vote of between Party X's vote in the district and Party Z
parties X, Y and Z in all villages together, all in the block election. [All voters cast their votes
blocks together and district elections are 15:8:15, and all votes are valid]
7:5:5 and 1:2:1 respectively. Find the total votes a) 2520
of party Y in the block election and Z in the b) 2240
district election? c) 2630
a) 2000 d) 2120
b) 3000 e) None of these
c) 2500

Directions (5 - 8): Study the following information carefully and answer the questions given below.
The pie chart shows the percentage distribution of the total number of staff in five different factories [A, B,
C, D, and E].

Click Here For Bundle PDF Course | support@guidely.in Page 2 of 12


Bank Po Mains PDF Course 2024
Quantitative Aptitude Day -12 (Eng)

The pie chart shows the percentage distribution of the total number of graduate staff in five different
factories [A, B, C, D, and E].

Note –
I. R-Q=8%. P is 50% more than Q.

Click Here For Bundle PDF Course | support@guidely.in Page 3 of 12


Bank Po Mains PDF Course 2024
Quantitative Aptitude Day -12 (Eng)

II. the Ratio of Q and S is 1:2.


III. The Total Number of graduate staff in all factories together is 5600.
IV. The total number of non-graduate staff in E is 660.
5) Out of the total number of staff in C, T% are ratio of the number of staff using private
female and the rest are male. Out of total male transport in B and D is 29:48. Find the difference
(4R-5) % are graduates. Find the possible value between the number of employees using public
of non-graduate female staff in C? transport in B and D together and number of
I. 9R+55 employees using private transport in B and D
II.7S+25 together?
III.9T+1 a) 2560
a) Only I b) 2740
b) only I and II c) 2140
c) Only I and III d) 2470
d) only II and III e) None of these
e) only III
8) The total number of staff in F is R% more than
6) Out of the total graduates in A, (Q+13)% are the number of staff in E and the total number of
officers and the rest are clerks. The average staff in G is (P+2)% more than the number of
salary of an officer is Rs.45000 and the average graduate staff in E. 25% of the total number of
salary of a clerk is Rs.29000. Average salary of employee of F and G together work in night shift.
all non-graduate staff is Rs.22000. Find the Find the possible value of the total number of
approximate average salary of all the staff of A? staff in F and G together work in night shift?
a) Rs.22884 I.50.25S
b) Rs.29854 II. 4.5P
c) Rs.29544 III.75.375(Q+4)
d) Rs.28844 a) only I
e) Rs.29484 b) only I and III
c) only II and III
7) The ratio of the number of staff using public d) only I and II
transport in factories B and D is 5:12 and the e) None of these

Directions (09 - 12): Study the following information carefully and answer the questions given below.

Click Here For Bundle PDF Course | support@guidely.in Page 4 of 12


Bank Po Mains PDF Course 2024
Quantitative Aptitude Day -12 (Eng)

The given bar graph shows the amount of investment by A, B, and C, in three different business [P, Q,
and R] and the difference between the amount of investment by A and B and B and C in each business is
also given.

9) In business P, after 6 months, 8 months and Rs.2000. The ratio of profit share of A, B and C
10 months A, B and C invest Rs.8000, Rs.6000 is 108:105:116. Find the value of n?
and Rs.2000 respectively. If after one year the a) 6
total profit of P is Rs. 11041 Find the share of b) 4
profit of B? c) 8
a) z-260 d) 2
b) 2x+360 e) 7
c) y+660
d) x+y-2560 11) All the person invests their money for 12
e) 2x+240 months and the total profit in P, Q and R is
10) In business Q, after 4 months A invests Rs.8100, Rs.9065 and Rs. 11000. Find which of
Rs.6000 more and after 2 months B invests the following is true?
Rs.5000 more and after n months C withdraws a) The sum of the share of A in all three is 7852.

Click Here For Bundle PDF Course | support@guidely.in Page 5 of 12


Bank Po Mains PDF Course 2024
Quantitative Aptitude Day -12 (Eng)

b) The sum of the share of B in all three is 9975. scored by C is 15:16. D scored 6.66% more
c) The difference of share of A and C in P and Q marks than C in math. Marks of C in math are
is 654. 25% more than B in chemistry. The ratio of
d) The difference of share of C in P and Q is 458. marks of A in math and physics is 16:17. The
e) None of these ratio of marks scored by B in physics and
chemistry is 7:6. Total marks scored by C in all
12) Total profit from Business P is Rs.13500. the subjects together is 230. The ratio of marks
With his share, A bought a watch and then sold it of C and D in physics is 5:6.
at 20% profit. Find the possible selling price of 13)If the marks of A in English is 20% more than
the watch? the marks of A in math. The ratio of marks of A
I.5.6X-7600 and B in English is 4:3. Marks of B in Biology is
II.4.5X-9900 25% more than his marks in English. Which of
III.X+Y the following can be the ratio of marks of the
a) only I and II biology of B and A?
b) only II and III I. 9:10. II. 9:11, III. 5:6
c) only I a) only II
d) all three b) only III
e) None of these c) only I
d) only I and II
Directions (13- 16): Study the following e) only II and III
information carefully and answer the questions
given below. 14) The ratio of marks of A and E in Math is 4:3.
There are four students: [A, B, C and D]. Each The average marks of B and E in chemistry is 72
student scores different marks in three different and the average marks of C and E in physics is
subjects: math, physics, and chemistry. The ratio 78. Find the total marks scored by E?
of marks of A and B in math is 16:17. The total a) 245
marks in each subject are 100. In chemistry, A b) 258
scored 20 marks less than his score in physics. c) 256
Marks of D in chemistry are 16.66% more than d) 225
marks of B in the same subject. The marks of B e) 268
in math are 13.33% more than the marks of C in
physics. Marks of C in Chemistry are 33.33% 15) If A got x% percentage marks in three
more than the same scored by B in chemistry. subjects and B got y% percentage marks in three
The ratio of marks in physics and chemistry

Click Here For Bundle PDF Course | support@guidely.in Page 6 of 12


Bank Po Mains PDF Course 2024
Quantitative Aptitude Day -12 (Eng)

subjects. Find the percentage of marks C and D Quantity: II If Mark up percentage of item B is
get? 60% above the cost price and the discount
a) y%, x% amount is Rs.80 then find the profit percentage?
b) (y+5)%, (x+3.34)% A. Quantity: I < Quantity: II
c) (x+2.5)%, (x-2.5)% B. Quantity: I ≥ Quantity: II
d) (x+3.66)%, (y-1.66)% C. Quantity: II ≥ Quantity: I
e) None of these D. Quantity: I > Quantity: II
E. Quantity I = Quantity II or relation can't be
16) established
Quantity: I Total marks scored by all the students
together in physics is what percent of the total 18) The ratio of the present age of A and B is
marks scored by all the students together in 2:3. The ratio of the age of A after 5 years and
maths? the present age of C is 5:7. B is 5 years younger
Quantity: II Total marks scored by all the than C.
students together in chemistry is what percent of Quantity: I If the Age of D is the average age of A
the total marks scored by all the students and B then find the average age of C and D?
together in maths? Quantity: II If the age of E is 2 years younger
A. Quantity: I < Quantity: II than B and the Age of F is 3 years younger than
B. Quantity: I ≥ Quantity: II C. Find the average age of E and F?
C. Quantity: II ≥ Quantity: I A. Quantity: I < Quantity: II
D. Quantity: I > Quantity: II B. Quantity: I ≥ Quantity: II
E. Quantity I = Quantity II or relation can't be C. Quantity: II ≥ Quantity: I
established D. Quantity: I > Quantity: II
E. Quantity I = Quantity II or relation can't be
17) The ratio of the cost price of items A and B is established
4:5. Item A and B is marked up by 60% and 70%
above the cost price. Item A and B sold at Rs.40 19) SERIES I and SERIES II follow the same
and Rs.60 discount. The profit percentage of logic. Find the value of D?
item A is 56.66%. Series I – 158, 290, 446, 628, 838, A
Quantity: I If Mark up percentage of item A is Series II – A, B, C, D, E
50% above the cost price and the discount a) 1478
amount is the same then find the profit b) 1548
percentage? c) 1684
d) 1984

Click Here For Bundle PDF Course | support@guidely.in Page 7 of 12


Bank Po Mains PDF Course 2024
Quantitative Aptitude Day -12 (Eng)

e) None of these a) 1940


b) 1480
20) Find the Sum of the missing number of the c) 1570
series? d) 1640
Series I –78,180,408,840, ? e) None of these
Series II- 145,155,166,179,196, ?
Click Here to Get the Detailed Video Solution for the above given Questions
Or Scan the QR Code to Get the Detailed Video Solutions

Answer Key with Explanation


1. Answer: C So, the total votes of party Z in the village
The total number of blocks in A is 4. election = 8000-2000-3200=2800
The total number of villages in A is 4*2=8
The total number of registered voters in A is 2. Answer: E
8*1050=8400 Total villages in B are = 5*2=10
Total cast vote is 8400*25/100 + 2900 The total population in B is = 10*1300=13000
+2900+100=8000 Total valid vote in village of B = 13000*90*90/

Total votes party X in block election = [100*100] =10530

8000*30/100=2400 Total votes got by party Y in all the elections

Total votes of party Y in block election = [8000- together


2400]/2=2800 = Total votes of party Z in block = (30% of (10530*3)
election. =9477

Total votes of party X in village election = 4400- Total votes got by party Z in all the elections

2400=2000 together – From the given data, we couldn’t find


Total votes of party Y in village election = 6000- votes got by Z in district elections.
2800=3200 Answer: Can’t be determined

Click Here For Bundle PDF Course | support@guidely.in Page 8 of 12


Bank Po Mains PDF Course 2024
Quantitative Aptitude Day -12 (Eng)

3. Answer: B From the 2nd pie chart, R%+Q%=100-16-22-


Total voters in district C is = 6*2*8000=9600 30=32
The total cast votes in C is 9600*3/4=7200 And ATQ, R-Q=8%
Total valid votes of village elections of all villages So, R= [32+8]/2=20% and Q=[32-8]/2=12%
together = 7200-400=6800 P=12*150/100=18%
Total valid votes of village elections of all villages S=12*2/1=24%
together = 7200-1600=5600 From 1st pie chart, T=100%-18%-12%-20%-
Total valid votes of village elections of all villages 24%=26%
together = 7200-1200=6000 The total number of graduate staff in E is =
Let party X vote in village, block and district 5600*30/100=1680
elections is 15a, 8a and 15a respectively. The total number of staff in E is =
Let party Y votes in village, block and district 1680+660=2340
elections are 7b, 5b and 5b respectively. So, the total number of staff in all company
Let party Z votes in village, block and district together is
election is c, 2c and c respectively. =[2340/26]*100=9000
So, 15a+7b+c=6800, 8a+5b+2c=5600 and 5. Answer: C
15a+5b+c=6000 Total female in C is = 1800*26/100=468
By solving the above equations, we get a=200, So, male is = 1800-468=1332
b=400, c=1000 So, male graduate is = 1332*75/100=999
Required sums = 5*400+1000=3000 So, the female graduate is =1232-999=233
So, female non- graduate is = 468-
233=235=9R+55=9T+1
4. Answer: A
Total registered voters in D is = 3*2*1400=8400 6. Answer: D
Total number of people in D is = Total officer is = 896*25/100=224
8400*100/80=10500 Total clerk is = 896-224=672
Total female voters is = So, total salary is =
[10500*2/5]*70/100=2940 224*45000+672*29000+544*22000
Total male voters is = 8400-2940=5460 =41536000
Required difference = 5460-2940 =2520 So, average salary =
[41536000/1440]=28844.44=28844
Directions (5 - 8):
7. Answer: B

Click Here For Bundle PDF Course | support@guidely.in Page 9 of 12


Bank Po Mains PDF Course 2024
Quantitative Aptitude Day -12 (Eng)

Let number of staff use public transport for B and Or, Z=5000
D is 5x and 12x. So, X=[5000+3000]/4=8000/4=2000
The number of staff use private transport of B Now, Y=3*2000/2=3000
and D is 29y and 48y.
So, 5x+29y=1260(i) and 12x+48y=2160,
x+4y=180(ii)
By solving (i) and (ii) we get x =20 and y=40.
9. Answer: C
So, the total number of staff from B and D using
Investment ratio of A, B and C,
public transport is
[12000*6+20000*6]:[18000*8+24000*4]:[24000*
= 100+240=340
10+26000*2]
The total number of staff from B and D use
=192:240:292=48:60:73
private transport is
So, share of profit of B is =
= 1160 + 1920 = 3080
11041*60/181=3660=y+660
So, required difference 3080 - 340=2740
8. Answer: B
10. Answer: D
The total number of staff in F is =
So, their investment ratio
2340*120/100=2808
[14000*4+20000*8] :
Total number of staffs in G is =
[15000*6+20000*6]:[20000*(6+n)+18000*(12-
1680*120/100=2016
(6+n)] = 108:105:116
So, the total number of staff in F and G is =
108 : 105 : n+114 = 108 : 105 : 116
2808+2016=4824
n+114 = 116
The total number of employees working night
n=2
shifts is = 4824*25/100
=1206=50.25S=75.375(Q+4)
11. Answer: B
Invest ratio of A, B and C in P = 12:18:24=2:3:4
Directions (09 - 12):
Invest ratio of A, B and C in Q =
From the graphs, we can see in business P, [9X-
14:15:20=14:15:20
6X]=2Y
Invest ratio of A, B and C in R =
Or, 3X=2Y
10:18:16=10:18:16
Again, in business Q, 3Z-[8X-2000] =1000,
So, the sum of share of B is
In business R, 3Z+3000-2Z=4X
=8100*3/9+9065*15/49+11000*18/44=9975
Or, Z+3000=4X
So, B is true.
Now, 3Z-[2*(Z+3000)-2000] =1000

Click Here For Bundle PDF Course | support@guidely.in Page 10 of 12


Bank Po Mains PDF Course 2024
Quantitative Aptitude Day -12 (Eng)

12. Answer: C 90*6/5=108 - not possible


Invest ratio of A, B and C in P = 12:18:24=2:3:4
So, A’s share is = 13500*2/9=3000 14. Answer: D
So, the selling price of watch is Total marks of E is = 80*3/4+72*2-60+78*2-
3000*120/100=3600=5.6X-7600 75=225

Directions (13- 16): 15. Answer: B


Let marks of C in physics and chemistry be 15x A’s percentage = [230/300]*100=76.66%
and 16x, respectively. B’s percentage = [215/300]*100=71.66%
So, the mark of B in chemistry is 16x*100/133.33 C’s percentage =
= 12x. [230/300]*100=76.66%=[y+5]%
So, the mark of C in math is 12x*125/100 = 15x. D’s percentage =
So, 15x+16x+15x=230 [240/300]*100=80%=[x+3.34]%
Or, x=230/46=5
Marks of C in physics and chemistry are 15* 5 = 16. Answer: D
75 and 16* 5 = 80. Quantity I
So, marks of B in chemistry are 12 * 5 = 60. Required percentage =
So, the mark of C in math is 75. [(85+70+75+90)/(80+85+75+80)]*100=100%
Marks of B in math are = 75*113.33/100 = 85. Quantity II
Marks of A in math are = 85*16/17 = 80. Required percentage =
Marks of A in physics are = 80*17/16 = 85. [(65+60+80+70)/(80+85+75+80)]*100
Marks of A in chemistry are = 85-20 = 65. =85.9375%
Marks of B in Physics are 60*7/6 = 70. Quantity I>Quantity II
Marks of D in math are = 75*106.66/100 = 80.
Marks of D in chemistry are 60*116.66/100 = 70. 17. Answer: A
Marks of D in Physics are = 75*6/5 = 90. Let the cost price of A and B is 400x and 500x
13. Answer: C So, 400x*160/100 -40 =400x*156.66/100
Marks of A in English is = 80*120/100=96 Or, 13.36x=40
Marks of B in English is = 96*3/4=72 Or, x=40/13.36=3
Marks of B in Biology is = 72*125/100=90 So, the cost price of item A is 400*3=1200 cost
So, from I, marks of A in biology are = price of item B is 500*3=1500
90*10/9=100  possible Quantity I
90*11/9=110  not possible

Click Here For Bundle PDF Course | support@guidely.in Page 11 of 12


Bank Po Mains PDF Course 2024
Quantitative Aptitude Day -12 (Eng)

So, selling price of A is = 1200*150/100 - Quantity II


40=1760 Age of E is = 30-2=28 years
So, profit percentage = [1760- Age of F is = 35-3=32
1200]*100/1200=46.66% Average age of E and F is = [28+32]/2=30 years
Quantity II Quantity I = Quantity II
Selling price of item B is = 1500*160/100 -
80=2320 19. Answer: B
So, profit percentage = [2320- Series I – 158, 290, 446, 628, 838, A
1500]*100/1500=54.66% 158+121+11=290, 290+144+12=446,
Quantity I < Quantity II 446+169+13=628, 628+196+14=838,
838+225+15=1078=A
18. Answer: E Series II – A=1078, B=1078+121+11=1210,
Let the ages of A and B be 2x and 3x. C=1210+144+12=1366, D=1366+169+13=1548
Age of C is 3x+5
So, 2x+5/3x+5=5/7 20. Answer: A
Or, 14x+35=15x+25 Series I –78,180,408,840, ?
Or, x=10 78*2+24=180, 180*2+48=408, 408*2+24=840,
So, the age of A and B is 20 and 30 years 840*2+48=1728
respectively. Series II- 145,155,166,179,196, ?
Age of C is 30+5=35 years. 145+1+4+5=155, 155+1+5+5=166,
Quantity I 166+1+6+6=179, 179+1+7+9=196,
Age of D is = [20+30]/2=25 years 196+1+9+6=212
Average age of C and D is = [25+35]/2=30 years So, the required sum = 1728+212=1940

Click Here For Bundle PDF Course | support@guidely.in Page 12 of 12


Bank Po Mains PDF Course 2024
English Day - 12

English Language
Directions (1-4): In the questions given below, B. As the sun dipped below the horizon, the city's
four statements are given that are labelled as skyline became a mesmerizing tapestry of
(A),(B),(C) and (D), with one statement being twinkling lights.
grammatically and contextually incorrect. You C. The old oak tree in the park provided shade
are required to choose that sentence which is and solace to those seeking refuge from the
erroneous. If all sentences are correct, then scorching summer sun.
choose ‘option E’ as your answer. D. With a backpack full of camping gear and a
1. sense of adventure, the journey embarked into
A. A sudden downpour transformed the dusty the heart of the wilderness.
streets into glistening ribbons of water, bringing A. A
relief to the parched earth and rejuvenating the B. B
surrounding landscape. C. C
B. After a long day at work, he sank into his D. D
favourite armchair, sips on a hot cup of tea, and E. All are correct
savours the quiet moments of solitude.
C. In the heart of the bustling city, a small park 3.
provided a peaceful oasis where people could A. The gentle chirping of the insects on the
escape the urban chaos and unwind amidst summer night provided a soothing backdrop to
nature. their campfire stories.
D. The ancient ruins of the Mayan civilization B. The vibrant flowers in the garden swayed in
stood as a testament to the ingenuity and the breeze, creating a symphony of colour and
architectural prowess of a bygone era. motion.
A. A C. With determination and a clear plan in mind,
B. B she embarked a journey to achieve her long-held
C. C dreams.
D. D D. As the first snowflake touched the ground, a
E. All are correct sense of wonder and anticipation filled the
children's hearts.
2. A. A
A. The aroma of freshly baked cookies wafted B. B
through the house, tempting everyone to take a C. C
sweet break. D. D
E. All are correct

Click Here For Bundle PDF Course | support@guidely.in Page 1 of 12


Bank Po Mains PDF Course 2024
English Day - 12

outcomes was the decision not to exercise the


4. Council’s power to tax extra neutral alcohol
A. The scent of blooming flowers and the (ENA) used for alcoholic liquor. With alcohol for
chirping of birds signalled the arrival of spring in human consumption still outside the GST net, the
the quiet, suburban neighbourhood. indirect tax levy on ENA or high strength potable
B. With a canvas and a palette of vibrant colours, alcohol — a key ingredient — could not be set off
she set out to create a masterpiece that would against State levies on the final product. Industry
expressed her deepest emotions. had been seeking for clarity on this vexed issue
C. The sun dipped below the horizon, casting for years, with courts taking varying positions.
long shadows across the tranquil meadow. It is heartening that the Council, that met just
D. After a long day of hiking, they gathered twice in 2022, has met four times this year, and
around the campfire, shared stories, and roasted thrice in just four months, even if a few agenda
marshmallows under the starry night sky. items pertained to fixing anomalies in recent
A. A decisions. With the age norms for the president
B. B and members of the long-awaited GST Appellate
C. C Tribunals now harmonised with other tribunals —
D. D a clearly avoidable oversight — one hopes they
E. All are correct will become operational soon. For consumers
and producers, however, the biggest matter of
Directions (5-11): Read the given passage concern should be the Council’s resolve to meet
carefully and answer the following questions. at a future date exclusively to discuss what
Some words are highlighted to help you locate Finance Minister Nirmala Sitharaman termed
them while answering the questions. ‘perspective planning’ on the GST Compensation
The Goods and Services Tax (GST) Council last Cess and what kind of surcharge it could be
Saturday lifted the haze on about a dozen tax replaced with. Originally packaged as a time-
treatment ambiguities, some of which have bound levy on top of a ‘Good and Simple Tax’ to
lingered since the indirect tax regime’s launch in compensate States for revenue losses for the
July 2017, such as the tax on corporate and first five years of GST, the COVID-19 pandemic’s
personal guarantees for bank loans. It slashed hit on tax collections had triggered an extension
the GST on molasses from 28% to 5%, with a of the Cess levied on so-called demerit goods
view to lower cattle feed costs and ease up cash such as aerated drinks, tobacco products and
flows for sugar mills so they may pay farmers’ automobiles, till March 2026. Discouraging some
dues faster. Rate tweaks and spring-cleaning sin goods may be desirable. However, ringing in
clarifications apart, one of the significant a new cess must not be done in isolation, but as

Click Here For Bundle PDF Course | support@guidely.in Page 2 of 12


Bank Po Mains PDF Course 2024
English Day - 12

a part of the broader rationalisation of GST’s 7. According to the passage, what is one of the
complex multiple-rate structure. That key issues with the current GST regime that
rationalisation exercise, initiated two years ago, needs attention, leading to discussions in the
unfortunately remains off the table despite robust passage?
revenue inflows in recent times. Frequent tweaks A. The need to promote the consumption of
of irritants aside, the GST regime needs a holistic demerit goods, despite their negative impact on
reform plan, including a road map to bring in health.
excluded items such as electricity, petroleum and B. The extension of Cess on all goods,
alcohol. particularly in light of the ongoing COVID-19
5. What is one of the noteworthy results of the pandemic.
most recent GST Council meeting, as indicated C. The complexity of the multiple-rate structure,
in the passage? which requires simplification and rationalization.
A. The reduction of GST on molasses from 5% to D. The absence of clarity on the taxation of ENA,
28%. a critical ingredient in alcoholic liquor.
B. The Council's choice not to impose taxes on E. The increase in tax collections during the
extra neutral alcohol (ENA). pandemic, necessitating a reevaluation of
C. The decision to tax corporate guarantees for revenue sources.
bank loans.
D. The clarification of tax treatment ambiguities 8. Which action, as per the author, is considered
related to GST. positive or encouraging?
E. The inclusion of alcohol for human A. The Council has decided to discontinue
consumption in the GST net. meetings due to various anomalies.
B. The Council has harmonized age norms for
6. Why was the Cess on demerit goods extended other tribunals.
until March 2026, as mentioned in the passage? C. The implementation new policies without
A. To discourage the consumption of sin goods. discussion.
B. To provide additional revenue for the D. The Council has decided to increase the age
government. limits for GST Appellate Tribunal members.
C. To simplify the GST's multiple-rate structure. E. The increased frequency of meetings by the
D. To compensate states for revenue losses due GST Council in the current year.
to the GST.
E. To promote the consumption of aerated drinks 9. What is the tone of the given passage?
and tobacco products. A. Critical
B. Derisive

Click Here For Bundle PDF Course | support@guidely.in Page 3 of 12


Bank Po Mains PDF Course 2024
English Day - 12

C. Obsequious I. Despite, but also


D. Sarcastic II. In spite of, and also
E. Hypothetical III. Meanwhile, but also
IV. As a result of, on the other hand
10. What is the antonym of the word A. Only II
‘ANOMALIES’ as used in the passage? B. Only III
A. Irregularities C. Only I
B. Conformities D. Both I and IV
C. Discrepancies E. None of these
D. Inconsistencies
E. Aberrations 13.
A. John thought he had made a mistake
11. What is the meaning of the phrase ‘off the B. his decision turned out to be the right one in
table’ as used in the passage? the end
A. No longer relevant C. it pleasantly surprised everyone.
B. Still under consideration I. since, but
C. Highly recommended II. hence, and on the other hand
D. Currently in progress III. conversely, but
E. Open for discussion IV. but, and as a matter of fact
Directions (12-15): In the following questions, A. Only III
three phrases/sentences are given (A, B and C) B. Only IV
followed by four connectors (I, II, II and IV). C. Only I
Choose the option that represents the correct D. Both II and IV
combination of connectors that can be used to E. None of these
join the given sentences without changing their
original meaning. If none of the connectors is 14.
correct, choose option E as your answer. A. the rain was pouring down relentlessly from
12. the overcast sky,
A. the challenging economic conditions, B. I decided to go for a run in the park,
B. the company not only managed to maintain its determined to stay committed to my daily
market share exercise routine
C. expanded its global operations, launching new C. I soon regretted it when I got completely
products and ultimately securing its position as a soaked.
market leader A. Although, but

Click Here For Bundle PDF Course | support@guidely.in Page 4 of 12


Bank Po Mains PDF Course 2024
English Day - 12

B. Nevertheless, but 16. The government's decision to revise tax


C. Due to, in contrast policies has ___________ a substantial debate
D. Besides, subsequently among economists, with some arguing that the
A. Only III changes could ___________ economic growth.
B. Only IV A. stimulated
C. Both I and IV B. overhauled
D. Both I and II C. prompted
E. None of these D. hinder
E. stimulate
15. F. impair
A. Eager to enjoy the festivities and spend time A. AD
with friends, I had a strong desire to attend the B. BF
party. C. CE
B. My responsibilities were piling up, and my D. CD
workload had reached a point where it was E. None of these
simply impossible to set them aside.
C. I had no choice but to decline the invitation 17. The company's commitment to sustainability
with a heavy heart. goes beyond its environmental efforts; it also
I. Nonetheless, so extends to its _______ practices and its _______
II. Meanwhile, provided that in local community initiatives.
III. Furthermore, yet A. ethical
IV. Conversely, thus B. profitable
A. Only I C. innovative
B. Only III D. engagement
C. Both I and IV E. alienation
D. Both II and III F. participation
E. None of these A. AE
Directions (16-20): The given sentences carry B. AD
two blanks each which can be filled with the C. BF
words from the choices given below them. Read D. CE
them carefully and choose the most appropriate E. None of these
combination of words that makes the sentence
complete and meaningful. 18. The company's commitment to research and
development is commendable; it continuously

Click Here For Bundle PDF Course | support@guidely.in Page 5 of 12


Bank Po Mains PDF Course 2024
English Day - 12

invests in cutting-edge technology to ________ its F. forge


products and ________ its market position. A. AD
A. streamline B. BE
B. enhance C. CF
C. jeopardize D. Both BE and CF
D. bolster E. None of these
E. stagnate
F. sustain
A. AE 20. The chef's dedication to using fresh, locally-
B. CD sourced ingredients is what ________ the quality
C. BD of the restaurant's dishes and ________ its
D. CF reputation for excellence.
E. None of these A. elevates
B. sustains
19. The author's ability to craft compelling C. stabilizes
characters and weave intricate plotlines is what D. amplifies
________ readers to immerse themselves in the E. erodes
story and ________ a deep emotional connection. F. tarnishes
A. entices A. AD
B. fascinates B. BE
C. captivates C. CF
D. repels D. Both AD and CF
E. sever E. None of these

Click Here For Bundle PDF Course | support@guidely.in Page 6 of 12


Bank Po Mains PDF Course 2024
English Day - 12

Click Here to Get the Detailed Video Solution for the above given Questions
Or Scan the QR Code to Get the Detailed Video Solutions

Answer Key with Explanation


1. Answer: B The introductory phrase "With a backpack full of
The error is in sentence B in the wrong usage of camping gear and a sense of adventure" is
the verb forms. meant to describe the person or group
Parallelism or parallel construction is a embarking on the journey, but it's not properly
grammatical and stylistic rule that requires the connected to the subject of the sentence.
elements in a series or list to have the same The correct sentence is:
grammatical form. With a backpack full of camping gear and a
The parallelism rule for this sentence would sense of adventure, they embarked on a journey
require consistency in verb tense for the items in into the heart of the wilderness.
the series. In the corrected sentence, all verbs In this corrected version, the introductory phrase
should be in the past tense to maintain parallel clearly modifies the subject "they," and the
structure. modifier is properly connected to the subject,
Thus, the correct sentence is: resolving the dangling modifier problem. Thank
"After a long day at work, he sank into his you for pointing that out.
favourite armchair, sipped on a hot cup of tea, Therefore, option D is the correct answer.
and savoured the quiet moments of solitude."
Therefore, option B is the correct answer. 3. Answer: C
The error lies in sentence C as did not use the
2. Answer: D preposition ‘on’ after the verb ‘embarked’.
The error in the sentence D is a misplaced The word "embarked" is often followed by the
modifier, leading to a dangling construction. preposition "on" when it is used to mean "to start
or begin a journey or a significant endeavour."

Click Here For Bundle PDF Course | support@guidely.in Page 7 of 12


Bank Po Mains PDF Course 2024
English Day - 12

When you say someone "embarked on" neutral alcohol (ENA) when it is used in the
something, it indicates that they have started or production of alcoholic liquor.
undertaken a particular course of action, project, Therefore, option B is the correct answer.
or journey.
For example, She embarked on a new career in 6. Answer: D
medicine. In the passage, it is mentioned that the Cess on
Thus, the correct sentence is: demerit goods (such as aerated drinks, tobacco
With determination and a clear plan in mind, she products, and automobiles) was originally
embarked on a journey to achieve her long-held introduced as a time-bound levy to compensate
dreams. states for revenue losses during the first five
Therefore, option C is the correct answer. years of the Goods and Services Tax (GST)
regime. This compensation was necessary
4. Answer: B because the GST was expected to lead to
The error in sentence B is in the wrong usage of revenue losses for states initially.
modal-verb agreement. However, due to the impact of the COVID-19
Modal verbs like "would," "could," "should," pandemic on tax collections, the Cess on
"might," and "can" are followed by the base form demerit goods was extended until March 2026.
of the main verb (the infinitive form without "to"). This extension was primarily aimed at
Thus, the correct sentence is: compensating states for the revenue losses they
With a canvas and a palette of vibrant colours, experienced.
she set out to create a masterpiece that would Therefore, the correct answer is option D.
express her deepest emotions.
Therefore, option B is the correct answer. 7. Answer: C
In the passage, the author discusses the
5. Answer: B complexities of the current Goods and Services
In the given passage, it is mentioned that “Rate Tax (GST) regime, emphasizing that there is a
tweaks and spring-cleaning clarifications apart, need for reform and simplification. This includes
one of the significant outcomes was the decision addressing the issue of a multiple-rate structure,
not to exercise the Council’s power to tax extra which has been a subject of concern. The
neutral alcohol (ENA) used for alcoholic liquor.” passage suggests that the complex multiple-rate
Thus we can understand that the passage structure needs attention, and it is one of the key
mentions a significant decision made by the GST issues with the GST system that is highlighted in
Council, which is not to impose taxes on extra the passage.

Click Here For Bundle PDF Course | support@guidely.in Page 8 of 12


Bank Po Mains PDF Course 2024
English Day - 12

Therefore, option C is the correct answer.  Derisive: Mocking or scornful in a


disrespectful manner.
8. Answer: E  Obsequious: Excessively obedient or
It is mentioned in the passage that “The author servile, often to gain favor.
considers this action to be positive or  Sarcastic: Using irony or mock praise to
encouraging because it signifies that the GST convey contempt or humor.
Council is actively addressing issues and  Hypothetical: Involving a supposition or
challenges.” The phrase ‘it is heartening’ means assumption, not necessarily based on
‘The phrase "it is heartening" means providing fact.
comfort, encouragement or cheer to someone. It
is often used to express gratitude or relief. 10. Answer: B
The author considers this action to be positive or "Anomalies" refers to irregularities or deviations
encouraging because it signifies that the GST from what is considered normal or expected.
Council is actively addressing issues and Let’s learn the meanings of the given words:
challenges.  Irregularities: Departures from the norm or
Therefore, option E is the correct answer. standard.
 Conformities: Acts of complying with rules
9. Answer: A or standards.
The passage contains a critical assessment of  Discrepancies: Differences or
various aspects of the Goods and Services Tax inconsistencies between two or more
(GST) regime, including its complexity, the need things.
for rationalization, and the extension of the Cess  Inconsistencies: Lack of harmony or
on demerit goods. The author points out areas uniformity; contradictions.
where there is room for improvement and reform  Aberrations: Departures from what is
in the GST system. considered normal or typical.
Overall, the tone combines the presentation of Thus, we can see that ‘conformities’ is the
information with a critical assessment of the antonym of the given word.
current GST system and a cautious sense of Therefore, option B is the correct answer.
optimism for future improvements.
The meanings of the given words: 11. Answer: A
 Critical: Expressing disapproval or The phrase "off the table" is an idiomatic
pointing out flaws and shortcomings. expression that means an option or possibility is
no longer being considered or is not currently

Click Here For Bundle PDF Course | support@guidely.in Page 9 of 12


Bank Po Mains PDF Course 2024
English Day - 12

available for discussion or consideration. It belief that he made a mistake, his decision
signifies that a particular idea or proposal has turned out to be the right one, and this was
been excluded or dismissed from consideration. pleasantly surprising. The "as a matter of fact"
Therefore, option A is the correct answer. part adds an emphasis on the surprising aspect.
Thus, the sentence formed is:
12. Answer: C John thought he had made a mistake but his
The phrase "Despite" effectively connects the decision turned out to be the right one in the end
introductory phrase to the subsequent actions and as a matter of fact, it pleasantly surprised
and outcomes, showing a contrast between the everyone.
challenging conditions and the company's Therefore, option B is the correct answer.
achievements. "But also" is an appropriate
connector that indicates that in addition to 14. Answer: D
maintaining market share, the company ‘Nevertheless, although, but’ all three are
expanded its operations and secured its position contrasting connectors and are used to connect
as a market leader. two opposite ideas or sentences. Here, ‘although
The other options are not suitable connectors for and nevertheless’ come at the beginning of
the given sentence or do not maintain the sentence A, to convey the idea that despite the
intended meaning and coherence of the adverse weather conditions, the person decided
passage. to go for a run.
Thus, the sentence formed is: ‘But’ is used to connect sentences B and C to
Despite the challenging economic conditions, the highlight the contrast between the person’s
company not only managed to maintain its determination to exercise and the regret that
market share but also expanded its global followed as a consequence of their decision.
operations, launching new products and Thus, the sentence formed is:
ultimately securing its position as a market Although/ nevertheless the rain was pouring
leader. down relentlessly from the overcast sky, I
Therefore, option C is the correct answer. decided to go for a run in the park, determined to
stay committed to my daily exercise routine but I
13. Answer: B soon regretted it when I got completely soaked.
"But" introduces a contrast, and "and as a matter Therefore, option D is the correct answer.
of fact" is used to provide additional information
or clarification. In this context, this combination 15. Answer: A
works well. It shows that despite John's initial

Click Here For Bundle PDF Course | support@guidely.in Page 10 of 12


Bank Po Mains PDF Course 2024
English Day - 12

"Nonetheless" is used to indicate a contrast or changes in tax policies could have a negative
contradiction between statements I and II. In this effect on economic growth. "Hinder" is the
context, it conveys the idea that despite the correct word for this context, as it means to
strong desire to attend the party, there were create obstacles or restrictions that slow down or
obstacles (piling responsibilities and an impede progress.
overwhelming workload) that made it Therefore, option D is the correct answer.
impossible.
"so" is used to show the cause-and-effect 17. Answer: B
relationship between the increasing The first blank should be filled with a word that
responsibilities and the necessity to decline the describes the company's practices as being in
party invitation. It conveys that because the line with moral principles or ethical standards.
workload became unmanageable, the person "Ethical" is the appropriate word for this context.
had no other option but to decline. The second blank should be filled with a word
Thus, the sentence formed is: that indicates the company's active involvement
Eager to enjoy the festivities and spend time with or participation in local community initiatives.
friends, I had a strong desire to attend the party. "Engagement" is the correct word for this
Nonetheless, my responsibilities were piling up, context.
and my workload had reached a point where it Therefore, option B is the correct answer.
was simply impossible to set them aside. So, I
had no choice but to decline the invitation with a 18. Answer: C
heavy heart The first blank should be filled with a word that
Therefore, option A is the correct answer. indicates the company's efforts to improve its
products. "Enhance" is the correct word in this
16. Answer: D context as it means to improve, elevate, or make
In the given sentence, the first blank should be better.
filled with a word that describes the The second blank should be filled with a word
government's decision as a catalyst for or the that suggests strengthening or fortifying the
cause of the substantial debate among company's market position. "Bolster" is the right
economists. "Prompted" is the appropriate word word for this context as it means to support or
here, as it means to cause or initiate a reinforce.
discussion or action. Therefore, option C is the correct answer.
In the second blank, you need a word that
indicates that some economists believe that the

Click Here For Bundle PDF Course | support@guidely.in Page 11 of 12


Bank Po Mains PDF Course 2024
English Day - 12

19. Answer: C The word ‘elevates’ in this context, means ‘to


The word ‘captivates’ means to attract and hold raise or lift up, and it is used to emphasize how
the interest and attention of someone. In the the chef's dedication significantly improves or
context of the sentence, it's used to describe raises the quality of the restaurant's dishes’. It
how the author's ability to craft compelling conveys the idea that the use of fresh, locally-
characters and intricate plotlines attracts and sourced ingredients takes the quality to a higher
holds the interest of readers. level.
The word ‘forge’ in this context, means to create The word "amplifies" means to increase the
or develop a deep emotional connection. It's strength, intensity, or significance of something.
used to convey that the author's storytelling skills In this sentence, it is used to emphasize how the
lead readers to form or create a strong and deep chef's dedication enhances or intensifies the
emotional connection with the story or its restaurant's reputation for excellence.
characters. It conveys the idea that the restaurant's excellent
Therefore, option C is the correct answer. reputation becomes even more prominent or
significant due to the chef's commitment to high-
20. Answer: A quality ingredients.
Therefore, option A is the correct answer.

Click Here For Bundle PDF Course | support@guidely.in Page 12 of 12


Bank Po Mains PDF Course 2024
Reasoning Day -13 (Eng)

Reasoning Aptitude
Directions (1-5): A string of letters are given as 2) How many letters are there in the English
input. Some conditions are given in four different alphabetical series between the sixth letters from
steps. Study the following information carefully both ends in step III?
and answer the given questions. a) Eight
Input: D G A F L I W P E Y S U H C Z O M J X B b) Fifteen
RKNTVQ c) Twelve
Step I: The first half of the letters is written in d) Three
reverse order after the second half of the letters. e) None of these
Step II: All the letters are changed to their
corresponding reverse letter as per the 3) How many consonants from the second half of
alphabetical series. the English alphabetical series are in step IV,
Step III: If the letters which come in the second which are immediately followed by a vowel?
half of the alphabetical series are immediately a) Four
followed by a vowel, then the letters are changed b) Two
to the second previous letter as per the c) Three
alphabetical series, else if the letters which come d) One
in the first half of the alphabetical series are e) None
immediately preceded by a vowel, then the
letters are changed to the third succeeding letter 4) Which of the following element is eight from
as per the alphabetical series. the right end in step IV?
Step IV: All the letters that appeared more than a) B
once are dropped from the series and the b) F
remaining letters are written in reverse order. c) P
Step IV is the final output of the given input. d) E
1) Which of the following letter is fifth to the left of e) None of these
the second vowel from the right end in step II?
a) I 5) _____ is fourth to the ____ of the letter which is
b) M _____ from the left end of step IV.
c) B a) Z, left, sixth
d) K b) F, right, Second
e) None of these c)G, Right, Seventh
d) Both a and c
e) Both a and b

Click Here For Bundle PDF Course | support@guidely.in Page 1 of 10


Bank Po Mains PDF Course 2024
Reasoning Day -13 (Eng)

Directions (6-10): Study the following information a) Only III


carefully and answer the given questions. b) All I, II, and III
A school conducts eight matches of four different c) Both II and III
sports on the sports day. The matches were held d) Both I and III
between 8 AM to 6 PM. Two matches for cricket, e) None of these
two matches for volleyball, three matches for
football, and one match for badminton. Each 8) Which of the following match was held
match was held for one hour. There are two immediately before Tea break?
breaks between the matches viz.- 1 PM – 2 PM a) Football
lunch break and tea break at 4 PM-5 PM. b) Cricket
Note: No two consecutive matches were held for c) Badminton
the same sport. d) None
One of the Cricket matches was held after the e) Volleyball
lunch break. Two Football matches were held at
a gap of 120 minutes. None of the football 9) Football is not played at which of the following
matches were held immediately after and time?
immediately before lunch break. Badminton a) 5 PM-6 PM
match was held immediately after one of the b) 11 AM-12 AM
cricket matches but not held after the lunch c) 2 PM-3 PM
break. Only one match was held between both d) 8 AM-9 AM
Volleyball matches. e) None of these
6) Badminton match was held at which of the
following time? 10) How many matches were held before the first
a) 12 PM-1 PM cricket match?
b) 9 AM-10 AM a) Three
c) 3 PM-4 PM b) One
d) 10 AM-11 AM c) Two
e) None of these d)Four
e) None
7) Which of the following match was held
between Lunch and Tea break? Directions (11-15): Study the following
I. Cricket information carefully and answer the questions
II. Football given below.
III. Volleyball

Click Here For Bundle PDF Course | support@guidely.in Page 2 of 10


Bank Po Mains PDF Course 2024
Reasoning Day -13 (Eng)

Certain number of persons from the same family b) Fifteen


of three generations are sitting in a row facing c) Twelve
north. Information about only a few persons are d) Seventeen
known. Not more than three known persons are e) Ten
sitting together. Some of the person’s relations
may or may not be known. 13) How K is related to the one who sits
B, who is the only daughter-in-law of K, sits five immediate right of Y?
places away from R. R sits second to the right of a) Mother-in-law
K. As many persons sit between B and K as b) Grandmother
between K and G, who is the only daughter of K. c) Aunt
G sits third from one of the extreme ends. L, who d) Nephew
is the only son of G, sits third to the right of the e) None of these
one who sits immediate left of G.L sits fourth to
the left of M, who is married to B. The one who 14) Four of the following five are alike in a certain
sits third to the right of M is the father of G. Only way based on the given arrangementand thus
one person sits between M and W, who is the form a group. Which one of the followingdoes not
father of L. Only six persons sit between W and belong to the group?
Q, who is the only daughter of M. Q and Y sit a) PW
together. Only four persons sit between Y and Z, b) UM
who sits at the extreme end of the row. As many c) QB
persons sit between Z and K as between Q and d) WK
U. U sits four places away from P, who is the e) MR
sibling of W’s spouse’s niece.
11) ______ is the _______ of the one who sits 15) If Y is the sister-in-law of R, who has no
fourth to the left of _____. siblings, then how Y is related to the one who
a)W, Uncle, R sits third to the right of U?
b) P, Son, Q a) Maternal Aunt
c) G, Mother, M b) Brother
d) Both A and C c) Sister
e) Both A and B d) Daughter
e) None of these
12) How many persons are sitting to the right of
B’s sister-in-law?
a) Eight

Click Here For Bundle PDF Course | support@guidely.in Page 3 of 10


Bank Po Mains PDF Course 2024
Reasoning Day -13 (Eng)

Directions (16-20):Read the given passage 17) Which of the following can be a probable
carefully and answer the questions based on the ‘Course Of Action’ to pacify the anxious people?
same respectively a) All the protestors should be jailed so that more
In scenes reminiscent of the Arab Spring, (A) people don’t join the protests
citizens stormed the residences of Sri Lanka’s b) The protestors should be promised with free
president and prime minister in Colombo, (B) food and basic facilities for all of them
forcing both leaders to announce that they would c) Present heads of the country should resign
quit their respective offices. President Gotabaya with immediate effect
Rajapaksa, the main target of the protests that d) A new government should take charge and try
erupted in April over food and fuel shortages and to restore the trust of the people
have continued since, promised to resign by e) The protestors should be given employment
Wednesday. The Speaker of Sri Lanka’s which would ensure consistent money flow
parliament, Yapa Abeywardena, is expected to
take over as acting president and prime minister, 18) Which option can be marked as an
and facilitate, possibly, the setting up of a ‘Inference’ as per the contents of the given
national unity government. Gotabaya’s paragraph?
resignation could bring down the temperature a) The people of Sri Lanka have lost trust in their
and help the authorities to persuade the leadership
protestors to return home. Though the marches b) Gotabaya’s presence in the government has
and sit-ins in Galle Face and elsewhere have sparked ire
been largely peaceful, the recent violent events c) People of the island country have become
suggest that the people, battling acute shortage anxious
of essentials, including milk, baby food, petrol, d) Both (a) and (b)
are on the edge. Only a new leadership can win e) None of the given options
back public trust in government and steer the
island nation’s economy out of choppy waters. 19) Which of the given statements can most
16) Which of the given options provides the likely be taken as an ‘assumption’?
correct relationship between sentences A and B? a) The speaker takes over the roles of the
a) A is the cause and B is its effect estranged leaders till the government is formed
b) A is the effect and B is one of the causes b) An island country can only be ruled by a unity
c) Both A and B are causes government
d) Both A and B are effects of independent c) Violent protests were the only way left before
causes the people to show their agony
e) A is the effect and B is its cause

Click Here For Bundle PDF Course | support@guidely.in Page 4 of 10


Bank Po Mains PDF Course 2024
Reasoning Day -13 (Eng)

d) The last resort to save the economy was to b) A new political leadership is needed to steer
oust their leaders out of power Sri Lanka out of present impasse
e) All statements can be assumed c) The violent protests are a result of the
negligence of the government
20) Which of the following can be ‘concluded’ d) The food scarcity is a proof that the country
from the given paragraph? was being mismanaged
a) It is only the citizens who can bring the country e) Graft had taken the centre stage and the
out of the severe trouble economy was in doldrums while the government
enjoyed
Click Here to Get the Detailed Video Solution for the above given Questions
Or Scan the QR Code to Get the Detailed Video Solutions

Answer Key with Explanation


Direction (1-5): For Step II: All the letters are changed to their
1) Answer: C corresponding reverse letter as per the
2) Answer: C alphabetical series.
3) Answer: C Step II: X A L N Q C Y I P M G E J S FH B V K D
4) Answer: D ROUZTW
5) Answer: D For Step III: If the letters which come in the
We have: second half of the alphabetical series are
Input: D G A F L I W P E Y S U H C Z O M J X B immediately followed by a vowel, then the letters
RKNTVQ are changed to the second previous letter as per
For Step I: The first half of the letters are written the alphabetical series, else if the letters which
in reverse order after the second half of the come in the first half of alphabetical series are
letters. immediately preceded by a vowel, then the
Step I: C Z O M J X B R K N T V Q H U S Y E P letters are changed to the third succeeding letter
WILFAGD as per the alphabetical series.

Click Here For Bundle PDF Course | support@guidely.in Page 5 of 10


Bank Po Mains PDF Course 2024
Reasoning Day -13 (Eng)

Step III: V A O N Q C W I P M G E M S F HB V  Two Football matches were held at a gap


KDPMUZTW of 120 minutes.
For Step IV: All the letters that appeared more  None of the football matches were held
than once are removed and the remaining letters immediately after and immediately before
are written in reverse order. lunch break.
Step IV: T Z U D K B H F S E G I C Q N O A That means, in case (1) Cricket match
was held immediately after the lunch
Direction (6-10): break, in case (2) Cricket match was held
6) Answer: D two matches after the lunch break, in
7) Answer: D case (3) Cricket match was held four
8) Answer: E matches after the lunch break.
9) Answer: C Based on the above given information we have:
10) Answer: B

Again, we have:
 Badminton match was held immediately
after one of the cricket matches but not
held after the lunch break.
That means, badminton match must be held at
10 AM.
We have:  Only one match was held between both
 One of the Cricket matches was held after Volleyball matches.
the lunch break. That means, case (2) is not valid.

Click Here For Bundle PDF Course | support@guidely.in Page 6 of 10


Bank Po Mains PDF Course 2024
Reasoning Day -13 (Eng)

Based on the above given information we have: 13) Answer: B


14) Answer: A (Only two persons sit between the
given pair of persons, except option A)
15) Answer: A

We have:
 B, who is the only daughter-in-law of K,
sits five places away from R.
Again, we have:  R sits second to the right of K.
Since, three football matches were held, and That means, in case (1) B sits fifth to the
none of the football match was held immediately left of R, in case (2) B sits fifth to the right
before or after lunch break, thus case (2) and of R.
case (3) is not valid.  As many persons sit between B and K as
Based on the above given information we have: between K and G, who is the only
daughter of K.
 G sits third from one of the extreme ends.
Based on the above given information we have:

Linear Arrangement:

Again, we have:
Direction (11-15):  L, who is the only son of G, sits third to
11) Answer: C the right of the one who sits immediate
12) Answer: B left of G.

Click Here For Bundle PDF Course | support@guidely.in Page 7 of 10


Bank Po Mains PDF Course 2024
Reasoning Day -13 (Eng)

 L sits fourth to the left of M, who is


married to B.
For Blood relation:
That means in case (2) M sits immediate
left of K, case (1) is not valid.
Based on the above given information we have:

Again, we have:
 Only four persons sit between Y and Z,
who sits at the extreme end of the row.
 As many persons sit between Z and K as
between Q and U.
For linear arrangement: That means, in case (2) U sits immediate
right of L, in case (2a) U sits immediate
left of L.
 U sits four places away from P, who is the

Case (1) is not valid as no place for M. sibling of W’s spouse’s niece.

Again, we have: That means, in case (2) P sits immediate

 The one who sits third to the right of M is left of G, case (2a) is not valid.

the father of G. Based on the above given information we have:

 Only one person sits between M and W,


who is the father of L.
Since not more than three known persons
are sitting together, thus W doesn’t sit
adjacent to R.
 Only six persons sit between W and Q,
who is the only daughter of M. Linear arrangement:
 Q and Y sit together.
That means, in case (2) Y sits immediate
right of R, in case (2a) Y sits third to the
right of R.
Based on the above given information we have:

Click Here For Bundle PDF Course | support@guidely.in Page 8 of 10


Bank Po Mains PDF Course 2024
Reasoning Day -13 (Eng)

Case(2a) is not valid as not more than three It has been given in the paragraph that the
known persons are sitting together. people have become anxious as they have no
access to even the basic necessities so as per
16) Answer: E the content above, it has been said that a new
The first sentence ie; A talks about the crowd leadership should quickly take charge. Option (c)
that barged into the residences of the President could be correct had the issue been stopping the
and the PM. If we wonder, what made them do protests. But the issue asked is mentioned at the
the same? We get our answer for this is B which end of the paragraph. (e) becomes superfluous
says that the citizens wanted that these two looking at the situation given in the paragraph.
leaders should leave their offices. So, we can So, the correct answer is option (d).
say that; because of B  A took place…
We can clearly understand that, A is the effect 18) Answer: D
and B is the cause. We have two options, (b) An Inference is something that can be deduced
and (e) to confuse from. on the basis of some known fact. It is not
If we go with (b) which says B is one of the something that is directly given in the paragraph.
causes, the other causes than this one should This becomes the reason that we cancel the
also be mentioned which is not the case. B is the third option as it has been discussed directly at
only immediate cause that has been quoted. multiple places in the paragraph.
So, this makes option (e) the best choice for the Towards the end of the paragraph, it has been
answer. given that a new government can restore the
trust of the people to inspire them to stop their
17) Answer: D violent protests. From this, we can infer that the
A Course of Action should always be inclined first option should be correct.
towards lessening the problem and never an It has been given that Gotabaya’s resignation
extreme step. The option that one chooses would bring down the anger. From this we can
should either solve the issue given in the context infer that his presence in the government has
or at least lessen the intensity of the same. sparked a rage in the country.
The first option is an extreme step which should Thus, of the three options, we can clearly see
be avoided as it won’t make any difference. that the first two can be inferred and the third
Option (b) is not a solution as it would put a lot of can be left out. So, the correct answer is option
pressure on the country and increase the (d).
problem indirectly.
19) Answer: A

Click Here For Bundle PDF Course | support@guidely.in Page 9 of 10


Bank Po Mains PDF Course 2024
Reasoning Day -13 (Eng)

An assumption should be chosen in the reverse


order. We must start with the options and try to 20) Answer: B
find that if the particular option were assumed, A conclusion is logically deduced based on the
would that lead us to any conclusion given in the contents of the given passage. The whole
paragraph or the sentences given. Like for paragraph revolves around one single idea that
example; the issues that have led to protests can be
If we take the first option to be an assumption, addressed only with the change in the
we can very well conclude the point given in the government. The people have lost trust in their
paragraph that the speaker Yapa would be governance as they have been left at the mercy
taking over the charge till the government is of no one.
formed. So, this is correct. The first option is superfluous and can be ruled
Option (b) cannot be taken as an assumption as out. Option (c) is correct as given in the
nothing about the same or related to the same paragraph but it cannot be taken as the
has been clearly mentioned. It has nowhere conclusion. Same goes for option (d).
been mentioned that the violent protests were We have no clue about graft and corruption
the only way out. Even the statement in (d) which makes it wrong for us to mark option (e).
doesn’t count for an assumption as it has directly So, it can be eliminated.
been discussed as an issue in the paragraph. Thus, the correct answer is option (b).
So, the best would be to mark option (a) as the
answer.

Click Here For Bundle PDF Course | support@guidely.in Page 10 of 10


Bank Po Mains PDF Course 2024
Quantitative Aptitude Day -13 (Eng)

Quantitative Aptitude

Directions (01 - 05): Study the following information carefully and answer the questions given below.
There are five companies A, B, C, D and E. Each company has two offices in the city Mumbai and Delhi.
Total number of employees of Mumbai and Delhi office of each company and difference of employees of
Mumbai and Delhi office are given in bar graphs. There are three posts [HR, IT, and Marketing] in each
branch. Ratio of number of in HR, IT and Marketing of each office of each branch is given in table.

Notes – Number of employees in Mumbai office of company A and B is more than that of Delhi office and
Number of employees in Delhi office of company C, D and E is more than that of Mumbai office.

Click Here For Bundle PDF Course | support@guidely.in Page 1 of 11


Bank Po Mains PDF Course 2024
Quantitative Aptitude Day -13 (Eng)

1) Ratio of total number of male and female c) Difference between the number of employees
employees of Mumbai office of company A is 3:2, in marketing post of company E in Mumbai and
3:5 and 3:1 in HR, IT and Marketing posts. Ratio the Delhi office is 18.
of total number of male and female employees of d) Difference of average number of employees in
the Delhi office of company A is 2:3, 1:3 and 3:2 IT post all companies together in Mumbai office
in HR, IT and Marketing posts. Find the and average number of employees in Marketing
difference between the total number of males post all companies together in Mumbai office is
and total number of females of company A? 15 .
a) 16 e) None of these
b) 19
c) 12 4)
d) 18 Quantity I: Total number of employees in
e) 24 Marketing post in Delhi office of companies A, B
and C together is what percent of total number of
2) Find which one is true? employees in IT post of companies B, C and D
I. Difference in total employees in IT posts in together in Mumbai office?
Mumbai of all companies together and total Quantity II: Total number of employees in IT post
employees in IT posts In Delhi of all companies in Delhi office of companies A, B and C together
together is 14. is what percent of total number of employees In
II. Total employees in the Marketing post of all HR post of companies B, C and D together in
companies together are 306. Mumbai?
III. Average number of employees in the Delhi a) Quantity I > Quantity II
office of all companies together is 120. b) Quantity I ≥ Quantity II
a) Only I and II are true c) Quantity II > Quantity I
b) Only I is true d) Quantity II ≥ Quantity I
c) Only I and are III true e) Quantity I = Quantity II or Relation cannot be
d) All are true establish
e) All are false ed

3) Find which one is true? 5) Find the difference between the total number
a) Ratio of number of employees in HR post of of employees in companies A, B and C together
company C in Mumbai and Delhi office is 2:3. in the Mumbai office and the total number of
b) Total number of employees in HR post in employees in companies C, D and E together in
Mumbai office of all companies is 300. the Delhi office?

Click Here For Bundle PDF Course | support@guidely.in Page 2 of 11


Bank Po Mains PDF Course 2024
Quantitative Aptitude Day -13 (Eng)

a) 0 d) 3
b) 1 e) None of these
c) 2

Directions (06 - 10): Study the following information carefully and answer the questions given below.
The line graph shows the percentage of registered voters who cast their vote, the percentage of invalid
votes out of total cast votes, and the percentage of party A's votes out of a valid vote in five cities. The
total number of registered voters in each city is 30000. There is only party A and Party B contest in the
election.

6) Find the sum of the difference of votes got by and the average number of valid votes of all five
parties A and B in all cities together? cities together?
a) 20152 a) 2150
b) 26530 b) 2650
c) 23820 c) 2840
d) 21300 d) 2370
e) 26520 e) 2850

7) Find the difference between the average 8) In city F, the total number of populations is
number of cast votes of all five cities together 35000, and the number of registered voters is
the same as mentioned in the question. The ratio

Click Here For Bundle PDF Course | support@guidely.in Page 3 of 11


Bank Po Mains PDF Course 2024
Quantitative Aptitude Day -13 (Eng)

of valid votes of cities F and R is 4:3 and party A respectively. The speed of boat P, Q, and R in
got 55% of the valid vote then find the number of still water is x km/hr, y km/hr, and z km/hr
votes party B got in city F? respectively. On Monday, boat P goes 66 km
a) 11250 downstream and 56 km upstream in 7 hours and
b) 10800 88 km downstream and 28 km upstream in 6
c) 14200 hours. On Tuesday, boat Q goes 44 km
d) 13000 downstream and 60 km upstream in 8 hours and
e) None of these 88 km downstream and 70 km upstream in 11
hours. On Wednesday, boat R goes 64 km
9) Find the ratio of the total number of people downstream and 32 km upstream in 4 hours and
who did not cast their vote in all cities together 96 km downstream and 64 km upstream in 7
and the number of invalid votes in all cities hours.
together? 11) On Tuesday, boat P goes 5z km downstream
a) 200:79 and boat R goes 8c km upstream. Find the
b) 212:25 difference in time taken by two boats?
c) 241:26 a) 5(4/9)hrs
d) 241:22 b) 3(4/9)hrs
e) None of these c) 1(4/9)hrs
d) 2(4/9)hrs
10) Find the difference between the percentage e) 4(4/9)hrs
of party B's votes out of the total registered
voters in city P and the percentage of party A's 12) Find the ratio of the total distance covered by
votes out of the total registered voters in city Q? boat Q on Monday in b hours downstream and
a) 17.75% the total distance covered by boat R on Monday
b) 15.2% in c hours upstream?
c) 17.8% a) 2:3
d) 16.5% b) 4:5
e) None of these c) 5:6
d) 3:4
Directions (11 - 15): Study the following e) 1:2
information carefully and answer the questions
given below. 13) On Monday,Boat P goes from point M to N
The speed of the stream of river on Monday, downstream of the river. O is the middle point of
Tuesday, and Wednesday is a, b, and c km/hr M and N. From M to O, speed of the stream is

Click Here For Bundle PDF Course | support@guidely.in Page 4 of 11


Bank Po Mains PDF Course 2024
Quantitative Aptitude Day -13 (Eng)

25% less than the original speed. If the distance 16)


between point M and N is 88 km, then find the Quantity I: Total surface area of a sphere is 2464
approximate total time taken by the boat? m2. Find the volume of the sphere?
a) 5 hrs Quantity II: The sum of the height and radius of a
b) 4 hrs cone is 28 m and the ratio of radius and height of
c) 3 hrs the cone is 11:3. Find the volume of the cone?
d) 2 hrs Quantity III: The ratio of the side of the cube and
e) None of these length of the cuboid is 2:5. Volume of the cube is
64 m. Ratio of length and breadth and the ratio of
14) Speed of boat S is 25% more than the speed length and height of the cuboid is 5:3 and 5:7.
of boat of Q in still water. Find the distance cover Find the volume of the cuboid?
by boat S on Wednesday in (a+2) hours a) Quantity I <Quantity II >Quantity III
downstream? b) Quantity I> Quantity II =Quantity III
a) 168 km c) Quantity I< Quantity II< Quantity III
b) 172 km d) Quantity I> Quantity II> Quantity III
c) 180 km e) Quantity I> Quantity II <Quantity III
d) 182 km
e) None of these 17)
Quantity I: The ratio of age of A and B is 2:3.
15) Boat Q and Boat R are travelling in opposite After 5 years age of B is 32 years. Age of C is 5
direction towards each other from a certain years more than A. Find the average age of A, B,
distance on Tuesday. If boat Q goes downstream and C?
and boat R goes upstream and they meet after 7 Quantity II: The sum of age of A and B is 38
hours then find the distance between boat Q and years and age of B is 5 years older than C whose
R initially? age is 13 years. Find the age of A?
a) 280 km Quantity III: The age of the son is 1/3rd of the
b) 220 km age of the father and the age of the mother is
c) 256 km thrice of her daughter. If the sum of the age of
d) 250 km the father and mother is 84 years and both of
e) None of these their ages are equal. Find the sum of the age of
the daughter and son?
Direction (16-20) – Each question has three a) Quantity I >Quantity II >Quantity III
quantities. Calculate the value of each quantity b) Quantity I>Quantity II< Quantity III
and compare it. c) Quantity I =Quantity II< Quantity III

Click Here For Bundle PDF Course | support@guidely.in Page 5 of 11


Bank Po Mains PDF Course 2024
Quantitative Aptitude Day -13 (Eng)

d) Quantity I< Quantity II <Quantity III Quantity II: An item isbought at Rs.1250 then
e) Quantity I >Quantity II= Quantity III marked the price 50% above the buying price
and then gave a 10% discount on the marked
18) price, find the profit percentage?
Quantity I: A person has Rs.12400. He invests Quantity III: The selling price of the item is
40% in the scheme which provides a 10% rate of Rs.1580. The shopkeeper is given a 20%
interest and the rest invests in another scheme discount on the marked price and the marked up
which provides a 5% rate of interest. Find the percentage is 40%. Find the profit percentage?
total interest earned by the person after 5 years if a) Quantity I <Quantity II <Quantity III
both are invested in SI? b) Quantity I= Quantity II >Quantity III
Quantity II: Rs.4852.85 c) Quantity I <Quantity II> Quantity III
Quantity III: A person invests Rs.1200 at 20% d) Quantity I >Quantity II> Quantity III
rate of compound interest for 2 years and e) Quantity I >Quantity II <Quantity III
Rs.6000 invests at 25% rate of compound
interest for 2 years. Find the total interest earned 20) f(x)=x3-3x2+3x-4
by the person? Quantity I: Find the value of f(x) when x=3?
a) Quantity I =Quantity II =Quantity III Quantity II: Find the value of f(x) when x=-2.5?
b) Quantity I >Quantity II >Quantity III Quantity III: Find the value of f(x) when x=1.5?
c) Quantity I <Quantity II< Quantity III a) Quantity I <Quantity II >Quantity III
d) Quantity I >Quantity II =Quantity III b) Quantity I< Quantity II< Quantity III
e) Quantity I < Quantity II >Quantity III c) Quantity I <Quantity II =Quantity III
d) Quantity I >Quantity II< Quantity III
19) e) Quantity I =Quantity II <Quantity III
Quantity I: The ratio of the cost price and selling
price of the item is 7:11. Find the profit
percentage of the item?

Click Here For Bundle PDF Course | support@guidely.in Page 6 of 11


Bank Po Mains PDF Course 2024
Quantitative Aptitude Day -13 (Eng)

Click Here to Get the Detailed Video Solution for the above given Questions
Or Scan the QR Code to Get the Detailed Video Solutions

Answer Key with Explanation


Directions (01 - 05):
Number of employee in Mumbai office of
company A is [220+60]/2=140
Number of employee in Delhi office of company
A is [220-60]/2=80
Number of employees in HR post of Mumbai
Office =140*3/7=60
Number of employees in IT post of Mumbai
office =140*2/7=40
Number of employees in Marketing post of
Mumbai office =140*2/7=40
Number of employees in HR post of Delhi Office
=80*3/8=30
Number of employees in IT post of Delhi office
=80*2/8=20
Number of employees in Marketing post of Delhi
office =80*3/8=30
Similarly, we can calculate other values also.
1) Answer: C
Total male in company A is
=60*3/5+40*3/8+40*3/4+30*2/5+20*1/4+30*3/5=
116
Total female in company A is

Click Here For Bundle PDF Course | support@guidely.in Page 7 of 11


Bank Po Mains PDF Course 2024
Quantitative Aptitude Day -13 (Eng)

=60*2/5+40*5/8+40*1/4+30*3/5+20*3/4+30*2/5= Quantity I
104 Required percentage = [(30+40+20) /
So, difference is = 116-104 = 12 (60+20+40)]*100 = 75%
Quantity II
2) Answer: D Required percentage =
I. Difference of total employees in IT post in [(20+20+60)/(80+60+60)]*100 = 50%
Mumbai of all companies together and total
employees in IT post in Delhi of all companies 5) Answer: A
together is Required difference = [140+160+100] –
= [40+60+20+40+30]-[20+20+60+40+36]=14. [120+160+120] =0
II. Total employees in Marketing post of all
companies together Directions (06 - 10):
=40+20+20+40+20+30+40+20+40+36=306. In city P,
III. Average number of employees in Delhi office The number of total registered voters is 30000.
of all companies together is = The total number of cast votes is
[80+120+120+160+120]/5=120. 30000*80/100=24000
The total number of people who did not cast vote
3) Answer: E is 30000-24000=6000
a) Ratio of number of employees in HR post of The total number of valid votes is
company C in Mumbai and Delhi office is 3:2. 24000*90/100=21600
b) Total number of employees in HR post in the The total number of invalid votes is 24000-
Mumbai office of all companies is 320. 21600=2400
c) Difference between the number of employees The number of votes gets by party A is
in marketing post of company E in Mumbai and 21600*60/100=12960
the Delhi office is 16. The number of votes gets by party B is 21600-
d) Difference of average number of employees in 12960=8640
IT post all companies together in Mumbai office Similarly, we can calculate other cities also.
and average number of employees in Marketing
post all companies together in Mumbai office is
10.
So, all are false.

4) Answer: A

Click Here For Bundle PDF Course | support@guidely.in Page 8 of 11


Bank Po Mains PDF Course 2024
Quantitative Aptitude Day -13 (Eng)

The speed of the stream of river R on Monday,


Tuesday, and Wednesday is a, b, and c km/hr
respectively. The speed of boats P, Q, and R in
still water is x km/hr, y km/hr, and z km/hr
So, we can say,
66/(a+x) + 56/(x-a) =7 --------I
88/(a+x) + 28/(x-a) =6 --------II
By solving I and II we get, x=18 km/hr and a=4
km/hr.
6) Answer: C
44/(y+b) + 60/(y-b) =8 --------III
Required sum
88/(y+b) + 70/(y-b) =11 --------IV
=[12960-8640]+[13965-5985]+[9900-
By solving III and IV we get, y=16 km/hr and b=6
8100]+[13770-9180]+[15390-10260]= 23820
km/hr.
64/(z+c) +32/(z-c) =4 --------V
7) Answer: D
96/(z+c) + 64/(z-c) =7 --------VI
Required difference
By solving V and VI we get, z=24 km/hr and c=8
= [24000+21000+22500+25500+27000]/5 –
km/hr.
[21600+19950+18000+22950+25650]/5
11) Answer: C
=24000-21630=2370
Required difference = 120/(18+6)-64/(24-
6)=1(4/9)hrs
8) Answer: B
B’s party vote is [18000*4/3]*45/100=10800
12) Answer: D
Required ratio =[16+4]*6 : [24-4]*8=120:160=3:4
9) Answer: A
Required ratio=[6000+9000+7500+4500+3000]:
13) Answer: B
[2400+1050+4500+2550+1350]
The speed stream from M to O is 4*75/100=3
=30000:11850=200:79
So, required time is =
[44/(18+3)]+[44/(18+4)]=4.09=4 hours.
10) Answer: A
Required difference =[13965/30000]*100–
14) Answer: A
[8640/30000]*100=17.75%
The speed of boat S is =16*125/100=20 km/hr.
Required distance = [20+8]*[4+2]=28*6=168 km
Directions (11 - 15):

Click Here For Bundle PDF Course | support@guidely.in Page 9 of 11


Bank Po Mains PDF Course 2024
Quantitative Aptitude Day -13 (Eng)

15) Answer: A 18) Answer: E


Total distance of boat Q and R initially is = Quantity I:
[16+6]*7+[24-6]*7=280 km Total interest earned
= [12400*40/100]* 5*10
16) Answer: D /100+[12400*60/100]*5*5/100
Quantity I: =Rs.4340
So, 4*22/7*r*r=2464, r=14m Quantity II: Rs.4852.5
So, the volume is 22/7*14*14*14*4/3=11498.66 Quantity III:
m3 Total interest earned
Quantity II: 1200*[1+20/100]2+6000*[1+25/100]2-
Radius is 28*11/14=22m, height is 28-22=6m 7200=Rs.3903
So, volume is =1/3*22/7*22*22*6=3042.28m3 Quantity I < Quantity II >Quantity III
Quantity III:
The side of the cube is 4m 19) Answer: D
So, the length of the cuboid is 4*5/2=10m Quantity I:
The breadth of the cuboid is 10*3/5=6m Profit percentage = [4/7]*100=57.14%
The height of the cuboid is 10*7/5=14m Quantity II:
So, volume is 10*6*14=840m3 Selling price =
Quantity I> Quantity II> Quantity III 1250*150*90/(100*100)=Rs.1687.5
Profit percentage = [(1687.5-
17) Answer: B 1250)/1250]*100=35%
Quantity I: Quantity III:
Age of B is 32-5=27 years. Cost price of item is
Age of A is 27*2/3=18 years 1580*[100/80]*[100/140]=1410.71
Age of C is 18+5=23 years So, profit percentage [1580-
So, the average age is [27+18+23]/3=22.66 1410]*100/1410=12.05%
years Quantity I >Quantity II> Quantity III
Quantity II:
Age of Ais 38-[13+5]=20 years 20) Answer: D
Quantity III: Quantity I:
The Sum of the age of Son and daughter is So, f(x)=3*3*3-3*(3)2+3*3-4 when x=3,
84/3=28 years Or, f(x)=5
Quantity I>Quantity II< Quantity III Quantity II:

Click Here For Bundle PDF Course | support@guidely.in Page 10 of 11


Bank Po Mains PDF Course 2024
Quantitative Aptitude Day -13 (Eng)

So, f(x)=(-2.5)3-3(-2.5)2+3*(-2.5)-4=-45.875 when So, f(x)=(1.5)3-3*(1.5)2+3*(1.5)-4=-2.875


x=-2.5 So, Quantity I >Quantity II < Quantity III
Quantity III:

Click Here For Bundle PDF Course | support@guidely.in Page 11 of 11


Bank Po Mains PDF Course 2024
English Day - 13

English Language
Directions (1-5) : Rearrange the following five e) E
sentences (A), (B), (C), (D) and (E) in the proper
sequence to form a coherent paragraph and then 2) Choose from the below options the
answer the questions given below. SYNONYM of the word PROMOTE as given in
(A) The alliance operates on the principle of the sentence B.
collective defence/A, stating that an attack a) Stimulate
against one member is/B considered an attack b) demote
against to all, leading to/C joint action to deter c) discourage
and respond to potential threats/D. d) dishonour
(B) Formed in 1949, NATO is a political and e) desist
military alliance that aims to promote collective
defence, preserve peace, and safeguard the 3) Fill in the blank given in sentence C and
freedom of its member nations. complete it.
(C) NATO, the North Atlantic Treaty a) commemoration
Organization, stands as a cornerstone of b) contemplation
international security and __________ . c) connotation
(D) a pivotal role in promoting stability and d) cooperation
peace in /P Europe and beyond, undertaking e) confrontation
missions /Q throughout its history, NATO has
played /R management, and disaster relief 4) Rearrange the parts of the sentence in D to
operations /S such as peacekeeping, crisis /T form a meaningful sentence.
(E) Comprising 30 member countries from North a) TQSPR
America and Europe, NATO fosters close b) QPTRS
diplomatic ties and military cooperation among its c) STRQP
members to address a wide range of security d) PQRST
challenges. e) RPQTS
1) Find which part of sentence ‘A’ has an error in
it and mark that as your answer. Mark option (e) 5) Which of the following is the correct sequence
if no error is found. of rearrangement that forms a meaningful
a) A coherent paragraph ?
b) B
c) C a) CDEAB
d) D b) CBEAD

Click Here For Bundle PDF Course | support@guidely.in Page 1 of 10


Bank Po Mains PDF Course 2024
English Day - 13

c) BCEAD of creating fireworks igniting a sense of


d) AECDB
spread to different childlike wonder in our
e) EDBCA
parts of the world, hearts.
each culture
Directions (6-10): In the following questions two
a) A-F
columns are given containing three
b) A-E
Sentences/phrases each. In the first column,
c) B-E
sentences/phrases are A, B and C and in the
d) A-E and C-D
second column the sentences/phrases are D, E
e) None of these
and F. A sentence/phrase from the first column
may or may not connect with another
7)
sentence/phrase from the second column to
COLUMN 1 COLUMN 2
make a grammatically and contextually correct
sentence. Each question has five options, four of A. While conjunctivitis D. dirty hands, using
which display the sequence(s) in which the is generally not a contaminated eye
sentences/phrases can be joined to form a severe condition, it makeup, or swimming in
grammatically and contextually correct sentence. can be highly water with harmful
If none of the options given forms a correct contagious, bacteria.
sentence after combination, mark option (e), i.e. necessitating
“None of these” as your answer.
B. These infections E. self-medicate eye
6)
can lead to more infections, as improper
COLUMN 1 COLUMN 2
severe symptoms like treatment can
A. From brilliant D. create entertainment severe pain, blurred exacerbate the condition
cascades of sparkles and began incorporating vision, sensitivity or delay proper care.
to thunderous them into religious
C. Eye infections can F. to light, and even
explosions that ceremonies and festive
be caused by various vision loss if left
reverberate in the events.
factors, including untreated.
B. The history of E. they were initially poor hygiene,
fireworks dates back invented by alchemists touching the eyes
to ancient China, searching for the elixir of with
where immortality.

C. Over time, the art F. against the darkness, a) A-D

Click Here For Bundle PDF Course | support@guidely.in Page 2 of 10


Bank Po Mains PDF Course 2024
English Day - 13

b) B-F A. In an age D. simple pleasure of


c) A-F and B-D
dominated by traditional games, the
d) B-F and C-D
screens and great outdoors offers an
e) None of these
technology, the expansive canvas for
allure of outdoor boundless fun and
8) games remains as adventure.
COLUMN 1 COLUMN 2 strong as ever,
A. By being D. technology's benefits
B. From soccer and E. and the thrill of
positive role and ensuring healthy real-
basketball to cricket scoring a point can
models and world interactions is a
and baseball, team create unforgettable
displaying qualities challenge parents face in
sports have a unique memories with friends
such as kindness, the digital age.
way of fostering and family.
B. It is also crucial E. resilience, and honesty, camaraderie,
for parents to parents impart invaluable teaching
remember that no life lessons that extend far
C. Whether it's the F. bring people together,
one is infallible, beyond the classroom.
excitement of team encouraging physical
and making
sports, the thrill of activity and imaginative
mistakes
running freely on play in equal measure.
C. The constant F. ample time to children open fields, or the
juggling act can sometimes leave
a) A-F
between work, parents feeling
b) C-D
household overwhelmed and c) B-D and C-F
responsibilities, exhausted.
d) B-E and C-D
and giving e) None of these
a) A-E and C-F
b) A-D and C-E 10)
c) B-F and C-D COLUMN 1 COLUMN 2
d) B-E and C-F A. Adulteration, a D. affects human health
e) None of these deceptive practice but also tarnishes the
that involves adding reputation of businesses
9) inferior or harmful and industries.
COLUMN 1 COLUMN 2

Click Here For Bundle PDF Course | support@guidely.in Page 3 of 10


Bank Po Mains PDF Course 2024
English Day - 13

substances to food, ensuring a tidy _______ that would attract


customers and make their shopping experience
beverages, or other
more enjoyable.
products, poses a
a) count, place
B. The history of E. adulteration has b) arrange, display
adulteration dates expanded to a wide range c) disturb, present
back centuries, of products, including d) keeping, look
where unscrupulous food items, e) organized, exhibit
individuals sought to pharmaceuticals,
profit at the expense cosmetics, and even 12) Typewriting classes were in high demand
fuels. during the pre-computer era, as they offered
valuable skills for ______office professionals and
C. Contaminated F. serious threat to public
typists, ensuring fast and ______ document
food and beverages health and undermines
creation.
can lead to a host of the integrity of industries
a) demotivated, inexact
health issues, and businesses.
b) beginning, furious
ranging from mild
c) aspiring, accurate
discomfort and
d) disinterested, improper
a) A-F
e) energetic, false
b) B-E
c) B-D and C-F
13) Free meals in the government schools aimed
d) B-E and C-F
to _______ the issue of malnutrition among
e) None of these
underprivileged children, ensuring they received
_______ nutrition for their overall well-being.
Directions (11-15) : Each of the following
a) address, adequate
questions has two blanks, each blank indicating
b) fixed, good
that a word/phrase has been omitted. Choose
c) solve, insufficient
the set of words/phrases that best fits the given
d) complicate, satisfaction
blanks according to the context of the sentence.
e) preclude, fair
Few alphabets are given to help you locate the
blanks and fill them.
14) She worked really hard to manage both her
_______job and her responsibilities as a single
11) The storekeeper was instructed to _______
parent, proving her determination and dedication
the newly arrived merchandise on the shelves,
to ______the best for her child.

Click Here For Bundle PDF Course | support@guidely.in Page 4 of 10


Bank Po Mains PDF Course 2024
English Day - 13

a) demanding, provide 17) The medicines at the store were neatly


b) easy, take arranged (A) on the shelves, with a wide
c) simple, deliver assortment (B) of prescription drugs, over-the-
d) new, lend counter remedies, and health supplements
e) teaching, refuse catering (C) to various healthcare needs (D).
a) A-B, B-assorted
15) The launch of the restaurant was a grand b) A-D, D-equipments
______, with celebrities, food critics, and locals c) B-C, A-arranging
alike attending the event to _____the delectable d) C-D, C-caters
offerings and celebrate the culinary journey. e) No changes required
a) event, tasted
b) launch, enjoy 18) Exams are arriving (A) up, and there's a mix
c) opening, dislike of nervousness and determination among the
d) affair, savour students as they diligently (B) prepare and
e) party, oppose achieve (C) to perform at their best and revise
(D) their academic goals.
Directions (16-20) : The following questions a) A-C, D-revised
given below have four words that have been b) B-C, C-achieving
highlighted as they might’ve been placed at the c) B-D, A-arrived
wrong positions. One of these words might also d) C-D, A-coming
be incorrect and need a replacement. Read the e) No changes required
same carefully and mark the appropriate option
as the answer. 19) Ballpoint pens are better choice (A) for
16) My friend is celebration (A) married this everyday (B) writing tasks due to their smooth ink
month, and the excitement in the air is palpable flow, reliability, and convenience, making them a
(B) as we eagerly anticipate getting (C) their love popular suit (C) among students and
and witnessing (D) the beginning of a beautiful professionals alike (D).
new chapter in their lives. a) A-C, C-suited
a) A-B, D-encountering b) A-D, B-every
b) B-D, C-readily c) B-C, C-suited
c) A-C, A- celebrating d) C-D,B-every
d) C-D, A-celebrating e) No changes required
e) No changes required 20) The OMR sheet given (A) in the examination

Click Here For Bundle PDF Course | support@guidely.in Page 5 of 10


Bank Po Mains PDF Course 2024
English Day - 13

was carefully designed with printed bubbles b) B-C, C-corresponding


allowing (B) to each question, correspond (C) c) B-D, B- allow
candidates to mark their answers precisely and d) C-D, B-allowed
facilitating (D) efficient evaluation. e) No changes required
a) A-D, A-gave
Click Here to Get the Detailed Video Solution for the above given Questions
Or Scan the QR Code to Get the Detailed Video Solutions

Answer Key with Explanation


1) Answer: C The word stimulate is similar in meaning to the
Part C of the sentence has an error in it. word ‘promote’
Replace ‘against to all’ with ‘against all’ to make The other words wither have different meaning
the given sentence grammatically correct and or they are the opposite of ‘promote’
meaningful.
The corrected sentence : The alliance operates Demote - to move somebody to a lower position
on the principle of collective defence, stating that or level
an attack against one member is considered an Dishonour - lack or loss of honour
attack against all, leading to joint action to deter Desist - stop doing something; cease or abstain
and respond to potential threats.
3) Answer: D
2) Answer: A Cooperation is the right word that fits the given
The meaning of promote - to encourage blank correctly.
something Commemoration - something that is done to
Stimulate - to make something active or more remember officially and give respect to a great
active person or event
Contemplation - deeply or seriously thoughtful.

Click Here For Bundle PDF Course | support@guidely.in Page 6 of 10


Bank Po Mains PDF Course 2024
English Day - 13

Connotation - an idea expressed by a word in come after the introduction and before the actual
addition to its main meaning work done by NATO. As, sentence ‘E’ spoke
Confrontation - a fight or an argument about the security challenges ‘A’ is the correct
The sentence: NATO, the North Atlantic Treaty sentence that follows ‘E’. The final sentence or
Organization, stands as a cornerstone of the last sentence is ‘D’ which seems to be a
international security and cooperation. concluding sentence representing the actions of
NATO throughout history.
4) Answer: E
The correct sequence of rearrangement from the 6) Answer: C
given option is option (e) RPQTS. B-E forms a complete sentence which is both
R is the starter sentence and the remaining are a meaningful and correct.
continuation of one another, P follows R joining The other options have a combination of
the sentence. Next comes Q which is a statements/fragments which do not combine to
continuation of P. Among S and T, S is the form a complete sentence.
concluding sentence because it has the last The sentence formed : The history of fireworks
fragment “and…”. So, TS forms the last two dates back to ancient China, where they were
sentences. initially invented by alchemists searching for the
The sentence : Throughout its history, NATO elixir of immortality.
has played a pivotal role in promoting stability
and peace in Europe and beyond, undertaking 7) Answer: D
missions such as peacekeeping, crisis Both B-F and C-D are the correct combinations
management, and disaster relief operations. which combine to form a complete sentence
which is meaningful and correct.
5) Answer: B The other options are incorrect as those
CBEAD is the right sequence of rearrangement combinations of fragments/statements do not
that forms a meaningful paragraph. form a proper sentence.
C is the first sentence, as it has the full form of The sentences formed :
NATO and also the basic/simple definition or B-F : These infections can lead to more severe
purpose of NATO. ‘B’ follows ‘C’ as it talks about symptoms like severe pain, blurred vision,
the formation of NATO and gives more details on sensitivity to light, and even vision loss if left
its purpose. Sentence ‘E’ follows CB and comes untreated.
before both A and D because it talks about the C-D : Eye infections can be caused by various
members and the diplomatic ties which should factors, including poor hygiene, touching the

Click Here For Bundle PDF Course | support@guidely.in Page 7 of 10


Bank Po Mains PDF Course 2024
English Day - 13

eyes with dirty hands, using contaminated eye 10) Answer: A


makeup, or swimming in water with harmful A-F is the correct combination that forms a
bacteria. complete sentence.
The other given options are wrong as they don’t
8) Answer: A contain statements that form a meaningful
A-E forms a complete meaningful sentence sentence.
similarly the combination C-F forms a complete The sentence formed : Adulteration, a deceptive
sentence with meaning. Fragments B and D do practice that involves adding inferior or harmful
not combine with the other statements to substances to food, beverages, or other
generate a sentence and hence are discarded. products, poses a serious threat to public health
The sentences formed : and undermines the integrity of industries and
A-E : By being positive role models and businesses.
displaying qualities such as kindness, resilience, 11) Answer: B
and honesty, parents impart invaluable life Arrange and display are the right words that fit
lessons that extend far beyond the classroom. both the given blanks respectively to complete
C-F : The constant juggling act between work, the meaning of the sentence.
household responsibilities, and giving ample First blank - this blank needs a verb in the first
time to children can sometimes leave parents form - count, arrange and disturb are the verbs
feeling overwhelmed and exhausted. in the first form - so eliminate options d and e.
Disturb completely inappropriate to the given
9) Answer: B context - eliminate option c
The fragment C and D combine to form a Count on the shelves is wrong - so eliminate
meaningful and complete sentence. option a
Even though the other statements also revolve The left out and the correct option is ‘b’.
around the same topic they do not combine with The sentence : The storekeeper was instructed
each other to form a sentence that is meaningful to arrange the newly arrived merchandise on the
and correct. shelves, ensuring a tidy display that would
The sentence formed : Whether it's the attract customers and make their shopping
excitement of team sports, the thrill of running experience more enjoyable.
freely on open fields, or the simple pleasure of
traditional games, the great outdoors offers an 12) Answer: C
expansive canvas for boundless fun and Aspiring and accurate are the two words that fit
adventure. the given blanks respectively.

Click Here For Bundle PDF Course | support@guidely.in Page 8 of 10


Bank Po Mains PDF Course 2024
English Day - 13

First blank - aspiring and energetic are the only The sentence : Free meals in the government
words that fit the context of the sentence, the schools aimed to address the issue of
other words create a negative impact on the malnutrition among underprivileged children,
sentence, so discard options a, b and d. ensuring they received adequate nutrition for
Second blank - again ‘false’ creates a negative their overall well-being.
impact but the sentence is in a positive tone
where professionals learnt typewriting to develop 14) Answer: A
their skills. So, to match the context ‘accurate’ is The correct pair is “demanding and provide”.
the correct word. First blank - All the given words fit the first blank
The sentence : Typewriting classes were in high but ‘easy and simple’ are opposite in context of
demand during the pre-computer era, as they the given sentence and hence options b and c
offered valuable skills for aspiring office are discarded.
professionals and typists, ensuring fast and Among a,d and e - the option which has the
accurate document creation. correct word to fill the second blank is option(a) -
‘provider the best’
13) Answer: A ‘Lend the best’ and ‘refuse the best’ are
Address and adequate are the correct words and incorrect and inappropriate and hence not the
hence option (a) is the right answer for this right pick
question. The sentence : She worked really hard to
The first blank needs a verb in the first form - manage both her demanding job and her
discard option b responsibilities as a single parent, proving her
The government scheme is in place to help the determination and dedication to provide the best
people and children so ‘complicate and preclude’ for her child.
are incorrect words that can be used - eliminate
options d and e 15) Answer: D
Preclude - to prevent something from happening Affair and savour are the right words.
or somebody from doing something; to make Savour - to taste or smell with pleasure
something impossible First blank - launch and opening do not fit the
Between options a and c - look at the second first blank because they are redundant and
words adequate and insufficient - adequate is conveying the same meaning repeatedly - so
the right word that fits the second blank in discard options b and c
accordance with the context of the sentence. Out of options a,d and e - ‘to tasted’ is wrong,
Option a is the correct answer. similarly ‘to oppose’ is inappropriate to the

Click Here For Bundle PDF Course | support@guidely.in Page 9 of 10


Bank Po Mains PDF Course 2024
English Day - 13

context of the sentence, hence ‘savour’ is the The sentence: Exams are coming up, and
correct word. there's a mix of nervousness and determination
The sentence : The launch of the restaurant was among the students as they diligently prepare
a grand affair, with celebrities, food critics, and and revise to perform at their best and achieve
locals alike attending the event to savour the their academic goals.
delectable offerings and celebrate the culinary
journey. 19) Answer: A
‘Are better choice’ is wrong instead it must be
16) Answer: C ‘are a better choice' so definitely word (A) is
First of all the word ‘celebration’ is incorrect, misplaced, swap it with word C to make the
instead it must be celebrating. And the positions sentence correct.
of the words A and C must be interchanged to The word ‘suit’ is still inappropriate, change it to
make the given sentence all correct and ‘suited’ to make the words and the respective
meaningful. sentence correct and meaningful.
The sentence: My friend is getting married this The sentence: Ballpoint pens are better suited
month, and the excitement in the air is palpable for everyday writing tasks due to their smooth ink
as we eagerly anticipate celebrating their love flow, reliability, and convenience, making them a
and witnessing the beginning of a beautiful new popular choice among students and
chapter in their lives. professionals alike.

17) Answer: E 20) Answer: B


The given four words are correctly placed and Swap the words B and C to make the sentence
are appropriate too, hence there is no need to correct and meaningful. The word ‘correspond’ is
swap or replace them. incorrect, instead it must be ‘corresponding’ to
make the sentence more accurate and
18) Answer: D meaningful.
‘Arriving up’ is completely wrong and hence The sentence: The OMR sheet given in the
needs replacement of the word itself. examination was carefully designed with printed
Out of the given options ‘coming’ is the right bubbles corresponding to each question,
word to put in place (A) and replace words C and allowing candidates to mark their answers
D to make the sentence correct. precisely and facilitating efficient evaluation.

Click Here For Bundle PDF Course | support@guidely.in Page 10 of 10


Bank Po Mains PDF Course 2024
Reasoning Day -14 (Eng)

Reasoning Aptitude
Directions (1-5): In each group of questions II. No Pine is Teak
below there are three conclusions followed by III. All Maples are Oak is not a Possibility
five statements. You have to choose the correct Statements:
set of statements that logically satisfy the given I. Only a few Maples are Pine, All Pines are Oak,
conclusions. Only Oak is Teak, No Cedar is Maple
1. Conclusions: II. Only a few Cedars are Pine, No Oak is Pine,
I. Some Emeralds are Sapphire is a possibility Some Oaks are not Teak, All Maples are Teak
II. Some Pearl is not Sapphire III. Some Teaks are Oak, Only a few Oaks are
III. All Diamonds can be Ruby Pine, All Pines are Cedar, No Maple is Teak
Statements: IV. No Maple is Teak, Only Maple is Pine, Some
I. Only a few Pearls are Diamonds. All Emeralds Cedars are Maple, Some Teaks are not Oak
are Diamonds. Some Sapphires are Ruby. No V. All Cedars are Pine, Some Teaks are Oak, All
Diamond is Sapphire Pines are Maples, Only a few Maples are Teak
II. Some Diamonds are Ruby. No Ruby is a) Both I and V follow
Sapphire. Only a few pearls are Sapphire. All b) Only II follows
Emeralds are Diamonds. c) Both I and IV follow
III. Only a few Emeralds are Pearl. All Pearls are d) Only I, IV and V follow
Ruby. No Ruby is Sapphire. Only Sapphire is e) All follows
Diamond.
IV. Some Diamonds are Emerald. All Emeralds 3. Conclusions:
are Pearl. Only a few Pearls are Ruby. No Ruby I. All Quartz can be Marble
is Sapphire. II. All Marbles are Granite is a possibility
V. Some Rubies are Diamond. Only Pearl is III. Some Tiles are Concrete is a possibility
Emerald. No Diamond is Pearl. Only a few Statements:
Sapphires are Ruby. I. Only a few Concretes are Granites, Some
a) All follow Granites are Tiles, No Tile is Marble, Only
b) Both I and III follow Concrete is Quartz
c) Both II and V follow II. Only a few Granites are Marbles, Some
d) Both II and IV follow Marbles are not Tiles, All Tiles are Quartz, Some
e) None follows Concretes are Granites
III. All Marbles are Quartz, All Tiles are Granites,
2. Conclusions: No Quartz is Granite, Only a few Granites are
I. Some Cedars are not Oak is a possibility Concrete

Click Here For Bundle PDF Course | support@guidely.in Page 1 of 12


Bank Po Mains PDF Course 2024
Reasoning Day -14 (Eng)

IV. Only a few Concretes are Marble, Some c) Both II and IV follow
Marbles are Tiles, Some Tiles are not Granite, All d) Both IV and V follow
Quartz are Granite e) None Follows
V. Some Marbles are Tile, Only a few Tiles are
Quartz, All Quartz are Granite, No Granite is 5. Conclusions:
Concrete I. All Monsoons is winter
a) None follows II. All Summer being spring is a possibility
b) Both II and III follow III. Some Winter is not autumn
c) Only II, IV and V follow Statements:
d) All follows I. Only a few Spring is Summer, Some Summer
e) Both II and IV follow is Winter, No Autumn is Winter, All Autumn is
Monsoon
4. Conclusions: II. Some Winter is Autumn, All Autumn is
I. No Tree is Creeper Summer, No Summer is Monsoon, Only a few
II. Some Shrubs are Tree is a Possibility Spring is Monsoon
III. Some Herbs are not Climber III. No Monsoon is Spring, Some Spring is
Statements: Winter, Only a few Winter is Autumn, Only
I. Only a few Shrubs are Climber, All Climbers Autumn is Summer
are Tree, No tree is Herb, Some Creepers are IV. All Monsoon is Winter, Some Winter is
Herb Autumn, Only a few Autumn is Spring, No Spring
II. All Shrubs are Climber, Some Climbers are is Summer
Creeper, Only a few Creepers are Herb, Some V. Some Autumn is Summer, Only a few
Herbs are not Tree Summer is Spring, No Spring is Monsoon, No
III. All Trees are Herb, Only a few Herbs are Winter is Monsoon
Shrub, Only Shrub is Climber, Some Creepers a) Both I and V follow
are Shrub b) Both II and IV follow
IV.Some Trees are Herb, Only a few Herbs are c) Both I and III follow
Shrub, Only Shrub is Creeper, No Climber is d) All follow
Shrub e) None follows
V. Some Climbers are Creepers, Only a few Directions (6-10): Study the following information
Creepers are Shrub, No Shrub is Herb, All Herbs carefully and answer the given questions.
are Tree Ten boxes are placed one above the other in a
a) All Follow stack in an Amazon warehouse where the
b) Only IV follows bottommost box is numbered 1 and the box

Click Here For Bundle PDF Course | support@guidely.in Page 2 of 12


Bank Po Mains PDF Course 2024
Reasoning Day -14 (Eng)

above 1 is numbered 2 and so on till the topmost 6. How many boxes are having an even number
box is numbered 10. Each box contains different of toys and which of the following toy is in the
types of toys viz., P, Q, R, S, T, U, V, W, X and box which has the highest odd number of toys?
Y. Each box has different number of toys. a) 7, R
Note: The number of toys in box 1 is equal to the b) 7, Q
multiple of the box number of the box which is c) 6, P
placed at the topmost position, The number of d) 6, R
toys in box 2 is equal to the multiple of the box e) 7, S
number of the box which is placed at the second
topmost position and so on till the number of toys 7. The number of boxes between the box which
in a box 10 is equal to the multiple of the box has 52 toys and Q is ______ the number of boxes
number of the box which placed at the between W and the box which has 40 toys.
bottommost positions. a) Two less than
Only four boxes are placed between the box b) Three more than
which has 66 toys and V. The box which has 66 c) Three less than
toys is not placed at the topmost position. The d) Two more than
box which has 35 toys which is placed above V e) One more than
and three boxes below W. As many boxes
placed below the box which has 49 toys as 8. The_____ toy is in the box 7 and the _____ toy
above X, which is not placed at the bottommost is in the box 5 respectively.
position. The box which has 50 toys contains Y, a)X, U
which is placed below W. The number of boxes b) X, T
placed between Y and the box which has 52 toys c) P, T
is two more than the number of boxes placed d) P, Q
between the boxes which has 37 toys and T. R is e) Q, W
placed immediately below the box which has 40
toys. More than four boxes are placed between R 9. If the Toys are arranged in alphabetical order
and P, which is not placed at the topmost from top to bottom, then how many boxes remain
position. The box which contains U has 42 toys. unchanged in their position?
The box which has a square number of toys a) One
neither contains neither S nor T. The remaining b) None
boxes are having 28 and 72 toys. c) Two
d) Three
e) More than three

Click Here For Bundle PDF Course | support@guidely.in Page 3 of 12


Bank Po Mains PDF Course 2024
Reasoning Day -14 (Eng)

opposite to the one who sits adjacent to H. More


10. Which of the following statements is/are true than one person sits between H and V, who
according to the given arrangement? faces the same direction as M faces. The
I. Box 7 has 52 toys and it contains X number of persons sitting between V and R is
II. Box 6 has odd number of toys and it is two two more than the number of persons sitting
boxes below the box which has number of toys between S and N. R does not sit adjacent to N.
that is multiple of 11 The one who sits opposite to R sits three places
III. The number of boxes above V is two less away from K. More than four persons sit between
than the number of boxes below P L and P. P sits opposite to E, who sits adjacent
a) Only I and III to K. Only two persons sit between F and T, who
b) All are true faces H. The number of persons sitting to the left
c) Only II and III of E is one less than the number of persons
d) None is true sitting to the right of J, who faces Q. D and W sit
e) Only I and II in the same row but face the opposite direction.
D sits fifth from one of the extreme ends.
Directions (11-15): Study the following 11. Which of the following person sits fourth from
information carefully and answer the given the west end of each row according to the final
questions. arrangement?
Twenty persons from D to W are sitting in three I. The one who faces H
parallel rows. There are three rows i.e., row1, II. The one who sits immediate left of N
row2, and row3 such that row2 is in the north of III. The one who sits diagonally opposite to R
row3 and row1 is in the north of row 2. Five IV. The one who sits fifth to the left of H
persons are sitting in row 1 and facing south a) Only I and II
whereas five persons are sitting in row 3 and b) Only II and III
facing north. Ten persons are sitting in row 2 in c) Only I, II and IV
such a way that five persons from the east in row d) Only IV
2 face north towards the persons in row 1 and e) Only I and IV
five persons from the west in row 2 face south
towards the persons in row 3. 12. How many persons are facing north whose
M sits third from one of the extreme ends. S, who name starts with a vowel?
faces south, sits second to the right of the one a) Two
who faces M. I, who sits diagonally opposite to S, b) Three
sits fourth to the right of O. As many persons sit c) Four
to the left of O as to the left of G, who sits d) None

Click Here For Bundle PDF Course | support@guidely.in Page 4 of 12


Bank Po Mains PDF Course 2024
Reasoning Day -14 (Eng)

e) One “X # Y (45)” means “X is 33m north of Y”


“X $ Y (14)” means “X is 22m south of Y”
13. If U is related to D and T is related to V in a “X & Y (37)” means “X is 24m west of Y”
certain way, then who among the following “X @ Y (21)” means “X is 26m east Y”
person is related to G? Statement: J $ L(12); K @ L(15); I # M(56); M $
a) V K(17); O # Q(32); Q @ K(16); O & P(35); N & J
b) The person who sits extreme right end in Row (30)
2 16. What is the direction of I with respect to O
c) The person who sits second left of the one and what is the shortest distance between K and
who faces T O?
d) D a) West, 29m
e) Q b) Northwest, 28m
c) Southwest, 28m
14. Four of the following five are alike in a certain d) Northwest, 29m
way based on the given arrangement and thus e) Southwest, 29m
form a group. Which one of the following does
not belong to that group? 17. Match the following list I and list II according
a) K, the one who sits extreme left end in Row 1 to the final arrangement:
b) S, the one who sits third to the left of W
c) O, Q
d) F, the one who faces H
e) R, the one who sits second to the right of N

15. As many persons sit to the ____ of Q as to


a) A2, B3, C1, D4
the left of ____.
b) A3, B2, C4, D1
a) Right, E
c) A1, B4, C2, D3
b) Left, the one who faces P
d) A3, B1, C4, D2
c) Right, the one who sits immediate right of D
e) A4, B2, C3, D1
d) Left, K
e) Left, V
18. If R $ K (12), then what is the direction of R
with respect to N and what is the distance
Directions (16-20): Study the following
between R and N?
information carefully and answer the given
a) Southeast, 37m
questions.

Click Here For Bundle PDF Course | support@guidely.in Page 5 of 12


Bank Po Mains PDF Course 2024
Reasoning Day -14 (Eng)

b) Southeast, 20m c) Southeast


c) East, 20m d) Southwest
d) East, 37m e) South
e) Cannot be determined
20. How many points are to the southwest of O?
19. If the letters are changed to the next letter as a) Three
per the English alphabetical series, then what is b) Five
the direction of P with respect to L? c) More than five
a) Northeast d) Four
b) Northwest e) Two
Click Here to Get the Detailed Video Solution for the above given Questions
Or Scan the QR Code to Get the Detailed Video Solutions

Answer Key with Explanation


1. Answer: D 2. Answer: C

Click Here For Bundle PDF Course | support@guidely.in Page 6 of 12


Bank Po Mains PDF Course 2024
Reasoning Day -14 (Eng)

7. Answer: C
3. Answer: C 8. Answer: A
9. Answer: D
10. Answer: E
Final arrangement:

We have,
 Only four boxes are placed between the
box which has 66 toys and V. The box
4. Answer: B
which has 66 toys is not placed at the
topmost position.
Applying the above condition, there are three
possibilities.

5. Answer: E
None Follows

Directions (6-10):
6. Answer: B

Click Here For Bundle PDF Course | support@guidely.in Page 7 of 12


Bank Po Mains PDF Course 2024
Reasoning Day -14 (Eng)

Again we have, Again we have,


 The box which has 35 toys which is  The box which has 50 toys contains Y,
placed above V and three boxes below which is placed below W.
W.  The number of boxes placed between Y
 As many boxes placed below the box and the box which has 52 toys is two
which has 49 toys as above X, which is more than the number of boxes placed
not placed at the bottommost position. between the boxes which has 37 toys and
Applying the above conditions, Case 3 is T.
eliminated because there is no possibility to Applying the above conditions, there is one extra
place the box which has 35 toys have to possibility.
place above V.

Click Here For Bundle PDF Course | support@guidely.in Page 8 of 12


Bank Po Mains PDF Course 2024
Reasoning Day -14 (Eng)

Again we have,
 R is placed immediately below the box
which has 40 toys.
 More than four boxes are placed between
R and P, which is not placed at the
topmost position.
Applying the above conditions, there is one extra
possibility.

Again we have,
 The box which contains U has 42 toys.
 The box which has a square number of
toys neither contains S nor T.
 The remaining boxes are having 28 and
72 toys.

Click Here For Bundle PDF Course | support@guidely.in Page 9 of 12


Bank Po Mains PDF Course 2024
Reasoning Day -14 (Eng)

Applying the above conditions, Case 1, Case Directions (11-15):


2 and Cases 2a(1) are eliminated because 11. Answer: C
there is no possibility to place the box which 12. Answer: B
has 28 and 72 toys. Hence, Case 2a gives a 13. Answer: C
final arrangement. 14. Answer: E
15. Answer: D
Final arrangement:

We have,
 M sits third from one of the extreme ends.
 S, who faces south, sits second to the
right of the one who faces M.
 I, who sits diagonally opposite to S, sits
fourth to the right of O.
 As many persons sit to the left of O as to
the left of G, who sits opposite to the one
who sits adjacent to H.
Applying the above conditions, there are two
possibilities.

Click Here For Bundle PDF Course | support@guidely.in Page 10 of 12


Bank Po Mains PDF Course 2024
Reasoning Day -14 (Eng)

Again we have,
Again we have,
 More than four persons sit between L and
 More than one person sits between H and
P.
V, who faces the same direction as M
 P sits opposite to E, who sits adjacent to
faces.
K.
 The number of persons sitting between V
 Only two persons sit between F and T,
and R is two more than the number of
who faces H.
persons sitting between S and N.
 R does not sit adjacent to N.
 The one who sits opposite to R sits three
places away from K.

Click Here For Bundle PDF Course | support@guidely.in Page 11 of 12


Bank Po Mains PDF Course 2024
Reasoning Day -14 (Eng)

Applying the above conditions, Case 1 is Y (14) + 8 = 22


eliminated because there is no possibility to  “X $ Y (14)” means “X is 22m south of Y”
place P and L. Y (37) -13 = 24
Again we have,  “X & Y (37)” means “X is 24m west of Y”
 The number of persons sitting to the left Y (21) +5 = 26
of E is one less than the number of  “X @ Y (21)” means “X is 26m east Y”
persons sitting to the right of J, who faces From the above statement, we have:
Q.  J $ L (12); J is (12 +8) = 20m south of L
 D and W sit in the same row but face the  K @ L (15); K is (15 +5) = 20m east of L
opposite direction.  I # M (56); I is (56 -12) = 44m north of M
 D sits fifth from one of the extreme ends.  M $ K (17); M is (17 +8) = 25m south of K
Applying the above conditions, Case 2 gives a  K $& O; K is southwest of O
final arrangement.  O # Q (32); O is (32 -12) = 20m north of Q
 Q @ K(16);Q is (16 +5) =21m east of K
 O & P (35); O is (35 –13) = 22m west of P
 N & J (30); N is (30 –13) = 17m west of J

Directions (16-20):
16. Answer: E
17. Answer: B
18. Answer: D
19. Answer: A
20. Answer: C
Y (45) – 12 = 33
 “X # Y (45)” means “X is 33m north of Y”

Click Here For Bundle PDF Course | support@guidely.in Page 12 of 12


Bank Po Mains PDF Course 2024
Quantitative Aptitude Day -14 (Eng)

Quantitative Aptitude

Directions (01 - 04): Study the following information carefully and answer the questions given below.
The given table shows the total surface area and curved surface area of
cylinder, cone, sphere, cube and cuboid. [Take approximate value always]

1) The ratio of the radius of given cylinder and 3) The radius of the cone is ______ more than
another cylinder is 4:3 and the ratio of the height the radius of the sphere and the volume of the
of the given cylinder and another cylinder is 3:5. sphere is ____ more than the volume of the cone.
Find the difference between the volume of the Find which of the following value satisfy the
two cylinder? blank?
a) 3582 m3 a) 8 m, 2540 m3
b) 3642 m3 b) 7 m, 2259 m3
c) 3234 m3 c) 9 m, 2520 m3
d) 3842 m3 d) 11 m, 2549 m3
e) 3684 m3 e) None of these

2) Side of a square is 20% more than the side of 4) The Ratio of the radius of the sphere and the
a cube and the length of a rectangle is the radius of the circle is 2:3. The Area of the circle
average of the length and height of a cuboid and is L m2more than the total surface area of the
the width of the rectangle is 25% more than the cube. The Base of the triangle is √L m and the
breadth of the cuboid. Find the sum of the area height of the triangle is same as the radius of the
of the square and rectangle? [In cuboid , length > sphere. Find the area of the triangle?
height > breadth and length – breadth is 20] a) 42 m2
a) 445 m2 b) 58 m2
b) 645 m2 c) 64 m2
c) 654 m2 d) 48 m2
d) 554 m2 e) None the of these
e) 664 m2

Click Here For Bundle PDF Course | support@guidely.in Page 1 of 11


Bank Po Mains PDF Course 2024
Quantitative Aptitude Day -14 (Eng)

5) Total number of populations in village A is II.4800,35,4230


_______. The Ratio of the male and female III.6400,24,8776
population in village A is 5:3. The number of Find which of the is satisfy the blank?
male populations increased by 20% and the a) both I and II
number of female populations increased by b) only I
____% so that the total population of the village c) only II
now is _____. d) only III
I.3200,25,3900 e) None of these

Directions (06 - 09): Study the following information carefully and answer the questions given below.
The given table shows the speed (km/hr), length (m), time taken to cross a post and time taken to cross
platform P of five different trains [A, B, C, D and E].

6) Length of platform P is 25% less than the cross each other in 43.5 sec when moving in the
length of platform Q. Time taken by train C to same direction. Find which of the following is
cross platform Q is t sec. In (t+21) sec train A true?
cross platform S. Find the length of platform S? a) Speed of train F = Speed of train D
a) 445 b) Speed of train F/2 > speed of train D
b) 475 c) Speed of train F > 1.5*speed of train D
c) 460 d) Speed of train F< speed of train D
d) 485 e) Speed of train F> speed of train D
e) 560
8) Train A crosses platform R in ______ sec and
7) The Ratio of length of train D and F is 9:7. train B crosses platform T in ______ sec. Sum of
Train D and F cross each other when moving in length of platform R and platform T is _____.
same direction is 80 sec. Train F and train E Find which of the following is true?

Click Here For Bundle PDF Course | support@guidely.in Page 2 of 11


Bank Po Mains PDF Course 2024
Quantitative Aptitude Day -14 (Eng)

I.20,16, 310 c) only II


II. 25, 18, 425 d) only III
III. 32, 30, 485 e) None of these
a) only I and II
b) only I 10) The ratio of the speed of boat A and boat B
c) only II and III in still water is 5:6. Downstream speed of boat A
d) only III in river P is 30 km/hr and the upstream speed of
e) None of these boat A in the same river is ______. Boat B covers
_____ km in 10 hours in downstream and _____
9) Train B cross platform P in m sec and train C km in 7 hours in upstream.
crosses double the length of platform P in n sec. I. 20, 350, 175
Find the possible time taken by train E and D to II. 10,170, 125
cross each other when moving in same III. 18, 180, 120
direction? Find which of the following is satisfy?
I.5n+7 a) only I
II.2[m+n]+13 b) only II
III.5m+11 c) only III
a) both I and III d) all true
b) only I and II e) None of these

Directions (11 - 14): Study the following information carefully and answer the questions given below.
The pie chart shows the percentage distribution of the number of seminars [software + hardware]
organized by five different colleges [A, B, C, D and E] in 2022.

Click Here For Bundle PDF Course | support@guidely.in Page 3 of 11


Bank Po Mains PDF Course 2024
Quantitative Aptitude Day -14 (Eng)

Note – I. 2P+3Q+9R=58, II. 5P+4Q+7R=57, III.8P+9Q+4R=63 IV. S=2P. Total number of seminars
organized in five colleges is 300. The Ratio of software and hardware seminars organized in each college
is 2:1.
11) Total number of software seminars organized number of seminars organized by males and the
by colleges A, B and C is ______ and the total total number of seminars organized by female
number of hardware seminars organized by employees.
colleges C, D and E is ______. The Sum of two a) Q+R
blanks is _______. b) R+S
Find which of the following is true? c) P+Q
a) 14S, 30S, 190 d) P+S
b) 11R, 13Q, 200 e) P+R
c) 24R, 35P, 190
d) 28P, 38Q, 200 13) The ratio of the number of software seminars
e) none of these organized by E and F is 2:3, and the ratio of the
number of hardware seminars organized by D
12) Out of the total software seminars organized and F is 5:7. Find the possible value of the total
by College C, 40% were hosted by males and number of seminars organized by F?
the rest were hosted by females. Out of the total I.30R
hardware seminars organized by College C, 60% II.50S
were hosted by males and the rest were hosted III.75P
by females. Find the difference between the total a) all false

Click Here For Bundle PDF Course | support@guidely.in Page 4 of 11


Bank Po Mains PDF Course 2024
Quantitative Aptitude Day -14 (Eng)

b) all true The cost price of item A is Rs. 6X. The ratio of
c) only I cost price of items A and B is 3:4. The selling
d) only II price of item A is Rs. 72 more than the selling
e) None of these price of item E when both items are sold at 20%
profit. The ratio of the selling price of items A and
14) In 2023, college A wasincreased software C when item A is sold at 25% profit and item C is
seminars by 5.55% and decrease hardware sold at 35% profit is 25:42. Cost price of item C
seminars by 5.55%. In the same year, college B is Rs. 7Y. The cost price of item D is [6Z+20].
increased both hardware and software seminars The cost price of item E is 120. The cost price of
by 8.33%. Find the total number of seminars item D is Rs. 40 more than the cost price of item
organised by colleges A and B in 2023. C.
a) 94 16) Find which one is true?
b) 92 I. When item A is sold at Rs.6Y then the profit
c) 93 percentage is 20%.
d) 91 II. When item B is sold at 9X then the profit
e) None of these percentage is 12.5%.
III. when item C is sold at 6Z then the profit
15) Sum of the present age of A and B is _____ percentage is 10%.
years. The Ratio of age A after 3 years and age a) only III
of B after 2 years is ______ . The Age of C is b) only I
20% more than the A and the age of D is 20% c) only II
more than B. The Sum of Age of C and D is d) all three
______. e) only II and III
Find which of the following is satisfy the blank?
a) 85, 4:5, 92 17) When item C is sold at 25% profit. Find
b) 75, 7:8, 89 possible selling price of item C?
c) 45, 5:4, 64 I.7Z
d) 55, 7:8, 66 II.10Y-50
e) None of these III.12X-10
a) only I true
Directions (16 - 17): Study the following b) only II true
information carefully and answer the questions c) all false
given below. d) all true
e) only III

Click Here For Bundle PDF Course | support@guidely.in Page 5 of 11


Bank Po Mains PDF Course 2024
Quantitative Aptitude Day -14 (Eng)

Directions (18-20): Study the following e) None of these


information carefully and answer the questions 19) The difference between the number of
given below. laptops and phones sold in A is m; the difference
The ratio of the number of laptops sold in shops between the number of phones sold in B and D
A and B is 7:8. The ratio of the number of laptops is n; and the difference between the number of
and phones sold in Shop C is 7:8. The number of laptops sold in B and D is p. Find the value of
phones sold in shop B is double the number of mn+mp.
phones sold in A. The number of phones sold in a) 159
A is 20% less than the number of laptops sold in b) 147
B. The number of laptops sold in D is 28.57% c) 158
more than the number of laptops sold in C. The d) 185
ratio of the number of laptops sold in B to the e) None of these
number of phones sold in D is 8:5. The total
number of items sold in D is 79, and the total 20) The ratio of 5G and 4G phones sold in A is
number of phones sold in B and C together is 5:3. The ratio of 5G and 4G phones sold in B is
112. 3:5. The ratio of 5G and 4G phones sold in C is
18) The ratio of the number of laptops sold in D 2:1. The total number of 5G phones sold in A, B,
and E is 9:11, and the ratio of the number of and C together is (approx.)what percent of the
phones sold in C and E is 8:9. Find the total number of 4G phones sold in A, B, and C
difference between the total number of items together?
sold in E and B. a) 108%
a) 7 b) 112%
b) 5 c) 115%
c) 6 d) 117%
d) 9 e) None of these

Click Here For Bundle PDF Course | support@guidely.in Page 6 of 11


Bank Po Mains PDF Course 2024
Quantitative Aptitude Day -14 (Eng)

Click Here to Get the Detailed Video Solution for the above given Questions
Or Scan the QR Code to Get the Detailed Video Solutions

Answer Key with Explanation


1) Answer: C = [22/7] *28*28*21 – [22/7] *35*21*21
The radius of the cylinder and height are r and h = 51744-48510=3234 m3
respectively.
So, 2*[22/7]*r*h=3696 –(I) and 2) Answer: D
2*[22/7]*r*[r+h]=8624 ----(II) The side of the cube is a.
By divide (II) from I The total surface area of the cube is 6a2= 1350.
So, [r+h]/h=8624/3696 So, a2=1350/6=225
Or, r+h=2.33h So, a=15
Or, r=1.33h The total surface area of a cuboid is 2*[lb+bh+lh]
So, from I = 1744.
1.33h2=3696*7/(22*2)=588 And the curved surface area of a cuboid is
Or, h2=588/1.33=441 2h*[l+b]=1296.
Or, h=+21,-21 Where l, b, and h are the length, breadth, and
So, h=21 height of the cuboid, respectively. [l>h>b]
So, r=21*1.33=27.9=28 2lb=1744-1296=448, lb=224.
Another method: And l=20+b
2πr(h+r) - 2πrh = 8624 – 3696 Or, b*[b+20]=224
2πr2 = 4928 Or, b2+20b-224=0
r = 28 Or, b2+28b-8b-224=0
The radius of another cylinder is = 28*3/4=21 Or, (b+28)(b-8)=0, b=-28, b=8.
Height of another cylinder is = 21*5/3=35 So, the breadth is 8.
So, the difference in volume of two cylinder Length is 20+ 8 = 28.

Click Here For Bundle PDF Course | support@guidely.in Page 7 of 11


Bank Po Mains PDF Course 2024
Quantitative Aptitude Day -14 (Eng)

Height is = [1296/(36*2)] = 18. The total surface area of the sphere is =


So, the side of the square is = 15*120/100=18. 4*22/7*r2=2464
Rectangle length = [28+18]/2=46/2=23 Or, r2= [2464*7/(22*4)]=196, r=14
The width of the rectangle is equal to So, The radius of the circle is = 14*3/2=21
8*125/100=10. Area of circle = 22/7* 21*21=1386
So, the sum of the area is So, L=1386-1350=36
18*18+23*10=324+230=554 sq m. So, The base of the triangle is √36 =6
The Height of the triangle is = 14
3) Answer: B So, area = ½ * 6 *14=42 sq m
For sphere
The total surface area of the sphere is = 5) Answer: B
4*22/7*r2=2464 For value I,
Or, r2= [2464*7/(22*4)]=196, r=14 Male in village A is = 3200*5/8=2000
For cone, Female in village A is = 3200-2000=1200
Total surface area is = 22/7 * r *[r+l]=3300 Total population now = 2000*120/100 +
Curved surface area is = 22/7 * r *l=1914 1200*125/100=3900
So, (l+r)/l=3300/1914 Similarly, we can check others value also and
Or, l+r=1.7241l we see only I is satisfy.
Or, r=0.7241l
So, r*l=1914*7/22=609 Directions (06 - 09):
Or, l*[0.7241l]=609 The speed of train E is 72 km/hr = 72* 5/18 = 20
Or, l2=609/0.7241=841, l=29 m/sec.
So, r= [1914*7/[22*29]=21 Let the length of platform P be l.
So, h2=29*29-21*21=400, h=20 So, [l+260]/20=22
So, the radius of cone is 21-14= 7 more than Or, l=440-260=180 m.
radius of sphere Now, the speed of train A is 72 km/hr = 72* 5/18
Volume of sphere is = = 20 m/sec.
22/7*[14*14*14]*4/3=11498.66=11499 Let the length of train A be m.
Volume of cone is = 1/3 * 22/7* 21*21*20=9240 So, (m+180)/20=21
So, difference = 11499-9240=2259 Or, m=420-180=240 m.
The speed of train B is 250/10=25 m/sec.
4) Answer: A The speed of train C is 108 * 5/18 = 30 m/sec.
For sphere Length of train C is = 30*6=180 m

Click Here For Bundle PDF Course | support@guidely.in Page 8 of 11


Bank Po Mains PDF Course 2024
Quantitative Aptitude Day -14 (Eng)

The speed of train D is = [225+ 180]/27 = 15 =97=5n+7=5m+11


m/sec.
10) Answer: A
6) Answer: C For value of I,
Length of platform Q is = 180*100/75=240 The speed of boat A is [30+20]/2=25km/hr
So, t= [240+180]/30=420/30 =14 sec So, the speed of boat B is = 25*6/5=30km/hr
So, In [14+21]=35 sec train A cross = 35*20=700 The speed of the stream is 30-25=5km/hr
m So, boat B cover [30+5]*10=350 km in 10 hours
So, the length of platform S is = 700-240=460 m in downstream
Boat B cover [30-5]*7=175 km in 7 hours in
7) Answer: D upstream.
Length of train F is = 225*7/9=175 Similarly, we can check others' values also.
Let speed of train F is s. We can see only I is satisfying.
So, [175+225]/[15 ~ s]=80
So, [15~s]=400/80=5 Directions (11 - 14):
So, s= 20 or 10 I. 2P+3Q+9R=58, II. 5P+4Q+7R=57,
Now, [260+175]/(20-20) III.8P+9Q+4R=63
Or, [260+175]/(20-10)=43.50 By solving above equation, we get, P=2, Q=3,
So, speed of train F is 10 m/sec. R=5.
So, S=2*2=4.
8) Answer: B The Percentage distribution of number of
Let check option I, seminars in A, B, C, D and E is 6*3=18%,
Length of platform R, 20*20-240=160 = 12% , (2*3*5)=30%,
Length of platform T, 16*25-250=150 (2+3)*5=25%, 3*5=15% respectively.
So, sum is 160+150=310 The Number of seminars organized by A is =
Similarly, we can check other values also. 300*18/100=54
So, only I is satisfy. The Number of software seminars organized by
A is = 54*2/3=36
9) Answer: A THe Number of hardware seminars organized by
So, m = [180+250]/25=17.2 A is = 54-36=18
So, n=[180*2 +180]/30=18
So, train E and D cross each other in
[225+260]/[20-15]

Click Here For Bundle PDF Course | support@guidely.in Page 9 of 11


Bank Po Mains PDF Course 2024
Quantitative Aptitude Day -14 (Eng)

=24*108.33/100 + 12*108.33/100=39
So, the required sum is = 55+39=94

15) Answer: D
Let's check value of d,
Let the age of A after 3 years and age of B after
11) Answer: C 2 years is 7x and 8x.
The total number of software seminars So, 7x-3+8x-2=55
organized by colleges A, B and C is Or, 15x=55+5=60
= [36+24+60] =120 Or, x=60/15=4
The Total number of hardware seminars Age of A is =28-3 = 25 years
organized by colleges C, D and E is Age of B is = 32-2=30 years
= [30+25+15] =70 So, sum of C and D is = 25*120/100 +
So, the sum is 120+70=190 30*120/100=30+36=66
So, it is satisfied.
12) Answer: D
The total number of seminars organized by Directions (16 - 17):
males is The Cost price of item E is Rs.120.
= 60*40/100+30*60/100=24+18=42 The Selling price of item E when sold at 20%
The total number of seminars organized by profit is = 120*120/100=144
females is The Selling price of item A when sold at 20%
= 60*60/100+30*40/100=36+12=48 profit is = 144+72=216
So, required difference = [48-42]=6 So, the cost price of item A is =
216*100/120=180=6X, X=30
13) Answer: A So, the cost price of item B is = 180*4/3=240
The number of seminars organized by F is So, the selling price of item C when sold at 35%
= [30*3/2]+[25*7/5]=45+35=80is not equal to profit is
30R, 50S, 75P = [180*125/100]*42/25 =225*42/25=378
So, the cost price of item C is =
14) Answer: A 378*100/135=280=7Y, Y=40
Total number of seminars organised by A Cost price of D = 280+40=320=6Z+20,
=36*105.55/100 + 18*[100-5.55]/100=38+17=55 Z=300/6=50
Total number of seminars organized by B 16) Answer: C

Click Here For Bundle PDF Course | support@guidely.in Page 10 of 11


Bank Po Mains PDF Course 2024
Quantitative Aptitude Day -14 (Eng)

I. When item A sold at Rs.6Y then the profit The number of laptops sold in A and B is 7*5=35
percentage is 20%. and 8*5=40.
So, [240-180]*100/180=33.33% The number of phones sold in A is =6.4*5=32.
II. When item B sold at 9X then the profit The number of phones sold in B is =12.8*5=64.
percentage is 12.5%. Let the number of laptops and phones sold in C
[270-240]*100/240=12.5% be 7*6 = 42 and 8*6 = 48.
III. when item C is sold at 6Z then the profit The number of laptops sold in D is 9*6=54.
percentage is 10%. The number of phones sold in D is 5*5=25.
[300-280]*100/280=7.14% 18) Answer: E
The total number of items sold in E is 54*11/9 +
17) Answer: D 48*9/8 = 66+54 = 120.
So, required selling price = 280*125/100=350 So, the difference is = 120 – 40 – 64 = 16.
=12X-10=10Y-50=7Z
19) Answer: A
Directions (18-20): So, m=35-32=3.
Let the number of laptops sold in A and B be 7x So, n=64-25=39
and 8x. So, p=54-40=14
The number of phones sold in A is So, m*n + m*p = 3*39 + 3*14 = 159.
8x*80/100=6.4x.
The number of phones sold in B is 6.4x*2=12.8x. 20) Answer: B
The number of laptops and phones sold in C is The total number of 5G phones sold is 32*5/8 +
7y and 8y. 64*3/8 + 48*2/3.
The number of laptops sold in D is 7y*9/7=9y. = 20+24+32=76
The number of phones sold in D is 8x*5/8=5x. The total number of 4G phones sold is =
So, 9y+5x=79 and 12.8x+8y=112. [32+64+48] – 76 = 68.
By solving the above equation, we get x = 5 and So, the required percentage is [76/68]*100 =
y = 6. 112%.

Click Here For Bundle PDF Course | support@guidely.in Page 11 of 11


Bank Po Mains PDF Course 2024
English Day - 14

English Language
Directions (1-5): The given questions carry two 3.
sentences in which three words each have been Experts (A) noted that the dam, about 70
highlighted. These words might not be in their kilometres (44 miles) to the east of the city of
correct positions. You must choose an option Kherson, was (B) believed to be in disrepair and
which carries the correct pairs of words that (C) vulnerable to collapse
when interchanged would form correct and Besides a vigilant (D) approach like being
meaningful sentences. mindful of unusual odours, discolouration, or (E)
1. changes in food texture, cutting-edge technology
High global oil prices have (A) measure can play a pivotal role in (F) ensuring food
consequences for India that (B) imports 85% of safety.
its (C)transactions. A. A-D, B-F
The current (D) account—which is the B. B-E, A-D, C-F
broadest (E) seriousof India’s goods and C. A-E, C-D, B-F
services (F) requirements—slipped into a deficit D. B-D, A-C
A. A-E, B-D E. No swapping needed
B. B-D, C-F
C. A-E, C-F 4.
D. A-E, B-D, C-F Govt can’t do the (A) heavy lifting on recovery
E. No swapping needed alone, but the (B)transactionsfor business (C)
needs to get easier.
2. The current (D) account—which is the
The external affairs minister also (A)dangerthat broadest (E) measure of India’s goods and
India is making significant economic (B) impact services (F) environment.
that has been recognised (C)rang. A. A-D
Amid the disaster response, artillery shelling B. C-E
(D) globallyout as people scrambled to get out of C. B-F
the (E) saidzone, climbing onto military (F) trucks D. B-E
or rafts. E. No swapping needed
A. B-D, A-E, C-F
B. A-F, C-E 5.
C. C-D, B-F Punjab and Haryana farmers are (A) worsento
D. A-E, C-D paddy and a (B) importscycle that makes stubble
E. No swapping needed (C) hookedan imperative.

Click Here For Bundle PDF Course | support@guidely.in Page 1 of 12


Bank Po Mains PDF Course 2024
English Day - 14

High global oil prices have serious B. IV, III, I, I, II.


consequences for India that (D) cultivation85% of C. III, V, IV, II, I.
its requirements and will (E)burningthe current D. I, II, III, IV, V.
account (F) situation in FY22. E. None of the combination is correct.
A. A-E, B-D, C-F
B. Only A-E 7. One of the achievements of the NarendraModi
C. B-D, C-E government is the network of excellent highways
D. A-F, B-D, C-E across the country, which____________ matching
E. No swapping needed driving skills, especially as car manufacturersare
developing more powerful engines and foreign
Directions (6-10): In the following questions, a car makersare bringing their best wares. That
passage has been given with multiple blanks. can make for a deadly combination.And drivers
You are required to choose the appropriate word must _____________ that with great speedcomes
for each blank from the given options. great responsibility. The ______________ with
6. Science has been a powerful ____________for strapping onseatbelts is of recent origin. While
humanity’s progress. It has become those in the front seats do itwilly-nilly, backseat
the____________ of human follies and also passengers ______________ this safety measure.
theremedial tool that rescues us from Untilthe accident which claimed billionaire Cyrus
theirconsequences. Anti-microbial resistancehas Mistry in September 2022 on the Mumbai
become a major _____________ to human Ahmedabad Expressway, the seatbeltdid not get
survivaland well-being, as several multi- its due importance. Neither was it widely
drugresistant microbes have ____________ knownthat airbags do not ___________ if the
throughantibiotic misuse. The prospect of seatbelt is not strapped on.
findingpotent novel antibiotics ____________ dim I. Compliance
tillit was announced recently that II. Deploy
artificialintelligence (AI) helped to identify a III. Require
newantibiotic (again) that is highly IV. Remember
effectiveagainst the pathogenic bacterium. V. Ignore
I. Appeared A. III, IV, I, V, II.
II. Emerged B. V, II, I, IV, III.
III. Propellant C. IV, III, I, II, V.
IV. Threat D. II, III, V, IV, I.
V. Instrument E. None of the combination is correct.
A. V, IV, I, I, II.

Click Here For Bundle PDF Course | support@guidely.in Page 2 of 12


Bank Po Mains PDF Course 2024
English Day - 14

8. Indian education system_____________ the but also_______________ to the overallwell-being


required flexibilityover the years, which is of communities.By being _______________ in
verymuch required to take it tothe next level and foodsafety practices, one can significantlyreduce
make it truly global in approach andpracticality. the _________________of foodborne illnessesand
In Budget 23,the government has focusedon the their associated economicand social burdens.
_______________ of educationand skills I. Fatal
according tothe aptitude of the youthand the II. Paramount
demands of thefuture.It is for the first time thatan III. Contribute
equal ______________ is beinggiven to both IV. Proactive
education andskilling as part of the V. Incidence
neweducational policy, whichunburdens the A. IV, I, III, V, II.
students fromthe ______________ regulations B. II, I, V, III, IV.
andprocesses of the past. Thenew technological C. I, IV, II, V, III.
revolutionin recent years is helpinga lot in the D. III, V, II, I, IV.
process by creatingnew types of classroomswith E. None of the combination is correct.
unrestricted access to all,_______________ the
considerationslike region and distance. 10. It has been known for nearly a decade now
I. Obsolete that to_____________ the cops, drug traffickers
II. Lacks are ______________ to tradingon marketplaces
III. Blurring on the darknet — an Internet platformthat uses
IV. Emphasis secret alleys on The Onion Router (ToR),
V. Reorientation provides________________ to the traffickers and
A. IV, II, III, V, I. their customersand is, thus, difficult to detect and
B. II, V, III, I, IV. ____________. Theirmodus operandi involves
C. III, IV, V, III, I. clandestinely _____________ customerson social
D. II, V, IV, I, III. media, dealing in cryptocurrency andusing
E. None of the combination is correct. courier services for delivery.
I. Luring
9. Contaminated foodscan cause a__________ II. Anonymity
outcome.From posing a risk to life and III. Resorting
causingsevere permanent damage tothe body, it IV. Evade
is of _____________importance to identify V. Penetrate
foodhazards and ensure 360-degree food safety. A. V, III, II, IV, I.
Suchmeasures not only protectindividual health B. IV, III, II, V, I.

Click Here For Bundle PDF Course | support@guidely.in Page 3 of 12


Bank Po Mains PDF Course 2024
English Day - 14

C. IV, I, II, IV, III. could be transformative for energy-intensive


D. II, V, III, I, IV. sectors in India, starting with renewable power
E. None of the combination is correct. expansion to 450 GW. (I) With open source
technologies, India (1)/could incorporate
Directions (11-17): Read the following passage innovative materials and (2)/processes to
and answer the following questions. Some words decarbonise industry, transport and buildings,
are given in bold to help you answer some of the (3)/the biggest emitters, apart to power.(4)
questions. Many developed countries tend to view India’s
The U.S.-India Climate and Clean Energy reluctance to commit to a net zero emissions
Agenda 2030 Partnership raises expectations target as recalcitrance, but the climate change
that the coming decade will see sustained crisis originated not here but in the industrialised
financial and technological cooperation between world, which has used up much of the world’s
the two countries to cut greenhouse gas carbon space. A forward-looking policy should,
emissions. At the Leaders Summit on Climate therefore, envision green development anew,
organised by U.S. President Joe Biden, the providing funding and green technologies as
world’s attention was focused on countries compensation for the emissions space lost by
responsible for the highest carbon emissions. poorer countries.
India ranks third, behind the U.S. and China, This is a win-win game, since it would aid
although its per capita CO2 emissions are less sustainable development, boost employment,
than 60% of the global average, as Prime clean up the environment and, crucially, help all
Minister Narendra Modi pointed out. There is little countries emerge ……………. (II) from the
confidence in a pandemic-stricken world, pandemic. British Prime Minister Boris Johnson,
however, that future growth pathways will be who announced enhanced ambition at the
aligned away from fossil fuels. summit for Britain to cut carbon emissions by
The International Energy Agency, in fact, expects 78% by 2035 over 1990 levels, advanced the
a dramatic rise in emissions as countries race to agenda by calling for climate funding by rich
shake off the impact of the coronavirus, as they nations to exceed the decade-old goal of $100
did after the 2008 financial crisis. Yet, the years billion. For the India-U.S. agreement to yield
to 2030, as President Biden put it, are part of a results, Mr. Biden would have to persuade
“decisive decade”, and action to scale up funding industry and research institutions at home to
and innovation can help all countries move closer share knowledge and subsidise transfer of
to keeping global warming well below 2°C or technologies. He has won commendations for
even 1.5°C, as the Paris Agreement envisages. steering America around from the science-
There are many aspects to the bilateral pact that deprived Trump years and announcing enhanced

Click Here For Bundle PDF Course | support@guidely.in Page 4 of 12


Bank Po Mains PDF Course 2024
English Day - 14

ambition: cuts in emissions by 50% to 52% by 13. Which of the following statements correctly
2030 over 2005 levels. But much of his climate mention the conflict of ideas between Internal
effort will rely on executive authority, rather than Energy Agency and President Biden?
bipartisan support. With political will on both (I) Drastic rise in emissions is expected in
sides, the engagement with India can become a countries amid shaking off the pandemic impact
model. (II) The Paris Agreement envisages climate
11. According to the author, the U.S.-India actions taken by the countries at individual level
Climate and Clean Energy Agenda 2030 (III) The years till 2030 will be a decisive decade
Partnership arose what expectations? with bringing down the global warming through
(I) The partnership will witness financial innovation and scaled-up funding
cooperation between the two nations A. Only I
(II) It will boost the technological collaboration B. Only III
between US and India C. Both I and II
(III) The partnership will lead to collaborative D. Both I and III
climate actions between both the countries E. All of I, II, and III
A. Only I
B. Only III 14. What steps should be taken to yield the
C. Both I and II results from the India-US agreement?
D. Both II and III (I) Persuading industry and research facilities to
E. All of I, II, and III share knowledge
(II) Political will from both the countries
12. Which of the following statements can be (III) Climate effort of Biden will rely on executive
inferred from the first paragraph? authority and not on bipartisan support
A. The Leaders Summit on Climate aimed at the (IV) Subsidization of technologies’ transfer will
nations having the highest carbon emissions have to be encouraged
B. The rank of India is 4th followed by China and A. Both I and II
US B. Both II and III
C. The per capita CO2 emission of India is less C. Both I and IV
than 60% D. Both II and IV
D. It’s unlikely to align away the growth from E. All of the above
fossil fuels 15. Which of the following is the most opposite
E. Both A and C in meaning to the word ‘RECALCITRANCE’ as
highlighted in the given passage?
A. Resistant

Click Here For Bundle PDF Course | support@guidely.in Page 5 of 12


Bank Po Mains PDF Course 2024
English Day - 14

B. Obstinate B. Meticulous, preferring


C. Intransigent C. Careful, excluding
D. Cooperative D. Lax, including
E. Non-Compliant E. Careless, counted as

16. Which of the following words can fill in the 19. What ______ nearly 4,000 years ago in the
blank II to make the sentence grammatically court of the Kauravas is still happening because
correct and contextually meaningful? basic human nature has ________ changed.
A. Peddle A. Happened, hardly
B. Hulking B. Came about, not much
C. Healthier C. Materialised, bare
D. Incendiary D. Hit upon, slightly
E. None of these E. None of these

17. The italicized sentence I is divided into four 20. While Assam is the most _________ example,
parts. Find the part which contains an error. If several states in the Northeast have witnessed a
there is no error in any of the parts; mark your significant ________ from across the border.
answer as E. A. Serious, migrants
A. 3 B. Obvious, influx
B. 2 C. Evident, exodus
C. 4 D. Apparent, arrival
D. 1 E. Both (c) and (d)
E. No Error
21. This fact is _________ since about 96% of
Directions (18-22): The questions given below China’s container trade with Europe went
carry sentences in which words from the options ________ sea routes.
given below them can be filled to complete the A. Importance, across
same grammatically and meaningfully. You must B. Venerable, haywire
choose the most appropriate option for each C. Crucial, via
sentence as your answer. D. Paramount, though
18. Our regulators have had a record of ______ E. None of these
and poor oversight over unlisted companies’
_________ startups.
A. Negligible, of

Click Here For Bundle PDF Course | support@guidely.in Page 6 of 12


Bank Po Mains PDF Course 2024
English Day - 14

22. A _________ search operation was set in B. Massive, imploded


motion and hours later the vessel was found C. Tremendous, splintered
_________ with deceased individuals. D. Vast, burnt
A. Evacuation, mended E. Immense, sunk
Click Here to Get the Detailed Video Solution for the above given Questions
Or Scan the QR Code to Get the Detailed Video Solutions

Answer Key with Explanation


1. Answer: C said should come after the external affairs
The word serious should come before minister which makes A-E one correct pair to be
consequences as it acts as a perfect adjective interchanged.
that defines the nature of the consequences. The wordglobally should come after recognised.
This makes A-E one good pair to be swapped. The phrase rang out means to be heard loudly
The options carrying the same are (a), (c) and and clearly… This means that the word in C
(d). should come in D.
The wordrequirements should come in C as 85% This makes option (d) the most logical answer
of the required oil is imports is being mentioned choice.
in the line. This meaning can only be understood
when the words in C and F are interchanged as 3. Answer: E
well. The given words are in their correct positions
This makes option (c) the most logical answer and need no interchange.
choice. This makes option (e) the most logical answer
choice.
2. Answer: D
The words that should be swapped between 4. Answer: C
themselves are given in option (d) as, the word

Click Here For Bundle PDF Course | support@guidely.in Page 7 of 12


Bank Po Mains PDF Course 2024
English Day - 14

The word environment should come in C the other words are grammatically incorrect.
because the environment for business is being Because of the misuse, microbes have emerged.
talked about. Then, the remaining word “appeared” should be
This tells us that the words in B and F need to be filled in the fifth blank.
interchanged. Therefore, option C is the correct answer.
Therefore, the correct answer is option (c).
7. Answer: A
5. Answer: A Here, the passage is about driver’s safety and
The hints: the first sentence is expressed in a positive note.
 Stubble – burning(is a method of So, we cannot use the word “ignore” and the
removing paddy crop residues from the word “compliance” which is a noun, so “require”
field to sow more). will be placed here. The second sentence says
 Hooked to - very interested and that drivers should go in speed with great
enthusiastic about something responsibility, they have to keep this in mind, for
 Cultivation – cycle (one crop being sown that, the word “remember” should be used. Here,
after the other) the word “the” is present which will precede the
These hints tell us that the pairs given in the first noun usually so it will take “compliance” in the
option are correct and (a) should be marked the blank. The fourth statement says backseat
answer. passengers should also use this safety measure.
From this, we can infer “ignore” should be placed
6. Answer: C here. The fifth blank will take “deploy” as the
Propellant – a fuel force or something that answer.
propels. Therefore, option A is the correct answer.
Here, the blank requires a word that describes
the word “science”, because “science” is helping 8. Answer: D
humans to get progress. So, the word The second sentence says something is still
“propellant” should be used. The second needed to move to the next level, so we can
statement indicates that it works as a remedial infer that flexibility is missing in the Indian
tool from overcoming the consequences, so the Education System, so the word “lacks” should be
word “instrument” should be used. The third used in the first blank. In 2023 Budget,
statement says anti-microbial resistance is government has focused to reshape this, so the
becoming something to human survival and well- word “reorientation” should be used. The third
being, here the word “threat” should be filled, as statement says that this is the first time

Click Here For Bundle PDF Course | support@guidely.in Page 8 of 12


Bank Po Mains PDF Course 2024
English Day - 14

government is considering both, so it has given blank as the word “detect” is used. The word
importance, for that the word “emphasis” should “anonymity” should be used in the third blank as
be used. Here, the fourth blank requires a word the word “provides” require a noun to complete
that should describe the word “regulation” an the sentence. How the customers get involved
adjective is needed so the word “obsolete” into it, they are “lured” by social media.
should be used. The word “blurring” should be Therefore, option B is the correct answer.
used in the fifth blank.
Therefore, option D is the correct answer. 11. Answer: C
Refer to the introductory lines of the first
9. Answer: E paragraph, you can draw the hint from the lines,
Here, the blanks should be filled with the The U.S.-India Climate and Clean Energy
answers in the sequence I, II, III, IV and V. Agenda 2030 Partnership raises expectations
Because, eating contaminated food will result to that the coming decade will see sustained
deadliest consequence. For that, the word “fatal” financial and technological cooperation between
would come. To avoid this, we need to give the two countries to cut greenhouse gas
importance to food safety, for that the word emissions.
“paramount” should be used. If the importance is Hence, C is the right answer choice where I and
given to food, it will not only help individuals but II are the only correct statements.
also communities, for that “contribute” would be
correct. The fifth blank requires a noun so 12. Answer: E
“incidence” should be placed and “proactive” Read the lines of first paragraph, you can extract
should be placed at fourth blank. the hint from the below quoted lines,
Therefore, option E is the correct answer. At the Leaders Summit on Climate organised by
U.S. President Joe Biden, the world’s attention
10. Answer: B was focused on countries responsible for the
Luring – tempt. highest carbon emissions. India ranks third,
Anonymity – the condition of being unknown. behind the U.S. and China, although its per
Resorting – The act of turning to a person or capita CO2 emissions are less than 60% of the
thing for help or as a means of achieving global average, as Prime Minister Narendra
something. Modi pointed out. There is little confidence in a
Here, only the first and the fourth blank will have pandemic-stricken world, however, that future
base form verbs, as the word “to” precede. The growth pathways will be aligned away from fossil
word “penetrate” should be placed in the fourth fuels.

Click Here For Bundle PDF Course | support@guidely.in Page 9 of 12


Bank Po Mains PDF Course 2024
English Day - 14

Hence, E is the right answer choice in which A climate effort will rely on executive authority,
and C are the only correct statements and can rather than bipartisan support. With political will
be inferred from the first paragraph of the given on both sides, the engagement with India can
passage. become a model.
Hence, E is the right answer choice where all the
13. Answer: D given statements stand true as per the given
Read the lines of second paragraph, you can information in the passage.
take the hint from the below quoted lines,
The International Energy Agency, in fact, 15. Answer: D
expects a dramatic rise in emissions as Recalcitrance: having an obstinately
countries race to shake off the impact of the uncooperative attitude towards authority or
coronavirus, as they did after the 2008 financial discipline.
crisis. Yet, the years to 2030, as President Biden Antonym: Apologetic, Cooperative
put it, are part of a “decisive decade”, and action Resistant: offering resistance to something or
to scale up funding and innovation can help all someone.
countries move closer to keeping global warming Obstinate: stubbornly refusing to change one's
well below 2°C or even 1.5°C, as the Paris opinion or chosen course of action, despite
Agreement envisages. attempts to persuade one to do so.
Hence, D is the right answer choice. Intransigent: unwilling or refusing to change
one's views or to agree about something.
14. Answer: E Non-Compliant: failing to act in accordance with
For the right answer, read the lines of thelast a wish or command.
paragraph, you can draw the hint from the below Hence, D is the right answer choice where
quoted lines, ‘Cooperative’ is the antonym of the given word
. For the India-U.S. agreement to yield results, ‘Recalcitrance’.
Mr. Biden would have to persuade industry and
research institutions at home to share 16. Answer: C
knowledge and subsidise transfer of ‘Healthier’ can fill in the blank II to make the
technologies. He has won commendations for sentence grammatically correct and contextually
steering America around from the science- meaningful.
deprived Trump years and announcing Peddle: promote (an idea or view) persistently or
enhanced ambition: cuts in emissions by 50% to widely
52% by 2030 over 2005 levels. But much of his Hulking: large, heavy, or clumsy

Click Here For Bundle PDF Course | support@guidely.in Page 10 of 12


Bank Po Mains PDF Course 2024
English Day - 14

Incendiary: tending to excite Materialised means to become real; to happen…


Hence, option C is the most viable answer bare means uncovered. Both these words make
choice. no sense in the given sentence.
Hit upon means discover or think of something,
17. Answer: C especially by chance… This word cancels the
Here, the error is in the4th part of the sentence. fourth option as well.
‘From’ should be used instead of ‘to’ after ‘apart’ The only option carrying both the correct words
to make the sentence grammatically correct and is (a).
contextually meaningful. Hence, C is the right
answer choice where 4th part of the italicized 20. Answer: B
sentence is erroneous. The first option can be ruled out as the word
migrants does not fit in grammatically.
18. Answer: D The word obvious means easily seen or
The word negligible means very small and understood; clear. Influx means large numbers
therefore not important. This can be taken as of people or things arriving suddenly… (this fits
correct but the second word of makes no sense as it describes people coming from across the
in the context. The word meticulous is same as border). Exodus is the opposite of influx as it
careful but both cannot fill the first blank as they means the mass departure of people. Apparent
will go contextually incorrect. The choice for the means the same as obvious or evident but
second blank in the last option makes it arrival is not fit here. It is best used with respect
incorrect. to railway stations or airports etc.
Lax means not having high standards; not strict. The only pair of words that fits correctly can be
The words given in option (d) fill the sentence found in option (b).
correctly.
21. Answer: C
19. Answer: A The first blank can take the words in the third
The first blank can take the word happened as and the fourth options only and not the others.
the sentence itself gives a hint by using the The word venerable means highly respectable…
phrase ‘still happening’ in the later part of the The word haywire means erratic; out of control…
sentence. important could have been a better choice
The phrase came about also means happened grammatically for choosing the first option. The
but the choice for the second blank does not fit fourth option gets eliminated for the usage of the
in. word though…

Click Here For Bundle PDF Course | support@guidely.in Page 11 of 12


Bank Po Mains PDF Course 2024
English Day - 14

The best combination of words that can be found collapsed. This word describes the state of the
in the options hence, is (c). vessel mentioned in the sentence. Tremendous
is very large or great; very good but this carries a
22. Answer: B positive tone making it unfit for the sentence
Evacuation means the removal of persons or given above. Splintered means to split or rend
things from an endangered area. The sentence into long thin pieces.
carries the word search after the first blank Of all the given options, the pair of words that
which would make the given word redundant. make the sentence grammatically and
Mended means to repair something that is meaningfully correct is (b).
damaged or broken… imploded means

Click Here For Bundle PDF Course | support@guidely.in Page 12 of 12


Bank Po Mains PDF Course 2024
Reasoning Day -15 (Eng)

Reasoning Aptitude
Directions (1-5): Study the following information Gujarat. Both Baseball and the kabaddi matches
carefully and answer the given below. were held in the same state but not held in the
Eight trophies of different games viz. Cricket, same state as chess match. Final match of the
Chess, Kabaddi, Kho-Kho, Carrom board, game which trophy was kept second from the
Volleyball, Baseball and Hockey were kept in a bottom of the stack in arrangement 1 was held in
single stack in the college office room one above neither Karnataka nor Gujarat. Hockey match
another, but not necessarily in the same order. was not held in the same state as Kabaddi
The final of each match was held in different match. Both Volleyball and hockey matches were
states viz. Haryana, Karnataka and Gujarat. At held in the same state.
least two and not more than three games were Arrangement 3:
held in each state. These games were held in Final match of the game which trophy was kept
different years 1988, 1996, 2001, 2005, 2010, at the bottom of the stack in arrangement 1 was
2013, 2017 and 2021. held in an even numbered year. The difference
Arrangement 1: between the years in which chess and baseball
Only three trophies were kept between the matches were held is a multiple of 11. Cricket
trophies of Cricket and Carrom board. As many match was held immediately before baseball
trophies kept above cricket trophy as below match. The difference between the years of the
baseball trophy. Hockey trophy was kept matches held in Haryana other than cricket in
immediately below the cricket trophy. Baseball arrangement 2 is 12 years. Only three matches
trophy was not kept adjacent to Cricket trophy. were held between Carrom board and hockey,
Only two trophies were kept between the which was not held immediately after chess
trophies of hockey and Kabaddi. Neither kabaddi match. The difference between the years in
nor hockey trophy was kept adjacent to chess which Baseball and Kabaddi matches were held
trophy. Volleyball trophy was kept three trophies is a multiple of 7.
above Kho-Kho trophy, which was not kept at the 1. Volleyball match was held in which of the
bottom of the stack. following year as per the arrangement 3?
Arrangement 2: a) 1988
Cricket match was held in Haryana but not with b) 2005
chess. Final match of the game which trophy was c) 1996
kept immediately above baseball trophy in d) 2021
arrangement 1 was not held in Karnataka. Final e) 2013
match of the game which trophy was kept at the
top of the stack in arrangement 1 was not held in

Click Here For Bundle PDF Course | support@guidely.in Page 1 of 14


Bank Po Mains PDF Course 2024
Reasoning Day -15 (Eng)

2. As per arrangement2, in which state only two c) The match which was held immediately after
matches were held and name those two matches Carromboard match
respectively? d) Chess match
a) Haryana, Kho-Kho and Volley ball e) None of these
b) Karnataka, Baseball and Kabaddi
c) Haryana, Cricket and Hockey Directions (6-10): Study the following information
d) Gujarat, Carromboard and Kabaddi carefully and answer the given below.
e) Gujarat, baseball and cricket &@©≤*+!$#%=~^
Step I: Consider the first four prime numbers in
3. Which of the following game trophy was kept the number series, add “2” to each of these
three stacks below Kho-Kho trophy as per numbers and write all numbers one by one after
arrangement 1? every third special character from left to right in
a) Volleyball trophy the same order.
b) The trophy which is kept immediately above Step II: Pick out the letters from the English
Volleyball trophy alphabetical series which have its corresponding
c) Baseball trophy place value is a multiple of “6” and write all such
d) The trophy which is kept at the bottom of the letters in reverse alphabetical order (one by one)
stack from left to right after every second element in
e) Carromboard trophy the series thus formed after step I.
Step III: Pick out the vowels except I from the
4. Which of the following match was held in English alphabetical series, and write all such
Gujarat as per arrangement 2? vowels(one by one) in alphabetical order from
a) Kho-Kho right to left after fifth, twelve, fifteen and
b) Baseball nineteenth element from the right end in the
c) Chess series thus formed after step II.
d) Both a and b Step IV: The letters which have its place value of
e) Both b and c even number in the alphabetical series are
replaced by second succeeding letter and the
5. If Kabaddi match is related to chess match letters which have its place value of odd numbers
and baseball match is related to cricket match in are replaced by preceding letter, in the series
a certain way in arrangement 3, then which of thus formed after step III.
the following match is related to hockey match? Thus step IV is the final step and answer the
a) The match which was held in the year 2001 below questions according to the final step.
b) Volleyball match

Click Here For Bundle PDF Course | support@guidely.in Page 2 of 14


Bank Po Mains PDF Course 2024
Reasoning Day -15 (Eng)

6. Which of the following element is the ninth c) !


element to the left of the third consonant from the d) 4
right end? e) Z
a) @
b) 1 10. How many symbols are there in the series
c) Z which is/are immediately proceeded by a
d) T consonant and immediately followed by a
e) * number?
a) One
7. If first 16 elements are reversed, then how b) Two
many such letters are there in the series which is c) Three
either immediately preceded by or immediately d) More than three
followed by a symbol (but not both)? e) None
a) Six
b) Seven Directions (11-15): Study the following
c) Eight information carefully and answer the given
d) Five below.
e) Nine Eighteen persons from D to U are sitting in three
parallel rows viz. row-1, row-2, and row-3 such
8. If 9 is related to = and D is related to * in a that row-2 is to the north of row-3 and row-1 is to
certain way, then 7 is related to which of the the north of row-2. 5 persons are sitting in row-1
following element? where 9 persons are sitting in row-2 and 4
a) 4 persons are sitting in the row-3. Persons sitting in
b) ^ row-3 faces north. Persons sitting in row-1 face
c) 5 south. The first 5 persons sitting from west to
d) * east in row-2 face north and the last four persons
e) $ sitting from west to east in row-2 face south.
Note: All the persons sitting in row1 and row 3
9. If all the letters which are immediately followed are facing the persons sitting in row 2 each
by a symbol are dropped, then which of the other.
following element is sixth to the right of the fifth K sits opposite to the one who sits second to the
element from the left end? right of U, who neither sits at the end of the row
a) 5 nor facing north. Only three persons sit between
b) $ K and G. As many persons sit to the left of U as

Click Here For Bundle PDF Course | support@guidely.in Page 3 of 14


Bank Po Mains PDF Course 2024
Reasoning Day -15 (Eng)

to the right of M, where both of them facing e) All I, II and III


opposite directions. Only two persons sit
between L and the one who sits opposite to M. J 13. Who among the following person sits north-
sits north-west of L but doesn’t sit at the end of east of the one who sits immediate right of P?
the row. The number of persons sitting to the left a) O
of J is one less than the number of persons b) The one who sits second to the right of I
sitting to the right of S. E sits opposite to the one c) H
who sits second to the right of S. K doesn’t sit d) The one who sits second to the left of S
adjacent to E.Only three persons sit between P e) The one who sits immediate left of U
and T, who sits second to the right of I. The
number of persons sitting between T and L is 14. What is the position of O with respect to K?
one less than the number of persons sitting a) Fourth to the right
between F and O, who sits opposite to Q. N sits b) Immediate left
immediate left of Q. The number of persons c) Second to the right
sitting between N and D is one more than the d) Third to the left
number of persons sitting between H and R, who e) Fifth to the left
doesn’t sit at the end.
11. Who among the following person sits exactly 15. Four of the following five are alike in a certain
between I and the one who sits opposite to J? way as per the given arrangement and thus form
a) O a group. Find the one that does not belong to the
b) The one who sits second to the right of K group.
c) L a) QN
d) The one who sits immediate right of L b) LF
e) N c) PI
d) RM
12. Who among the following persons are sitting e) TE
opposite to each other?
I) MT Direction (16-20): Study the following information
II) LQ carefully and answer the questions given below.
III) UF There are eight units placed one above the other
a) Only I and II in a single stack. Each unit contains a book and
b) Only III some boxes. The lowermost unit is numbered as
c) Only II and III 1 and the unit immediately above it is numbered
d) Only I as 2 and so on. Eight different books viz. L, M, N,

Click Here For Bundle PDF Course | support@guidely.in Page 4 of 14


Bank Po Mains PDF Course 2024
Reasoning Day -15 (Eng)

O, P, Q, R, and S are placed in each unit, but not units between S and the unit with a height of
necessarily in the same order. There is a certain 45cm. Book N is placed immediately above the
number of boxes which are placed one above the unit which has half of the boxes of book P. Book
other in each unit and the total height of each P is placed above Book R which is placed
unit is between 20cm and 75cm with all the immediately above the unit with a height of
boxes in each unit having the same height. No 72cm. Book O is placed three units below the
unit has the same height and the same number unit which has 3 boxes. Book Q does not have
of boxes. the lowest number of boxes. The height of the
Note: unit with book S is 2cm more than the unit which
i) If the total height of the unit is 12cm and the has 4 boxes. One of the units is 24cm in height.
number of boxes in that unit is 3, then the height The height of the Unit with book S is less than
of each box will be 4cm. the height of the Unit with book M.
ii) The total height of each unit will be the multiple 16. Which of the following book has maximum
of the number of boxes in that unit. At least two boxes along with it?
boxes are there in each unit. a) Q
iii) The unit which has an odd number of boxes b) R
kept on an even number unit and vice versa. The c) The book which is kept immediately below S
total number of boxes in each unit is not more d) M
than 9. e) The book which is kept immediately above O
Book L is placed on an even-numbered unit but
below unit number five. Only two books are 17. What is the total number of boxes of the unit
placed between L and the unit with a height of with books O, M, and S?
32cm. Book M is placed two units below the unit a) 9
with five boxes which is kept above the unit with b) 14
a height of 32cm. The lowermost unit doesn’t c) 12
have a height of 32 cm. The number of units d) 10
above book M is the same as the number of units e) 8
below the unit with a height of 52cm. Book S is
placed three units above the unit which has 6 18. How many books are kept above the unit
boxes. The unit with a height of 35cm is placed with a height of 24cm?
two units below the unit which has 9 boxes. The a) One
height of the unit with book L is an even number. b) Two
The number of units between L and the unit c) Three
which has 7 boxes is the same as the number of d) Four

Click Here For Bundle PDF Course | support@guidely.in Page 5 of 14


Bank Po Mains PDF Course 2024
Reasoning Day -15 (Eng)

e) More than four 20. Which of the following statement is true?


a) Book S kept on an even-numbered unit
19. What is the height of unit 6? b) No unit is there between L and N
a) 30cm c) Book R has 2 boxes
b) 45cm d) The height of the unit which has book S is
c) 72cm 32cm
d) 35cm e) All statements are true
e) None of these
Click Here to Get the Detailed Video Solution for the above given Questions
Or Scan the QR Code to Get the Detailed Video Solutions

Answer Key with Explanation


Directions (1-5):
1. Answer: D
2. Answer: B
3. Answer: D
4. Answer: C
5. Answer: B
Final arrangement
Arrangement 1:

Arrangement 2:

Click Here For Bundle PDF Course | support@guidely.in Page 6 of 14


Bank Po Mains PDF Course 2024
Reasoning Day -15 (Eng)

Arrangement 3:

Again we have,
 Only two trophies were kept between the
trophies of hockey and Kabaddi.
 Neither kabaddi nor hockey trophy was
kept adjacent to chess trophy.
Arrangement 1:
Here Case2 and 4 get eliminated
We have,
 Only three trophies were kept between
the trophies of Cricket and Carrom board.
 As many trophies kept above cricket
trophy as below baseball trophy.
 Hockey trophy was kept immediately
below the cricket trophy.
 Baseball trophy was not kept adjacent to
Cricket trophy.
From the above condition, there are five
possibilities
Again we have,
 Volleyball trophy was kept three trophies
above Kho-Kho trophy, which was not
kept at the bottom of the stack.
So Case1 and 5 get eliminated, hence Case-3
shows the final arrangement

Click Here For Bundle PDF Course | support@guidely.in Page 7 of 14


Bank Po Mains PDF Course 2024
Reasoning Day -15 (Eng)

 Both Baseball and the kabaddi matches


were held in the same state but not held
in the same state as chess match.
 Final match of the game which trophy was
kept second from the bottom of the stack
in arrangement 1 was held in neither
Karnataka nor Gujarat.
Here Case1 and 1a get eliminated

Arrangement 2:
We have,
 Cricket match was held in Haryana but
not with chess.
 Final match of the game which trophy was
kept immediately above baseball trophy in
arrangement 1 was not held in Karnataka.
From the above condition, there are four
possibilities Again we have,
 Hockey match was not held in the same
state as Kabaddi match.
 Both Volleyball and hockey matches were
held in the same state.
So Case2a gets eliminated, hence Case-2
shows the final arrangement

Again we have,
 Final match of the game which trophy was
kept at the top of the stack in
arrangement 1 was not held in Gujarat.

Click Here For Bundle PDF Course | support@guidely.in Page 8 of 14


Bank Po Mains PDF Course 2024
Reasoning Day -15 (Eng)

Arrangement 3:
 Final match of the game which trophy was
kept at the bottom of the stack in
arrangement 1 was held in an even
numbered year.
 The difference between the years in
which chess and baseball matches were
held is a multiple of 11.
 Cricket match was held immediately
before baseball match.
From the above condition, there are three Again we have,

possibilities  The difference between the years in


which Baseball and Kabaddi matches
were held is a multiple of 7.
So Case-3 gets eliminated, hence Case-1 shows
the final arrangement

Again we have,
 The difference between the years of the
matches held in Haryana other than
cricket in arrangement 2 is 12 years.
 Only three matches were held between
Carromboard and hockey, which was not
held immediately after chess match.
Here Case-2 gets eliminated Direction (6-10):
We have:
&@©≤*+!$#%=~^
Step I: Consider the first four prime numbers in
the number series, add “2” from each of these

Click Here For Bundle PDF Course | support@guidely.in Page 9 of 14


Bank Po Mains PDF Course 2024
Reasoning Day -15 (Eng)

numbers and write all numbers one by one after Thus we have the following result:
every third special character from left to right in &@TZ©4TN≤*ND+5H!$#7Z%=~9
the same order. ^
For Example: 2+2=4, 3+2=5, 5+2=7, 7+2=9 6. Answer: D
Thus we have the following result: Final series
&@©4≤*+5!$#7%=~9^ &@TZ©4TN≤*ND+5H!$#7Z%=~9
Step II: Pick out the letters from the English ^
alphabetical series which have its corresponding -> Third consonant from the right end is D and
place value is a multiple of “6” and write all such ninth to the left of D is T
letters in reverse alphabetical order (one by one)
from left to right after every second element in 7. Answer: B
the series thus formed after step I. Final series
Then, we have the letters: F, L, R, X & @ T Z ©4T N ≤ * N D + 5H ! $ # 7 Z % = ~ 9 ^
Thus we have the following result: After reversing first 16 elements
&@X©4R≤*L+5F!$#7%=~9^ ! H 5 + DN *≤ N T 4 © ZT @& $ # 7 Z % = ~ 9 ^
Step III: Pick out the vowels except I from the
English alphabetical series, and write all such 8. Answer: E
letters(one by one)in alphabetical order from Final series
right to left after fifth, twelve, fifteen and & @ T Z © 4 T N ≤ * N D + 5 H !$ # 7 Z % = ~ 9
nineteenth element from the right end in the ^
series thus formed after step II. 9 is second to the right of = and D is second to
The vowels from the English alphabetical series the right of * in the same way 7 is related to $.
are: A, E O and U
Thus we have the following result: 9. Answer: A
&@UX©4RO≤*LE+5F!$#7A%=~9 Final series
^ & @ TZ © 4 T N ≤ * N D + 5 H ! $ # 7 Z % = ~ 9
Step IV: The letters which have its place value of ^
even number in the alphabetical series are After letters are dropped
replaced by second succeeding letter and the &@T©4T≤*N+5!$#7%=~9^
letters which have its place value of odd 4 is the fifth element from the left end and sixth
numbers are replaced by preceding letter, in the to the right of 4 is 5
series thus formed after step III.
U-21; X-24; R-18; O-15; L-12; E-5; F-6; A-1 10. Answer: B

Click Here For Bundle PDF Course | support@guidely.in Page 10 of 14


Bank Po Mains PDF Course 2024
Reasoning Day -15 (Eng)

Final series
& @ TZ © 4 T N ≤ * N D + 5H !$ # 7 Z % = ~ 9 ^
Z©4 and D+5

Direction (11-15):
11. Answer: D
12. Answer: B
13. Answer: E
14. Answer: B
15. Answer: C
Final arrangement

Again we have,
 As many persons sit to the left of U as to
the right of M, where both of them facing
opposite directions.
 Only two persons sit between L and the
one who sits opposite to M.

We have,
 K sits opposite to the one who sits second
to the right of U, who neither sits at the
end of the row nor facing north.
 Only three persons sit between K and G.
From the above condition, there are three
possibilities

Click Here For Bundle PDF Course | support@guidely.in Page 11 of 14


Bank Po Mains PDF Course 2024
Reasoning Day -15 (Eng)

persons sitting between F and O, who sits


opposite to Q.
 N sits immediate left of Q.
 The number of persons sitting between N
and D is one more than the number of
persons sitting between H and R, who
doesn’t sit at the end.
Again we have,
So Case-1 and 2 get eliminated, hence Case-3
 J sits north-west of L but doesn’t sit at the
shows the final arrangement
end of the row.
 The number of persons sitting to the left
of J is one less than the number of
persons sitting to the right of S.
 E sits opposite to the one who sits second
to the right of S.
 K doesn’t sit adjacent to E.

Directions (16-20):
16. Answer: E
17. Answer: D
18. Answer: C
19. Answer: B
20. Answer: A
Again we have,
Final Arrangement
 Only three persons sit between P and T,
who sits second to the right of I.
 The number of persons sitting between T
and L is one less than the number of

Click Here For Bundle PDF Course | support@guidely.in Page 12 of 14


Bank Po Mains PDF Course 2024
Reasoning Day -15 (Eng)

Again we have,
 The number of units above book M is the
same as the number of units below the
unit with a height of 52cm.
 Book S is placed three units above the
unit which has 6 boxes.
From the above condition, case-2a gets
eliminated.

We have,
 Book L is placed on an even-numbered
unit but below unit number five. Only two
books are placed between L and the unit
with a height of 32cm.
 Book M is placed two units below the unit
with five boxes which is kept above the
Again we have,
unit with a height of 32cm.
 The unit with a height of 35cm is placed
 The lowermost unit doesn’t have a height
two units below the unit which has 9
of 32 cm
boxes. The height of the unit with book L
From the above condition, there are three
is an even number.
possibilities.
 The number of units between L and the
unit which has 7 boxes is the same as the
number of units between S and the unit
with a height of 45cm

Click Here For Bundle PDF Course | support@guidely.in Page 13 of 14


Bank Po Mains PDF Course 2024
Reasoning Day -15 (Eng)

Again we have,
 Book N is placed immediately Again we have,
above the unit which has half of the  One of the units is 24cm in height.
boxes of book P.  The height of the Unit with book S is less
 Book P is placed above Book R than the height of the Unit with book M.
which is placed immediately above From the above condition, case1 shows the final
the unit with a height of 72cm. arrangement.
 Book O is placed three units below
the unit which has 3 boxes.
 Book Q does not have the lowest
number of boxes. The height of the
unit with book S is 2cm more than
the unit which has 4 boxes.
From the above condition, case2 gets
eliminated.

Click Here For Bundle PDF Course | support@guidely.in Page 14 of 14


Bank Po Mains PDF Course 2024
Quantitative Aptitude Day -15 (Eng)

Quantitative Aptitude

Directions (1-4): Study the following data 3) Series that follows a certain logic given below
carefully and answer the questions: and contains one wrong term. There is another
1) Given below is a number series that follows series that follows the same logic as in first
certain logic. If ‘P’ is the nth term and ‘Q’ is ‘n + series and starts with the wrong term of first
1’th term, then find the correct relationship series. Find the 3rd term of second series?
between ‘P’ and ‘Q’. Series I: 2, 14, 112, 672, 3360, 13440, 40320
Series: 8, 25, 76, 229, 688 A.784
A.Q = 3P – 1 B.6272
B.Q = 2P + 3 C.5376
C.Q = 3P + 1 D.896
D.Q = 2P – 1 E.4704
E.Q = 2P – 3
4) Two series I and II with different logics are
2) There are two series I and II. Logic in both the given below. Series I contains one wrong
series are different. element termed as A and series II contains one
Series I: 3, 12, 28, 53, 89, ……. missing element termed as B. Which of the
Series II: 259, 270, 283, 300, 319. following is not TRUE regarding A and B?
Which of the following will be the first term which Series I: 18480, 1680, 240, 48, 18, 8
is common in both the series? Series II: 200, 79, ?, 5, -4, -8
A.504 A.HCF of A and B is 6.
B.342 B.LCM of A and B is 90.
C.383 C. Total number of factors of A are 6.
D.283 D. Total number of factors of B are 8.
E.371 E.A > B

Directions (5-8): Study the following data carefully and answer the questions:
There are 6 shopkeepers A, B, C, D, E and F. Each shopkeeper purchased two types of bread i,e;
Banana bread and Garlic bread and sold only some of the breads.
The Pie chart given below shows the percentage distribution of the number of banana bread sold by each
shopkeeper on a particular day.

Click Here For Bundle PDF Course | support@guidely.in Page 1 of 12


Bank Po Mains PDF Course 2024
Quantitative Aptitude Day -15 (Eng)

The table given below is the average number of banana bread and that of garlic bread sold by
shopkeepers B, C, D, E and F.

Note:
1: Number of garlic bread sold by shopkeeper A is 40 more than that of banana bread sold by him.
2: Average number of banana bread and garlic bread sold by shopkeeper A is 90% of the number of
garlic bread sold by him.
5) Number of banana bread sold by C is 60% of total unsold bread (banana + garlic) of C are
the total banana bread purchased by him and the rotten, then find the number of unsold bread
number of garlic bread sold by C is 66(2/3) % of (banana + garlic) of C, which are not rotten?
total garlic bread purchased by him. If 20% of A.164

Click Here For Bundle PDF Course | support@guidely.in Page 2 of 12


Bank Po Mains PDF Course 2024
Quantitative Aptitude Day -15 (Eng)

B.120 C.50
C.180 D.25
D.148 E.45
E.152
9) In container A, contains the mixture of milk
6) Number of banana bread sold by C and F and water. The ratio of milk and water in
together is what percent less than the number of container A is 8:5. 39L mixture is taken out and
garlic bread sold by C and F together? put it on container B. when 4L milk and 5L water
A.34% is added in container A,then the ratio of milk to
B.42% water in the container A becomes 3:2.Find the
C.40% difference between the total milk in container A
D.32% and B together and total water in container A and
E.36% B together?
A.29 L
7) If 66(2/3) % of total bread (banana + garlic) B.20 L
sold by A are sold to males, out of which 40% C.22 L
are banana bread and 50% of total bread D.31 L
(banana + garlic) sold by D are sold to males, out E. None of these
of which 40% are banana bread, then find the
number of garlic bread sold by A and D together 10) If A started the business with initial
to males. investment of Rs. 6000 and after X months B
A.204 joined the business with initial investment of Rs.
B.216 9000. 3 months after B joining the business A
C.184 withdraws 1/4th of his initial investment. At the
D.198 end of the year A’s share is Rs. 3850 out of total
E.210 profit Rs. 8800. Find the value of X?
A.7 months
8) Shopkeeper B sold 82% of the total number of B.4 months
bread (banana + garlic) purchased by him and C.6 months
sold 80% of banana bread purchased by him. D.3 months
Find the number of unsold garlic bread of E.5 months
shopkeeper B.
A.40 11) Ratio of the radius of the cylinder to the
B.30 length of the rectangle is 1:2 and the breadth of

Click Here For Bundle PDF Course | support@guidely.in Page 3 of 12


Bank Po Mains PDF Course 2024
Quantitative Aptitude Day -15 (Eng)

the rectangle is 15% of the perimeter of the and speed ratio between Train B and Train A is
rectangle. If the height of the cylinder is half of 5:3. If the two Trains are travelling in opposite
the breadth of the rectangle and the curved direction they will meet after 10 seconds. Find
surface area of the cylinder is 528 cm2, then find the ratio between time taken by Train A and
the radius of the cylinder? Train B to cross the platform has length of
A.7 cm 220m?
B.14 cm A.7:8
C.3.5 cm B.9:7
D.21 cm C.9:5
E. Cannot be determined D.8:9
E.7:9
12) If the ratio between length of two Trains A
and B is 2:3. The speed of Train B is 180 kmph

Directions (13-16): Study the following information carefully and answer the questions given below.
The given table shows the average number of people who visit park A and B and the percentage of the
number of people who visit park B is more than that of A and the ratio of the number of people who visit
park A and C in two different seasons in five different states.

Click Here For Bundle PDF Course | support@guidely.in Page 4 of 12


Bank Po Mains PDF Course 2024
Quantitative Aptitude Day -15 (Eng)

13) What is the ratio of the total number of 15) The difference between the total number of
people who visit park C in summer season in people who visit park B and C in winter season in
Tamilnadu and Bihar together to the total number Bihar is what percent of the difference between
of people who visit park B in winter season in the number of people who visit park A and C in
Tamilnadu and Kerala together? summer season in Assam?
A.21:16 A.37.78%
B.22:17 B.39.10%
C.7:5 C.41.67%
D.11:8 D.43.56%
E. None of these E.45.89%

14) What is the difference between the number 16) What is the difference between the number
of people who visit park B and C together in of people who visit park B and C together in
winter and summer season in Assam? winter season in Manipur and the number of
A.2460 people who visit park A and C in summer season
B.2465 in Kerala?
C.2470 A.9000
D.2475 B.9200
E.2480 C.9100
D.9300
E.9400

Directions (17-20): Study the following data carefully and answer the questions:
There are 5 car manufacturing companies P, Q, R, S and T. Each company manufactures three types of
cars i,e; TUV, SUV and KUV.
The bar graph given below shows the number of SUVs manufactured by a company as percent of that of
TUVs manufactured by the respective company and also shows the average number of TUVs, SUVs and
KUVs manufactured by a company in a month.

Click Here For Bundle PDF Course | support@guidely.in Page 5 of 12


Bank Po Mains PDF Course 2024
Quantitative Aptitude Day -15 (Eng)

Note:
1: Ratio of the number of TUVs manufactured by P and R is 4: 5 respectively and the difference between
them is 37.5% of the number of TUVs manufactured by Q.
2: Ratio of the number of TUVs manufactured by R and S is 3: 2 respectively and the number of TUVs
manufactured by Q is 40.
3: Average number of TUVs manufactured by S and T is 65.
17) Cost of manufacturing one SUV in of the month and the ratio of the number of SUVs
companies P, R and S is Rs.5 Lakh, Rs.6 Lakh to that of KUVs manufactured by company Q in
and Rs.4 Lakh respectively, then find the the last 5 days is 5: 2, then find the ratio of
average total cost of manufacturing SUVs of number of SUVs to that of KUVs manufactured
these three companies in a month. by company Q in first 25 days. (Assume 30 days
A.Rs.30 million in the month)
B.Rs.36 million A.4: 7
C.Rs.24 million B.2: 3
D.Rs.40 million C.5: 9
E.Rs.32 million D.1: 2
E.3: 5
18) If 16(2/3) % of total KUVs manufactured by
company Q are manufactured in the last 5 days

Click Here For Bundle PDF Course | support@guidely.in Page 6 of 12


Bank Po Mains PDF Course 2024
Quantitative Aptitude Day -13 (Eng)

19) Total number of KUVs manufactured by 20) Total number of TUVs produced by
companies R and S together is what percent companies P and T together is what percent of
more than that of TUVs manufactured by these that of KUVs produced by these two companies
two companies together? together?
A.83% A.87.33%
B.76% B.91.33%
C.80% C.93.33%
D.85% D.85.33%
E.78% E.89.33%
Click Here to Get the Detailed Video Solution for the above given Questions
Or Scan the QR Code to Get the Detailed Video Solutions

Answer Key with Explanation


1) Answer: C 53 + 62 = 89
Logic in the series is: 89 + 72 = 138
(8 * 3) + 1 = 25 138 + 82 = 202
(25 * 3) + 1 = 76 202 + 92 = 283
(76 * 3) + 1 = 229 283 + 102 = 383
(229 * 3) + 1 = 688 Logic in series II:
Hence, 259 + 11 = 270
Q = 3P + 1 270 + 13 = 283
283 + 17 = 300
2) Answer: D 300 + 19 = 319
Logic in series I: 319 + 23 = 342
3 + 32 = 12 Hence, we can see that the first term which is
12 + 42 = 28 common in both the series is 283.
28 + 52 = 53

Click Here For Bundle PDF Course | support@guidely.in Page 7 of 12


Bank Po Mains PDF Course 2024
Quantitative Aptitude Day -15 (Eng)

3) Answer: A (d): Total number of factors of B = 8


Logic in the series is: (e): A < B
2 * 8 = 16 Hence, option (e) is not TRUE.
16 * 7 = 112
112 * 6 = 672 Directions (5-8):
672 * 5 = 3360 Let the number of banana bread sold by
3360 * 4 = 13440 shopkeepers A, B, C, D, E and F are ‘20x’, ‘12x’,
13440 * 3 = 40320 ‘15x’, ‘10x’, ‘25x’ and ‘18x’ respectively.
Wrong term = 14 So, number of garlic bread sold by shopkeeper A
New series: = 20x + 40
14 * 8 = 112 And average of number of banana bread and
112 * 7 = 784 garlic bread sold by shopkeeper A = (20x + 40) *
14, 112, 784 (90/100)
Hence, 3rd term of second series = 784 Now,
(20x + 20x + 40)/2 = (20x + 40) * (90/100)
4) Answer: E 40x + 40 = (20x + 40) * (9/5)
Logic in series I: 200x + 200 = 180x + 360
18480 ÷ 11 = 1680 x=8
1680 ÷ 7 = 240 Average of number of banana bread and garlic
240 ÷ 5 = 48 bread sold by shopkeeper A = (20 * 8 + 40) *
48 ÷ 3 = 16 (Not 18) (90/100) = 180
16 ÷ 2 = 8
A = 18 = 21 * 32
Logic in series II:
200 – 112 = 79
79 – 72 = 30
30 – 52 = 5
5 – 32 = -4
-4 – 22 = -8
B = 30 = 21 * 31 * 51
(a): HCF of 18 and 30 = 6
(b): LCM of 18 and 30 = 90 5) Answer: E
(c): Total number of factors of A = 6 Number of banana bread sold by C = 120

Click Here For Bundle PDF Course | support@guidely.in Page 8 of 12


Bank Po Mains PDF Course 2024
Quantitative Aptitude Day -15 (Eng)

So, the total number of banana bread, Total number of breads (banana + garlic) sold by
purchased by C: A = 160 + 200 = 360
120 * (100/60) = 200 Number of breads (banana + garlic) sold by A to
Number of garlic bread sold by C = 220 males = 66(2/3) % of 360 = 240
So, the total number of garlic bread, purchased Number of garlic breads sold by A to male =
by C: 60% of 240 = 144
220 * (300/200) = 330 Total number of breads (banana + garlic) sold by
Total number of breads (banana + garlic), D = 80 + 160 = 240
purchased by C: Number of breads (banana + garlic) sold by D to
200 + 330 = 530 males = 50% of 240 = 120
Total number of unsold bread (banana + garlic) Number of garlic breads sold by D to male =
of C: 60% of 120 = 72
530 – 120 – 220 = 190 Number of garlic breads sold by A and D
Since 20% of total unsold bread (banana + together to males = 144 + 72 = 216
garlic) of C are rotten.
So, the number of unsold bread (banana + 8) Answer: B
garlic) of C, which are not rotten: Total number of breads (banana + garlic) sold by
80% of 190 = 152 B = 96 + 150 = 246
So, total number of breads (banana + garlic)
6) Answer: A purchased by B:
Number of banana bread sold by C = 120 246 * (100/82) = 300
Number of banana bread sold by F = 144 Number of banana breads sold by B = 96
Total number of banana bread sold by C and F So, number of banana breads purchased by B =
together = 120 + 144 = 264 96 * (100/80) = 120
Number of garlic bread sold by C = 220 And number of garlic breads purchased by B =
Number of garlic bread sold by F = 180 300 – 120 = 180
Total number of garlic breads sold by C and F Number of garlic breads sold by B = 150
together = 220 + 180 = 400 So, number of unsold garlic breads of B = 180 –
Required percentage = [(400 – 264)/400] * 100 = 150 = 30
34%
9) Answer: A
7) Answer: B Let milk and water in container A is 8x and 5x
respectively.

Click Here For Bundle PDF Course | support@guidely.in Page 9 of 12


Bank Po Mains PDF Course 2024
Quantitative Aptitude Day -15 (Eng)

In 39L milk and water is 39*(8/13)=24 and Radius of the cylinder = 7 * 4/2 = 14 cm
39*(5/13)=15L respectively.
So, we can say, (8x-24+4)/(5x-15+5) = 3/2 12) Answer: B
Or, 16x-40=15x-30 The speed of Train B = 180 kmph = 180 * 5 / 18
Or, x=10 L = 50 m/s
So, in container A amount of milk is = 80- The speed of Train A = 50 * 3 / 5 = 30 m/s
24+4=60 L Length of two trains = 10 * (50 + 30) = 800
In container A amount of water is = 50-15+5=40 Length of Train A = 800 * 2 / 5 = 320m
L Length of Train B = 800 * 3 / 5 = 480m
Required difference = (60+24) – (40+15) = 29 L Time taken by Train A to cross the platform has
10) Answer: D length of 220m = (320 + 220) / 30 = 18s
At the end of the year, Time taken by Train B to cross the platform has
A’s share =Rs. 3850 length of 220m = 700 / 50 = 14s
B’s share = Rs. 4950 Required ratio = 18:14 = 9:7
Profit ratio of A to B = 7:9
(6000*X + 6000*3 + 4500*(9-X)) / (9000*3 + Directions (13-16):
9000(9-X)) = 7/9 Tamilnadu in winter season:
(1500x + 58500) / (10800 – 9000x) = 7/9 Number of people visit park A and B = 12900 * 2
765x = 2295 = 25800
X=3 A + A * 115/100 = 25800
A = 12000
11) Answer: B B = 25800 – 12000 = 13800
Ratio of the breadth and perimeter of the C = 5/4 * 12000 = 15000
rectangle =15:100 = 3:20 Similarly we can find the remaining values.
Breadth of the rectangle = 3x
Perimeter of the rectangle = 20x
Radius of the cylinder = y
Length of the rectangle = 2y
2 * (3x + 2y) = 20x
2y = 7x
y = 7x/2 13) Answer: A
2 * 22/7 * 7x/2 * 3x/2 = 528 Required ratio = (22400 + 19600):(13800 +
x=4 18200)

Click Here For Bundle PDF Course | support@guidely.in Page 10 of 12


Bank Po Mains PDF Course 2024
Quantitative Aptitude Day -15 (Eng)

= 42000:32000 Since, the average number of TUVs


= 21:16 manufactured by S and T is 65.
So, number of TUVs manufactured by T = (2 *
14) Answer: D 65) – 50 = 80
Required difference = (15000 + 16875) – (13800
+ 15600)
= 2475

15) Answer: C
Required % = (14000 – 12500)/(15600 – 12000)
* 100
=1500/3600*100
= 41.67%

17) Answer: A
16) Answer: B
Number of SUVs manufactured by company P in
Difference = (18000 + 28800) - (17600 + 20000)
a month = 72
= 9200
So, total cost of manufacturing SUVs of
company P in a month = 72 * 500000 = Rs.36
Directions (17-20):
million
Since, number of TUVs manufactured by Q = 40
Number of SUVs manufactured by company R in
So, difference between number of TUVs
a month = 60
manufactured by P and R = 37.5% of 40 = 15
So, total cost of manufacturing SUVs of
Since, the ratio of number of TUVs
company R = 60 * 600000 = Rs.36 million
manufactured by P and R is 4: 5 respectively.
Number of SUVs manufactured by company S in
So, number of TUVs manufactured by P = 15 *
a month = 45
4/ (5 – 4) = 60
So, total cost of manufacturing SUVs of
And number of TUVs manufactured by R = 60 *
company S = 45 * 400000 = Rs.18 million
(5/4) = 75
Required average = (36 + 36 + 18)/3 = Rs.30
Since, the ratio of number of TUVs
million
manufactured by R and S is 3: 2 respectively.
So, number of TUVs manufactured by S = 75 *
18) Answer: D
(2/3) = 50
Total number of KUVs manufactured by
company Q in the month = 60

Click Here For Bundle PDF Course | support@guidely.in Page 11 of 12


Bank Po Mains PDF Course 2024
Quantitative Aptitude Day -15 (Eng)

So, number of KUVs manufactured by company 105 + 115 = 220


Q in first 25 days of the month = 83(1/3) % of 60 Total number of TUVs manufactured by
= 50 companies R and S together:
Number of SUVs manufactured by company Q in 75 + 50 = 125
last 5 days of the month = 16(2/3) % of 60 * (5/2) Required percentage = [(220 – 125)/125] * 100 =
= 25 76%
Total number of SUVs manufactured by
company Q in the month = 50 20) Answer: C
So, number of SUVs manufactured by company Total number of TUVs produced by P and T
Q in the first 25 days of the month = 50 – 25 = 25 together = 60 + 80 = 140
Required ratio = 25: 50 = 1: 2 Total number of KUVs produced by P and T
together = 48 + 102 = 150
19) Answer: B Required percentage = (140/150) * 100 =
Total number of KUVs manufactured by 93.33%
companies R and S together:

Click Here For Bundle PDF Course | support@guidely.in Page 12 of 12


Bank Po Mains PDF Course 2024
English Day - 15

English Language
Directions (1-7): Given below are a few rubbing it in. It can cause your eyes then become
questions based on the passage. You have to red, irritated and watery," said Dr Gopal Pillai. To
read the passage carefully and answer the given prevent dry eyes, wash your eyes frequently,
questions accordingly. If none of the options are suggested Dr Pillai. "After about every few
correct then choose option E as your answer. minutes, voluntarily, close your eyes tightly and
Summer has its own set of infections that are then open them. Watch your eyes more often as
spread rampantly if a proper lifestyle is not it will help lubricate the eyeballs," he said. To
followed. Our eyes go through a great deal of combat dryness, use tear substitutes like eye
stress and pressure, especially due to the drops as well. Dr Sarang Goel, Medical Director,
summer sun that is bright and scorching. Cases Ayu Health Network of Hospitals, told
of ocular diseases like conjunctivitis are IndiaToday.in that rehydration is important for
reportedly heavily in the hot months and taking eye health.
necessary precautions is of utmost importance. "Rehydration is extremely important in summer.
Eyes are quite sensitive to the ultraviolet (UV) Avoid any chance of dehydration as it can affect
rays of the sun. In summer, prolonged exposure the eyes. While applying moisturiser, rub it
to UV modifies the lens proteins, leading to around your eyes. Wear a hat when out in the
various problems like cataract formation and sun or use an umbrella. Most importantly do not
worsening eyesight. UV can also damage the rub your eyes," said Dr Sarang Goel.
retina with a high risk of cancer like basal cell Another major problem in summer is ocular
carcinoma and squamous cell carcinoma. Dr diseases like conjunctivitis, keratitis,
Gopal S Pillai, Clinical Professor, Head of endophthalmitis, cellulitis and stye, among
Opthalmology, Amrita Hospital, Kochi, told others. Dr Pillai said that conjunctivitis is one of
IndiaToday.in people should wear UV the most common eye problems in the hot
sunglasses to protect their eyes. This will also months.
prevent dryness in the eyes, caused by the sun "Conjunctivitis spreads via human touch.
and more screen exposure. Someone could use an infected towel or even a
"Dryness is a major problem in eye health these door knob that has been touched by an infected
days, especially among children who study for a person. In a classroom, one child could infect
long time and those who are in front of the another child if they have conjunctivitis. It's a viral
screens. Many people look at their screens for 8- infection, so it spreads rampantly," the expert
14 hours a day which can cause extreme said.
dryness. In summer, dryness is a feeling in which Ideally, a healthy well balanced diet is essential
a person puts mud in their eyes and starts for eye health along with that we must have

Click Here For Bundle PDF Course | support@guidely.in Page 1 of 9


Bank Po Mains PDF Course 2024
English Day - 15

antioxidants like beta carotene, vitamins C and 2. Which of the following option is incorrect as a
E, Omega 3 and zinc. These help in preventing result of prolonged exposure to UV in the
the development of serious eye conditions. Avoid summer?
unprocessed foods on a daily basis," Dietician A. UV can also damage hair follicles and causes
Jyoti Khaniojh, Dietetics, Nutrition And Dietetics, hairfall.
Max Super Speciality Hospital, Patparganj, New B. UV can also damage the retina with a high risk
Delhi, told IndiaToday.in. of cancer like basal cell carcinoma and
When we eat a meal plate, which is colourful like squamous cell carcinoma
a rainbow, we get all sorts of antioxidants in our C. prolonged exposure to UV modifies the lens
diet. proteins, leading to various problems like
Since our childhood, we have been taught that cataract formation and worsening eyesight
an apple a day keeps the doctor away. Likewise, D. Both C&D
carrots are known to be best for the eyes. It is E. None of the above
rich in beta-carotene and vitamin, which protects
the eye against infections," Jyoti Khaniojh said. 3. Which of the following option is the synonym
Besides this, add lemon and citrus fruits to your of “VOLUNTARILY”?
diet for vitamin C intake. Nuts and seeds that A. willingly
contain vitamin E prevent cataracts and age- B. reluctantly
related macular degeneration. C. unwillingly
Salmon fish is a rich source of omega-3 and this D. unintentionally
healthy fat helps in prevent eye dryness as it is E. None of the above
great for retina health.
Egg yolks contain vitamin A, zinc, lutein and 4. Which of the following is not one of the
zeaxanthin and zinc, which help in age-related reasons given by Dr. Pillai to prevent dry eyes?
degeneration of eyes. Zinc helps in night A. You should wash your eyes frequently
improvement of night vision. B. After about every few minutes, voluntarily,
close your eyes tightly and then open them
1. Which of the following option is the antonym of C. Watch your eyes more often as it will help
RAMPANTLY? lubricate the eyeballs
A. unbridled D. Wash your eyes with hygienic soap frequently
B. unhindered E. None of the above
C. unhampered 5. Which of the following is not an example of
D. restrained ocular diseases?
E. None of the above A. conjunctivitis

Click Here For Bundle PDF Course | support@guidely.in Page 2 of 9


Bank Po Mains PDF Course 2024
English Day - 15

B. cellulitis A. a
C. endophthalmitis B. b
D. orthopthalmitis C. c
E. None of the above D. d
E. e
6. Why does an infected towel or even a door
knob is susceptible to conjunctivitis? 9. While the rapidly growing Indian aviation
A. Because it spreads only via human touch sector offers ample opportunities (a)/ for private
B. Because it spreads only via clothes carriers to launch operations (b)/, he remains an
C. Because it spreads only via saliva unforgiving space (c)/, considering the number of
D. Because it is a viral infection and it spreads airlines that have been forced to shut shop due
rampantly to financial troubles (d). No error (e).
E. None of the above A. a
B. b
7. Which of the food item is not mentioned by Dr. C. c
jyoti khanioj for a healthier eye? D. d
A. carrot E. e
B. lemon juice
C. selmon fish 10. In November 2019 (a)/, the bankruptcy court
D. zinga fish ordered a corporate insolvency resolution
E. None of the above process for the grounded regional low-cost air
carrier, Air Costa (b)/, aviation regulator
Directions (8-12): Given below are a few Directorate General of Civil Aviation (DGCA) in
questions divided into five parts which may or June 2017 suspended the flying license for Air
may not contain errors in them. You have to find Costa (c)/, making it permanently stop operations
the part containing errors in them. If none of the (d). No error (e).
parts contains error then choose option E as A. a
your answer. B. b
8. Banga, 63, was nominated for the post by US C. c
President Joe Biden (a)/ in late February and D. d
was the sole contender (b)/ to replace departing E. e
World Bank chief David Malpass (c)/, an 11. Sahara Airlines was first established on Sept
economist and former US Treasury official during 20, 1991 (a)/, and later begun its operation after
the Trump administration (d). No error (e). 2 years on December 3, 1993 (b)/. Two Boeing

Click Here For Bundle PDF Course | support@guidely.in Page 3 of 9


Bank Po Mains PDF Course 2024
English Day - 15

737-200 aircraft served as Sahara Airlines (c)/. company (b) indicated that it could lose more
On October 2, 2000, the airline was rebranded to subscribers in the current quarter.
Air Sahara, it controlled 12 per cent of India's One of the reasons behind growing losses could
domestic flight market at its peak (d). No error be the sharp fall in subscribers in the South Asia-
(e). focused Disney+ Hotstar after it lost the (c)
A. a streaming rights to the popular Indian Premier
B. b League (IPL) cricket matches.
C. c A. Only a
D. d B. Only b
E. e C. Only c
D. Both a&b
12. The World Bank has been led by an E. None of the above
American (a)/ since its founding at the end of
World War Two (b)/, while the International 14. While Go First had (a) blamed P&W for its
Monetary Fund has been led by an European financial troubles and bankruptcy filing, the US
(c)/. Banga, who was born in India and spent his engine maker said earlier that the airline’s (b)
early career there, has been a US citizen since claim was “unfounded” as it had not paid for the
2007 (d). No error (e). maintenance and lease charges for years, which
A. a (c) penultimately led to the necessary
B. b suspension of services.
C. c A. Only a
D. d B. Only b
E. e C. Only c
D. Both a&c
Directions (13-16):Given below are a few E. None of the above
questions with three words highlighted which
may or may not used appropriately as per the 15. While a (a) surge in traffic in India, the
context of the sentence. You have to find the world’s third-biggest aviation market in the world,
word which is used out of the context of the led to record jet orders, the (b) failure of two
sentence. If all words are appropriately used major airlines – Kingfisher Airlines in 2012 and
then choose option E as your answer. Jet Airways in 2019 – have made lessors (c)
13. Disney+ lost a (a) whopping 4 million comprehensive about the market.
subscribers in just the second quarter. The A. Only a

Click Here For Bundle PDF Course | support@guidely.in Page 4 of 9


Bank Po Mains PDF Course 2024
English Day - 15

B. Only b D. who expect lower food prices to aid the


C. Only c decline
D. Both a&b E. No improvement is required
E. None of the above
18. Taiwanese electronics giant Foxconn has
16. The Go First bankruptcy decision has also bought a huge tract of land on the outskirts of
made lessors nervous about another financially- Indian tech hub Bengaluru, the key Apple
troubled airline, SpiceJet. While SpiceJet said it supplier said in a filing as it looks to diversify
has no plans of filing for insolvency, lessors are production towards China.
worried that a similar move by the airline could A. he looks to diversify production away from
(a) spoil their (b) respect of (c) repossessing China
planes. B. she looks to diversify production away from
A. Only a China
B. Only b C. them looks to diversify production toward
C. Only c China
D. Both b&c D. it looks to diversify production away from
E. None of the above China
E. No improvement is required
Directions (17-20): Given below are few question
with a highlighted phrase which may or may not 19. Apple have making its own push into India
need improvement. You have to find the correct and chief executive Tim Cook last month opened
replacement of the phrase if required. If no its first two retail stores in the world's most
improvement is required then choose option E as populous country.
your answer. The California-based firm is betting big on the
17. India's headline retail inflation in May is likely nation of 1.4 billion people -- home to the
to fall further towards 4% — the midpoint of the second-highest number of smartphone users in
central bank's target and a level last seen in the world, after China.
January 2021, according to a few economists, A. Apple have been making its own push into
which expect lower food prices to add the India
decline. B. Apple have been made its own push into India
A. who expect lower food prices to add the C. Apple has been making its own push into
decline India
B. who expect lower food prices to aid the rise D. Apple has been made its own push into India
C. who expect lower food prices to add the rise E. No improvement is required

Click Here For Bundle PDF Course | support@guidely.in Page 5 of 9


Bank Po Mains PDF Course 2024
English Day - 15

A. Adani Ports has been floated a tender of up to


20. Adani Ports had floated a tender of up to B. Adani Ports had been floated a tender of up to
$130 million of 3.375% 2024 maturity dollar- C. Adani Ports was floated a tender of up to
denominated bonds late last month, as it seeks D. Adani Ports were floated a tender of up to
to boost investor confidence after the group's E. No improvement is required
shares were pummelled earlier this year by a US
short-seller's report.
Click Here to Get the Detailed Video Solution for the above given Questions
Or Scan the QR Code to Get the Detailed Video Solutions

Answer Key with Explanation


1. Answer: D Head of Opthalmology, Amrita Hospital, Kochi,
Here, rampant means showing no sign of being told IndiaToday.in people should wear UV
under control and restrained means just the sunglasses to protect their eyes. This will also
opposite to it whereas all other options are prevent dryness in the eyes, caused by the sun
synonyms of rampant. So, option D is the correct and more screen exposure. We can say from the
answer. options that hair fall is nowhere mentioned in the
passage. So, option A is the correct answer.
2. Answer: A
Here, from the lines, In summer, prolonged 3. Answer: A
exposure to UV modifies the lens proteins, Here, voluntarily means of one’s own free will
leading to various problems like cataract and here willingly is the only synonym whereas
formation and worsening eyesight. UV can also all other options are antonyms. So, option A is
damage the retina with a high risk of cancer like the correct answer.
basal cell carcinoma and squamous cell
carcinoma. Dr Gopal S Pillai, Clinical Professor, 4. Answer: D

Click Here For Bundle PDF Course | support@guidely.in Page 6 of 9


Bank Po Mains PDF Course 2024
English Day - 15

Here, from the lines, Dryness is a major problem rampantly," the expert said. We can say that
in eye health these days, especially among option D is the correct answer.
children who study for a long time and those who
are in front of the screens. Many people look at 7. Answer: D
their screens for 8-14 hours a day which can Here, as per the lines, When we eat a meal
cause extreme dryness. In summer, dryness is a plate, which is colourful like a rainbow, we get all
feeling in which a person puts mud in their eyes sorts of antioxidants in our diet. Since our
and starts rubbing it in. It can cause your eyes childhood, we have been taught that an apple a
then become red, irritated and watery," said Dr day keeps the doctor away. Likewise, carrots are
Gopal Pillai. To prevent dry eyes, wash your known to be best for the eyes. It is rich in beta-
eyes frequently, suggested Dr Pillai. "After about carotene and vitamin, which protects the eye
every few minutes, voluntarily, close your eyes against infections," Jyoti Khaniojh said. Besides
tightly and then open them. Watch your eyes this, add lemon and citrus fruits to your diet for
more often as it will help lubricate the eyeballs," vitamin C intake. Nuts and seeds that contain
he said. We can say that option D is the correct vitamin E prevent cataracts and age-related
answer. macular degeneration. Salmon fish is a rich
source of omega-3 and this healthy fat helps in
5. Answer: D prevent eye dryness as it is great for retina
Here, as per the lines, Another major problem in health. Egg yolks contain vitamin A, zinc, lutein
summer is ocular diseases like conjunctivitis, and zeaxanthin and zinc, which help in age-
keratitis, endophthalmitis, cellulitis and stye, related degeneration of the eyes. Zinc helps in
among others. Dr Pillai said that conjunctivitis is the night improvement of night vision. We can
one of the most common eye problems in the hot say that option D is the correct answer.
months. We can say that option D is the correct
answer. 8. Answer: E
Here, in this question, there is no error. So,
6. Answer: D option E is the correct answer.
Here, as per the lines, Conjunctivitis spreads via
human touch. Someone could use an infected 9. Answer: C
towel or even a door knob that has been touched Here, the error lies in part c as the use of he is
by an infected person. In a classroom, one child inappropriate as here in this question there is no
could infect another child if they have gender-specific thing so he or she cannot be
conjunctivitis. It's a viral infection, so it spreads

Click Here For Bundle PDF Course | support@guidely.in Page 7 of 9


Bank Po Mains PDF Course 2024
English Day - 15

used as per the context, instead it should have Here, ultimately should have been used in place
been used. So, option C is the correct answer. of penultimately as ultimately means in the end,
and here ultimately fits best whereas
10. Answer: C penultimately means next to the last which is
Here, the error lies in part c as the use of for is inappropriate as per the context of the sentence.
inappropriate before Air Costa as of should have So, option C is the correct answer.
been used instead of for. Of shows possession
and here license was possessed by the 15. Answer: C
company so it should have been used. So, Here, apprehensive should have been used here
option C is the correct answer. in place of comprehensive as comprehensive
means including everything or nearly everything
11. Answer: B that is connected to a particular subject whereas
Here, the error lies in part b as the use of begun apprehensive means worried or afraid that
is inappropriate, and began should have been something bad may happen and here failure of
used here because the sentence is in past airlines is the main focus of the sentence. So,
simple tense and here the second form of the option C is the correct answer.
verb should have been used. So, option B is the
correct answer. 16. Answer: B
Here, prospects should have been used in place
12. Answer: C of respects which is contextless here as per the
Here, the error lies in part c as the use of an context of the sentence. Prospects mean a
before European is inappropriate as an is used possibility that something might happen whereas
before words which begins with a vowel sound, respect means polite behavior or care towards
and European sounds like European and somebody and here former fits as per the
therefore an cannot be used before it, instead a context of the sentence. So, option B is the
should have been used. So, option C is the correct answer.
correct answer.
17. Answer: D
13. Answer: E Here, which should be replaced with who as
Here, none of the highlighted words are here economists word is used which is a living
inappropriately used. So, option E is the correct being and for them who should have been used
answer. there and add should have been replaced with
14. Answer: C aid as add means to put something together and

Click Here For Bundle PDF Course | support@guidely.in Page 8 of 9


Bank Po Mains PDF Course 2024
English Day - 15

aid means help and here latter fits best. So, Here, have should be replaced with has been as
option D is the correct answer. the sentence is in present perfect continuous
tense and Apple is a singular subject and
18. Answer: D therefore we will use a singular helping verb
Here, towards should have been replaced with which is has been. So, option C is the correct
away from as land has been bough in India answer.
which means the company has bought land
away from China and therefore option D fits 20. Answer: E
best. So, option D is the correct answer. Here, there is no need for any improvement. So,
option E is the correct answer.
19. Answer: C

Click Here For Bundle PDF Course | support@guidely.in Page 9 of 9


Bank Po Mains PDF Course 2024
Reasoning Day -16 (Eng)

Reasoning Aptitude
Directions (1-5): Study the following information a) T
carefully and answer the given questions. b) The one who joined on June 7th
Nine persons – O, P, Q, R, S, T, U, V, and, W c) The one who joined two persons after S
joined the bank in three different months viz.- d) O
March, June, and August on three different dates e) V
– 4th, 7th, and 9th. Only one person joined on one
date of each month. All persons like different 2) Who among the following person joined on 7th
shoe brands viz.- Mochi, Puma, Reebok, Bata, August?
Relaxo, Nike, Red Chief, Liberty, and Adidas. a) The one who likes Bata
The one who likes Bata joined on an even b) The one who joined two persons after U
numbered date in a month having an odd c) The one who likes Puma
number of days. Only three persons joined d) V
between the one who likes Bata and W, who e) P
joined immediately after the one who likes Nike.
R joined four persons before the one who likes 3) Four of the following five are alike in a certain
Red Chief. At least one but not more than three way based on the given arrangement and thus
persons joined between the one who likes Bata form a group. Which one of the following does
and R. Both R and O like neither Nike nor not belong to the group?
Adidas. S joined three persons before the one a) T
who likes Adidas. Both O and S like neither Bata b) Q
nor Nike. As many persons joined before the one c) W
who likes Adidas as after O. As many persons d) R
joined between the one who likes Red Chief and e) S
O as between P and the one who likes Relaxo.
The one who likes Relaxo joined immediately 4) How many persons joined between U and the
after U, who joined on an even numbered date. P one who likes Reebok?
and the one who likes Mochi joined in the same a) As many persons joined before the one who
month. Only three persons joined between T and likes Nike
the one who likes Mochi. More than two persons b) Three
joined between the one who likes Liberty and Q, c) As many persons joined between V and the
who likes neither Red Chief nor Reebok. one who likes Reebok
1) Who among the following person likes d) Four
Relaxo? e) None

Click Here For Bundle PDF Course | support@guidely.in Page 1 of 10


Bank Po Mains PDF Course 2024
Reasoning Day -16 (Eng)

digital sum of the outcome from top to bottom


5) Which of the following statements is/are not respectively.
true as per the given arrangement? Arrangement 1:
a) T joined in August D, I, R, Z, T, E, O, F, K, U, G, P, and V are to be
b) The one who likes Puma joined immediately filled against the outcomes. Only one letter is
after V placed along with each outcome.
c) Three persons joined before U I. All vowels are placed in reverse alphabetical
d) Either B or C order along with each outcome whose digital
e) All the above statements are true sum is a prime number from top to bottom
respectively.

Directions (6-10): Study the following information II. All consonants which come before M are

carefully and answer the given questions. placed in reverse alphabetical order along with

There are nine levels marked one to nine from each outcome whose digital sum is an even

bottom to top respectively. There are thirteen number from top to bottom respectively.

outcomes which are placed in these nine levels. III. All the remaining letters are placed with the

At least one and at most two outcomes are remaining outcomes from bottom to top in

placed on each level. No two levels have the alphabetical order. (If more than two outcomes

same outcome. Two dice are thrown and their are at the same level, then the digital sum of the

outcomes are given below: outcome is highest given first preference)

(6, 4), (6, 3), (4, 5), (1, 2), (1, 5), (1, 1), (3, 2), (5, Arrangement 2:

3), (3, 3), (5, 2), (4, 4), (6, 6), and (5, 5). All the letters are placed in a row from the left

Condition: end based on the descending order of their

I) If the digital product of the outcome is digital sum of the outcome and facing north. (If

even(more than 16), then the respective the outcome of letters are same, then give

outcomes are arranged in descending order and preference in alphabetical order).

placed at an even numbered level from top to 6) Which of the following letter is placed second

bottom respectively. to the right of the letter which is fifth to the left of

II) If the digital product of the outcome is odd, G in arrangement 2?

then the respective outcomes are arranged in a) Z

ascending order and placed at an odd numbered b) The letter which is placed immediate left of E

level from top to bottom respectively. c) The letter which is placed fourth from the left

III) The remaining outcomes are placed on an end

even numbered level in ascending order of the d) O


e) F

Click Here For Bundle PDF Course | support@guidely.in Page 2 of 10


Bank Po Mains PDF Course 2024
Reasoning Day -16 (Eng)

a) U, 8, K
7) ______ is the outcome of the letter which is b) F, 6, D
_______ to the right of V in arrangement 2. c) G, 7, I
a) (1, 1), Fourth d) Both B and C
b) (4, 5), Second e) I, 7, O
c) (1, 5), Third
d) (6, 4), Fifth Directions (11-15): Study the following
e) None of these information carefully and answer the given
questions.
8) Which of the following letters are placed at Sixteen persons from A to P are sitting around
even numbered levels? the two concentric square tables in such a way
I. TRD that eight persons are sitting at the inner table
II. RUF facing away from the centre whereas eight
III. FRT persons are sitting at the outer table facing
IV. OTK towards the centre. One person sits at each
a) Only III and IV corner whereas one person sits in the middle of
b) Only III each side of both tables.
c) Only I, III, and IV Note: I. Consecutive alphabetically named
d) Only I, II and IV persons are neither sitting at the same table nor
e) None facing each other.
II. If A is facing B, then both are facing each
9) Four of the following five are alike in a certain other at different tables, if A sits opposite to B,
way based on the given arrangement 2 and thus then both are sitting at the same table.
form a group. Which one of the following does H sits third to the right of the one who is facing C,
not belong to the group? who sits at the corner of the inner table. One
a) The letter which is placed third to the right of G person sits between C and E, who sits opposite
b) F to G. The one who is facing G sits third to the left
c) The letter which is placed second to the right of N. Only three persons sit between F and D,
of P who sits adjacent to H. As many persons sit
d) E between F and J as between K and M, when
e) O counted from the left of both F and K. K sits
opposite to O, who neither sits adjacent to C nor
10) _____ letter is placed at level _____ and E. Both M and I don’t sit adjacent to E. Only
immediately above ______. three persons sit between A and I. The number

Click Here For Bundle PDF Course | support@guidely.in Page 3 of 10


Bank Po Mains PDF Course 2024
Reasoning Day -16 (Eng)

of persons sitting between I and O is one less a) N


than the number of persons sitting between B b) The one who sits third to the left of D
and J, when counted from the right of both B and c) The one who faces I
I. Two persons sit between B and P. d) Both A and B
11) What is the position of C with respect to the e) Either B or C
one who faces J?
a) Immediate right 15) Who among the following persons are sitting
b) Second to the left between the one who faces I and B, when
c) Third to the left counted from the left of B?
d) Second to the right I. O
e) Immediate left II. The one who sits second to the left of L
III. The one who faces C
12) How many persons are sitting between A a) Only II
and the one who faces N, when counted from the b) Only I and III
right of A? c) Only III
a) Two d) All I, II, and III
b) One e) Only II and III
c) Three
d) More than three Directions (16-20): Study the following
e) None information carefully and answer the given
questions.
13) Four of the following five are alike in a certain In a certain code language following statements
way based on the given arrangement and thus are coded as follows:
form a group. Which one of the following does “Burn Are More Empty Close” is coded as “#%+
not belong to the group? @%¥ #$¥ @^* @^¥”.
a) L “Went All File Above Equal” is coded as “@^¥
b) The one who sits second to the right of A @^© #$£ #%£ #%¥”.
c) N “Blue Copy Age Down Today” is coded as “@%+
d) The one who sits third to the left of B #$¥ @^+ @^* @^¥”.
e) O Note: (All the given codes are three symbols
codes only)
14) If K is related to F, similarly D is related to O, 16) How “went” is coded in the given code
then who among the following person is related language?
to E? a)#$¥

Click Here For Bundle PDF Course | support@guidely.in Page 4 of 10


Bank Po Mains PDF Course 2024
Reasoning Day -16 (Eng)

b) @^© d) Either A or B
c)@^¥ e) Either B or C
d)@%¥
e) @^* 19) What may be the code of “Run Today
Away”?
17) Which of the following phrase is coded as a) @%* @%+ #^+
“#$¥ #%£”? b)@$* #%* #^+
a) Age Equal c)@$* @%+ #^+
b) Down Empty d) Either A or B
c) Are Equal e) None of these
d) Both A and C
e) None of these 20) What is the code of “Empty File”?
a) #*^ @^£
18) How “More Copy” is coded in the given code b) #%+ @^¥
language? c) #*^ @^+
a) @^¥ @^+ d) @%^ #^¥
b) @^* #$¥ e) None of these
c) @^¥ #$¥
Click Here to Get the Detailed Video Solution for the above given Questions
Or Scan the QR Code to Get the Detailed Video Solutions

Answer Key with Explanation


Direction (1-5): 4. Answer: A
1. Answer: B 5. Answer: B
2. Answer: C
3. Answer: D(All the persons joined in the month
having 31 days except option D)

Click Here For Bundle PDF Course | support@guidely.in Page 5 of 10


Bank Po Mains PDF Course 2024
Reasoning Day -16 (Eng)

Again, we have:
 R joined four persons before the one who
likes Red Chief.
 At least one and not more than three
persons joined between the one who likes
Bata and R.
 Both R and O like neither Nike nor
Adidas.
That means, in case (1) R joined on 9
March, in case (2) R joined on 7 June, in
case (2a) R joined on 4 June.
Based on the above given information we have:

We have:
 The one who likes Bata joined on an even
numbered date in a month having an odd
number of days.
 Only three persons joined between the
one who likes Bata and W, who joined
immediately after the one who likes Nike.
That means, in case (1) the one who likes
Bata joined on 4 March, in case (2) the
one who likes Bata joined on 4 August. Again, we have:
Based on the above given information we have:  S joined three persons before the one
who likes Adidas.
 Both O and S like neither Bata nor Nike.
 As many persons joined before the one
who likes Adidas as after O.
Since, R doesn’t like Adidas.
That means, in case (2) S joined on 4
March, in case (2a) R joined on 9 June,
case (1) is not valid.
Based on the above given information we have:

Click Here For Bundle PDF Course | support@guidely.in Page 6 of 10


Bank Po Mains PDF Course 2024
Reasoning Day -16 (Eng)

Case (1) is not valid as O doesn’t like Bata.


Again, we have:
 As many persons joined between the one
who likes Red Chief and O as between P
and the one who likes Relaxo. Case (2a) is not valid as the one who likes
 The one who likes Relaxo joined Relaxo joined immediate after U.
immediately after U, who joined on an Again, we have:
even numbered date.  More than two persons joined between
That means, in case (2) U joined on 4 the one who likes Liberty and Q, who likes
June, case (2a) is not valid. neither Red Chief nor Reebok.
 P and the one who likes Mochi joined in Thus, S likes Liberty and O likes Reebok.
the same month. Based on the above given information we have:
 Only three persons joined between T and
the one who likes Mochi.
That means, W likes Mochi and T likes
Bata.
Based on the above given information we have:

Click Here For Bundle PDF Course | support@guidely.in Page 7 of 10


Bank Po Mains PDF Course 2024
Reasoning Day -16 (Eng)

Direction (6-10):
6. Answer: B
7. Answer: C
8. Answer: A
9. Answer: A(All the letters are placed in the
middle of the row except option A)
10. Answer: D
Arrangement 1:

Arrangement 1:
D, I, R, Z, T, E, O, F, K, U, G, P, and V are to be
filled against the outcomes. Only one letter is
placed along with each outcome.
I. All vowels are placed in reverse alphabetical
order along with each outcome whose digital
sum is a prime number from top to bottom
respectively.
Thus, U, O, I, and E are placed.
Arrangement 2: II. All consonants which come before M are
placed in reverse alphabetical order along with
each outcome whose digital sum is an even
number from top to bottom respectively.
We have:
Thus, K, G, F, and D are placed.
Outcome: (6, 4), (6, 3), (4, 5), (1, 2), (1, 5), (1, 1),
III. All the remaining letters are placed with the
(3, 2), (5, 3), (3, 3), (5, 2), (4, 4), (6, 6), and (5,
remaining outcomes from bottom to top in
5).
alphabetical order. (If more than two outcomes
Even digital products: 36, 24, 20, 18,
are at the same level then the digital sum of the
Odd Digital products: 1, 5, 9, 15, 25
outcome is highest given first preference)
Digital sum: 3, 5, 7, 8
Thus, the remaining letters: P, R, T, V, and Z are
placed
Based on the above conditions we have:

Click Here For Bundle PDF Course | support@guidely.in Page 8 of 10


Bank Po Mains PDF Course 2024
Reasoning Day -16 (Eng)

We have:
 H sits third to the right of the one who is
facing C, who sits at the corner of the
inner table.
Arrangement 2:
 One person sits between C and E, who
All the letters are placed in a row from the left
sits opposite to G.
end based on the descending order of their
That means, in case (1) E sits second to
digital sum of the outcome and facing north. (If
the left of C, in case (2) E sits second to
the outcome of letters are same, then give
the right of C.
preference in alphabetical order).
 The one who is facing G sits third to the
Based on the above given condition we have:
left of N.
Based on the above given information we have:

Direction (11-15):
11. Answer: C
12. Answer: B
13. Answer: A(All the persons are sitting in the
middle of the table except option a) Again, we have:

14. Answer: D  Only three persons sit between F and D,

15. Answer: C who sits adjacent to H.


Since, the persons in alphabetical order
are neither sitting at the same table nor
sits facing each other.
That means, in case (1) & case (2) D sits
immediate right of H.

Click Here For Bundle PDF Course | support@guidely.in Page 9 of 10


Bank Po Mains PDF Course 2024
Reasoning Day -16 (Eng)

 As many persons sit between F and J as Case (1) & case (2) are not valid as no place
between K and M, when counted from the available for I and A.
left of both F and K. Again, we have:
 K sits opposite to O, who neither sits  Two persons sit between B and P.
adjacent to C nor E. That means, P sits immediate left of F.
 Both M and I don’t sit adjacent to E. Based on the above given information we have:
Since, we have to consider the number of
persons between F and J, thus J and K
can’t sit at the same table.
That means, in case (1a) M is facing D, in
case (1) M sits immediate right of C, in
case (2) M sits immediate left of C.
Based on the above given information we have:

Direction (16-20):
16. Answer: B
17. Answer: D
18. Answer: A
Again, we have:
19. Answer: C
 Only three persons sit between A and I.
20. Answer: B
 The number of persons sitting between I
We have:
and O is one less than the number of
For the first symbol: If the word starts with a
persons sitting between B and J, when consonant then it is coded as ‘@’, else it is
counted from the right of both B and I.
coded as ‘#’.
That means, in case (1a) I sits immediate
For the second symbol: the second symbol
right of C, case (1) & case (2) are not
represents the number of letters in the word.
valid.
For 3  $, 4  ^, and 5  %
Based on the above given information we have:
For the third symbol: the third symbol represents
the last letter of the word.
Y  +, N  *, E  ¥, and L  £

Click Here For Bundle PDF Course | support@guidely.in Page 10 of 10


Bank Po Mains PDF Course 2024
Quantitative Aptitude Day -16 (Eng)

Quantitative Aptitude

Directions (01 - 04): Study the following information carefully and answer the questions given below.
There are five showrooms [A, B, C, D, and E]. In each showroom different numbers of diesel, petrol, CNG
and electric cars sold. The given charts show the difference between the number of diesel and electric,
diesel and CNG, petrol and electric, petrol and electric, and CNG and electric cars sold in showrooms A,
B, C, D, and E respectively. The number of diesel cars sold in each shop is the maximum and the
number of electric cars sold in each shop is the minimum.

NOTE – I. The total number of cars sold in A is 420. The number of CNG cars sold in A is double the
number of electric cars sold in A. The number of petrol cars sold in A is 50% more than the number of
CNG cars sold in A.
II. The number of Petrol cars sold in A is 20% more than the number of petrol cars sold in B. The number
of diesel cars sold in B is 22.22% less than the number of diesel cars sold in A. The ratio of the number of
CNG and Electric cars sold in B is 7:5.
III. The total number of petrol and electric cars sold in C is 200. The number of CNG cars sold in C is 3/2
of the number of electric cars sold in C. The number of Diesel cars sold in C is 166.66% more than the
number of electric cars sold in C.

Click Here For Bundle PDF Course | support@guidely.in Page 1 of 13


Bank Po Mains PDF Course 2024
Quantitative Aptitude Day -16 (Eng)

IV. Difference between the number of diesel and CNG cars sold in D is 120, the sum of CNG and electric
cars sold in D is 120. The ratio of petrol and diesel cars sold in D is 4:5 respectively.
1) The number of diesel cars sold in E is 10% 3) Difference between the number of petrol cars
less than the same in D. The sum of Diesel and and CNG cars sold in A is _________ % of the
CNG cars sold in E is 240 and the difference difference between the number of petrol cars and
between the number of diesel and petrol cars electric cars sold in D and the Difference
sold in E is 60. The ratio of the Top model and between the number of petrol cars and CNG cars
the base model of all type's cars sold in E is 5:3. sold in B is ________ % of the difference between
The average price of the Top model car and the number of petrol cars and electric cars sold
Base model car sold in E is 15 lakh and 12 lakhs in C.
respectively. Find the total revenue (in lakhs) Find which of the following satisfies the blank?
earn by E? a) 37.5%, 33.33%
a) 2540 b) 33.33%, 37.5%
b) 8520 c) 62.5%, 37.5%
c) 5550 d) 66.66%, 33.33%
d) 6500 e) None of these
e) 7500
4) The number of XUV sold in C and D is more
2) Find which of the following is/are true? than the number of sedan cars sold in C and D
I. k is the difference between the number of out of the total number of diesel and petrol cars
diesel and petrol cars sold in C and l is the sold in C and D respectively. The difference
difference between the number of CNG and between the number of XUV and sedan petrol
Electric cars sold in C. So k2+k*l+l2=2900. cars and diesel cars out of total diesel and petrol
II. Sum of electric cars sold in A, B and C cars sold in C is 40 respectively and Difference
together is 150. between the number of XUV and sedan petrol
III. Difference between the total number of cars cars and diesel cars out of total diesel and petrol
sold in C and D is 40. cars sold in D is 40 respectively. The number of
a) all false XUVs sold in C and D together is what percent of
b) all true the number of sedan cars sold in C and D
c) only II and III together?
d) only II a) 164%
e) only III b) 154%
c) 157%
d) 132%

Click Here For Bundle PDF Course | support@guidely.in Page 2 of 13


Bank Po Mains PDF Course 2024
Quantitative Aptitude Day -16 (Eng)

e) None of these less than the number of students who liked both
math and physics but not chemistry. The number
5) A shopkeeper marked up an article by _______ of students who like only physics and math but
% above the cost price and give discount of not chemistry is 15. The number of students who
______% and make profit of 20%. The cost price like only physics is 30. The number of students
of the article is ________. who passed only Math is three times the number
Find which of the following satisfies the blank? of students who like both math and physics but
I.60%, 25% ,240 not chemistry. The number of students who like
II.50%, 20%,360 both chemistry and math but not physics is three
III. 75%, 15%, 420 times the number of students who passed all
a) Only I and II three exams, which is equal to 6 and the number
b) Only II and III of students like all three subject is 5. The number
c) only I and III of students who passed both math and chemistry
d) both I and II but not physics is 7. The ratio of the number of
e) None of these students like only math to the number of students
passing only Math, the number of students who
Directions (06 - 09): Study the following like only Math to the number of students who
information carefully and answer the questions pass only chemistry, the number of students who
given below. like only chemistry and the number of students
There is a survey taken among the students. who passed only chemistry is 7:9, 7:5 and 8:5
Each student likes at least one subject respectively.
chemistry, physics, and math. Each student 6) Ratio of the number of male and female
passed at least one subject. The number of students who like only chemistry is 3:5
students who passed both physics and chemistry respectively and the ratio of the number of male
but not math is 33.33% less than the number of and female students who passed only chemistry
students who like both chemistry and physics but is 3:2 respectively . Find the difference between
not Math. The number of students who passed the number of female students who like only
only physics is 37.5% more than the number of chemistry and the number of female students
students who like only chemistry. The number of who passed only chemistry?
students who like both chemistry and math but a) 10
not physics is 50% more than the number of b) 19
students who like both chemistry and physics but c) 15
not math. The number of students who passed d) 13
both physics and math but not chemistry is 40% e) 11

Click Here For Bundle PDF Course | support@guidely.in Page 3 of 13


Bank Po Mains PDF Course 2024
Quantitative Aptitude Day -16 (Eng)

7) Quantity: I The total number of students who 9) x is the percentage of the number of students
like physics is what percent of the number of who like all three subjects out of the total number
students who passed in Physics? of students who passed all three subjects and y
Quantity: II The total number of students who like is the percentage of the number of students who
math is what percent of the number of students passed only physics out of the number of
who passed in Math? students who passed only math. Find the value
A. Quantity: I < Quantity: II of 2x+3y?
B. Quantity: I ≥ Quantity: II a) 533.32
C. Quantity: II ≥ Quantity: I b) 543.2
D. Quantity: I > Quantity: II c) 582.33
E. Quantity I = Quantity II or relation can't be d) 592.37
established e) None of these

8) Difference between the number of students 10) A can complete a work alone in 40 days and
who like both chemistry and physics but not math B can complete the same work alone in 30 days.
and the number of students who passed both When A work alone but takes a break one day
math and physics but not chemistry is _______ after every 5 days of work, then the total work
and the difference between the number of completed in n days. When B works alone but
students who like both chemistry and math but takes a break one day after every 5 days of
not physics and the number of students who work, total work is completed in m days. D
passed both chemistry and physics but not math completes the work in m+25 days and C
is _______. ;Which is 1/3rd of the Multiply of completes n-23 days. Find the time taken by C
values of two blanks is ______. and D together to complete the work?
Find the possible value to satisfy the blank? a) 120/7
a) 12, 10 , 40 b) 145/7
b) 3, 10, 12 c) 120/9
c) 12, 4, 16 d) 120/11
d) 10, 3, 10 e) None of these
e) None of these

Directions (11 - 14): Study the following information carefully and answer the questions given below.
The given pie chart shows the percentage distribution of the speed of five different boats [A, B, C, D and
E] in still water.

Click Here For Bundle PDF Course | support@guidely.in Page 4 of 13


Bank Po Mains PDF Course 2024
Quantitative Aptitude Day -16 (Eng)

The given pie charts show the percentage distribution of the speed of streams of five different rivers [R,
S, T, U and V].

Note – The speed of boat B in still water is 18 km/hr and the downstream speed of boat D in river V is 57
km/hr.
11) Boat A covers d km distance downstream in the following given boats in upstream in river U
river R and then the same distance upstream in can cover a distance 2d km within n hours?
n hours. The same boat covers d+60 km I. Boat D
downstream in river T in 5 hours. Find which of II. Boat E
III. Boat A

Click Here For Bundle PDF Course | support@guidely.in Page 5 of 13


Bank Po Mains PDF Course 2024
Quantitative Aptitude Day -16 (Eng)

a) only I and II e) None of these


b) both I and III
c) only I 14) Find which of the following is/are true?
d) all three I. Boat B covers 48 km downstream in river U
e) none and 64 km downstream in river S in 3.32 hours.
II. Boat C covers 88 km downstream in river U
12) River S and River T meet at point M and and 54 km downstream in river T in 4.3 hours.
move towards N WHICH IS considered as the III. Boat E covers 84 km downstream in river U
name of river U. O and P are two points of river and 104 km downstream in river U in 3.41 hours.
S and T respectively. Distance from M to O and a) Both I and III
P is 70 km and 80 km respectively. The distance b) Both I and II
from M to N is 75 km. Boat C and D start the c) All three
same journey towards M from O and P d) only I
respectively and then reached N. Find the e) None of these
difference in time taken by boats C and D to
reach N from their starting point? 15) Jar A contains _______ litres of mixture of
[ All the distance covered downstream] milk and water, the percentage of Milk is 60%.
a) 1.78 Jar B contains _____ litres of a mixture of milk
b) 2.23 and water, in a ratio of 5:3. 40% mixture of
c) 2.01 mixture A and 60% of mixture B is mixed in C.
d) 1.42 Difference of milk and water in C is ______.
e) 1.98 Find which of the following satisfies the blank?
a) 150, 120, 22.6
13) In river T, boat D goes a certain distance b) 100, 96 , 12.6
downstream and then returns and covers some c) 124, 88, 17.6
part of the distance in upstream in total 9 hours. d) 120, 80 , 21.6
The same distance that boat D covers in e) None of these
upstream, boat B can cover in 17.5 hours in
upstream in river V. Find the time taken by boat Directions (16 - 19): Study the following
D to return to the starting point? information carefully and answer the questions
a) 16.42 hours given below.
b) 17.14 hours 16) Find the profit share of B in business 1? [ A
c) 17.35 hours and B invest in each business at different times]
d) 21.33 hours

Click Here For Bundle PDF Course | support@guidely.in Page 6 of 13


Bank Po Mains PDF Course 2024
Quantitative Aptitude Day -16 (Eng)

Statement I: The profit share of A in Business 2 Statement I: Sum of the age of A and B after 5
is 20% more than the profit share of A in years is 85 years. The ratio of age of C and E is
Business 1. 5:3. Sum of the age of C and D is 54 years.
Statement II: Total share of profit in business 1 is Statement II: The sum of age of A and E is 58
6800. B invest 10 months in each business. years and the average age of A and D is 32
Statement III: B invests Rs.14000 in business2. years.
The total profit share of A in business1 and 2 is Statement III: D is 6 years older than E. Ratio of
Rs.5500. age between C and D is 5:4. Difference in age
a) All III together necessary between B and C is 5 years.
b) only I a) only I
c) only I and III b) all three are necessary to answer
d) I and II together c) only III
e) only III d) only I and II
e) None of these
17) Find the volume of the cuboid? [ take pi = 3]
Statement I: The length of the cuboid is equal to 19) Find the compound interest after 2 years?
the height of the cylinder whose volume is Statement I: The rate of interest is 30%.
25137. The curved surface area of the cylinder is Statement II: If the same principle invests at
2394. simple interest for 3 years at the same interest
Statement II: The breadth of the cuboid is equal rate, then the interest amount is Rs.10800.
to the radius of the cone whose volume is 486. Statement III: For the same principle invests the
The breadth of the cuboid is 4 more than the amount of compound interest at R% for 2 years
radius of the sphere. The ratio of the length to is Rs.5280.
the breadth of the cuboid is 3:2 a) both I and II
Statement III: The height of the cuboid is equal to b) Only II
the radius of the sphere whose volume is 10976 c) only I and III
m. d) only III
a) only II e) None of these
b) only I
c) only I and III 20) Length of train A is ________ m and crosses
d) II and III together a platform in ________ sec. The length of train B
e) only III is double the length of the platform and the
speeds of train A and B is 90 km/hr and 72 km
18) Find the sum of the age of B and C?

Click Here For Bundle PDF Course | support@guidely.in Page 7 of 13


Bank Po Mains PDF Course 2024
Quantitative Aptitude Day -16 (Eng)

per hour respectively. Train B crosses the b) 480, 14, 40


platform in ________ sec. c) 240, 16, 36
Find which of the following satisfies the blank? d) 320, 18, 48
a) 320, 24, 42 e) None of these
Click Here to Get the Detailed Video Solution for the above given Questions
Or Scan the QR Code to Get the Detailed Video Solutions

Answer Key with Explanation


Directions (01 - 04): The number of petrol cars sold in A is
Let the number of petrol and diesel cars sold in 2x*150/100 = 3x.
D be 4y and 5y respectively. Or, x+x+140+3x+2x=420
The number of CNG cars sold in D is 5Y-120. Or, 7x = 420-140 = 280, and x = 40.
The number of Electric cars sold in D is 120- So, the number of electric cars sold in A is 40,
5y+120=240-5y. and the number of CNG cars sold in A is 2* 40 =
The number of electric cars sold in D is 4Y-120. 80.
So, 240-5y=4y-120 The number of diesel cars sold in A is x + 140 =
Or, 9y = 360, y = 40. 40 + 140 = 180.
The number of petrol and diesel cars sold in D is The number of petrol cars sold in A = 3x = 3*40
4* 40 = 160 and 5* 40 = 200, respectively. = 120.
The number of CNG cars sold in D is 5y-120 = The number of petrol cars sold in B is 120 *
200-120 = 80. 100/120 = 100.
The number of electric cars sold in D is 4y- The number of diesel cars sold in B is 180 *
120=160-120=40. [100-22.22]/100 = 180* [100-22.22]/100 = 140.
Let the number of electric cars sold in A be x and The Number of CNG cars sold in B is 140-
the number of CNG cars sold in A be 2x. 70=70.
The number of diesel cars sold in A is x+140.

Click Here For Bundle PDF Course | support@guidely.in Page 8 of 13


Bank Po Mains PDF Course 2024
Quantitative Aptitude Day -16 (Eng)

The number of electric cars sold in B is 70*5/7 = So, II is true.


50. III. Difference between the total cars sold in C
The number of petrol cars sold in C = and D is
[200+80]/2 = 140. = [200+160+80+40] – [160+140+90+60]=30.
The Number of electric cars sold in C = 140- So, III is false
80=60
The Number of CNG cars sold in C is 60*3/2 = 3) Answer: B
90. Value for first blank = [40/120] *100=33.33%
The Number of Diesel cars sold in C is 60 * Value of for second blank = [30/80]*100=37.5%
266.66 / 100=160
1) Answer: C 4) Answer: A
The Number of diesel cars sold in E is Number of XUV sold in C = [160+40]/2 +
200*90/100=180 [140+40]/2 =100+90=190
The Number of CNG cars sold in E is = 240- Number of XUV sold in D = [200+40]/2 +
180=60 [160+40]/2 = 120+100=220
The Number of Petrol cars sold in E is = 180- Total number of sedan cars sold in C and D =
60=120 [300-190]+ [360-220] =110+140=250
The Number of electric cars sold in E is = 60- So, required percentage = [410/250] *100=164%
20=40
Total cars sold in E is = 180+120+60+40=400 5) Answer: A
So, total revenue is For I, the cost price is 240, marked price =
= [400*5/8]*15 + [ 240*160/100=384
400*3/8]*12=250*15+150*12=5550 lakh Selling price = 384*75/100=288
So, profit percentage = [288-240]*100/240=20%
2) Answer: D So, it's satisfying.
k is the difference between the number of diesel Similarly, we can check others' values also,
and petrol sold in C, so k= 160-140=20, Only I and II satisfy.
l is the difference between the number of CNG
and Electric cars sold in C, so I= 90-60=30 Directions (06 - 09):
So, 202+20*30+302=1900. The number of students who passed in all three
So, I is false subjects is 6.
II. Sum of the number of electric cars sold in A, B Number of students who like all three subjects is
and C together is =40+50+60= 150. 5.

Click Here For Bundle PDF Course | support@guidely.in Page 9 of 13


Bank Po Mains PDF Course 2024
Quantitative Aptitude Day -16 (Eng)

Number of students who like only physics and


math but not chemistry is 15.
Number of students who passed only math =
15*3 = 45.
The Number of students who like only math is
45*7/9 = 35.
Number of students who passed only chemistry
= 35*5/7 = 25
Number of students who like only chemistry =
Venn diagram of the number of students who
25*8/5 = 40
like the subjects.
Number of students who passed only physics =
40* 137.5/100 = 55
Number of students who like only physics = 30.
Number of students who passed both math and
physics but not chemistry is = 15*60/100=9
Number of students who like both chemistry and
Math but not physics is 6* 3 = 18.
Number of students who like both physics and
chemistry but not math
=18*100/150=12
Number of students who passed both chemistry
and math but not physics is 7. 6) Answer: C
The number of students who passed both Required difference = 40*5/8 – 25 *2/5= 25-
chemistry and physics but not math is = 10=15
12*66.66/100=8
VENN DIAGRAM of the number of students who 7) Answer: A
passed Quantity I
Required percentage =
[(30+12+15+5)/(8+55+6+9)]*100=79.48%
Quantity II
Required percentage =
[(35+18+15+5)/(7+45+6+9)]*100=108.95
Quantity I < Quantity II

Click Here For Bundle PDF Course | support@guidely.in Page 10 of 13


Bank Po Mains PDF Course 2024
Quantitative Aptitude Day -16 (Eng)

Directions (11 - 14):


8) Answer: E The speed of the boat B in still water is 18 km/hr.
Possible value for 1st blank is = [12-9]=3 So, total value of 1st pie chart is =
Possible value for 2nd blank is =[18-8]=10 [18/12]*100=150
So, possible value for 3rd blank is = [10*3]/3=10 So, the speed of boat D is = 150*30/100=45
So, the speed of a stream of river V is 57-45=12
9) Answer: A km/hr
So, x =[5/6]*100=83.33% So, total value of 2nd pie chart is =
So, y=[55/45]*100=122.22% [12/15]*100=80
So, 2*83.33 + 122.22*3=533.32

10) Answer: A
A completed in 40 days and B was completed in
30 days.
So, total work = LCM of 40 and 30 =120
11) Answer: E
A’s one days work is = 120/40=3 and B’s one
So, we can say, d/39 + d/ 15 = n
days work is = 120/30=4
2nd case,
So, when A takes a break after 5 days, means in
[d+60]/[27+24] = 5
6 days A completes 5*3=15 units of work, so,
Or, d=255-60=195
total work will complete
So, 195/39 +195 /15 = 5 +13 =18 =n
= [6*8-1]=47 days
Time taken by boat D to cover 2d distance in
So, when B takes a break after 5 days, means in
upstream in river U is = [2*195/(45-17.6]=14.23
6 days B completes 5*4=20 units of work, so, the
Time taken by boat E to cover 2d distance in
total work will complete
upstream in river U is = [2*195/(37.5-17.6]=19.59
= [6*6-1]=35 days
Time taken by boat A to cover 2d distance in
So, D takes 35+25=60 days to complete the
upstream in river U is = [2*195/(27-17.6]=41.49
work and C takes 47-23=24 days to complete
So, none of the given boats can cover the
the work.
distance in n hour.
The efficiency of C and D is 120/24=5 and
120/60=2 respectively.
12) Answer: D
So, C and D together can complete the work in =
Time taken by boat C = [70/[22.5+14.4]]
120/7 days
+[75/[22.5+17.6]
=1.89+1.87=3.76

Click Here For Bundle PDF Course | support@guidely.in Page 11 of 13


Bank Po Mains PDF Course 2024
Quantitative Aptitude Day -16 (Eng)

Time taken by boat D = [80/[45+24]] So, Quantity of Milk in C =72*40/100


+[75/[45+17.6] +50*60/100=58.8
=1.15+1.19=2.34 Quantity of Water in C = 48*40/100 +
So, difference = 1.42 30*60/100=37.2
So, the difference = 58.8-37.2=21.6
13) Answer: B
Distance that boat D covers in Upstream is = 16) Answer: A
17.5*[18-12] =105 From statement I, if the profit amount of A in
So, 105 km distance covered by boat D in business 1 is 5x then the profit share of business
upstream in river T in =105/[45-24]=5 hours 2 of A is 6x.
So, in 4 hours downstream boat cover = From statement II, the total profit share in
[45+24]*4=276 business 1 is 6800.
So, the total time taken by boat D to return to the From III, the total profit share of A is 5500
starting point From I and III, the profit share of A in business 1
=4+276/[45-24]=17.14 hours is = 5500*5/11=2500
From III, we can say the profit share of B in
14) Answer: A business 1 is 6800-2500=4300
I. Boat B covers 48 km downstream in river U All three statements together are necessary.
and 64 km downstream in river S in
=[48/(18+17.6)] +[64/(18+14.4)]= 3.32 hours. 17) Answer: D
II. Boat C covers 88 km downstream in river U From a single statement, we cannot get the
and 54 km downstream in river T in 9 hours answer.
=[88/(22.5+17.6)]+[54/(22.5+24]=3.35. From III we get the radius of the sphere from the
III. Boat E covers 84 km downstream in river U volume of the sphere.
and 104 km downstream in river U in 4/3*3*r*r*r=10976
=[84/(37.5+17.6)+104/[37.5+17.6]= 3.41 hours. So, r=14 m
And the height of the cuboid is equal to the
15) Answer: D radius of the sphere.
Let's check options D, From II we know that the breadth is 4 more than
Quantity of Milk in Jar A, 120*60/100=72 the height cuboid and the ratio of length and
Quantity of Water in Jar A 120-72=48 breadth is also given in II.
Quantity of Milk in Jar B = 80*5/8=50 Breadth is 14+4=18 m
Quantity of Water in Jar B = 80-50=30 Length is 14*3/2=21

Click Here For Bundle PDF Course | support@guidely.in Page 12 of 13


Bank Po Mains PDF Course 2024
Quantitative Aptitude Day -16 (Eng)

So, volume = 18*21*14 So, all the statements together are necessary
So, from II and III together we can calculate the 19) Answer: A
answer. From each single statement, we cannot calculate
the answer.
18) Answer: B But from I and II we get the answer.
From I, we get the sum of the age of A and B If the principal amount is p.
and the sum of the age of C and D. So, p*30*3/100=10800
Sum of the present ages of A and B = 85-10=75 Or, p=10800*100/90=12000
The sum of ages of C and D is = 54
From II, the Sum of the age of A and E is 58 and 20) Answer: A
the sum of the age of A and D is 32*2=64 Let's check option A,
From I and III, present age of C is 54*5/9=30, The length of train A is 320 m
present age of D is 24. The speed of train A is 90*5/18=25 m/sec
present age of E = 24-6=18 So, the length of the platform = 25*24-320=280
From II, we HAVE sum OF the ages A and E. So, the length of train B is 560
So, the present age of A is 58-18=40 The speed of train B is 72*5/18=20 m/sec
From I we get the sum of the present age of B So, required time = 3*280/20=42 sec
and A. So, satisfied.
So, the age of B is 75-40=35 Similarly, we can check others' values also.

Click Here For Bundle PDF Course | support@guidely.in Page 13 of 13


Bank Po Mains PDF Course 2024
English Day - 16

English Language
Directions (1-5): In the following questions three issued in Chawl land
columns are given containing three
several places scam case.
Sentences/phrases and three connectors. A
across the
sentence/phrase from the first column may or country
may not connect with another sentence/phrase
A. a-II-f
from the second column by using the given
B. c-III-d
connectors to make a grammatically and
C. b-I-e
contextually correct sentence. Each question has
D. a-II-f and c-III-d
five options, each of which display the
E. a-II-f, b-I-e and c-III-d
sequence(s) in which the sentences/phrases can
be joined to form a grammatically and
2.
contextually correct sentence.
1. Column I Column II Column III

Column I Column II Column III a) Indian I. or d) they want to


Railways carry goods
a) The I. despite d) heavy rain
announced new weighing more
Enforcement and the release
luggage rules, than the
Directorate of dam water
as a result of standard fixed
conducted into rivers.
which train by the railway.
searches at
passengers will
some locations
now have to
in Mumbai
either limit their
b) Tamil Nadu's II. in e) the luggage,
government connection independence
b) Railways II. such that e) will have to
school's with day event and
have fixed a pay more for
headmistress stated that her
limit on luggage carrying extra
refused to hoist religion
according to baggage while
the national disallowed her
each coach, travelling.
flag to do so.
c) Passengers III. if f) heavy goods
will now have to ranging from
c) Flood alerts III. due to f) the Rs 1,034
pay extra 40 kg to 70 kg
have been crore Patra

Click Here For Bundle PDF Course | support@guidely.in Page 1 of 10


Bank Po Mains PDF Course 2024
English Day - 16

can be kept in averages.


the train A. b-I-f and c-III-e
compartment. B. b-I-f, a-II-d and c-III-e
A. a-I-e and b-II-f C. b-I-f
B. c-III-d D. a-II-d
C. b-II-f and c-III-d E. c-III-e
D. a-I-e and c-III-d
E. a-I-e, b-II-f and c-III-d 4.

Column I Column II Column III


3.
a) India on I. Indeed d)the number
Column I Column II Column III
Wednesday of active cases
a) Of 7,239 I. and d)with the most recorded 9,062 came down to
cities, India is severe fresh COVID-19 1,05,058.
home to 18 of increase in infections that
the 20 cities PM2.5 pollution took its tally to
from 2010 to 4,42,86,256,
2019
b) In case a II. while e) the airlines
b) In the 20 II. yet e) China has passenger does have been
cities with seen the not comply with advised again
highest PM2.5 greatest the directions, on August 16
exposures, improvements. to strictly
residents in comply with
cities from India, COVID-19
Nigeria, Peru, protocol inside
the aircraft.
c) India and III. whereas f) Bangladesh
Indonesia have are exposed to c) The rise in the III. f) strict action
seen the most PM 2.5 levels number of subsequently will be taken
severe increase that are COVID-19 by airlines
in PM2.5 several-fold cases against the
pollution, higher than the passenger
global
A. b-III-f

Click Here For Bundle PDF Course | support@guidely.in Page 2 of 10


Bank Po Mains PDF Course 2024
English Day - 16

B. a-II-d and c-I-e D. a-I-e, b-III-f and c-II-d


C. a-II-d, b-III-f and c-I-e E. a-I-e
D. a-II-d
E. c-I-e Directions (6-10): Below are given sentences
that follow a word highlighted in bold above
5. them. These sentences have been made from

Column I Column II Column III words that are directly related to the word
highlighted. One of the sentences uses a word
a) India’s I. though d)would last for that may be an antonym or unrelated to the given
ambitious a month. word. Find that sentence and mark the same as
project to your answer:
translocate 6. Snag:
African cheetahs A. The biggest impediments to trade are no
has missed an longer tariffs but non-tariff barriers, such as
unofficial regulations or licences.
deadline of B. They must overcome a number of obstacles
August 15, before the restaurant can be opened.
C. Life itself, without the assistance of colleges
b) In February, II. which e) sources say
and universities, is becoming an advanced
India’s High the animals are
institution of learning.
Commissioner likely to arrive
D. My youngest sister is always more of a
to Namibia, within this year.
hindrance than a help when she tries to assist
Prashant
me with a task.
Agrawal,
E. Difficulty of communication was still a barrier,
c) The cheetahs III. however f) had stated and technology was still highly limited.
from Namibia that the
were currently in cheetahs were 7. Affluence:
medical expected to A. Because the movie star made over 10 million
quarantine arrive in India dollars a film, she was able to maintain an
by “mid-2022.” opulent lifestyle

A. b-III-f B. Hiding the lucre in many different accounts,


B. c-II-d the mobsters kept a watchful eye on their funds

C. a-I-e and c-II-d

Click Here For Bundle PDF Course | support@guidely.in Page 3 of 10


Bank Po Mains PDF Course 2024
English Day - 16

C. Many things in the physical world that we think 10. Cacophony:


of as scarce are not really scarce and just A. General Greene spoke finally, his voice jarring
presently beyond our ability to capture her out of her thoughts.
D. If you wish to play for the Indian team, you B. The heavy door made him jump with a grating
should make them wonder that you have such an sound.
elite class. C. Owls soar through the night sky, screeching
E. We have got plenty of wood today from the out their presence.
forest. D. The space inside the wagons was a din of
mules and man.
8. Incorrigible: E. The performers were elated by euphony of
A. Most of our family members are habitual of applause.
reading newspaper in the morning.
B. He was feeling guilty and depressed, Directions (11-15): In each of the following
repentant and scared. questions a sentence has been given with some
C. Tracey is a chronic worrier who gets upset highlighted group of words/phrases which may or
with everything with the wink of an eye may not be grammatically correct. Sentence is
D. We need to remember that God’s wrath does then followed by three alternatives. You are
burn against impenitent sinners required to choose the grammatically correct
E. These range from practice and mock drills to option that best replaces the incorrect phrase.
hardcore leaders who were keeping them in 11. He tried pulling the shawl over our eyes by
discipline. hiding the profit in a separate account, but we
were quick to catch onto his scheme.
9. Malign: a) Pulling the shroud over our eyes
A. She is always upfront and direct when it b) Pulling the wool over our eyes
comes to correcting me as she knows it will be c) To deceive
beneficial for me to listen to her. A. Only (c)
B. The evil queen was portrayed by the author as B. Both (a) and (b)
malignant and powerful C. Only (b)
C. You say she has become very spiteful since D. Both (b) and (c)
the last few weeks E. Only (a)
D. His hostile gaze met hers
E. The former vamp’s baleful look remained but 12. Prohibition will not curb an alcohol craving for
his eyes were blue, not red. alcohol; it will only make an addict seek out
clandestine ways to consume it.

Click Here For Bundle PDF Course | support@guidely.in Page 4 of 10


Bank Po Mains PDF Course 2024
English Day - 16

a) Alcoholic craving for alcohol 15. Instead of abating this sordid situation
b) Alcohol craving for alcoholic through additional fund allocation and
c) Alcoholic’s craving for alcohol appropriate policy intervention, the government is
A. Both (a) and (c) moving in the opposite direction, promoting and
B. Both (b) and (c) enriching rich oligarchy.
C. Only (a) a) Enrich richer oligarchs
D. Only (b) b) Enriching rich oligarchs
E. Only (c) c) Enrich rich oligarchy
A. Only (a)
13. Without the necessary infrastructure for B. Both (a) and (c)
rehabilitation medication and therapy, no ban, C. Both (b) and (c)
however, stringent will keep the teetotalers away D. Only (c)
from the bottle. E. Only (b)
a) Tippler away from the bottle
b) Boozer away from the bottle Directions (16-20): In the following questions,
c) Abstainer away from the bottle three sentences are given, which may or may not
A. Both (b) and (c) be free from error, you have to identify which
B. Only (b) sentence is free from error and mark it as your
C. Only (c) answer
D. Both (a) and (b) 16.
E. Only (a) 1. Economic growth and rapid urbanization are
major cause of increasing waste generation.
14. In most countries, diplomacy has become 2. He said that people were so obsessed with
hostage to domestic populist agendas with taking mobile phone footage that they miss the
diplomats left to play the organ grinder monkey. actual experience of the moment.
a) Organ grinder’s monkey 3. Freedom of expression and the freedom to
b) Grinder organ’s monkey practice and propagate a person’s own religion
c) Organ’s grinder monkey are constitutional endowed rights in this multi-
A. Only (b) religious country.
B. Both (b) and (c) A. Only 1
C. Only (a) B. Only 3
D. Only (c) C. Both 1 and 2
E. Both (a) and (b) D. Both 2 and 3
E. All are incorrect

Click Here For Bundle PDF Course | support@guidely.in Page 5 of 10


Bank Po Mains PDF Course 2024
English Day - 16

17. 19.
1. Not believing in the deity’s existence is as 1. If these numbers were updated, it would have
much an opinion as believes in it; the latter is enable the states to issue new ration cards over
religion while the former is, perhaps, philosophy. time
2. In an ideal arrangement, neither side should 2. Beside that, the food ministry has, in eight
threaten the other, and neither side should feels years, annulled some 47 million ghost or
insulted by the other’s presence. duplicate ration cards (170 million people) across
3. Any hostility between the two sides can laid the states under the PDS
bare the inclinations of society at that point of 3. Newly-enrolled beneficiaries, who have not
time. yet seeded their cards to the online portal, don’t
A. Only 1 receive their quotas.
B. Only 2 A. Only 2
C. Only 3 B. Only 3
D. Both 1 and 3 C. Both 1 and 2
E. All are incorrect D. Both 1 and 3
E. All are incorrect
18.
1. The bilateral meetings between leaders in 20.
Samarkand could shape the course of the region 1. One-third of the country’s land mass are
and the world far more than the UN described as flooded, with Sindh and Balochistan
2. Secularism functions more as tolerance and provinces the worst affected.
peaceful coexistence than as a strict separation 2. The larger point is that this is the third major
of the State from religion natural catastrophe that have impacted the
3. There is little chances, then, of any coherent, country in less than two decades
common position emerging from the SCO 3. If international responses to each of these
summit. three visitations are to be ranked, the 2005
A. Only 1 earthquake had evoked the strong reactions.
B. Only 2 A. Only 1
C. Both 1 and 3 B. Only 2
D. Both 2 and 3 C. Only 3
E. All are incorrect D. Both 1 and 2
E. All are incorrect

Click Here For Bundle PDF Course | support@guidely.in Page 6 of 10


Bank Po Mains PDF Course 2024
English Day - 16

Click Here to Get the Detailed Video Solution for the above given Questions
Or Scan the QR Code to Get the Detailed Video Solutions

Answer Key with Explanation


1. Answer: D We use “or” to connect two or more possibilities
The correct match is- a-II-f and c-III-d or alternatives.
a-II-f- The Enforcement Directorate conducted b-II-f- Railways have fixed a limit on luggage
searches at some locations in Mumbai in according to each coach, such that heavy goods
connection with the Rs 1,034 crore Patra Chawl ranging from 40 kg to 70 kg can be kept in the
land scam case. train compartment.
We use “in connection with” for expressing a We use “such that” to give reason.
relationship of one thing with another. c-III-d Passengers will now have to pay extra if
c-III-d- Flood alerts have been issued in several they want to carry goods weighing more than the
places across the country due to heavy rain and standard fixed by the railway.
the release of dam water into rivers. We use “if”to introduce a possible or unreal
We use “due to” to show cause-and-effect situation or condition.
relationships and function in the same way. Hence option E is the correct answer.
Hence option D is the correct answer.
3. Answer: A
2. Answer: E The correct match is- b-I-f and c-III-e
The correct match is- a-I-e, b-II-f and c-III-d b-I-f- In the 20 cities with highest PM2.5
a-I-e- Indian Railways announced new luggage exposures, residents in cities from India, Nigeria,
rules, as a result of which train passengers will Peru, and Bangladesh are exposed to PM 2.5
now have to either limit their luggage, or will levels that are several-fold higher than the global
have to pay more for carrying extra baggage averages.
while travelling.

Click Here For Bundle PDF Course | support@guidely.in Page 7 of 10


Bank Po Mains PDF Course 2024
English Day - 16

We use “and” conjunction between words, We use “which” conjunction to connect two
phrases etc in order to connect them together. unequal parts, e.g., dependent and independent
c-III-e- India and Indonesia have seen the most clauses.
severe increase in PM2.5 pollution, whereas Hence option C is the correct answer.
China has seen the greatest improvements.
We use “whereas” to indicate a contrast between 6. Answer: C
two facts or ideas. The word snag is used to define a small difficulty
Hence option A is the correct answer. or disadvantage that is often unexpected or
hidden. The given sentences except the one in
4. Answer: D (c), use the synonyms of the word snag.
The correct match is- a-II-d The word assistance is a positive word that
a-II-d India on Wednesday recorded 9,062 fresh means to help in making something happen. So,
COVID-19 infections that took its tally to this sentence uses an antonym to the word given
4,42,86,256, while the number of active cases in bold.
came down to 1,05,058. Therefore, the best is to mark option (c) as the
We use “while” to show “during the time that; at answer.
the same time as” and as a “whereas (indicating
a contrast).” 7. Answer: C
Hence option D is the correct answer. Affluent means prosperous, extremely rich and
also abundant. Of the given sentences above,
5. Answer: C only (c) ฀ scarce is the antonym of affluent as it
The correct match is- a-I-e and c-II-d means scanty or available in less quantity. Let’s
India’s ambitious project to translocate African understand the usages of some related words:
cheetahs has missed an unofficial deadline of Opulent means luxurious
August 15, though sources say the animals are Elite is used to denote aristocracy or superiority
likely to arrive within this year. Lucre hints at money earned dishonestly
“Though/although” are subordinating
conjunctions used to connect a subordinate 8. Answer: B
clause to a main clause. Incorrigible is something that is unable to be
The cheetahs from Namibia were currently in changed or corrected.
medical quarantine which would last for a month. The above statements are quite easy to
understand as (a) talks about being habitual

Click Here For Bundle PDF Course | support@guidely.in Page 8 of 10


Bank Po Mains PDF Course 2024
English Day - 16

which is similar to incorrigible. Also, hardcore is 11. Answer: D


similar in meaning as well. The right phrase is “pull the wool over our eyes”
(d) talks about impenitent sinners means the which means to trick or deceive someone, to
ones who are not remorseful for their acts or are hide the truth.
shamelessly audacious. This is also in a way a
synonym to the highlighted word. The word 12. Answer: E
chronic means severe and this is rule out as The right structure is “alcoholic’s craving for
well. alcohol”. Alcoholic is a person suffering from
So, (b) happens to be the best as repentant is alcoholism. And to show the possession of an
one who feels shameful or is feeling sincere alcoholic person on alcohol (‘s) is used.
regret for his actions.
13. Answer: D
9. Answer: A “Teetotaler” or “Abstainer” are persons who
‘Mal’ is a negative prefix which denotes that never drink alcohol. But the context of the
something unfriendly, harmful or bad is being sentence is for those who consume alcohol; we
described. So, malign means evil or ill-willed. use “tippler” or “boozer” for a habitual drinker of
Clearly we know that a positive word will be an alcohol.
antonym to the above so option (a) beneficial is 14. Answer: C
the obvious answer for us. The right saying is “organ grinder’s monkey”
Malignant (dangerous, harmful), Spiteful (mean, which means someone who is closely
unfriendly, hurtful), Baleful (unfriendly, harmful), associated with a powerful person, and acts on
Hostile (malicious and confrontational) are all their behalf but has no real power themselves.
similar words that carry a negative tone.
15. Answer: E
10. Answer: E The right structure is “enriching rich oligarchs”
Cacophony is a harsh and loud sound or Rule of parallelism will be applied here, so
screaming voice that is irritating to the ears. The “enrich” is not grammatically right.
words grating, screeching, jarring and din are “Oligarchy” is a form of government
related as they all carry a similar meaning. characterized by a rule of a few people or
Euphony which means pleasant to the ears or a families. But we need a plural form of “oligarch”
likeable sound is the obvious unrelated word in which is “oligarchs” which means rulers in an
the above so that’s the answer. oligarch.

Click Here For Bundle PDF Course | support@guidely.in Page 9 of 10


Bank Po Mains PDF Course 2024
English Day - 16

16. Answer: E (3) – ‘chance’ will be replaced by ‘chances’, ‘little’


(1) – ‘cause’ will be replaced by ‘causes’, the is an uncountable noun, and it is used with
subject of the noun is ‘economic growth and singular form. So, ‘chance’ is incorrect here
rapid urbanization’ which is plural, so the noun
should be used in plural form 19. Answer: B
(2) – ‘miss’ will be replaced by ‘missed’, the (1) – according to the rule of conditional
structure of the sentence is in past tense, so the sentence (if +simple past, subject + would +V1)
verb should be according to its tense. So, ‘miss’ is used, so ‘would have’ is incorrect here, only
is incorrect here ‘would’ will be used.
(3) – ‘constitutional’ will be replaced by (2) – ‘beside’ will be replaced by ‘besides’,
‘constitutionally’, we need an adverb to qualify ‘beside’ means – at the side of; next of, whereas
the verb (endowed), ‘constitutional’ is an ‘besides’ means – in addition to; apart from. The
adjective. So, ‘constitutional’ will be replaced context of the sentence talks about the additional
information, so, ‘besides’ will be used here
17. Answer: A
(2) – ‘feels’ will be replaced by ‘feel’, model verb 20. Answer: E
‘should’ doesn’t follow the verb using ‘e/es’ with (1) – ‘are’ will be replaced by ‘is’; the subject of
it. So, ‘feels’ is incorrect here the verb is ‘one-third of the country’s land’ which
(3) – ‘laid’ will be replaced by ‘lay’, the structure is a single unite, so the verb should be singular.
of the sentence is in present tense, so the verb So, ‘are’ is incorrect here
will be used according to the tense (2) – ‘have’ will be replaced by ‘has’, the subject
(Lay (V1) – Laid (V2) – Laid (V3)) of the verb is ‘natural catastrophe’ which is
singular, so the verb will be used according to
18. Answer: B the subject. So, ‘has’ will be used here
(1) – ‘can’ will be used after ‘the UN’, here the (3) – the context of the sentence talks about the
comparison is between the efforts of the leaders occurrence of three natural calamities, in which
who can shape the course of the region, which is 2005’s earthquake evoked the strongest
the leaders in Samarkand could shape it far reaction. Superlative degree ‘strongest’ will be
more the UN can. So, ‘can’ will be used after ‘the used here.
UN’

Click Here For Bundle PDF Course | support@guidely.in Page 10 of 10


Bank Po Mains PDF Course 2024
Reasoning Day -17 (Eng)

Reasoning Aptitude
Directions (1-5): Study the following statements Plane ^ Ship
and then decide which of the given conclusions Bus % Plane
logically does not follow from the given Bus $ Auto
statements disregarding the commonly known Conclusions:
facts. I) All ships can never be auto
“A@B” means “Some A is not B” II) All buses being tempo is not a possibility
“A&B” means “Only a few B is A” III) Some cars are not auto is a possibility
“A%B” means “At least some B is A” IV) Some vans cannot be plane
“A^B” means “All B is A” a) Both conclusions I and II don’t follow
“A+B” means “Only A is B” b) Only conclusion I doesn’t follow
“A$B” means “No A is B” c) Only conclusion II doesn’t follow
1. Statements: d) Only conclusions I, II and IV don’t follow
Milk @ Almond e) None follows
Almond & Nuts
Milk $ Water 3. Statements:
Almond ^ Cookie Thread % Rope
Nuts & Acid Jute ^ Thread
Acid $ Alkaline Cloth & Jute
Conclusions: Cloth $ Fibre
I) All nuts cannot be alkaline Fibre + Ribbon
II) No cookie being milk is a possibility Rope $ Cotton
III) All nuts can never be cookie Conclusions:
IV) Some water is definitely not almond I) All fibres cannot be thread
a) Both conclusions II and IV don’t follow II) Some jute can be cotton
b) Only conclusion III doesn’t follow III) All clothes can never be ribbon
c) Both conclusions I and III don’t follow IV) Some ropes are definitely not cloth
d) Only conclusion IV doesn’t follow a) Both conclusions I and IV don’t follow
e) Both conclusions III and IV don’t follow b) Only conclusion III doesn’t follow
c) Only conclusions I, II and IV don’t follow
2. Statements: d) Only conclusion IV doesn’t follow
Tempo + Van e) Both conclusions III and IV don’t follow
Tempo & Car
Car @ Ship 4. Statements:

Click Here For Bundle PDF Course | support@guidely.in Page 1 of 11


Bank Po Mains PDF Course 2024
Reasoning Day -17 (Eng)

Layer & Part Directions (6-10): Study the following information


Slice % Layer carefully and answer the given questions.
Portion ^ Slice A certain number of persons from the same
Piece & Portion family of three generations sit in a linear row
Piece $ Total facing the north directions. Information about
Lump % Total only a few of them is known.
Conclusions: Only four persons sit between N and O, who is
I) All portions can be total the child of J and is not married. N sits second
II) No slice being total is a possibility from one of the extreme ends. I sits adjacent to
III) Some layers cannot be piece N, who is the only son of B. The one who is the
IV) All lump is part is not a possibility wife of B is the only daughter of J, who has only
a) Both conclusions I and III don’t follow two children. Only three persons sit between H
b) Only conclusion II doesn’t follow and O. The number of persons sitting between
c) Only conclusions I, III and IV don’t follow the one who is the brother-in-law of O and H is
d) Only conclusion I doesn’t follow one more than the number of persons sitting
e) All conclusions I, II, III and IV don’t follow between I and L, who sits immediate right of H. B
is the child of L and doesn’t have any siblings.
5. Statements: The one who is the husband of L sits fourth to
Clock + Alarm the left of the one who is the son-in-law of J and
Watch % Clock sits at one of the extreme ends. A sits sixth to the
Time $ Watch left of T and is the brother of H. Only one person
Seconds & Time sits between T and the one who is the father of
Minutes ^ Seconds N. T is the daughter-in-law of B. A is the father-
Minute $ Hour in-law of I. As many persons sit to the left of A as
Conclusions: to the right of J.
I) Some alarms can never be minutes. 6. If V is the wife of J, then the relationship
II) Some second is watch is not a possibility between the one who sits third to the right of L
III) All time cannot be hour and V is same as the relationship between __
IV) No hour is clock and __.
a) Both conclusions I and II don’t follow a) H, T
b) Both conclusions II and IV don’t follow b) N, H
c) Only conclusion IV doesn’t follow c) L, B
d) Only conclusions I, II and IV don’t follow d) A,T
e) None follows e) None of these

Click Here For Bundle PDF Course | support@guidely.in Page 2 of 11


Bank Po Mains PDF Course 2024
Reasoning Day -17 (Eng)

7. What is the sum of the number of persons b) N is the father of the one who sits immediate
sitting in the row and the number of persons right of J.
sitting between O and J? c) H is the sister-in-law of L
a) 36 d) Only five persons sit to the left of the one who
b) 28 is the son-in-law of B
c) 41 e) O is the maternal uncle of the one who is the
d) 33 spouse of T.
e) 56
Directions (11-15): Study the following
8. If X sits sixth to the left of N, then what is the information carefully and answer the given
position of X with respect to the one who is the questions.
paternal grandmother of N? Seven persons - L, M, N, O, P, Q and R live on
a) Immediate left seven different floors of a seven-storey building
b) Third to the right where the lowermost floor is numbered one and
c) Sixth to the left the floor immediately above it is numbered two
d) Second to the right and so on. Each of them joined a company in
e) Eighth to the left different years. No two persons joined in the
same year and no one joined after 2004.
9. Which among the following pair of persons are The one who joined the company in 1989 lives
nephew and uncle respectively? three floors above M. Only one person lives
I) HB between M and L, who doesn’t live above the
II) NO fourth floor. The number of floors above L is one
III) LB more than the number of floors below R. The
a) Only II difference between the years in which the one
b) Only I and III who lives on the topmost floor and R joined the
c) Only II and III company is 16 years. Only four floors are
d) Only I between the one who joined in 1995 and Q, who
e) All I, II and III didn’t join in 1989. As many floors above Q as
below N, who joined three years after P. The
10. Which of the following statement(s) is/are difference between the years in which the one
true as per the given arrangement? who lives immediately below N and N joined the
a) Only one person sits between B and the one company is 6 years. There is a gap of seven
who is the son of I. years between the years in which P and L joined
the company. The difference between the years

Click Here For Bundle PDF Course | support@guidely.in Page 3 of 11


Bank Po Mains PDF Course 2024
Reasoning Day -17 (Eng)

in which O and M joined the company is 22 a) Three more


years. One of the persons joined the company in b) Two more
1982. c) One less
11. If the experience of each person is calculated d) Two less
from the base year 2023, then what is the sum of e) Three less
the experience of the one who lives on the
topmost floor and N? 15. In which of the following option the first
a) 38 person joined three years after the second
b) 80 person?
c) 72 a) LR
d) 52 b) RQ
e) 65 c) LO
d) ML
12. What is the difference between the years in e) PM
which the one who lives immediately below O
and M joined the company? Directions (16-20): Study the following
a) 5 years information carefully and answer the given
b) 10 years questions.
c) 8 years Raj, Dev, Hari, Sam, James and Dino went for
d) 17 years the mountain trucking to take the photo of the
e) 13 years mountain range. A different team of two persons
took the photo of six mountain range viz.
13. What is the sum of the floor number of P and Himalayas, Aravalli, Vindhya, Western Ghats,
the difference between the years in which R and Hindu Kush and Garhwal. No two teams take the
Q joined the company? photo of the same mountain range.
a) 10  Each person takes a photo of only two
b) 30 mountain ranges
c) 16  James, Sam and Dev are not combined as a
d) 12 team while taking photo.
e) 22  James did not take a photo of Himalayas.
 Raj was not the member of the team who took
14. The number of floors above the one who the photo of Garhwal and Vindhya.
joined in 1995 is ___ than the number of floors  Hari is one person in the team who took a
below P. photo of Aravalli.

Click Here For Bundle PDF Course | support@guidely.in Page 4 of 11


Bank Po Mains PDF Course 2024
Reasoning Day -17 (Eng)

 One of the persons, who took a photo of e) None of these


Vindhya, also a part of taking a photo of another
mountain range when pairing with James 18) Which of the following combination is true?
 Raj and Sam, Dev and Dino were not a) Dino – Dev – Garhwal
combined as a team while taking a photo b) Hari – Dino – Vindhya
 Neither Dino nor Raj takes the photo of c) James – Dev – Aravalli
Western Ghats. d) Dino – Sam – Vindhya
 No same person was member of the team who e) More than one combination is true.
took a photo of both the Western Ghats and
Himalayas. 19) Which of the following person is one of the
 Dino and James successfully photographed as team members of the one who took the photo of
a team. Garhwal?
16) Which among the following pair of person a) Dev
took the photo of Hindu Kush? b) Sam
a) James – Hari c) Hari
b) Dino – Sam d) Raj
c) Raj – James e) James
d) Sam – James
e) None of these 20) Which of the following pair of persons form a
successful team?
17) Sam took the photo of which of the following a) Dino – Hari
mountain Ranges? b) Dev – Hari
a) Vindhya & Aravalli c) Hari – James
b) Aravalli & Western Ghats d) James – Sam
c) Himalayas & Garhwal e) None of these
d) Western Ghats & Vindhya

Click Here For Bundle PDF Course | support@guidely.in Page 5 of 11


Bank Po Mains PDF Course 2024
Reasoning Day -17 (Eng)

Click Here to Get the Detailed Video Solution for the above given Questions
Or Scan the QR Code to Get the Detailed Video Solutions

Answer Key with Explanation


Directions (1-5):
1) Answer: D

4) Answer: C
2) Answer: A

5) Answer: D

3) Answer: D

Directions (6-10):
6) Answer: C

Click Here For Bundle PDF Course | support@guidely.in Page 6 of 11


Bank Po Mains PDF Course 2024
Reasoning Day -17 (Eng)

7) Answer: B Again, we have


8) Answer: D  Only three persons sit between H and O.
9) Answer: A  The number of persons sitting between
10) Answer: E the one who is the brother-in-law of O and
Final arrangement H is one more than the number of
persons sitting between I and L, who sits
immediate right of H.
 B is the child of L and doesn’t have any
siblings.
 The one who is the husband of L sits
We have,
fourth to the left of the one who is the son-
 Only four persons sit between N and O,
in-law of J and sits at one of the extreme
who is the child of J and is not married.
ends.
 N sits second from one of the extreme
After applying the above conditions case-2 and
ends.
case-2a get eliminated because L’s husband
 I sit adjacent to N, who is the only son of
doesn’t sit at one of the extreme ends.
B.
 The one who is the wife of B is the only
daughter of J, who has only two children.
From the above conditions, there are four
possibilities

Again, we have
 A sits sixth to the left of T and is the
brother of H.
 Only one person sits between T and the
one who is the father of N.
 T is the daughter-in-law of B. A is the
father-in-law of I.
 As many persons sit to the left of A as to
the right of J.

Click Here For Bundle PDF Course | support@guidely.in Page 7 of 11


Bank Po Mains PDF Course 2024
Reasoning Day -17 (Eng)

After applying the above conditions case-1 gets From the above conditions, there are four
eliminated because there is no possibility to possibilities
place J, hence case-1a shows the final
arrangement.

Again, we have
Directions (11-15):
 The difference between the years in
11) Answer: C
which the one who lives on the topmost
12) Answer: E
floor and R joined the company is 16
13) Answer: D
years.
14) Answer: B
 Only four floors are between the one who
15) Answer: E
joined in 1995 and Q, who didn’t join in
Final arrangement
1989.
 As many floors above Q as below N, who
joined three years after P.
 The difference between the years in
which the one who lives immediately
below N and N joined the company is 6
years.
After applying the above conditions, case-2 and
case-3 get eliminated because there is no
possibility to place N in case 3 and no possibility
We have,
to place P in case 2.
 The one who joined the company in 1989
lives three floors above M.
 Only one person lives between M and L,
who doesn’t live above the fourth floor.
 The number of floors above L is one more
than the number of floors below R.

Click Here For Bundle PDF Course | support@guidely.in Page 8 of 11


Bank Po Mains PDF Course 2024
Reasoning Day -17 (Eng)

20. Answer: B

Again, we have
 There is a gap of seven years between
the years in which P and L joined the We have:

company.  Each person takes a photo of only two

 The difference between the year in which mountain ranges

O and M joined the company is 22 years.  James, Sam and Dev are not combined

 One of the persons joined the company in as a team while taking photo.

1982.  Raj and Sam, Dev and Dino were not

After applying the above conditions, case-4 gets combined as a team while taking photo

eliminated because the difference between the  Dino and James successfully

year in which O and M joined the company is 22 photographed as a team.

years is not satisfied. Hence case-1 shows the From the above given conditions, the possible

final arrangement. pairs are,


(Raj, Dev), (Raj, Hari), (Raj, James), (Raj, Dino),
(Dev, Hari), (Hari, Sam), (Hari, James), (Hari,
Dino), (Sam, Dino) and (Dino and James)
I) Dino and James is one of the pair
Given that, each person takes the photo of only
two mountain ranges. Thus, Sam is paired with
only two persons, (i.e.) Hari and Sam, Sam and
Dino is another pair.
II) Hari and Sam is one of the pair
Directions (16-20): III) Sam and Dino is one of the pair
16. Answer: C Again, Dino is already paired with two persons,
17. Answer: D which is Sam and James. So, (Raj and Dino),
18. Answer: D (Hari and Dino) pair gets eliminated
19. Answer: E

Click Here For Bundle PDF Course | support@guidely.in Page 9 of 11


Bank Po Mains PDF Course 2024
Reasoning Day -17 (Eng)

(Raj, Dev), (Raj, Hari), (Raj, James), (Raj, Dino),


(Dev, Hari), (Hari, Sam), (Hari, James),
(Hari,Dino), (Sam, Dino) and (James, Dino).
Dev is paired with only two persons. So, another
two pair is (Raj, Dev) and (Dev, Hari)
IV) Raj and Dev is one of the pair
V) Dev and Hari is one of the pair
Now, Hari is already paired with two persons, Again we have,
which is Dev and Sam. So (Hari and Raj), (Hari  Raj was not the member of the team who
and James) pair get eliminated took the photo of Garhwal and Vindhya.
(Raj, Dev), (Raj, Hari), (Raj, James), (Dev, Hari),  One of the persons, who took a photo of
(Hari, Sam), (Hari, James), (Sam, Dino) and Vindhya, also a part of taking a photo of
(James, Dino). another mountain range when pairing with
(Raj, Dev), (Raj, James), (Dev, Hari), (Hari, James
Sam), (Sam, Dino) and (James, Dino). Thus, the person paired with James is Dino and
Remaining pair is (Raj and James) Raj, but raj does not take the photo of Vindhya
VI) Raj and James is one of the pair Range. So Dino is the one who took the photo of
Again, we have Vindhya.
 Hari is one person in the team who took a
photo of Aravalli.
From the above conditions, there are two
possibilities

Again, we have
 James did not take a photo of Himalayas.
 No same person was member of the team
who took a photo of both the Western
Ghats and Himalayas.
Again, we have
No common person should be between the pair
 Neither Dino nor Raj takes the photo of
of persons who took the photo of Western Ghats
Western Ghats.
and Himalayas, So case 1 gets eliminated. Thus,
case 2 gives the final arrangement.

Click Here For Bundle PDF Course | support@guidely.in Page 10 of 11


Bank Po Mains PDF Course 2024
Reasoning Day -17 (Eng)

Click Here For Bundle PDF Course | support@guidely.in Page 11 of 11


Bank Po Mains PDF Course 2024
Quantitative Aptitude Day -17 (Eng)

Quantitative Aptitude

Directions (1-4): Study the following data b) 265 cubic feet


carefully and answer the questions: c) 245 cubic feet
There are two cubical boxes A and B and two d) 235 cubic feet
cuboidal boxes C and D. The side (in feet) of e) 275 cubic feet
each cubical box is different and the length,
width, and height (in feet) of each cuboidal box 3) If the cost of painting the walls of boxes is ₹15
are different. per square feet, then find the average of total
The length of box C is equal to the side of box A, cost of painting box B (except the floor), the total
the width of box D is equal to the side of box B cost of painting box C (except the floor) and the
and the height of box C is half of its length. The total cost of painting box D (except the floor)?
width of boxes C and D are equal. The height of a) ₹2420
box D is 4 feet less than its width and the length b) ₹2220
of box D is 10 feet. The volume of box D is 120 c) ₹2320
cubic feet, and the volume of box B is 24 cubic d) ₹2620
feet more than the volume of box C. e) ₹2520
1) If the ratio of the volume of box A to the
volume of box B is M: N and the ratio of the 4) If the length, width and height of cuboidal box
volume of box C to the volume of box D is P: Q, E are respectively 125%, and 50% of
then find the value of (NQ – M – P) is divisible by that of box C, then find that which of the following
which of the following? statements is/are true?
a) 5 X: The difference between the volumes of boxes
b) 9 D and E is 40 cubic feet.
c) 7 Y: The ratio of the side of box B to the length of
d) Both (a) and (b) box E is 2: 5.
e) Both (b) and (c) Z: The average of the side of box A and the
height of box E is 5 feet.
2) If box A is filled with water up to the height of 5 a) Only Y
feet and box D is filled with water up to the height b) Only X and Z
of 1.25 feet, then find the difference between the c) Only Y and Z
volume of water filled in box A and the volume of d) None is true
water filled in box D? e) Only X
a) 255 cubic feet

Click Here For Bundle PDF Course | support@guidely.in Page 1 of 18


Bank Po Mains PDF Course 2024
Quantitative Aptitude Day -17 (Eng)

Directions (5-8): Study the following data carefully and answer the questions:
The data given below shows the number of students in two schools A and B in different age groups i,e;
(5-8) years, (9-12) years, (13-16) years and (17-20) years.
The bar graph given below shows the following data:

Note:
1: In school A, the number of students in the age group (5 – 8) years, is 10 less than those in the age
group (13 – 16) years.
2: In school B, the number of students in the age group (9 – 12) years, is equal to those in the age group
(17 – 20) years.
5) If the ratio of the total number of 5 to 20 years c) Only P
old students in school A to those in school B is d) Only R
M: N, then which of the following can’t be the e) Only Q and R
average of the number of students in school B, in
age group (9 – 12) years, those in age group (13 6) If the average of the number of students in
– 16) years and those in age group (17 – 20) age group (5 – 8) years in schools A, B and C is
years? 48 and the average of the number of students in
P: age group (9 – 12) years in schools A, B and C is
Q: 25(N – M) 70, then find that the number of students in
R: (M – 2)% of 10(N + 3) school C in age group (5 – 8) years is what per
a) Only P and Q cent of those in age group (9 – 12) years?
b) Only Q a) 51%

Click Here For Bundle PDF Course | support@guidely.in Page 2 of 18


Bank Po Mains PDF Course 2024
Quantitative Aptitude Day -17 (Eng)

b) 85% 8) Which of the following statements is/are true?


c) 34% P: The ratio of the total number of 5 to 12 years
d) 102% old students in school A to those in school B is 5:
e) 68% 4.
Q: If the average of the number of students in
7) In school B, if the ratio of boys to girls in age age group (13 – 16) years in school B and those
group (5 – 8) years is 3: 2, the ratio of boys to in school C is 90, then the average of the number
girls in age group (9 – 12) years is 5: 9, the ratio of students in age group (13 – 16) years in
of boys to girls in age group (13 – 16) years is 9: school A and those in school C will be 80.
11 and the ratio of boys to girls in age group (17 R: The difference between the total number of 9
– 20) years is 4: 3, then find the total number of 5 to 16 years old students in school A and those in
to 20 years old girls in school B? school B is 20.
a) 150 a) Only Q
b) 130 b) Only P and Q
c) 145 c) Only Q and R
d) 125 d) Only P
e) 140 e) Only R

Directions (9-12): Study the following data carefully and answer the questions:
Seven workers A, B, C, D, E, F and G are hired to complete a work and the efficiency of each worker is
different.
The hexagonal graph given below shows the following data:

Click Here For Bundle PDF Course | support@guidely.in Page 3 of 18


Bank Po Mains PDF Course 2024
Quantitative Aptitude Day -17 (Eng)

Note: If A works with 125% of its original efficiency and G works with of its original efficiency,
then A and G together will finish the work in 9.6 hours.
9) If B and C work with 50% of their respective Q: The average of time, in which A alone, C
original efficiencies and D and E work with 200% alone and D alone can complete the work, is 13
of their respective original efficiencies, then find hours.
the time, in which B, C, D and E together will R: The ratio of C’s efficiency to F’s efficiency is 4:
complete the work with their new efficiencies? 1.
a) a) Only Q
b) b) Only P and Q

c) c) Only Q and R

d) d) Only P
e) Only R
e)

11) Each work received wages in the ratio of


10) Which of the following statements is/are
their efficiencies. If the difference between B’s
true?
wages and D’s wages is ₹5400, then what is the
P: D and G together can complete the work in
difference between A’s wages and G’s wages?
hours.
a) ₹1018

Click Here For Bundle PDF Course | support@guidely.in Page 4 of 18


Bank Po Mains PDF Course 2024
Quantitative Aptitude Day -17 (Eng)

b) ₹3675 I: 4, X, 24, XY, 840


c) ₹9187.5 II: (X – 5), 6, (Y + 3), 90, 630
d) ₹3187 Find that which of the following statements is/are
e) ₹11187.5 true?
P: The 1st term of series II is 32.5% of the 2nd
12) Find the ratio of between the time, in which A term of series I.
and B together can complete the work, the time, Q: The value of X is of the value of Y.
in which C and D together can complete the work R: The average of the 4th term of series I and the
and the time, in which F and G together can 3rd term of series II is 79.
complete the work respectively? a) Only Q
a) 3: 2: 7 b) Only P and Q
b) 15: 8: 28 c) Only Q and R
c) 9: 5: 21 d) None is true
d) 5: 2: 11 e) Only R
e) 45: 28: 112
15) A series is given below:
Direction (13-15): Study the following data 16, 30, 58, 114, 226, 5X, (10X – Y)
carefully and answer the questions: If a new series is formed, whose 1st term is
13) A series is given below, in which the next and the other terms are obtained by adding 1 on
term is obtained by adding 1 on double of the the double of the previous term, then find the 3rd
previous term. term of the new series?
4, (x + y), 19, (8x – 1) a)
If a new series is formed, whose 1st term is (4y – b)
x) and follows the same logic as in the given
c)
series, then find the average of 5th term of the
d) Both (a) and (b)
given series and the 5th term of the new series?
e) Both (b) and (c)
a) 125
b) 140
Directions (16-19): Study the following data
c) 135
carefully and answer the questions:
d) 130
There are five mixtures A, B, C, D, and E of two
e) 120
types of tea P and Q. The table given below
shows the percent of quantity of each mixture out
14) Two series I and II are given below and both
of total quantity in all the five mixtures together.
the series follow the same pattern:

Click Here For Bundle PDF Course | support@guidely.in Page 5 of 18


Bank Po Mains PDF Course 2024
Quantitative Aptitude Day -17 (Eng)

Note: The cost of type P and type Q tea per kg is b) 7.5%


₹40 and ₹60 respectively and all the quantities c) 15%
are in kg. d) 12.5%
e) 20%

17) If the whole quantity of mixture D is mixed


with ‘x’ kg of mixture G to form a new mixture
such that per kg cost of new mixture becomes
₹ . If √(y2 – x2) = 60, then find the value of ‘x
+ y’.
a) 180
b) 160
c) 200
d) 240
e) 120

18) Two mixtures A and C are mixed in a certain

Some other information is also given: ratio to form a new mixture whose selling price

Profit percent earned when mixture E is sold at lies between ₹46.53 and ₹48.4 when sold at 10%

the cost of type Q tea is 31.25%. Mixture F is profit. Which of the following ratio(s) given below

formed after mixing mixtures A and D in the ratio can be the possible ratio in which mixtures A and

of 2: 3 while mixture G is formed after mixing C must be mixed?

mixtures B and C such that after selling mixture I: 1: 4

G at the cost of type P tea, loss percent incurred II: 2: 9

is . III: 3: 7
a) Only I
16) If in mixture F, 15 kg of type P is removed
b) Only I and III
and 15 kg of type Q tea is added, then the
c) Only II
quantity of type P tea in the final mixture
d) All I, II, and III
becomes 15 kg more than the quantity of type Q
e) Only II and III
tea in that mixture F. Find the profit percent
earned when the final mixture F is sold at the
19) The whole quantity of two mixtures X and Y
cost of ₹54 per kg.
are mixed together to form another mixture M (‘X’
a) 10%

Click Here For Bundle PDF Course | support@guidely.in Page 6 of 18


Bank Po Mains PDF Course 2024
Quantitative Aptitude Day -17 (Eng)

and ‘Y’ are the mixtures of the same per kg cost and B are quadratic in ‘y’. Choose the pair of
price among the mixtures A, B, C, D, and E). If statements in which ‘x > y’.
55 kg of another mixture N (ratio of type P to
type Q tea is 3: 8) is mixed with the whole
quantity of mixture M to form a final mixture, then
find the difference between type P and type Q
I: PA
tea in the final mixture.
II: PB
a) 45 kg
III: QA
b) 30 kg
IV: QB
c) 25 kg
a) I and II
d) 40 kg
b) II and IV
e) 35 kg
c) II and III
d) III and IV
20) Below given is a table that contains four
e) I and IV
quadratic equations numbered P, Q, A, and B
respectively. P and Q are quadratic in ‘x’ while A
Click Here to Get the Detailed Video Solution for the above given Questions
Or Scan the QR Code to Get the Detailed Video Solutions

Answer Key with Explanation


Directions (1-4): The width of box C = ‘y’ feet
Let the side of box A be ‘2x’ feet. And the height of box D = (y – 4) feet
So, the length of box C = 2x feet Since, the length of box D is 10 feet and its
And the height of box C = (2x/2) = ‘x’ feet volume is 120 cubic feet.
Let the side of box B be ‘y’ feet. So,
So, the width of box D = ‘y’ feet 10 * y * (y – 4) = 120

Click Here For Bundle PDF Course | support@guidely.in Page 7 of 18


Bank Po Mains PDF Course 2024
Quantitative Aptitude Day -17 (Eng)

y2 – 4y – 12 = 0 So, the volume of water filled in box A = 8 * 8 * 5


y2 – 6y + 2y – 12 = 0 = 320 cubic feet
y(y – 6) + 2(y – 6) = 0 Since, box D is filled with water up to the height
y=6 of 1.25 feet.
Since, the volume of box B is 24 cubic feet more So, the volume of water filled in box D = 10 * 6 *
than the volume of box C. 1.25 = 75 cubic feet
So, Required difference = 320 – 75 = 245 cubic feet
y3 – 2x2y = 24
216 – 12x2 = 24 3) Answer: C
x=4 Since, the side of box B = 6 feet
The side of box A = 2 * 4 = 8 feet So, the total surface area of box B to be painted
The side of box B = 6 feet = 5 * 62 = 180 square feet
The length of box C = 2 * 4 = 8 feet And the total cost of painting box B = 180 * 15 =
The width of box C = 6 feet ₹2700
The height of box C = 4 feet Since, the length, width and height of box C are
The length of box D = 10 feet 8 feet, 6 feet and 4 feet respectively.
The width of box D = 6 feet So, the total surface area of box C to be painted:
The height of box D = 6 – 4 = 2 feet (8 * 6) + 2(8 * 4 + 6 * 4) = 48 + 112 = 160 square
1) Answer: E feet
The volume of box A = 8 * 8 * 8 = 512 cubic feet And the total cost of painting box C (except the
The volume of box B = 6 * 6 * 6 = 216 cubic feet floor) = 160 * 15 = ₹2400
So, M: N = 512: 216 = 64: 27. Since, the length, width and height of box D are
The volume of box C = 8 * 6 * 4 = 192 cubic feet 10 feet, 6 feet and 2 feet respectively.
The volume of box D = 10 * 6 * 2 = 120 cubic So, the total surface area of box D to be painted:
feet (10 * 6) + 2(10 * 2 + 6 * 2) = 60 + 64 = 124
So, P: Q = 192: 120 = 8: 5 square feet
Now, the value of (NQ – M – P) = (135 – 64 – 8) And the total cost of painting box D (except the
= 63 floor) = 124 * 15 = ₹1860
And 63 are divisible by both 9 and 7. Required average = = ₹2320

2) Answer: C 4) Answer: B
Since, box A is filled with water up to the height Since, the length of box C = 8 feet
of 5 feet. So, the length of box E = 125% of 8 = 10 feet

Click Here For Bundle PDF Course | support@guidely.in Page 8 of 18


Bank Po Mains PDF Course 2024
Quantitative Aptitude Day -17 (Eng)

Since, the width of box C = 6 feet So,


SO, the width of box E = of 6 = 8 feet (5Y – 15) – (5X + 5) = 10
Since, the height of box C = 4 feet Y – X = 6 -----------(1)
So, the height of box E = 50% of 4 = 2 feet The number of students in school B in age group
From X: (9 – 12) years:
The volume of box D = 10 * 6 * 2 = 120 cubic
feet
The number of students in school B in age group
The volume of box E = 10 * 8 * 2 = 160 cubic
(17 – 20) years:
feet
Required difference = 160 – 120 = 40 cubic feet
Since, in school B, the number of students in
So, X is true,
From Y: age group (9 – 12) years, is equal to those in

Ratio of the side of box B to the length of box E age group (17 – 20) years.
So,
= 6: 10 = 3: 5
3(X + Y) – 14 = 10(Y – X) + 10
So, Y is not true.
From Z: From equation (1):

Average of the side of box A and the height of 3(X + 6 + X) – 14 = 10 * 6 + 10

box E: X = 11

So, Z is true.
Hence, only X and Z are true.

Directions (5-8):
The number of students in school A in age group
(5 – 8) years:

The number of students in school A in age group


(13 – 16) years:

Since, in school A, the number of students in


age group (5 – 8) years, is 10 less than those in
age group (13 – 16) years. 5) Answer: B

Click Here For Bundle PDF Course | support@guidely.in Page 9 of 18


Bank Po Mains PDF Course 2024
Quantitative Aptitude Day -17 (Eng)

The total number of 5 to 20 years old students in (3 * 70) – (90 + 70) = 50


school A: Required average = = 68%
60 + 90 + 70 + 50 = 270
The total number of 5 to 20 years old students in 7) Answer: A
school B: In school B,
50 + 70 + 100 + 70 = 290 The number of girls in age group (5 – 8) years =
So, M: N = 270: 290 = 27: 29 = 20
Now, the required average: The number of girls in age group (9 – 12) years =
= 45

From P: The number of girls in age group (13 – 16) years


= = 55
The number of girls in age group (17 – 20) years
So, P can be the required average.
= = 30
From Q:
25(N – M) = 25(29 – 27) = 50 So, the total number of 5 to 20 years old girls in

So, Q can’t be the required average. school B = 20 + 45 + 55 + 30 = 150

From R:
(M – 2)% of 10(N +3) = (27 – 2)% of 10(29 + 3) = 8) Answer: D

80 From P:

So, R can be the required average. Ratio of the total number of 5 to 12 years old

Hence, only Q can’t be the required average. students in school A to those in school B = (60 +
90): (50 + 70) = 150: 120 = 5: 4

6) Answer: E So, P is true.

Since, the average of the number of students in From Q:

age group (5 – 8) years in schools A, B and C is Since, the average of the number of students in

48. age group (13 – 16) years in school B and those

So, the number of students in age group (5 – 8) in school C is 90.

years in school C: So, the number of students in age group (13 –

(3 * 48) – (60 + 50) = 34 16) years in school C:

Since, the average of the number of students in (2 * 90) – 100 = 80

age group (9 – 12) years in schools A, B and C And the average of the number of students in

is 70. age group (13 – 16) years in school A and those

So, the number of students in age group (9 – 12) in school C:

years in school C:

Click Here For Bundle PDF Course | support@guidely.in Page 10 of 18


Bank Po Mains PDF Course 2024
Quantitative Aptitude Day -17 (Eng)

So, Q is not true. So, the time, in which A alone can finish the
From R: work with 125% of its original efficiency =
The difference between the total number of 9 to = 12t hours
16 years old students in school A and those in And the time, in which G alone can finish the
school B = (70 + 100) – (90 + 70) = 10 work with of its original efficiency =
So, R is not true. = 48t hours
Hence, only P is true. So,

Directions (9-12):
The ratio of A’s efficiency to B’s efficiency = A: B
= 1: 0.75 = 4: 3 t=1
Since, the ratio of B’s efficiency to C’s efficiency The time, in which A alone can complete the
= B: C = 1: 1.25 = 4: 5 work = 15 hours
So, the ratio of A’s efficiency to C’s efficiency = The time, in which B alone can complete the
(4 * 4): (3 * 5) = 16: 15 work = = 20 hours
Since, the ratio of C’s efficiency to D’s efficiency The time, in which C alone can complete the
= C: D = 1: 2 work = = 16 hours
So, the ratio of A’s efficiency to D’s efficiency = The time, in which D alone can complete the
(16 * 1): (15 * 2) = 16: 30 = 8: 15 work = = 8 hours
Since, the ratio of D’s efficiency to E’s efficiency The time, in which E alone can complete the
= D: E = 1: 0.25 = 4: 1 work = = 32 hours
So, the ratio of A’s efficiency to E’s efficiency =
The time, in which F alone can complete the
(8 * 4): (15 * 1) = 32: 15
work = = 32 hours
Since, the ratio of E’s efficiency to F’s efficiency
The time, in which G alone can complete the
= E: F = 1: 1
work = = 64 hours
So, the ratio of A’s efficiency to F’s efficiency =
9) Answer: C
32: 15
The time, in which B alone can complete the
Since, the ratio of F’s efficiency to G’s efficiency
work with 50% of its original efficiency =
= F: G = 1: 0.5 = 2: 1
= 40 hours
So, the ratio of A’s efficiency G’s efficiency = (32
The time, in which C alone can complete the
* 2): (15 * 1) = 64: 15
work with 50% of its original efficiency =
Let the time, in which A alone and G alone can
= 32 hours
complete the work be ‘15t’ hours and ‘64t’ hours.

Click Here For Bundle PDF Course | support@guidely.in Page 11 of 18


Bank Po Mains PDF Course 2024
Quantitative Aptitude Day -17 (Eng)

The time, in which D alone can complete the Hence, only Q is true.
work with 200% of its original efficiency =
= 4 hours 11) Answer: B
The time, in which E alone can complete the The ratio of B’s efficiency to D’s efficiency = 8 :
work with 200% of its original efficiency = 20 = 2 : 5
= 16 hours So, the wages received by B = =

Part of completed by B, C, D and E together in 1 ₹3600


hour: Since, the ratio of B’s efficiency to A’s efficiency
= 15: 20 = 3: 4
So, the wages received by A = =
So, the time, in which B, C, D and E together will
₹4800
complete the work with their new efficiencies:
Since, the ratio of B’s efficiency to G’s efficiency
= 64: 20 = 16: 5
So, the wages received by G = =
10) Answer: A
₹1125
From P:
The required difference = 4800 – 1125 = ₹3675
Part of work, which can be completed by D and
G together in 1 hour:
12) Answer: E
Part of work, which can be completed by A and
So, the time, in which D and G together can B together in 1 hour:
complete the work:

So, the time, in which A and B together can


So, P is not true. complete the work = hours
From Q: Part of work, which can be completed by C and
Average of time, in which A alone, C alone and D together in 1 hour:
D alone can complete the work:

So, the time, in which C and D together can


So, Q is true. complete the work = hours
From R: Part of work, which can be completed by F and
Ratio of C’s efficiency to F’s efficiency = 32: 16 = G together in 1 hour:
2: 1
So, R is not true.

Click Here For Bundle PDF Course | support@guidely.in Page 12 of 18


Bank Po Mains PDF Course 2024
Quantitative Aptitude Day -17 (Eng)

So, the time, in which F and G together can 120 ÷ 5 = 24


complete the work = hours In series II:
Required ratio: (Y + 3) = 90 ÷ 5 = 18
Y = 15
From equation (1):
X=8
13) Answer: C
Now, the series I: 4, 8, 24, 120, 840
The given series:
And the series II: 3, 6, 18, 90, 630
4, (x + y), 19, (8x – 1)
From P:
Since,
The 1st term of series II = 3
(x + y) = 2 * 4 + 1
The 2nd term of series I = 8
x + y = 9 ----------(1)
So, the required percentage = = 37.5%
And,
(8x – 1) = 2 * 19 + 1 So, P is not true.

x=5 From Q:

From equation (1): X = 8, Y = 15

y=4 So, X is of Y.

The given series = 4, 9, 19, 39 So, Q is true.

And the 5th term of the series = 2 * 39 + 1 = 79 From R:

Now, the 1st term of the new series = 4 * 4 – 5 = The 4th term of series I = 120

11 The 3rd term of series II = 18

The 2nd term of the new series = 2 * 11 + 1 = 23 The required average = = 69

The 3rd term of the new series = 2 * 23 + 1 = 47 So, R is not true.


The 4th term of the new series = 2 * 47 + 1 = 95 Hence, only Q is true.
And the 5th term of the new series = 2 * 95 + 1 =
191 15) Answer: D
So, the required average = = 135 Logic in the given series:
16 + 14 = 30

14) Answer: A 30 + 28 = 58

Since, both the series follow the same pattern: 58 + 56 = 114

So, in series II: 114 + 112 = 226

630 ÷ 7 = 90 226 + 224 = 450

And, in series I: 450 + 448 = 898

XY = 840 ÷ 7 = 120 ------------(1) Since, 5X = 450

Click Here For Bundle PDF Course | support@guidely.in Page 13 of 18


Bank Po Mains PDF Course 2024
Quantitative Aptitude Day -17 (Eng)

X = 90 Total cost of mixture C = 6500 – 4400 = ₹2100


And, (10X – Y) = 898 Total cost of mixture D = 7800 – 6500 = ₹1300
Y=2 Total cost of mixture E = 11000 – 7800 = ₹3200
So, the 1st term of the new series = = 11 Let quantity of type P and type Q tea in mixture
The 2nd term of the new series = 11 * 2 + 1 = 23 E is ‘e’ and ‘28x – e’ respectively.
The 3rd term of the new series = 23 * 2 + 1 = 47 According to the question:
From option (a): e * 40 + (28x – e) * 60 = 3200
2e + 84x – 3e = 160
e = 84x – 160 ……….. (1)
So, option (a) is correct.
131.25% of [e * 40 + (28x – e) * 60] = 28x * 60
From option (b):
21 * [e * 40 + (28x – e) * 60] = 16 * 28x * 60
2e + 84x – 3e = 64x
So, option (b) is correct. e = 20x …… (2)
From option (c): From (1) and (2):
20x = 84x – 160
So, option (c) is not correct. 64x = 160
Hence, both options (a) and (b) are correct. x = 2.5
e = 20x = 50
Directions (16-19):
Percent of total quantity of mixture A out of total
quantity = 16%
Percent of total quantity of mixture B out of total
quantity = 40 – 16 = 24%
Percent of total quantity of mixture C out of total
quantity = 60 – 40 = 20%
Percent of total quantity of mixture D out of total
quantity = 72 – 60= 12%
Let quantity of type P tea in mixtures A, B, C,
Percent of total quantity of mixture E out of total
and D is ‘a’, ‘b’, ‘c’, and ‘d’ respectively.
quantity = 100 – 72 = 28%
Total cost of mixture A = a * 40 + (40 – a) * 60 =
Let total quantity of mixture A, B, C, D, and E is
1800
‘16x’, ‘24x’, ‘20x’, ‘12x’, and ‘28x’ respectively.
a = 30
Total cost of mixture A = ₹1800
Total cost of mixture B = b * 40 + (60 – b) * 60 =
Total cost of mixture B = 4400 – 1800 = ₹2600
2600

Click Here For Bundle PDF Course | support@guidely.in Page 14 of 18


Bank Po Mains PDF Course 2024
Quantitative Aptitude Day -17 (Eng)

b = 50 Ratio of type P to type Q tea in mixture F = 4: 1


Total cost of mixture C = c * 40 + (50 – c) * 60 = Since loss percent incurred after selling mixture
2100 G is 5.88%.
c = 45 Let ratio of type P to type Q tea in mixture G is g:
Total cost of mixture D = d * 40 + (30 – d) * 60 = 1.
1300 According to the question:
d = 25
Total cost of mixture E = e * 40 + (70 – e) * 60 =
16 * [g * 40 + 1 * 60] = 17 * (g + 1) * 40
3200
2 * [g * 40 + 1 * 60] = 17 * (g + 1) * 5
80g + 120 = 85g + 85
5g = 35
g=7
Ratio of type P to type Q tea in mixture G = g: 1
= 7: 1
16) Answer: D
Ratio of type P to type Q in the mixture F = 4: 1
Let ratio of type P to type Q tea in mixture F is f: Let initial quantity of type P and type Q tea in
1. mixture F are ‘4f’ and ‘f’ respectively.
Part of type P tea in mixture A = 3/4 According to the question:
Part of type P tea in mixture D = 5/6 (4f – 15) – (f + 15) = 15
By the rule of alligation: 3f – 30 = 15
3f = 45
f = 15
Final quantity of type P tea in mixture F = 4f – 15
= 45 kg
Final quantity of type Q tea in mixture F = f + 15
= 30 kg
Cost price of final quantity of mixture F = 45 * 40
+ 30 * 60 = 1800 + 1800 = ₹3600
Selling price of final quantity of mixture F when
sold at ₹54 per kg = (45 + 30) * 54 = 75 * 54 =
4f + 4 = 5f ₹4050
f=4 Profit amount = 4050 – 3600 = ₹450

Click Here For Bundle PDF Course | support@guidely.in Page 15 of 18


Bank Po Mains PDF Course 2024
Quantitative Aptitude Day -17 (Eng)

Profit percent (450/3600) * 100 Hence, value of x + y


= 12.5% = 80 + 100
= 180
17) Answer: A
Total cost price of mixture D = 25 * 40 + 5 * 60 = 18) Answer: D
1000 + 300 = ₹1300 Per kg cost price of mixture A = (30 * 40 + 10 *
Per kg cost of mixture D = 1300/(25 + 5) = 60)/(30 + 10) = 1800/40 = ₹45
1300/30 = ₹130/3 Per kg cost price of mixture C = (45 * 40 + 5 *
Total cost of mixture G = 7x * 40 + x * 60 = 280x 60)/(45 + 5) = 2100/50 = ₹42
+ 60x = 340x Per kg cost price of final mixture when sold at
Per kg cost of mixture G = 340x/(7x + x) = ₹46.53 at 10% profit = 46.53 * (100/110) = ₹42.3
340x/8x = ₹85/2 Per kg cost price of final mixture when sold at
Per kg cost of final mixture = ₹42(8/11) = ₹48.4 at 10% profit = 48.4 * (100/110) = ₹44
₹470/11 Case 1: When cost price of final mixture is
By the rule of alligation: ₹42.3.
130/3 85/2 By the rule of alligation:
470/11 Mixture A Mixture C
Ratio in which mixtures D and G are mixed = 45 42
42.3
Ratio in which mixtures A and C are mixed = (42
– 42.3): (42.3 – 45) = 0.3: 2.7 = 1: 9
Percent of mixture A in the final mixture = (1/10)
So, the ratio in which mixture D and G are mixed * 100 = 10%
= 3: 8 Case 1: When cost price of final mixture is ₹44.
Hence, quantity of mixture G mixed in the By the rule of alligation:
mixture = (25 + 5) * (8/3) Mixture A Mixture C
x kg = 80 kg 45 42
x = 80 44
Now, Ratio in which mixtures A and C are mixed = (42
√(y2 – x2) = 60 – 44): (44 – 45) = 2: 1
√(y2 – 802) = 60 Percent of mixture A in the final mixture = (2/3) *
y2 – 6400 = 3600 100 = 66.67%
y2 = 10000
y = 100

Click Here For Bundle PDF Course | support@guidely.in Page 16 of 18


Bank Po Mains PDF Course 2024
Quantitative Aptitude Day -17 (Eng)

So, in order to make a mixture whose selling = 35 kg


price lies between ₹46.53 and ₹48.4, the
quantity of mixture A must lie between 10% and 20) Answer: B
66.67% of total quantity. P: x2 – 12x + 35 = 0
In statement I:Percent of mixture A in the final x2 – 7x – 5x + 35 = 0
mixture = (1/5) * 100 = 20% x(x – 7) – 5(x – 7) = 0
In statement II:Percent of mixture A in the final (x – 7)(x – 5) = 0
mixture = (2/11) * 100 = 18(2/11)% x = 7 and 5
In statement III:Percent of mixture A in the final Q: x2 – 19x + 88 = 0
mixture = (3/10) * 100 = 30% x2 – 8x – 11x + 88 = 0
Hence, all the 3 statements are TRUE. x(x – 8) – 11(x – 8) = 0
(x – 8)(x – 11) = 0
19) Answer: E x = 8 and 11
Per kg cost price of mixture A = ₹45 A: y2 – 14y + 33 = 0
Per kg cost price of mixture B = (50 * 40 + 10 * y2 – 3y – 11y + 33 = 0
60)/(50 + 10) = 2600/60 = ₹43.33 y(y – 3) – 11(y – 3) 0
Per kg cost price of mixture C = ₹42 (y – 3) (y – 11) = 0
Per kg cost price of mixture D = (25 * 40 + 5 * y = 3 and 11
60)/(25 + 5) = 1300/30 = ₹43.33 B: y2 + 9y – 52 = 0
Per kg cost price of mixture E = (50 * 40 + 20 * y2+ 13y – 4y – 52 = 0
60)/(50 + 20) = 3200/70 = ₹45.71 y(y + 13) – 4(y + 13) 0
So, mixtures X and Y will be mixtures B and D. (y + 13) (y – 4) = 0
Total quantity of type P tea in mixture M = 50 + y = 4 and -13
25 = 75 kg
Total quantity of type Q tea in mixture M = 10 + 5
= 15 kg
Total quantity of type P tea when 55 kg of
mixture N is mixed with mixture M = 75 + 55 *
(3/11) = 75 + 15 = 90 kg
Total quantity of type Q tea when 55 kg of
mixture N is mixed with mixture M = 15 + 55 *
(8/11) = 15 + 40 = 55 kg
Required difference = 90 – 55

Click Here For Bundle PDF Course | support@guidely.in Page 17 of 18


Bank Po Mains PDF Course 2024
Quantitative Aptitude Day -17 (Eng)

Hence, the statements PB and QB are TRUE.

Click Here For Bundle PDF Course | support@guidely.in Page 18 of 18


Bank Po Mains PDF Course 2024
English Day - 17

English Language
Directions (1-5) : Swap the words given in bold (if I. The GDP has seen a improve (A) upward
necessary) to make the given sentences trend, reflecting a steady growth in economic
meaningful and correct. activity and indicating (B) positive momentum for
1) the overall economy.
I. The furniture (A) was so soft and luxurious, it II. The country has to gradual (I) on exports by
felt like a gentle appeal (B) against my skin. implementing strategic trade policies, diversifying
II. The fabric (I) cushion is selected based on its (II) its export markets, and enhancing
durability, comfort, and aesthetic embrace (II) to competitiveness.
create the perfect balance of style and a) A-I
functionality. b) B-II
a) A-I c) Both a and b
b) B-II d) A-II
c) Both a and b e) No changes required
d) A-II
e) No changes required 4)
I. The baby was asleep, cradled (A) in the
2) exhaustion (B) of the crib, breathing softly and
I. Worshipping God is like a deeply (A) spiritual peacefully as dreams danced through its
journey, a connection to the divine that fills the innocent slumber.
heart with reverence and gratitude (B). II. The online session was so tiring, leaving me
II. The professor asked the students to be drained (I) and longing for a break from the
actively (I) engaged in the discussion, fostering screen-induced warmth (II).
(II) an environment of critical thinking and a) A-I
collaborative learning. b) B-II
a) A-I c) A-II
b) B-II d) Both a and b
c) Both a and b e) No changes required
d) B-I
e) No changes required 5)
I. The book has 15 chapters, each expressed (A)
3) into a different aspect of the protagonist's
transformative journey, creating a rich (B) and
multifaceted narrative.

Click Here For Bundle PDF Course | support@guidely.in Page 1 of 10


Bank Po Mains PDF Course 2024
English Day - 17

II.The author has delving (I) her profound c) Bihar, a state in eastern India, holds a
insights on human nature and the complexities of significant place in the country's history and
relationships through eloquent (II) prose that heritage also known for its ancient roots
resonates deeply with readers' emotions and d) While many factors contribute to the dynamics
experiences. of the tigers present in a region
a) A-I e) None of these
b) B-I
c) B-II 7) The Digital Data Protection Bill, 2023, was
d) Both a and c passed in the Lok Sabha on Monday and will
e) No changes required now have to be cleared by the Rajya Sabha. The
fresh iteration, which has undergone a few
Directions (6-10) : In each of these questions, a drafts, seems to have incorporated suggestions
paragraph is given that has a blank in it. Out of made to its 2022 version. Although it is not clear
the given options, only one sentence fits in with what the submissions were as the consultation
the context of the paragraph. Select that as your process was not brought to light by the
answer. government. __________.
6) Madhya Pradesh, for the second time in eight a) The highlight of the Bill is the provision that
years, reported 785 tigers, or about a fifth of the personal data of an individual may be processed
national count. The State reported a 50% rise in without the consent of the data principal
tigers since the last census, a figure bettered b) Data protection and privacy are essential
only by Bihar which has less than 10% of M.P’s rights in the digital age, ensuring that individuals
tigers______________________. M.P. over the have control over their personal information
years has perfected the approach of actively c) The Lok Sabha is the lower house of the
moving both tigers, as well as their prey, within Parliament of India, consisting of elected
the State to balance predator and prey representatives from various constituencies
population. across the country
a) Tigers, majestic and powerful predators, roam d) The highlight of the Bill is the provision that
the dense jungles with their distinctive orange personal data of an individual may be processed
coats and dark stripes only after the consent of the data principal
b) Madhya Pradesh, located in the heart of India, e) None of these
boasts a rich tapestry of history, culture, and
natural beauty 8) For the second time in three months,
Pakistan’s former Prime Minister Imran Khan has
been arrested.This arrest was after a conviction

Click Here For Bundle PDF Course | support@guidely.in Page 2 of 10


Bank Po Mains PDF Course 2024
English Day - 17

in a case involving the misappropriation of official c) The spacecraft entered lunar orbit on 5 August
gifts meant for the ‘Toshakhana’. He was last 2023
arrested on May 9 in the ‘Al Qadir’ trust case d) The primary issue was the five engines which
charge sheet.__________________. It means that were used to reduce the velocity.
he is now disqualified from Parliament for the e) None of these
next five years, and will not be able to stand for
election in polls due by November. 10) The Supreme Court on August 10
a) The latest conviction, which carries a sentence announced the launch of portal 'Suswagatam'.
of three years This would enable advocates, visitors, interns
b) Established in 1974, the Tosha­khana is a and others to register themselves online and get
department which stores precious gifts given to e-passes to enter the apex court. Suswagatam'
rulers is a web-based and mobile-friendly application
c) He transitioned into politics and became Prime that allows users to register themselves online
Minister after his party won the general elections and request for e-passes for various purposes
in 2018 such as attending court hearings, meeting
d) Conviction refers to a formal declaration by a advocates. ____________________
court of law that someone has been found guilty a) The portal issues electronic passes to the
of a criminal offence after a legal trial or people
proceeding b) The Supreme Court of India is the country's
e) None of these highest judicial court
c) The 'Suswagatam' portal was tested as a pilot
9) Indian Space Research Organisation (ISRO) project from July 25, 2023
has released two images taken by the d) Many such portals are available that help the
Chandrayaan-3 spacecraft._________________. functioning of courts easier
The photo was taken on July 14, when the e) None of these
Chandrayaan-3 mission was launched from the
Satish Dhawan Space Centre in Sriharikota.The Directions (11-15) : Rearrange the following five
second image is of the Moon captured by the sentences (A), (B), (C), (D) and (E) in the proper
Lander Horizontal Velocity Camera (LHVC) on sequence to form a coherent paragraph and then
board the spacecraft on August 6. answer the questions given below.
a) Chandrayaan-3 was launched on 14 July 2023 (A) And while monetary authorities are right in
from Satish Dhawan Space Centre laying the onus on the government for ensuring
b) The first image is of the Earth as viewed by timely supply side interventions to ‘limit the
the Lander Imager Camera severity and duration of such shocks’, a broader

Click Here For Bundle PDF Course | support@guidely.in Page 3 of 10


Bank Po Mains PDF Course 2024
English Day - 17

unmooring of price stability will undermine c) C


macroeconomic stability and growth. d) D
(B) RBI Governor Shaktikanta Das e) No error
acknowledged that while/A the likely short-term
nature of these shocks allowed for policymakers 13) Fill the given blank in sentence D.
to/B look a couple past of high inflation prints, a) remained
frequently/C recurring food price shocks risk b) unchanged
destabilising inflation expectations/D. c) altered
(C) Barely two months after projecting d) changed
Consumer Price Index-based inflation to average e) unnoticed
5.1% over the current fiscal year ending in March
2024, the MPC has raised its forecast for the 14) Replace the bold word in sentence E to
annual average by 30 basis points to 5.4%. make it correct and meaningful.
(D) The latest decision by the Reserve Bank of a) expansion
India’s Monetary Policy Committee (MPC) to b) climb
leave its policy rate _______ is a calculated risk c) growth
taken by the rate setting panel, especially when d) spike
viewed in light of the upward revision of its e) addition
inflation forecast.
(E) Conceding that a improvement in tomato 15) Which of the following is the correct
prices had contributed to a shock that had forced sequence of rearrangement ?
a revision in the headline inflation projection for a) eadcb
the July-September quarter by a ‘substantial’ 100 b) ceadb
basis points to 6.2%. c) bceda
11) Meaning of the word ‘Onus’ in sentence A. d) abdec
a) something that is one's duty or responsibility. e) dceba
b) onus is when all people come together
c) an official or person in high position Directions (16-20) : In the statements given
d) praising someone for his/her act below, a blank has been given. This needs to be
e) one’s unwanted commitment filled up using a correct word from the options
mentioned below. The statement so formed must
12) Spot the erroneous part in sentence B. be contextually and grammatically sound.
a) A 16) The curbs were to kick in with immediate
b) B effect, rattling the entire supply chain and ______

Click Here For Bundle PDF Course | support@guidely.in Page 4 of 10


Bank Po Mains PDF Course 2024
English Day - 17

fears of shortages and price surges, especially b) conquer


ahead of the festive season. c) construct
a) enhancing d) consensus
b) illuminating e) convince
c) igniting
d) deciding 19) All the claims of scientists having found a
e) running room-temperature superconductor so far have
failed to _______ independent scrutiny.
17) The government sought to suggest this was a) withstand
primarily driven by security concerns and b) standby
imported devices could be used for__________, c) stick with
just as mobile phones could have spyware. d) give up
a) survival e) stand up
b) surveillance
c) surpassing 20) A Supreme Court judgement in March said
d) surveying the panel should _______ the Prime Minister, the
e) surrendering Leader of the Opposition in Lok Sabha and the
Chief Justice of India.
18) The main goal of Ukraine and its western a) compromise
partners was to build ________ among major b) apprise
powers, especially in the Global South, on c) reprise
working towards a fair and durable peace. d) supervise
a) consult e) comprise

Click Here For Bundle PDF Course | support@guidely.in Page 5 of 10


Bank Po Mains PDF Course 2024
English Day - 17

Click Here to Get the Detailed Video Solution for the above given Questions
Or Scan the QR Code to Get the Detailed Video Solutions

Answer Key with Explanation


1) Answer: C The furniture cushion is selected based on its
Both A-I and B-II need to be swapped to make durability, comfort, and aesthetic appeal to
the sentences I and II correct and meaningful. create the perfect balance of style and
Swap fabric and furniture, similarly swap appeal functionality.
and embrace.
2) Answer: E
Furniture is an object that cannot be worn, All the labelled words are correct and
whereas fabric refers to the material and can be appropriate. Hence, no changes required.
felt on the skin - hence replacing fabric and
furniture. 3) Answer: A
Only words A and I need swapping. Improve and
Embrace - to put your arms around somebody gradual and placed inappropriately and must be
as a sign of love, happiness interchanged to make the given two sentences
Appeal - to be attractive or interesting to correct and meaningful. The other two words
somebody indicating and diversifying are correct.
In the sentence I -’ felt like gentle embrace’ In sentence I - 'gradual upward trend’ would be
would be the correct phrase correct - gradual(slow)
In the sentence II - ‘aesthetic appeal’ would be In sentence II - ‘the country has to improve on
the correct phrase exports’ would be correct
The sentences: The sentences:
The fabric was so soft and luxurious, it felt like a
gentle embrace against my skin.

Click Here For Bundle PDF Course | support@guidely.in Page 6 of 10


Bank Po Mains PDF Course 2024
English Day - 17

The GDP has seen a gradual upward trend, The sentences:


reflecting a steady growth in economic activity The book has 15 chapters, each delving into a
and indicating positive momentum for the overall different aspect of the protagonist's
economy. transformative journey, creating a rich and
The country has to improve on exports by multifaceted narrative.
implementing strategic trade policies, The author has expressed her profound insights
diversifying its export markets, and enhancing on human nature and the complexities of
competitiveness. relationships through eloquent prose that
resonates deeply with readers' emotions and
4) Answer: B experiences.
The words exhaustion and warmth must be
interchanged to make the sentences I and II 6) Answer: D
correct and meaningful. Sentence in option d fits the blank perfectly as
Exhaustion - a state of being incredibly tired the passage deals with the conservation of
Warmth - a fairly high temperature or the effect tigers, talking about tigers in general or about the
created by this, especially when it is pleasant involved states is not necessary here.
In sentence I - baby fell asleep by the warmth of D is more appropriate and suitable here.
the crib A - generic about tigers (not relevant)
In sentence II - the session was tiring and B - about Madhya Pradesh (not relevant)
exhausting C - about Bihar (not relevant)
The sentences: The only relevant sentence is D.
The baby was asleep, cradled in the warmth of
the crib, breathing softly and peacefully as 7) Answer: D
dreams danced through its innocent slumber. D is the correct sentence that is talking about the
The online session was so tiring, leaving me data protection bill and privacy.
drained and longing for a break from the screen- A is incorrect because it is well known that
induced exhaustion. usage of personal information without consent is
illegal.
5) Answer: A B is a general statement about data protection
Interchange ‘expressed’ and ‘delving’ to make and privacy
the sentence correct and meaningful. C conveys general information about the Lok
Delve - to search inside something sabha which is also irrelevant to the context of
the passage.

Click Here For Bundle PDF Course | support@guidely.in Page 7 of 10


Bank Po Mains PDF Course 2024
English Day - 17

8) Answer: A B - generic sentence about supreme court


Among the given statements the sentence which D - does not fit the context
completes the given passage is ‘a’ as it talks
about the recent conviction, the sentence which 11) Answer: A
follows the blank also talks about the conviction Onus means - one’s duty or responsibility
and punishment.
B - Toshakhanna is somehow related to the 12) Answer: C
context of the passage but the blank cannot be The error is in part c of the given sentence.
filled with this sentence as there is no continuity. The phrase ‘look a couple past of’ is completely
C - the sentence is about Imran Khan but in the wrong instead it must be ‘look past a couple of’.
passage given it is irrelevant to talk about his Look past - To see potential beyond obvious
D - definition of conviction is not really needed in flaws; to consider something more than
this context something else.
The sentence : RBI Governor Shaktikanta Das
9) Answer: B acknowledged that while the likely short-term
B is the right statement/sentence to fill the given nature of these shocks allowed for policymakers
blank because the sentence following the blank to look past a couple of high inflation prints,
talks about a photo/image which must be a frequently recurring food price shocks risk
continuation. So, B is the correct sentence when destabilising inflation expectations.
compared to the other given options.
A - launch date of chandrayaan 3 is 13) Answer: B
comparatively less relevant than sentence b ‘Unchanged’ is the correct word that fits the
C - also irrelevant because the passage talks given blank.
about images that ISRO released ‘To leave something’ conveys a meaning - to
D - talks about some issues related to the leave something undisturbed/to leave it the
spacecraft launch same way it is
Changed and altered are wrong, unnoticed and
10) Answer: C remained are also incorrect and does not convey
C is the correct sentence that completes the a proper meaning
passage as it revolves around the main idea of The sentence:
the passage - Suswagatam portal. The latest decision by the Reserve Bank of
A - redundant information as the passage India’s Monetary Policy Committee (MPC) to
already spoke about the same leave its policy rate unchanged is a calculated

Click Here For Bundle PDF Course | support@guidely.in Page 8 of 10


Bank Po Mains PDF Course 2024
English Day - 17

risk taken by the rate setting panel, especially 16) Answer: C


when viewed in light of the upward revision of its Out of the given options, igniting is the best word
inflation forecast. that fits the blank without changing the actual
meaning of the sentence.
14) Answer: D Igniting - to start burning or to make something
Improvement is an incorrect word and does not start burning
fit the given sentence. Enhancing - to improve something or to make
Among the given words ‘spike’ is the correct something look better (enhance fear would
word that fits the sentence without altering the sound wrong)
original meaning. Illuminating - helping to explain something or
Spike - A spike is a sudden and large move in make something clear (irrelevant)
the price of an asset Deciding and running are also inappropriate
The sentence : Conceding that a spike in tomato words and hence discarded
prices had contributed to a shock that had forced The sentence: The curbs were to kick in with
a revision in the headline inflation projection for immediate effect, rattling the entire supply chain
the July-September quarter by a ‘substantial’ and igniting fears of shortages and price surges,
100 basis points to 6.2%. especially ahead of the festive season.

15) Answer: E 17) Answer: B


The correct sequence of rearrangement is According to the given context ‘surveillance’ is
DCEBA. the correct word that fits the blank correctly.
D is the introductory sentence as it has got the As the context is related to security, surveillance
information on the latest decision of RBI. D is would be the best fit.
followed by C which conveys information The other given words have a different meaning
regarding the RBI’s forecast in the midst of and hence are not suitable.
which the RBI has left the policy rate unchanged. Survival - the act or fact of living
After DC, comes E which cites the reason for the Surveillance - the careful watching of somebody
inflation project - price of tomatoes. E is followed who may have done something wrong
by B which talks about the price shocks in detail Surpass - to do something better than
and what it leads to. Last sentence would be A, somebody/something else or better than
which adds more to the price shocks and expected
inflation projection.

Click Here For Bundle PDF Course | support@guidely.in Page 9 of 10


Bank Po Mains PDF Course 2024
English Day - 17

Survey - the process of collecting, analysing and Withstand is the correct word that fits the blank
interpreting data from many individuals making the sentence complete and meaningful.
Surrender - to stop fighting and admit that you Withstand - to be strong enough not to break,
have lost/to give somebody/something to give up
somebody else Standby - a thing or person that can be used if
The sentence: needed
The government sought to suggest this was Stick with - to continue using or doing
primarily driven by security concerns and Give up - to cease doing
imported devices could be used for surveillance, Stand up - to be or become vertical
just as mobile phones could have spyware. The sentence:
All the claims of scientists having found a room-
18) Answer: D temperature superconductor so far have failed to
Consensus is the right word that fits the blank. withstand independent scrutiny.
Consensus - agreement among a group of
people (build agreement would be the correct 20) Answer: E
phrase as it conveys a proper meaning) Comprise is the correct word that fits the given
Conquer - to take control (irrelevant) blank.
Consult - to ask somebody for some information Other words are discarded because of their
or advice (does not fit because different meaning as they don’t fit the context of the
meaning) sentence.
Construct - to build (not relevant) Compromise - an agreement
Convince - to succeed in making someone Apprise - inform or tell
believe something (does not fit the blank Reprise - repeat
properly) Supervise - to watch somebody/something to
The sentence: make sure that work is being done properly
The main goal of Ukraine and its western Comprise - to consist of
partners was to build consensus among major The sentence:
powers, especially in the Global South, on A Supreme Court judgement in March said the
working towards a fair and durable peace. panel should comprise the Prime Minister, the
Leader of the Opposition in Lok Sabha and the
19) Answer: A Chief Justice of India.

Click Here For Bundle PDF Course | support@guidely.in Page 10 of 10


Bank Po Mains PDF Course 2024
Reasoning Day -18 (Eng)

Reasoning Aptitude
Directions (1-5): Study the following information
carefully and answer the given questions.
Nine different mobile shops – A, B, D, G, I, J, M,
N and O run on different floors of a nine storey
building where the lowermost floor is numbered
one and the floor immediately above it is
a) I-A, II-D, III-B, IV-C
numbered two and so on. No two shops were run
b) I-B, II-C, III-A, IV-D
on the same floor.
c) I-B, II-A, III-C, IV-D
Note 1: The shop names which start with a vowel
d) I-A, II-B, III-D, IV-C
were not run on an odd-numbered floor of the
e) I-B, II-D, III-A, IV-C
building.
Note 2: The consecutive alphabetically named
3. Which among the following shop runs on the
shops were not run on the adjacent floors of the
prime numbered floor as per the given
building.
arrangement?
G runs below fourth floor. Only three shops run
I. The shop which runs two floors below I.
between G and D. I runs either three floors below
II. The shop which runs immediately above A.
or three floors above D. The number of floors
III. The shop which runs on the adjacent floor of
above I is two less than the number of floors
N.
below M. N neither runs adjacent floor of D nor
a) Only III
runs below third floor. At most two shops run
b) Only I and II
between N and O. A runs below J. The number
c) Only I and III
of floors between J and A is three less than the
d) Only II
number of floors between D and B.
e) Only II and III
1. N runs __ floors below D and __ floors above
O respectively?
4. The number of floors between B and N is
a) One; Three
same as the number of floors between __ and __
b) Three; Three
a) M and I
c) Three; Four
b) A and O
d) Two; Three
c) Both a and b
e) One; Two
d) D and I
e) Both a and d
2. Match the list 1 with list 2

Click Here For Bundle PDF Course | support@guidely.in Page 1 of 10


Bank Po Mains PDF Course 2024
Reasoning Day -18 (Eng)

5. The difference between the floor numbers of G c) Third to the left; Daughter-in-law
and A is equal to the difference between the floor d) Fourth to the right; Daughter-in-law
numbers of __ and __. e) Third to the left; Mother
a) M and N
b) D and G 7. The number of persons sitting between __ and
c) B and N __ is two less than the number of persons sitting
d) J and I between __ and __, when counted from the left of
e) None of these the first persons in the given pairs respectively.
a) H and Husband of N; Son-in-law of P and N
Directions (6-10): Study the following information b) Mother-in-law of A and W; N and Son of H
carefully and answer the given questions. c) Husband of H and A; K and P
Eight persons from the same family – A, C, F, H, d) Both a and b
K, N, P, and W are sitting around a circular table e) Both a and c
facing the centre with equal distance between
adjacent persons. Four married couples are 8.__ sits second to the left of the mother of W,
there in the family. who sits ___ to the left of the Mother of C?
Note: Neither the couples nor the persons of the a) A; Second
same gender are sitting adjacent to each other. b) K; Fifth
K sits third to the right of W’s father. C is the only c) P; Fourth
child of K’s brother. Only one person sits d) A; Third
between K and C’s grandmother(either from left e) K; Fourth
or right). A is the brother-in-law of W and sits
adjacent to C’s grandmother. N sits immediate 9. Which of the following pair of persons are
left of the one who sits opposite to A, who has no married couples?
siblings. The number of persons sitting between
H and C’s father is one more than the number of
persons sitting between C’s father and C, when
counted from the right of H and left of C. H is the
parent of C and daughter-in-law of F. P sits
second to the left of K’s brother.
6. What is the position of W with respect to F and
How F is related to H’s sister-in-law?
a) Third to the right; Mother-in-law
b) Second to the left; Mother

Click Here For Bundle PDF Course | support@guidely.in Page 2 of 10


Bank Po Mains PDF Course 2024
Reasoning Day -18 (Eng)

Rickshaw A travels for 6m towards the south


direction from the rickshaw stand to reach
stoppage 1. Then turns towards the left and
travels for 8m to reach stoppage 2. Then, from
stoppage 2 again turns towards the left and
travels for 4m to reach stoppage 3, from there it
turns towards the right and travels for 9m to
reach stoppage 4.
Rickshaw B travels for 4m towards the east
direction from the rickshaw stand to reach
stoppage 5, from there it takes a left turn and
a) I-C, II-D, III-B, IV-C travels for 13m to reach stoppage 6. Then turns
b) I-C, II-A, III-A, IV-D towards the right and travels for 6m to reach
c) I-B, II-A, III-C, IV-D stoppage 7. Stoppage 8 is 7m south of stoppage
d) I-D, II-B, III-D, IV-C 7.
e) I-C, II-A, III-D, IV-B Rickshaw C travels for 9m towards the north
direction from the rickshaw stand to reach
10. In which among the following pair of persons stoppage 9. Stoppage 10 is east of stoppage 9
the first person sits second to the left of the and south of stoppage 7. Stoppage 11 is
second person? northeast of stoppage 10 and 7m east of
a) The one who faces C, A stoppage 7.
b) F, The one who sits immediate right of N These stoppages are assigned different names
c) The one who is the sister-in-law of K, N according to the below given conditions:
d) The one who is the mother of W, P 1). If the distance between the consecutive
e) None of these stoppages reached by the rickshaw is a multiple
of three, then “P” is named to the second
Directions (11-15): Study the following stoppage reached by the rickshaw (Consider
information carefully and answer the given rickshaw stand as one stoppage).
questions. 2). If the distance between the stoppage and the
There are three rickshaws i.e. rickshaw A, rickshaw stand is an even number (calculate as
rickshaw B, and rickshaw C starts travelling from total distance), then stoppage is named as “Q”,
a common rickshaw stand. Each rickshaw travel else the stoppage is named as “K”.
to some stoppages. 3). If the distance between the consecutive
stoppages reached by rickshaw is an odd prime

Click Here For Bundle PDF Course | support@guidely.in Page 3 of 10


Bank Po Mains PDF Course 2024
Reasoning Day -18 (Eng)

number, then “R” is named to the first stoppage a) 36m


reached by rickshaw. b) 25m
4). If the stoppage is in northeast or south c) 49m
direction from the rickshaw stand, then the d) 16m
stoppage is named as “M”. e) 64m
5). If the stoppage is in southeast or north
direction from the rickshaw stand, then the 14. Find the shortest distance between the
stoppage is named as “N”. stoppages S10 and S11.
6). If the distance between the consecutive a) √65m
stoppages is a root value, then only either b) √63m
conditions 4 or 5 follows. c) 2√31m
11. The total distance between which of the d) 2√27m
following two stoppages is greater than 15 with e) 2√11m
respect to the final arrangement? (If the rickshaw
stand is named as Y) 15. Which of the following pairs of stoppages,
a) Stoppages 1 and 4 the first stoppage is in northeast of the second
b) Stoppages 6 and Y stoppage with respect to the final arrangement?
c) Stoppages 5 and 11 a)Stoppages 7 and 5
d) Stoppages Y and 10 b) Stoppages 11 and 9
e) All of the above c) Stoppages 6 and 10
d) Both a and b
12. What is the shortest distance between the e) Both a and c
starting point to its final destination of rickshaw
B? Directions (16-20): Each of the questions below
a) √135 m consists of statements below it. You have to
b) 6√34 m decide whether the data provided in the
c) 2√34 m statements are sufficient to answer the question
d) 6√37 m and mark the appropriate option as answer.
e) 4√37 m 16. Seven vegetables viz. A, B, C, D, E, F, and G
were cooked on different days from Monday to
13. Find the square value of the difference Sunday of the same week. G was cooked after
between the sum of the total distance between Thursday. Only two vegetables were cooked
S5 and S8 and the sum of the total distance between G and C. Which of the following
between S5 and S4.

Click Here For Bundle PDF Course | support@guidely.in Page 4 of 10


Bank Po Mains PDF Course 2024
Reasoning Day -18 (Eng)

vegetables was cooked on the second day of the d) One person sits between B and G. A sits
week? adjacent to G. C sits third to the right of F.
a) B was cooked immediately after C. The e) H sits opposite to B. Two persons between H
number of vegetables cooked before B is one and F. G sits second to the right of F.
less than the number of vegetables cooked after
E. 18. Eight persons – P, Q, R, S, T, U, V and W
b) D was cooked two days before F. Only one live on eight different floors of an eight storey
vegetable was cooked between C and D. Only building where the bottommost floor is numbered
two vegetables were cooked between F and A. B 1 and the topmost floor is numbered 8. R lives on
was cooked neither before E nor immediately one of the even numbered floors. Only three
after E. floors between R and T. Who lives two floors
c) As many vegetables cooked before C as after above T?
E. F was cooked three days before E. A was a) As many floors above T as below P. Only two
cooked adjacent to either E or F. floors between P and W. Q lives four floors
d) A was cooked two days after C. Only three above W. S lives adjacent floor of neither T nor
vegetables were cooked between A and E. W.
Atmost one vegetable was cooked between E b) Q lives two floors below T. As many floors
and F. above Q as below W. Only one floor between W
e) D was cooked adjacent to neither C nor G. and P.
Only three vegetables were cooked between D c) The number of floors between R and T is one
and E. F was cooked five days after A. more than the number of floors between R and
W. Q lives two floors above W. Not less than
17. Eight persons - A, B, C, D, E, F, G and H are three floors between W and S.
sitting around a circular table facing the centre. E d) Only two floors between R and W. Q lives four
sits third to the left of D. B sits second to the left floors below W. As many floors above W as
of E. Who sits second to the left of F? below P. U lives adjacent to P.
a) Two persons sit between B and F. C sits e) Q lives three floors below R. The number of
adjacent to B. More than two persons sit floors between R and Q is one less than Q and
between C and G, when counted from the right of W. P lives immediately above W. Only two floors
C. between P and S.
b) As many persons sit between D and B as E
and H. G sits second to the right of A. 19. Six persons – E, G, J, K, T and W have
c) I sits adjacent to B. G sits two places away different number of Toys. Atmost one person has
from H. less number of toys than W. Who among the

Click Here For Bundle PDF Course | support@guidely.in Page 5 of 10


Bank Po Mains PDF Course 2024
Reasoning Day -18 (Eng)

following person has the second highest number 20. There are six members in the family. Two
of Toys? married members and each married couple have
a) J has more toys than W but less than T. K has at least one child. How Q is related to R?
more toys than J and T but less than E. G has a) R is the mother of Q. U is the son of R’s only
more toys than T but less than E. son.
b) T has less toys than J but more than K. G has b) U is the son of Q’s brother. M is the only son
more toys than K but does not have the second of E, who is the father-in-law of T.
highest number of toys. c) T is the only daughter-in-law of E. M is the
c) T has more toys than J but less than K. G has father of U, who is the nephew of Q.
more toys than K. d) U is the daughter of Q’s brother. M is the only
d) Either a or b son of E, who is married to R.
e) None of these e) None of these

Click Here to Get the Detailed Video Solution for the above given Questions
Or Scan the QR Code to Get the Detailed Video Solutions

Answer Key with Explanation


Directions (1-5):
1. Answer: D
2. Answer: E
3. Answer: B
4. Answer: C
5. Answer: A
Final arrangement

Click Here For Bundle PDF Course | support@guidely.in Page 6 of 10


Bank Po Mains PDF Course 2024
Reasoning Day -18 (Eng)

We have,
 G runs below fourth floor.
 Only three shops run between G and D.
 I runs either three floors below or three
floors above D.
From the above conditions, there are three
possibilities

Again we have,
 A runs below J.
 The number of floors between J and A is
three less than the number of floors
between D and B.
From the above condition Case-2 gets
eliminated because the number of floors
Again we have, between J and A is three less than the number
 The number of floors above I is two less of floors between D and B. Hence Case-1 shows
than the number of floors below M. the final answer.
 N neither runs adjacent floor of D nor runs
below third floor.
 At most two shops run between N and O.
From the above condition Case-3 gets
eliminated because the number of floors above I
is two less than the number of floors below M.

Directions (6-10):
6. Answer: E

Click Here For Bundle PDF Course | support@guidely.in Page 7 of 10


Bank Po Mains PDF Course 2024
Reasoning Day -18 (Eng)

7. Answer: D From the above conditions, there are three


8. Answer: B possibilities
9. Answer: E
10. Answer: C
Final arrangement

Again we have,
 The number of persons sitting between H
and C’s father is one more than the
number of persons sitting between C’s
father and C, when counted from the right
of H and left of C.
 H is the parent of C and daughter-in-law
We have,
of F.
 K sits third to the right of W’s father.
 P sits second to the left of K’s brother.
 C is the only child of K’s brother.
Applying the above conditions, Case-2 gets
 Only one person sits between K and C’s
eliminated because C is not the father of W.
grandmother(either from left or right).
Case-1(a) gets eliminated because we cannot
 A is the brother-in-law of W and sits
place P. Hence, Case-1 shows the final answer.
adjacent to C’s grandmother.
 N sits to the immediate left of one who
sits opposite to A, who has no siblings.

Click Here For Bundle PDF Course | support@guidely.in Page 8 of 10


Bank Po Mains PDF Course 2024
Reasoning Day -18 (Eng)

Directions (16-20):
16. Answer: B

Directions (11-15):
11. Answer: E
12. Answer: C
13. Answer: B 17. Answer: D
14. Answer: A
15. Answer: D
Final arrangement

Click Here For Bundle PDF Course | support@guidely.in Page 9 of 10


Bank Po Mains PDF Course 2024
Reasoning Day -18 (Eng)

19. Answer: E

20. Answer: E
Clearly, none of the statements gives the relation
18. Answer: A
between Q and R.
Hence, option E is the correct choice.

Click Here For Bundle PDF Course | support@guidely.in Page 10 of 10


Bank Po Mains PDF Course 2024
Quantitative Aptitude Day -18 (Eng)

Quantitative Aptitude

Directions (01 - 05): Study the following b) 29.65%


information carefully and answer the questions c) 27.50%
given below. d) 15.35%
The table given below shows the number of e) 10.33%
mobiles [4G+5G] sold in shop A in five months
and also given the Percentage of the number of 2) Total number of mobile phones sold in shop B
4G mobiles sold in each month in shop A. in March is [(2M-N+O)/2] % more than that of
same month in shop A. If the ratio of the number
of 4G and 5G mobiles sold in shop B in March is
3:2 then find the total number of 5G mobiles sold
in shop A and shop B together in March?
a) 1452
b) 2952
c) 2150
d) 1664
e) 1894

3)
I. Number of 5G mobiles sold in April is ______
more than the number of 4G mobiles sold in
May.
Note: II. Number of 4G mobiles sold in February is ____
1. M+N+120=220 more than the number of 5G mobiles sold in
2. O% of 120 + 60% of 240=204 January.
3. 2M+O=50% of 260 Find which of the given option satisfy the above
1) If the number of 4G mobiles sold in June is blank?
increased by (N/4) % than the previous month a) 360,150
and the number of total mobiles sold in June is b) 360,120.
increased by (M/2) % than the previous month. c) 120,360
Find the percentage of increase in 5G mobiles d) 150,360.
sold in June than the previous month? e) None of these
a) 24.35%

Click Here For Bundle PDF Course | support@guidely.in Page 1 of 9


Bank Po Mains PDF Course 2024
Quantitative Aptitude Day -18 (Eng)

4) If the total number of 4G and 5G mobiles sold covered by car A in 6 hours. Car C can cover a
in all months is represented in the pie chart then distance of 255 km in 8.5 hours.
find the central angle made by the total number 6) Car A covers L km in (t+2) hours and car B
of 4G and 5G mobiles sold in May? covers M km in (t+1) hours. The value of L is 40
a) 600 km less than the distance between M and N.
b) 960 Then the distance covered by car C in 5t hours is
c) 1080 how much more than M km?
d) 720 a) 220km
e) None of these b) 260km
c) 250km
5) Number of 4G and 5G mobiles sold in June is d) 210km
840 and 680 respectively and the Number of 4G e) 280km
and 5G mobiles sold in July is 940 and 780
respectively. Find the difference between the 7) Car B covers (D+120) km in 5 hours and Car
total number of 4G mobiles sold in the first six E covers (2D+140) km in 15 hours. Find the
months and the total number of 5G mobiles sold average speed of car B and car E?
in the last six months till July? a) 145/3 km/hr
a) 20 b) 401/3 km/hr
b) 30 c) 332/3 km/hr
c) 50 d) 115/3km/hr
d) 80 e) 135/3 km/hr
e) None of these
8) Speed of car F is 20% more than the speed of
Directions (06 - 10): Study the following car D. Car F and Car C together covered some
information carefully and answer the questions distance in 5 hours. The total distance covered
given below. by these two cars is covered by car A in t hours.
The speed of car A is 33.33% more than the Find the value of t?
speed of car C. Speed of car C is 40% less than a) 11.75 hours
the speed of car B. A certain distance is covered b) 12.75 hours
by car D in 2a hours and same distance is c) 19.35 hours
covered by car A in 3a hours. Distance between d) 21.35 hours
two points P and Q is covered by car D in 3 e) None of these
hours. Distance between two points M and N is

Click Here For Bundle PDF Course | support@guidely.in Page 2 of 9


Bank Po Mains PDF Course 2024
Quantitative Aptitude Day -18 (Eng)

9) Car A start travelling at point P and car B at ____(n)____ days. They together complete 50%
point Q at 9 am towards Q and P respectively. of work in ___(o)___ days.
After some hours, they meet each other then car Find in which of the given options value of m, n,
B reached P and car A reached Q but car A and o satisfy the above condition.
returns to P. Find the difference between the I. m=1/4, n=8 and o=24/5
total time (in minutes) when both cars reached II. m=1/5, n=10 and o= 12
P? III. m=1/3, n=15 and o=10
a) 324 min a) Only I
b) 321 min b) Only III
c) 334 min c) Only I and III
d) 384 min d) Only II and III
e) None of these e) Only I and II

10) Point O and N is in the East and south 12) Cost price of the item is Rs.240. Shop
direction from point M. Distance between MN keeper marked up the item 60% above the cost
and MO is the same. Car A and D start their price then gave a discount D% and earn P%
journey with their original speed at the same time profit. Find the correct combination of values D
from M and they reached N and O respectively. and P in the given Column.
After reached their respective points, again they
start their journey from N and O and reached M
but both of them reached at same time. Find the
increased percent speed of car A when coming
back to M?
a) II-A
a) 50%
b) III-A
b) 100%
c) II-B
c) 200%
d) I-C
d) 75%
e) More than one option is correct
e) None of these

13) Speed of the boat in still water is m km/hr


Directions (11 - 15): Study the following
and the Speed of the stream is 4 km/hr. Boat can
information carefully and answer the questions
cover 48 km in 3 hours in upstream.
given below.
Find which of the given option is true from the
11) A can complete ___(m)____ part of work in 4
above data?
days and B can complete 1/3rd of work in

Click Here For Bundle PDF Course | support@guidely.in Page 3 of 9


Bank Po Mains PDF Course 2024
Quantitative Aptitude Day -18 (Eng)

I. The Speed of the car is (m+28) km/hr and car d) 40, 25


cover 144 km in 3 hours. e) More than one option is true.
II. The boat covers 116 km in 4.5 hours
downstream. Directions (16-20): Study the following
III. The distance covered by boat in 5 hours information carefully and answer the questions
downstream is the same as the distance covered given below. There are three rows, calculate the
by bus with the speed of 32 km/hour in 3 hours. roots of the equation and compare them with
a) Only II other columns. Check which one matches.
b) Only I 16)
c) Only III Equations Statements
d) Only I and II i) 5x2-18x+9=0 a) Sum of the roots is
e) None of these positive.
ii) 2x2-12x+18=0 b) Sum of roots is
14) Ratio of milk and water in a mixture is 3:2. negative.
Total quantity of the mixture is _____ liters and iii) 2x2+15x+13=0 c) Both roots are
40% mixture is taken out from the mixture then integers.
amount of milk in the remaining mixture is _____ a) ii)-b)
litres. b) i)-b)
Find which of the option is true which fill the c) i)-a) & iii) -b)
blank in same sequence? d) iii)-c)
a) 1050,378 e) None of these
b) 1430,512.8
c) 1200,430 17)
d) 1800,640 Equations Statements
e) None of these i) x2+16x+39=0 a) Sum of the roots is
positive.
15) Average age of A, B and C is ____ years. ii) x2+15x+56=0 b) Sum of roots is
Ratio of age of B and C is 3:2. Age of A after 5 negative.
years is 29 years. The difference in age between iii) x2-9x+14=0 c) Both the roots are
B and C is ______. positive.
Find which given option satisfies the blank? a) ii)-a)
a) 30, 15 b) i)-c)
b) 25, 18 c) i)-a) & ii) -c)
c) 28, 12

Click Here For Bundle PDF Course | support@guidely.in Page 4 of 9


Bank Po Mains PDF Course 2024
Quantitative Aptitude Day -18 (Eng)

d) iii)-c) negative.
e) None of these iii) x2+20x+75=0 c) Both the roots are
positive.
18) a) ii)-a)
Equations Statements b) i)-b)
i) x2 -56x+784=0 a) Sum of the roots is c) i)-b) & ii) -b)
positive. d) i)-c)
ii) x2+10x-171=0 b) Sum of roots is e) None of these
negative.
iii) 2x2+12x+16=0 c) Highest root is a 20)
perfect square Equations Statements
number. i) 4x2+2x-20=0 a) Sum of the roots is
a) ii)-a) positive.
b) i)-b) ii) 3x2-10x-13=0 b) Sum of roots is
c) ii)-c) & ii) -b) negative.
d) iii)-c) iii) 10x2-8x-2=0 c) Both the roots are
e) None of these positive prime
numbers.
19) a) ii)-a)
Equations Statements b) i)-a)
i) 2x2-19x+17=0 a) Sum of the roots is c) i)-a) & ii) -b)
positive. d) iii)-c)
ii) x2+16x+60=0 b) Both the roots are e) None of these

Click Here For Bundle PDF Course | support@guidely.in Page 5 of 9


Bank Po Mains PDF Course 2024
Quantitative Aptitude Day -18 (Eng)

Click Here to Get the Detailed Video Solution for the above given Questions
Or Scan the QR Code to Get the Detailed Video Solutions

Answer Key with Explanation


Directions (01 - 05):
2. O% of 120 + 60% of 240=204
Or, O=[204-144]*100/120=50
3. 2M+O=50% of 260
So, 2M=130-50=80, M=40
1. M+N+120=220
So, N=220-120-40=60
The Number of mobile phones sold in January is 1) Answer: C

1200. The total number of mobiles sold in June is

The Number of 4G mobile phones sold in 1200*120/100=1440

January is 1200*40/100=480 Number of 4G mobiles sold in June is

The number of 5G mobile phones sold in 720*[100+(N/4)]/100=828

January is 1200-480=720 The Number of 5G mobiles sold in June is 1440-

The Number of mobile phones sold in February 828=612

is 2600-1200=1400 So, increase percentage = [(612-

The Number of 4G mobiles sold in February is 480)/480]*100=27.5%

1400*60/100=840
The Number of 5G mobiles sold in February is 2) Answer: D

1400-840=560 Number of mobile phone sold in shop B in March

Similarly, we can calculate forother months also. is 1600*[100+(80-


60+50)/2]=1600*135/100=2160
The number of 5G mobiles sold in shop B in
March is 2160*2/5=864

Click Here For Bundle PDF Course | support@guidely.in Page 6 of 9


Bank Po Mains PDF Course 2024
Quantitative Aptitude Day -18 (Eng)

The total number of 5g Mobile sold in March in So, The ratio of the speed of cars B and C is 5:3.
shop A and B together is 864+800=1664 So, The speed of cars A, B, C, and D is 4:5:3:6.
The Speed of car C is 255/8.5=30 km/hr.
3) Answer: B So, The Speed of car A, B, and D is 4*10=40
I. Number of 5G mobiles sold in April is __(1080- km/hr, 5*10=50 km/hr, 6*10=60 km/hr.
720)=360___ more than the number of 4G Distance betweenP and Q is 60*3= 180 km
mobiles sold in May. Distance between M and N is 40*6=240 km
II. Number of 4G mobiles sold in February is 6) Answer: C
__(840-720)=120_ more than the number of 5G 40*(t+2)=L=240-40=200km
mobiles sold in January. Or, 40t=120, t=3
So, the answer is 360,120. M=50*4=200km
C cover distance in 5*3=15 hours is
4) Answer: A =15*30=450km
The central angle made by total mobiles sold in So, the difference is 450-200=250 km
May
= 7) Answer: D
[1200/(1200+1400+1600+1800+1200)]*360=600 So, D+120=5*50=250
Or, D=250-120=130
5) Answer: A So, the speed of car E is
Required difference [130*2+140]/15=400/15km/hr
= [480+840+800+720+720+840] – So, average speed is = (400/15 + 50)/2 =230/6 =
[560+800+1080+480+680+780] 115/3km/hr
= 20
8) Answer: B
Directions (06 - 10): The speed of car F is 60*120/100=72 km/hr.
The speed of car A is 33.33% more than the Total distance covered by car F and Car C is
speed of car of C. 5*(72+30)=510km
So, the speed ratio of car A and car C is 4:3. So, value of t= 510/40=12.75 hours
The Speed ratio of cars A and D is 2:3.
The Speed of car C is 40% less than the speed 9) Answer: A
of car B. So, car A cover 180*2=360 Km distance.
So, if the speed of car B is 100 then the speed of Total time taken by car A is = 360/40=9
car C is 60. hours=540 min

Click Here For Bundle PDF Course | support@guidely.in Page 7 of 9


Bank Po Mains PDF Course 2024
Quantitative Aptitude Day -18 (Eng)

Car B cover only 180 km. 13) Answer: B


So, time taken by car B is 180/50=3.6 hours=216 The Speed of the boat upstream is =48/3=16
min km/hr
The difference in time is 540-216=324 min The Speed of the boat in still water is
16+4=20km/hr=m.
10) Answer: C I. The Speed of the car is (m+28) km/hr and the
D cover full journey in (240/60) * 2 =8 hrs car cover 144 km in 3 hours.
A cover first half journey in (240/40) =6 hrs So, [20+28]*3=144km
Remaining hrs = 2 So, it is true.
Increase in speed of car A = 240/2 = 120km/hr II. The boat covers 116 km in 4.5 hours
Required increase in speed = (120-40)/40 * 100 downstream.
= 200% [20+4]*4.5=108 km, so it is false.
III. The distance covered by boat in 5 hours in
11) Answer: A downstream is the same as the distance covered
Now check the value of I, by bus with the speed of 32 km/hour in 3 hours.
So, m=1/4, A can complete in 4*4=16 days, Boat covers in 5 hours in downstream is
B can complete in 8*3=24 days 5*24=120 km
So, the total work is LCM of 16, and 24 is 48 Bus covers in 3 hours is 32*3=96 km
units. So, it is false,
The efficiency of A and B is 3 and 2 units/day Only I is true.
respectively.
A and B together complete 50% of work in 24/5 14) Answer: A
days. Let's check option a,
Options I is true. Similarly, we can check others The total quantity of the mixture is 1050 liters,
options so other two options are false. The amount of milk is 1050*3/5=630 liters
Left milk is 630*60/100=378 liters
12) Answer: E So, it is true. Similarly, we check other options
From the given options E is true. also. So, only A is true.
Marked price is 240*160/100=Rs.384
Selling price 384*75/100=288 15) Answer: C
Profit percentage = [(288-240)/240] *100=20% Let's check the option C,
Similarly, we can check for other values also. II- The average age of A,B, and C is 28years, total
A also satisfies the given condition. age is 28*3=84years

Click Here For Bundle PDF Course | support@guidely.in Page 8 of 9


Bank Po Mains PDF Course 2024
Quantitative Aptitude Day -18 (Eng)

The Sum of the age of B and C is 84-[29- or, (x-28)2=0, x=28, 28


5]=60years x2+10x-171=0
So, difference in age of B and C is 60/5=12years or, x2+19x-9x-171=0
So, it is true. or, (x+19)(x-9)=0, x=-19,9
2x2+12x+16=0
16) Answer: C 2x2+8x+4x+16=0
5x2-18x+9=0 Or, (x+4)(2x+4)=0, x=-4,-2
Or, 5x2-15x-3x+9=0 So, option C is true.
Or, (x-3)(5x-3)=0, x=3,5/3
2x2-12x+18=0 19) Answer: D
Or, 2x2-6x-6x+18=0 2x2-19x+17=0
Or, (x-3)(2x-6)=0, x=3,3 Or, 2x2-17x-2x+17=0
2x2+15x+13=0 Or, (x-1)(2x-17)=0, x=1,17/2
Or, 2x2+13x+2x+13=0 x2+16x+60=0
Or, (2x+13)(x+1)=0, x=-1,-13/2 or, x2+10x+6x+60=0
So, option c is true. or, (x+10)(x+6)=0, x=-10,-6
x2+20x+75=0
17) Answer: D or, x2+15x+5x+75=0
x2+16x+39=0 or, (x+15)(x+5)=0, x=-15,-5
or, x2+13x+3x+39=0 so, option D is true.
or, (x+13)(x+3)=0, x=-13,-3
x2+15x+56=0 20) Answer: A
or, x2+7x+8x+56=0 4x2+2x-20=0
or, (x+7)(x+8)=0, x=-7,-8 Or, 4x2+10x-8x-20=0
x2-9x+14=0 Or, (2x+5)(2x-4)=0, x=-5/2,2
or, x2-7x-2x+14=0 3x2-10x-13=0
or, (x-7)(x-2)=0, x=7,2 Or,3x2-13x+3x-13=0
so, option D is true. Or, (3x-13)(x+1)=0, x=13/3,-1
10x2-8x-2=0
18) Answer: C Or, 10x2-10x+2x-2=0
x2-56x+784=0 Or, (x-1)(10x+2)=0, x=1,-1/5
or, x2-2*28x+282=0 So, option A is true.

Click Here For Bundle PDF Course | support@guidely.in Page 9 of 9


Bank Po Mains PDF Course 2024
English Day - 18

English Language
Directions (1-5): Rearrange the following five 1) Which of the following is the FOURTH
sentences (A), (B), (C), (D) and (E) in the proper sentence after the rearrangement ?
sequence to form a coherent paragraph and then a) A
answer the questions given below. b) B
(A) Prior to this announcement, earlier in 2020, c) C
MEiTY notified rules stating that the Central d) D
government may allow Aadhaar authentication e) E
by requesting entities in the interest of good
governance, prevention of leakage of public 2) Which of the following is the FIRST sentence
funds and to promote ease of living. after the rearrangement ?
(B) The Registrar General of India (RGI) a) A
appointed under the Registration of Birth and b) B
Death Act, 1969 can now perform the yes or no c) C
Aadhaar authentication during registration of d) D
births and deaths in the country. e) E
(C) The State governments and Union Territory
administrations shall adhere to the guidelines 3) Which of the following sentences immediately
with respect to the use of Aadhaar authentication follows ‘sentence B’ in the rearranged sequence
as laid down by MEiTY. ?
(D) A gazette notification published in June said a) A
the Ministry of Electronics and Information b) B
Technology (MEiTY) has allowed the RGI office c) C
to use the Aadhaar database for authenticating d) D
the identity details provided during registration of e) E
births and deaths.
(E) The authentication may be on a voluntary 4) Which of the following is the SECOND
basis, for verification of Aadhaar number being sentence after the rearrangement ?
collected along with other details, as the case a) A
may be, for the purpose of establishing the b) B
identity of child, parent and the informant in case c) C
of deaths during registration of births or deaths”. d) D
e) E

Click Here For Bundle PDF Course | support@guidely.in Page 1 of 11


Bank Po Mains PDF Course 2024
English Day - 18

5) Which of the following is the LAST sentence 8. From June 1, India has made tests
after the rearrangement ? ___________ for cough syrups before they are
a) A exported. This step is the result of some reports
b) B regarding the contamination of cough syrups.
c) C A. propel
d) D B. contradictory
e) E C. remorse
D. mandatory
Directions (6-10): Given below are a few E. None of the above
questions with a blank in each of them. You have
to fill the blanks with appropriate word in order to 9. Mangoes priced at a _____________ Rs 2.5
make sentence contextually correct. If none of lakh per kg in the global market were stolen from
the options are correct then choose option E as a farm in Odisha’s Nuapada district shortly after
your answer. the farm owner shared photos of the fruit on
6. The Uttar Pradesh Madrassa Education Board social media.
has issued orders to celebrate Yoga Day in all A. stunning
aided and non-aided ___________ madrassas of B. staggering
the state on the occasion of International Yoga C. disgusting
Day. D. palpitating
A. recommended E. None of the above
B. primitive
C. inductive 10. Overall, India is the largest provider of
D. recognised generic drugs globally, ____________ over 50 per
E. None of the above cent of global demand for various vaccines,
about 40 per cent of generic demand in the US
7. Modi said the International Day of Yoga was and about 25 per cent of all medicines in the UK.
special this year as researchers at India's A. importing
research stations in the Arctic and Antarctica too B. mending
were _____________ in the celebrations. C. distracting
A. participating D. supplying
B. escalating E. None of the above
C. precipitating
D. mandating
E. None of the above

Click Here For Bundle PDF Course | support@guidely.in Page 2 of 11


Bank Po Mains PDF Course 2024
English Day - 18

Directions (11-15): Read each question to find e) E


out whether there is any grammatical error in it.
The error, if any, will be in one part of the 14. Students should always stay motivated and
sentence. Choose the option with that part as stop seeking/A motivation from outside instead
your answer. If there is no error, mark(e) as your one must/B always ask themselves one question
answer. time and again,/C “Did I take a forward step
11. Families who were caught together/A at the today that leads to my success ?”/D.No error/E
US-Mexico border and separated/B are now a) A
being offered an opportunity/C to reunite and b) B
stay together/D.No error/E c) C
a) A d) D
b) B e) E
c) C
d) D 15. During the pandemic the/A companies were
e) E providing more and more benefits to their/B
employees and the employees fell for the/C
12. He was right pretty much /A when he said furniture and gift hampers not knowing what was
almost all the people/B fail to change their coming for them/D.No error/E
behaviour and continue to/C blame the system a) A
for all their problems/D.No error/E b) B
a) A c) C
b) B d) D
c) C e) E
d) D
e) E Directions (16-17): A sentence is divided into five
parts. Two of these parts may have an error in it.
13. The human eardrum is able to receive/A and Choose the parts which have error in it. If there is
transmit signals across a radio/B frequency no error in the given statement, choose ‘No error’
range more greater than/C what we can hear as your answer.
with the human ear/D.No error/E 16. The commission amount was evenly
a) A distributed (A)/ among the two agents, following
b) B which (B)/ they were expected to report on their
c) C superior (C)/ to elaborate on the progress of their
d) D ongoing projects (D).

Click Here For Bundle PDF Course | support@guidely.in Page 3 of 11


Bank Po Mains PDF Course 2024
English Day - 18

a) A and D a) What we need to do is to focus on them, and


b) B and C to hone their skills in every possible way to get
c) B and D them acquainted with how they could
d) C and A revolutionize the society for good.
e) No error b) Education is the main ground of each student,
where they can develop their skills in their
17. One of the two administrative positions has chosen field.
(A)/ been opened for quite some time (B)/, and c) Not only does education train minds into a
you have made the right decision (C)/ by variety of subjects that will later on help them in
choosing to apply for the most efficient one (D)/. their preferred career, it also provides a safe
a) A and B place for students that they can consider as a
b) C and D second home.
c) B and D d) Moreover, the concept of learning is to
d) C and A prepare these young people for the world that
e) No error would not always cater to their comfort.
e) All the above
Directions (18-20): Directions: In each of the
following questions a short passage is given with 19. Since long time, many trains including metros
one of the lines in the passage missing and have been running on electricity. Indian Railways
represented by a blank. Select the best out of the trains are now almost running on electricity.
five answer choices given, to make the passage Electric bike, electronic car, electronic rikshaw
complete and coherent. are already in market. Now people should use
18. Unbeknown to others, the effectiveness of more electric vehicles in place of traditional
creating a progressive change in the society can petrol and diesel vehicles. __________________.
only be achieved if we start to believe in the Government has started campaigns to promote
capabilities of these young people. Empowering use of electric vehicles. Some rebate on taxes
the youth by means of training their leadership and subsidy on purchasing the electric vehicles,
and critical thinking skills approach many are also provided by the Government. Recently
beneficial factors to our main aim; to create a Delhi Government has launched ‘Switch Delhi’
step-forward action for the betterment of the campaign to promote the use of electric vehicles.
country. Young people can also become agents a) Dedicated social media handles were
of social change as much as the current leaders assigned to share updates on policy
of our generation. ________________________. implementation, provide information about
incentives, resolve grievances, and answer

Click Here For Bundle PDF Course | support@guidely.in Page 4 of 11


Bank Po Mains PDF Course 2024
English Day - 18

queries pertaining to EVs as part of the welcome respite. Other students may feel relief
campaign. from not having the pressure to look good or
b) As electric vehicle is cheaper in long run and meet certain social expectations, and shy or
also environment friendly, Government is trying anxious students may find it easier to reach out
to acquaint the people with the advantages of for teacher assistance. A lack of social activities
using electric vehicles using serious measures. such as clubs or sports teams can also allow
c) With the subsidy granted by the government students to focus more intensely on their
being reduced to 15 per cent, it is clear that the schoolwork and improve their study habits.
electric vehicle ecosystem in India is growing a) Children who have anxiety are often quiet and
rapidly and there is demand. obedient and do not talk about their fears or
d) HOP Electric Mobility Co-Founder & Chief worries explicitly.
Operating Officer Nikhil Bhatia supported the b) Additionally, classroom learning can be
government's move saying It was time for the EV expensive since the educational institutions
industry to stand on its own require expenses for resources, and that
e) All the above increases the cost of courses.
c) Many children have flourished during online
20. Students may welcome the lack of social learning because they aren’t feeling the social
pressure that comes with online learning. The pressure that comes with being on campus.
social aspect of the classroom learning can be a d) In the case of classroom learning, trainers
distraction for some students and anxiety- have the opportunity to observe and interact with
inducing for others. ____________. In particular, children, which enables the trainers to identify
children who may be the victims of bullying at the strengths and learning style of the children.
school have found online learning to be a e) All the above
Click Here to Get the Detailed Video Solution for the above given Questions
Or Scan the QR Code to Get the Detailed Video Solutions

Answer Key with Explanation

Click Here For Bundle PDF Course | support@guidely.in Page 5 of 11


Bank Po Mains PDF Course 2024
English Day - 18

1. Answer: C noticed well, ‘D’ is more of a gist of the whole


‘C’ is the fourth sentence after rearrangement. paragraph that is to be formed whereas ‘B’
The correct sequence is DBECA. contains specific information on the
The first or the introductory sentence is ‘D’ authentication mechanism. A paragraph always
because sentences A,C and E do not contain with generic information and moves to specific
any introduction, instead they are a continuation content. With this we conclude that ‘D’ is our first
of one another. So the options to pick the statement of the paragraph. Immediate to ‘D’
introductory sentence are two - B and D. If follows ‘B’ which holds the full form of RGI which
noticed well, ‘D’ is more of a gist of the whole is mentioned in the first sentence. After ‘B’
paragraph that is to be formed whereas ‘B’ comes ‘E’ which talks about the purpose of this
contains specific information on the authentication and the need. Fourth sentence
authentication mechanism. A paragraph always would be ‘C’ as it talks about who should adhere
with generic information and moves to specific to the announcement that has been made by the
content. With this we conclude that ‘D’ is our first ministry. Lastly, it is ‘A’ which goes back to 2020
statement of the paragraph. Immediate to ‘D’ when the ministry had an idea to allow Aadhar
follows ‘B’ which holds the full form of RGI which authentication to make life easier.
is mentioned in the first sentence. After ‘B’
comes ‘E’ which talks about the purpose of this 3. Answer: E
authentication and the need. Fourth sentence The correct sequence is DBECA.
would be ‘C’ as it talks about who should adhere The sentence which immediately follows ‘B’ is
to the announcement that has been made by the ‘E’.
ministry. Lastly, it is ‘A’ which goes back to 2020 The first or the introductory sentence is ‘D’
when the ministry had an idea to allow Aadhar because sentences A,C and E do not contain
authentication to make life easier. any introduction, instead they are a continuation
of one another. So the options to pick the
2. Answer: D introductory sentence are two - B and D. If
‘D’ is the first sentence after the rearrangement. noticed well, ‘D’ is more of a gist of the whole
The correct sequence is DBECA. paragraph that is to be formed whereas ‘B’
The first or the introductory sentence is ‘D’ contains specific information on the
because sentences A,C and E do not contain authentication mechanism. A paragraph always
any introduction, instead they are a continuation with generic information and moves to specific
of one another. So the options to pick the content. With this we conclude that ‘D’ is our first
introductory sentence are two - B and D. If statement of the paragraph. Immediate to ‘D’

Click Here For Bundle PDF Course | support@guidely.in Page 6 of 11


Bank Po Mains PDF Course 2024
English Day - 18

follows ‘B’ which holds the full form of RGI which when the ministry had an idea to allow Aadhar
is mentioned in the first sentence. After ‘B’ authentication to make life easier.
comes ‘E’ which talks about the purpose of this
authentication and the need. Fourth sentence 5. Answer: A
would be ‘C’ as it talks about who should adhere The last sentence after the rearrangement is ‘A’.
to the announcement that has been made by the The correct sequence is DBECA.
ministry. Lastly, it is ‘A’ which goes back to 2020 The first or the introductory sentence is ‘D’
when the ministry had an idea to allow Aadhar because sentences A,C and E do not contain
authentication to make life easier. any introduction, instead they are a continuation
of one another. So the options to pick the
4. Answer: B introductory sentence are two - B and D. If
‘B’ is the second sentence after rearrangement. noticed well, ‘D’ is more of a gist of the whole
The correct sequence is DBECA. paragraph that is to be formed whereas ‘B’
The first or the introductory sentence is ‘D’ contains specific information on the
because sentences A,C and E do not contain authentication mechanism. A paragraph always
any introduction, instead they are a continuation with generic information and moves to specific
of one another. So the options to pick the content. With this we conclude that ‘D’ is our first
introductory sentence are two - B and D. If statement of the paragraph. Immediate to ‘D’
noticed well, ‘D’ is more of a gist of the whole follows ‘B’ which holds the full form of RGI which
paragraph that is to be formed whereas ‘B’ is mentioned in the first sentence. After ‘B’
contains specific information on the comes ‘E’ which talks about the purpose of this
authentication mechanism. A paragraph always authentication and the need. Fourth sentence
with generic information and moves to specific would be ‘C’ as it talks about who should adhere
content. With this we conclude that ‘D’ is our first to the announcement that has been made by the
statement of the paragraph. Immediate to ‘D’ ministry. Lastly, it is ‘A’ which goes back to 2020
follows ‘B’ which holds the full form of RGI which when the ministry had an idea to allow Aadhar
is mentioned in the first sentence. After ‘B’ authentication to make life easier.
comes ‘E’ which talks about the purpose of this
authentication and the need. Fourth sentence 6. Answer: A
would be ‘C’ as it talks about who should adhere Here, recommended fits best as per the context
to the announcement that has been made by the of the sentence as it means to suggest that you
ministry. Lastly, it is ‘A’ which goes back to 2020 would find that thing or person good or useful

Click Here For Bundle PDF Course | support@guidely.in Page 7 of 11


Bank Po Mains PDF Course 2024
English Day - 18

and here it is said in the context of Yoga. So, something and here it is said for generic drug.
option A is the correct answer. So, option D is the correct answer.
Primitive means very simple and not developed. Mending means to repair something that is
Inductive means using particular facts and damaged or broken.
examples to form general rules and principles.
11. Answer: A
7. Answer: A The error is in part A of the sentence.
Here, participating fits best as per the context of Replace ‘who’ with ‘which’(even though family
the sentence as it means to take part or become consist of member/people it is an entity and not
involved in something and here it is said for a member by itself so it should be represented
Yoga day. So, option A is the correct answer. by ‘which’)
The corrected sentence: Families which were
8. Answer: D caught together at the US-Mexico border and
Here, mandatory fits best as per the context of separated are now being offered an opportunity
the sentence as it means compulsory and here it to reunite and stay together.
is said for cough syrup. So, option D is the
correct answer. 12. Answer: A
Propel means to push or drive usually forward or The error is in part A of the sentence.
onward. He was right - is the main idea.
Remorse means a feeling or sadness because Pretty much - nearly right(to agree that he was
you have done something wrong. right)
He was pretty much right - is the correct phrase.
9. Answer: B The corrected sentence : He was pretty much
Here, staggering fits best as per the context of right when he said almost all the people fail to
the sentence as it means something that you change their behaviour and continue to blame
find difficult to believe and here it is said for such the system for all their problems.
an unbelievably high prices of mangoes. So,
option B is the correct answer. 13. Answer: C
Part C is erroneous.
10. Answer: D More greater(redundancy in the degree of
Here, supplying fits best as per the context of the comparison) - it is wrong
sentence as it means to give or provide The corrected sentences:

Click Here For Bundle PDF Course | support@guidely.in Page 8 of 11


Bank Po Mains PDF Course 2024
English Day - 18

Replace ‘more’ with ‘far’ - The human eardrum is something that has happened and ‘report to’
able to receive and transmit signals across a means to give people information about what
radio frequency range far greater than what we you have seen, heard, done, etc. Since the two
can hear with the human ear. agents are conveying information about their
Simply remove more - The human eardrum is projects to their superior here, it is right to
able to receive and transmit signals across a replace ‘report on’ by ‘report to’.
radio frequency range greater than what we can Parts A and D are correct.
hear with the human ear. The right sentence is:
The commission amount was evenly distributed
14. Answer: E between the two agents, following which they
The given sentence is error free as it is were expected to report to their superior to
grammatically correct and meaningful. elaborate on the progress of their ongoing
projects.
15. Answer: B
Error is in part B of the sentence. 17. Answer: C
Replace ‘their’ with ‘its’. Company is an Part B is incorrect. ‘Has been open’ means that
entity(not a living thing) and usage of ‘their’ is the subject of this phrase was opened some time
inappropriate. ago and is still open. ‘Has been opened’ means
The corrected sentence: During the pandemic that the action of "being opened" is completed.
the companies were providing more and more Thus, the position is still open because the
benefits to their employees and the employees speaker has just applied for the job now. Thus,
fell for the furniture and gift hampers not knowing ‘has been opened’ should be replaced by ‘has
what was coming for them. been open’.
Part D is incorrect. When referring to two
16. Answer: B persons, places or things use the comparative
The word ‘among’ is incorrect here. ‘Among’ is form; when referring to more than two, use the
usually used with more than two subjects; superlative form. Thus, ‘most’ should be
‘between’ is used with exactly two subjects. replaced by ‘more.
Since it is clearly mentioned that there are only Parts A and C are right.
two people involved here, it is right to replace The right sentence is:
‘among’ by ‘between’ in part B. One of the two administrative positions has been
Part C of the sentence is incorrect because open for quite some time, and you have made
‘report on’ means to write or speak about

Click Here For Bundle PDF Course | support@guidely.in Page 9 of 11


Bank Po Mains PDF Course 2024
English Day - 18

the right decision by choosing to apply for the possible way to get them acquainted with how
more efficient one. they could revolutionize the society for good.

18. Answer: A 19. Answer: B


The sentence before the blank talks about how The right answer is option b. Because the
students could be agents of social change. sentences after the blank talks about how the
Option a bets fits the blank because the government is trying to bring in awareness
sentence explains how the skills of the youth among the public about electric vehicles. The
should be honed (made perfect) and how they sentence in option B introduces the same the
should be made acquainted (familiar) with how involvement of government for the first time.
they could revolutionize the society for good. Thus, option B is the right answer.
Here, revolutionize means to change radically. Option a talks about a particular campaign since
Thus, option a speaks about how the youth the word is preceded by ‘the’ and not ‘a’. ‘The
should be prepared to be the agents of social campaign’ denotes a campaign that has already
change that is mentioned in the sentence before been discussed and thus, option a is incorrect.
the blank. Thus, option a best suits that blank. Option c and d are incorrect because the
Options b, c, and d talk about education and the involvement of government since the passage
passage given in the passage does not talk before the blank has not mentioned anything
anywhere about the education of the youth. about the involvement of the government.
Thus, options b, c, and are incorrect. The right paragraph is:
The right passage is: Since long time, many trains including metros
Unbeknown to others, the effectiveness of have been running on electricity. Indian Railways
creating a progressive change in the society can trains are now almost running on electricity.
only be achieved if we start to believe in the Electric bike, electronic car, electronic rikshaw
capabilities of these young people. Empowering are already in market. Now people should use
the youth by means of training their leadership more electric vehicles in place of traditional
and critical thinking skills approach many petrol and diesel vehicles. As electric vehicle is
beneficial factors to our main aim; to create a cheaper in long run and also environment
step-forward action for the betterment of the friendly, Government is trying to acquaint the
country. Young people can also become agents people with the advantages of using electric
of social change as much as the current leaders vehicles using serious measures. Government
of our generation. What we need to do is to has started campaigns to promote use of electric
focus on them, and to hone their skills in every vehicles. Some rebate on taxes and subsidy on

Click Here For Bundle PDF Course | support@guidely.in Page 10 of 11


Bank Po Mains PDF Course 2024
English Day - 18

purchasing the electric vehicles, are also of classroom learning and do not focus on
provided by the Government. Recently Delhi elaborating on where children stand when it
Government has launched ‘Switch Delhi’ comes to online learning.
campaign to promote the use of electric vehicles. The right passage is:
Students may welcome the lack of social
20. Answer: C pressure that comes with online learning. The
The sentence after the blank begins with ‘in social aspect of the classroom learning can be a
particular’ which means the sentence is just distraction for some students and anxiety-
about to render information that is an addition to inducing for others. Many children have
the information stated in the previous sentence. flourished during online learning because they
The sentence after the blank states how children aren’t feeling the social pressure that comes with
who are victims of bullying at school, in being on campus. In particular, children who
particular, find online learning welcoming. Thus, may be the victims of bullying at school have
the sentence in the blank should speak about found online learning to be a welcome respite.
how children, in general, have grown to like Other students may feel relief from not having
online learning. Thus, option c is the right the pressure to look good or meet certain social
answer. expectations, and shy or anxious students may
Options a is incorrect it talks about children with find it easier to reach out for teacher assistance.
anxiety in general and states no direct A lack of social activities such as clubs or sports
information of the children’s opinion on online teams can also allow students to focus more
learning. intensely on their schoolwork and improve their
Option b and d are incorrect since they proceed study habits.
to describe the disadvantages and advantages

Click Here For Bundle PDF Course | support@guidely.in Page 11 of 11


Bank Po Mains PDF Course 2024
Reasoning Day -19 (Eng)

Reasoning Aptitude
Directions (1-5): Study the following information 1) Who among the following persons is not
carefully and answer the given questions. senior to Aden?
Nine persons – Adam, Aden, Arlo, Ezra, Evan, a) The one works two position junior to Arlo
Jack, Lucy, Luna, and Nora are working in the b) The one who works at immediately senior
real Estate company at different designations position of Vacant designation
such as Director, Chief Executive Officer(CEO), c) Evan
Chief Operating Officer(COO), Managing d) Nora
Director, Project Director, Deputy Director, e) The one who works three positions senior to
Regional Director, Admin Manager, Sales Lucy
Manager, and Sales Executive. The designations
are given in decreasing order such as Director is 2) Which of the following statements is/are true
the seniormost designation and Sales Executive as per the given arrangement?
is the juniormost designation. One of the I) Adam works as Sales Manager
positions is left Vacant. II) Only one person is designated between Arlo
Aden works as either Deputy director or Admin and the one who works as Chief Operating
Manager. Only two persons are designated Officer.
between Aden and Nora. At least two persons III) Lucy and Aden work as senior and junior
are junior to Nora. Ezra works three positions position of Vacant position respectively
senior to the one who works two positions junior IV) No one works junior to the one who works
to Lucy. At least two persons are senior to Ezra. two positions junior to Evan
Lucy neither works as Regional Director nor a) Only (III) and (IV)
Project Director. Jack works two positions Junior b) Only (I) and (II)
to the one who works immediately senior to c) Only (II) and (III)
Adam. The number of persons designated d) Only (II)
between Jack and Lucy is two less than the e) All (I), (II) and (III)
number of persons designated between Luna
and Ezra. Jack is designated one of the positions 3) How many persons are senior to the one who
junior to vacant post. The number of persons works two positions senior to Adam?
senior to the one who is Regional Director is a) None
same as the number of persons designated b) One
between Arlo and Adam. Evan is not designated c) Two
as Sales Manager. d) Three
e) Four

Click Here For Bundle PDF Course | support@guidely.in Page 1 of 10


Bank Po Mains PDF Course 2024
Reasoning Day -19 (Eng)

4) If Aden gets a promotion to one position, then Note-IV: Atmost one person lives in each flat and
which of the following designation does he work? atmost two persons live on each floor.
a) Director Note-V: Adjacent flats mean flat which is
b) Chief Executive Officer immediately above or immediately below and
c) Chief Operating Officer immediately left or right only considered.
d) Project Director Ira lives three floors above the one who lives
e) Managing Director immediately below the floor of Oma. Oma and Ira
are not living in the same type of flat. Leo lives
5) Four of the following five are alike in a certain one of the floors above Ira but does not live in
way based on the given arrangement and thus the same type of flat. The number of floors above
form a group. Which one of the following doesn’t Leo is one less than the number of floors below
belong to the group? Nat. Nat and Leo are living in the same type of
a) Director-Nora flat. Ema lives southeast flat of the one who lives
b) CEO-Ezra south-west flat of Kim. Kim does not live on the
c) Admin Manager-Luna odd numbered floor. The number of floors below
d) Project Director-Aden Ema is one less than the number of floors above
e) Deputy Director-Evan Joy. Joy and Oma are living in different type of
flats and on different floors. Only one floor gap
Directions (6-10): Study the following information between Kim and Lex, who lives below Kim. No
carefully and answer the given questions. one lives on the adjacent flat of Ira. Lex lives
Eleven persons - Ema, Gia, Ira, Joy, Kim, Leo, either on 4th or 2nd floor in flat A. As many floors
Lex, May, Nat, Nim and Oma live on six different between Nim and Ira as between vacant flat and
floors of a six storeyed building where the Gia. May lives in the same type of flat as vacant
lowermost floor is numbered 1, the floor above it flat but not live in the adjacent flat of vacant flat.
is numbered 2 and so on. One of the flats is Nim and Ema are not living on the same floor.
vacant. 6) Who among the following person lives west of
Note-I: Each floor has two flats viz., Flat A and the one who lives two floors above Lex?
Flat B where Flat A is to the west of Flat B. a) May
Note-II: Flat A of Floor 2 is immediately above b) The one who lives in Flat A on Floor four
Flat A of Floor 1 and immediately below Flat A of c) The one who lives in Flat A on Floor three
Floor 3 and so on. Similarly, Flat B of floor 2 is d) Joy
immediately above Flat B of Floor 1 and e) Gia
immediately below Flat B of Floor 3 and so on.
Note-III: Area of each flat on each floor is same. 7) As many floors above May as below_____

Click Here For Bundle PDF Course | support@guidely.in Page 2 of 10


Bank Po Mains PDF Course 2024
Reasoning Day -19 (Eng)

a) Ira Ten persons - L, M, N, O, P, Q, R, S, T and U


b) Nat belong to the same family. They bought a new
c) The one who lives on Floor 3 of flat B house in different years - 2001, 2003, 2005,
d) The one who lives on the same floor of Lex 2006, 2008, 2011, 2012, 2014, 2019, and 2022.
e) The one who lives on the same floor of Ema Only one person bought a house in each year.
N’s father bought three years before T’s only
8) Which of the following statements is/are true daughter. At least one person bought after T’s
as per the given arrangement? daughter. The number of persons bought before
a) May lives on the odd number floor N’s father is one more than the number of
b) Ira lives in a northeast flat of Nat persons bought after M, who is not N’s father. L
c) Nat and Vacant flat on the same floor is the only son of the one who is the father-in-law
d) Only one floor is between Oma and Leo of O’s spouse. N’s spouse bought three years
e) All the statements are true before the one who bought two years after R’s
son. Only one person bought between N’s
9) In which of the following floor and flat does the spouse and O. R’s only son is the brother-in-law
one lives who is immediate southwest of Leo? of N and vice versa. Neither O’s spouse nor T’s
a) Floor 1, Flat A spouse has a sibling. The brother-in-law of L is
b) Floor 2, Flat A the son-in-law of R. T’s daughter is the niece of
c) Floor 4, Flat A the one who is the daughter of R’s husband. Q is
d) Floor 5, Flat A the nephew of L, who is the only brother of T. L is
e) Floor 3, Flat A unmarried. The number of years difference
between T’s brother and O’s sister’s father-in-law
10) Which among the following pair of the person is one less than the number of years difference
live on the same floor? between P’s spouse and U’s father. U’s father
a) Ira, Leo did not buy in an odd numbered year. T’s spouse
b) Joy, Nim did not have a male kid. M’s brother-in-law is
c) May, Leo unmarried. The number of persons bought before
d) Lex, Oma L’s nephew is one more than the number of
e) Gia, Nat persons bought after S’s son. P did not buy in an
odd numbered year.
Directions (11-15): Study the following 11) Which of the following statement(s) is/are
information carefully and answer the given true as per the given arrangement?
questions. a) Q and U are siblings.
b) R’s daughter is the spouse of Q

Click Here For Bundle PDF Course | support@guidely.in Page 3 of 10


Bank Po Mains PDF Course 2024
Reasoning Day -19 (Eng)

c) All the statements are true 15) How many years difference between the one
d) Only two persons bought between O’s spouse who bought the house three years before T’s
and R father-in-law and the one who bought
e) T is the spouse of U’s father immediately after O’s spouse?
a) Fourteen
12) As many persons bought between N’s b) Twelve
spouse and P’s son as between O’s brother-in- c) Eight
law and _____ d) Nine
a) R’s spouse e) Ten
b) P’s spouse
c) N’s father Directions (16-20): Study the following
d) M’s daughter information carefully and answer the given
e) O’s spouse questions.
D&4%G27#E^38>LM@5A£6O9¥
13) Four of the following five are alike in a certain Step I: All such symbols which are immediately
way based on the given arrangement and thus preceded by a vowel and immediately followed
form a group. Which one of the following doesn’t by a number, are to be placed immediate right of
belong to the group? ‘L’ in the same order from left to right.
a) Q-N’s spouse Step II: All such letters which are immediately
b) L-T’s father-in-law preceded by a symbol and immediately followed
c) O- M’s son by a number are to be placed in alphabetical
d) R-U’s father order to the immediate right of ‘2’.
e) M-P Step III: All such numbers which are immediately
preceded and immediately followed by the same
14) If U and M interchange their positions and element type are to be placed immediate right of
R’s spouse and O’s mother did the same, with ‘^’ in ascending order.
respect to the new positions, who among the Note: Step 1 is followed by step II and step II is
following person bought the house three persons followed by step III
before O’s brother-in-law? Step III is the final step of the given input and
a) O answer the questions below based on the final
b) The one who bought immediately after Q step.
c) The one who is the son of O 16) How many such letters is/are there either
d) The one who is the daughter of N immediately preceded or immediately followed
e) M by a number?

Click Here For Bundle PDF Course | support@guidely.in Page 4 of 10


Bank Po Mains PDF Course 2024
Reasoning Day -19 (Eng)

a) Five c) One
b) Four d) More than three
c) Three e) None
d) Six
e) None of these 19) If all the letters immediately followed by
symbols are dropped, then which of the following
17) Which of the following element is tenth from element is twelfth from the left end?
the right end, if all the digits are dropped? a) M
a) L b) #
b) # c) 4
c) M d) 8
d) E e) None of these
e) None of these.
20) Find the odd one out.
18) How many such symbols are there in the a) E3#
final arrangement which is/are immediately b) 65@
preceded by a number and immediately followed c) %7G
by a consonant? d) M@O
a) Two e) 84^
b) Three
Click Here to Get the Detailed Video Solution for the above given Questions
Or Scan the QR Code to Get the Detailed Video Solutions

Answer Key with Explanation


Directions (1-5): 3) Answer: E
1) Answer: C 4) Answer: D
2) Answer: C

Click Here For Bundle PDF Course | support@guidely.in Page 5 of 10


Bank Po Mains PDF Course 2024
Reasoning Day -19 (Eng)

5) Answer: C(the designation is immediately  Ezra works three positions senior


higher position of the person, except option c) to the one who works two positions
Final Arrangement junior to Lucy.
 At least two persons are senior to
Ezra.
 Lucy neither works as Regional
Director nor Project Director.
 Jack works two positions Junior to
the one who is immediately senior
to Adam.
 The number of persons designated
between Jack and Lucy is two less
than the number of persons
designated between Luna and
We have,
Ezra.
 Aden works as either Deputy
 Jack is designated one of the
director or Admin Manager.
positions junior to vacant post.
 Only two persons are designated
From the above condition, Case-2, and Case-2a
between Aden and Nora.
get eliminated. Case1a has one more possibility.
 At least two persons are junior to
Nora.
From the above condition, there are four
possibilities.

Again we have,
 The number of persons senior to
the one who is Regional Director is
same as the number of persons

Again we have, designated between Arlo and


Adam.

Click Here For Bundle PDF Course | support@guidely.in Page 6 of 10


Bank Po Mains PDF Course 2024
Reasoning Day -19 (Eng)

 Evan is not designated as a Sales  Oma and Ira are not living in the same
manager. type of flat.
From the above condition, case 1 and Case-1a  Leo lives one of the floors above Ira but
get eliminated. Case-1a(1) shows the final does not live in the same type of flat.
arrangement.  The number of floors above Leo is one
less than the number of floors below Nat.
 Nat and Leo are living in the same type of
flat.
From the above condition, there are four
possibilities.

Directions (6-10):
6) Answer: B
Again we have,
7) Answer: E
 Ema lives southeast flat of the one who
8) Answer: C
lives southwest flat of Kim.
9) Answer: C
 Kim does not live on the odd numbered
10) Answer: D
floor.
Final Arrangement
 The number of floors below Ema is one
less than the number of floors above Joy.
 Joy and Oma are living in different type of
flats and on different floors.
 Only one floor gap between Kim and Lex,
who lives below Kim.
 No one lives on the adjacent flat of Ira.
 Lex lives either on 4th or 2nd floor in flat A.
 May lives in the same type of flat as
We have,
vacant flat but not live in the adjacent flat
 Ira lives three floors above the one who
of vacant flat.
lives immediately below the floor of Oma.

Click Here For Bundle PDF Course | support@guidely.in Page 7 of 10


Bank Po Mains PDF Course 2024
Reasoning Day -19 (Eng)

From the above condition, case-1 and case 4 get 14) Answer: D
eliminated. 15) Answer: D
Final Arrangement

Again we have,
 As many floors between Nim and Ira as
between vacant flat and Gia.
 Nim and Ema are not living on the same
floor.
From the above condition, case-3 gets
eliminated. Case-2 shows the final arrangement.

We have,
 N’s father bought three years before T’s
only daughter.
 At least one person bought after T’s
daughter.
 The number of persons bought before N’s
father is one more than the number of
Directions (11-15): persons bought after M, who is not N’s
11) Answer: E father.
12) Answer: B From the above condition, there are three
13) Answer: D (Both persons bought house possibilities.
either in odd or in even numbered year except
option d)

Click Here For Bundle PDF Course | support@guidely.in Page 8 of 10


Bank Po Mains PDF Course 2024
Reasoning Day -19 (Eng)

Again we have,
 L is the only son of the one who is the
father-in-law of O’s spouse.
 N’s spouse bought three years before the
one who bought two years after R’s son. Again we have,
 Only one person bought between N’s  The number of years difference between
spouse and O. T’s brother and O’s sister’s father-in-law is
 R’s only son is the brother-in-law of N and one less than the number of years
vice versa. difference between P’s spouse and U’s
 Neither O’s spouse nor T’s spouse has a father.
sibling.  .U’s father did not buy in an odd
 The brother-in-law of L is the son-in-law of numbered year.
R.  T’s spouse did not have a male kid.
 T’s daughter is the niece of the one who  M’s brother-in-law is unmarried.
is the daughter of R’s husband.  The number of persons bought before L’s
 Q is the nephew of L, who is the only nephew is one more than the number of
brother of T. persons bought after S’s son.
 L is unmarried.  P did not buy in an odd numbered year.
From the above condition, there are four From the above condition, Case-1, Case-2, and
possibility Case-2a get eliminated. Case-3 shows the final
arrangement.

Click Here For Bundle PDF Course | support@guidely.in Page 9 of 10


Bank Po Mains PDF Course 2024
Reasoning Day -19 (Eng)

D &4 % 2 E G 7 # 3 8 > L ^ £M @ 5 A6 O 9 ¥
For step III: All such numbers which are
immediately preceded and immediately followed
by the same element type are to be placed
immediate right of ‘^’ in ascending order.
D & % 2 E G 7 # 3 8 > L ^ 4 6 £M @ 5 A O 9 ¥
16. Answer: B
D & % 2 EG 7 # 3 8 > L ^ 4 6 £M @ 5 AO 9 ¥
Thus, four such letters are possible  E, G, A,
and O

17. Answer: B
D & % E G # > L ^ £M @ A O ¥
Thus, tenth element from the right end  #
Directions (16-20):
We have:
18. Answer: A
D&4%G27#E^38>LM@5A£6O9¥
D & % 2 E G 7 # 3 8 > L ^ 4 6 £M @ 5 A O 9 ¥
For step I: All such symbols which are
Thus, only two such symbols are possible >
immediately preceded by a vowel and
and £
immediately followed by a number, are to be
placed immediate right of ‘L’ in the same order
19. Answer: C
from left to right.
&%2EG7#38>^46£@5AO9¥
D & 4 % G 2 7 # E 3 8 > L ^ £M @ 5 A 6 O 9 ¥
Clearly, the twelfth element from the left end  4
For step II: All such letters which are
immediately preceded by a symbol and
20. Answer: D
immediately followed by a number are to be
placed in alphabetical order to the immediate
right of ‘2’.

Click Here For Bundle PDF Course | support@guidely.in Page 10 of 10


Bank Po Mains PDF Course 2024
Quantitative Aptitude Day -19 (Eng)

Quantitative Aptitude

Direction (1 – 5): Study the given information carefully and answer the given questions.
There are five students – Rajat, Anshul, Kanak, Sachin and Bhanu. All are attending examination X,which
consist of two papers (Paper 1 and Paper 2). Paper 1 for 100 marks and Paper 2 for 200 marks. Both
papers have different weightage in the final score. Paper 1 has 40% weightage and Paper 2 has 60%
weightage in the total weightage mark. The table given below shows the marks and weightage obtained
by each student in two papers and the total weightage Mark.

Note:
I. Total weightage mark= Weightage of obtained marks in Paper 1 + Weightage of obtained marks in
Paper 2
1. Naina’s final score is 20% more than the final b. 18.24%
score of Kanak. Naina obtained 180 marks on c. 10.66%
Paper 2. Find her obtained marks in Paper 1. d. 22.36%
a. 75 e. None of these
b. 80
c. 60 3. For the final score of the student, govt of India
d. 90 (GoI) announces Rs 500 for each percent above
e. None of these 60% to below 71%, Rs. 1000 for each percent
above 70% to below 76%, and Rs 2000 for each
2. If Anshul obtained 20 more marks in Paper 1 percent above 75%. Find the amount Sachin got
and the marks obtained by Anshul in Paper 2 from GoI.
remain the same. Find the percentage increase a. Rs.15000
in the final score of Anshul. b. Rs.12000
a. 12.36% c. Rs.18000

Click Here For Bundle PDF Course | support@guidely.in Page 1 of 14


Bank Po Mains PDF Course 2024
Quantitative Aptitude Day -19 (Eng)

d. Rs.21000 e. None of these


e. None of these
5. The sum of the total weightage marks of
4. If the marks obtained by Kanak in Paper 2are Sachin and Bhanu together is what percent more
increased by 20% and the total weightage mark than the sum of the total weightage marks of
of Kanak is 78, then find the percentage change Anshul and Kanak together.
in the marks obtained by Kanak in Paper 1. a. 15%
a. 30% b. 6%
b. 15% c. 12%
c. 25% d. 9%
d. 20% e. None of these

Direction (6 – 10): Study the given information carefully and answer the given questions.
There are five shopkeepers – A, B, C, D, and E who sells cold drink of three types – Pepsi, Thumbs Up,
and Coca-Cola. The bar graph given below shows the sum of Pepsi bottles and Thumbs Up bottles sold
by every shopkeeper. Also shows the difference between Thumbs Up bottles sold and Coca-Cola bottles
sold by each shopkeeper. The table given below shows the ratio of the Pepsi bottles sold to Coca-Cola
bottles sold by each shopkeeper. The number of Thumbs Up bottles sold by each shopkeeper is always
greater than the number of Coca-Cola bottles sold by each shopkeeper.

Click Here For Bundle PDF Course | support@guidely.in Page 2 of 14


Bank Po Mains PDF Course 2024
Quantitative Aptitude Day -19 (Eng)

6. If the sum of the total number of Coca-Cola container such that the ratio of Pepsi, Thumbs
bottles sold by A and C together is increased by Up, and Soda in the container is 12 : 9 : 13
d% then it will become 5 less than the sum of the respectively. Find the quantity of mixture in
total number of Thumbs Up bottles sold by D and vessel B is what percent of the quantity of
E together. Find the value of d. mixture in vessel A.
a. 12 a. 70%
b. 22 b. 50%
c. 15 c. 60%
d. 18 d. 75%
e. None of these e. None of these

7. There are 3 different quantity of Pepsi bottles 9. Shopkeeper B sold 52% of Pepsi, 58% of
sold by B such as 600 ml, 1 liter, and 2 liter and Thumbs Up, and 65% of Coca-Cola bottles he
its ratio is 5 : 13 : 8 respectively. Find the total has. Find the total number of all cold drink bottles
quantity of Pepsi sold by B in liters? together that B has.
a. 520 liter a. 4300
b. 480 liter b. 3700
c. 560 liter c. 6100
d. 640 liter d. 3500
e. None of these e. None of these

8. Tanu makes a mixture of Pepsi and Soda in 10. If all the bottles of cold drinks sold by E are 1
vessel A in the ratio 3 : 2 and another mixture of liter in quantity. The selling price of a 1-liter bottle
Thumbs Up and Soda in the ratio 9 : 5 in vessel of Pepsi ,Thumbs Up and Coca-Cola is Rs 55,
B. She mixes the mixture of both the vessel in a

Click Here For Bundle PDF Course | support@guidely.in Page 3 of 14


Bank Po Mains PDF Course 2024
Quantitative Aptitude Day -19 (Eng)

Rs 62 and Rs 60 respectively, Find the average c. Rs. 60.12


selling price of 1 bottle of cold drink sold by E. d. Rs. 54.68
a. Rs. 58.22 e. None of these
b. Rs. 64.12

Direction (11 – 15): Study the given information carefully and answer the given questions.
There are five villages- A, B, C, D, and E in which elections are held. The line graph given below shows
the difference between the winning candidate and runner-up candidate of elections in each village.

11. In village C, 80% of the population cast their number of voters in village D, if 15% of casted
votes, and 15% of the casted votes were votes are invalid, If all the people in that village
declared invalid. The winning candidate gets are eligible and all are casted their votes?
48% of the total casted votes. Find the a. 18300
population of village C. b. 17200
a. 32567 c. 15400
b. 33400 d. 16800
c. 36700 e. None of these
d. 39500
e. None of these 13. The winning candidate of village B gets
15900 more votes than the winning candidate of
12.In village D, the winning candidate gets 52% village E. The ratio of the number of votes got by
of total casted votes. Then Find the total the winning candidate of village E to the runner-

Click Here For Bundle PDF Course | support@guidely.in Page 4 of 14


Bank Po Mains PDF Course 2024
Quantitative Aptitude Day -19 (Eng)

up candidate of village B is 2:3. Find the number c. 2331


of votes got by the runner-up candidate of village d. 2731
E. e. None of these
a. 20142
b. 24012 Direction (16 – 18): Study the given information
c. 28412 carefully and answer the given questions.
d. 26114 In a township there are four blocks – A, B, C, and
e. None of these D. The dimension of all room/house in a block is
same but different from other blocks.
14. In village F, there are three candidates – P, The length of the rooms of block A is equal to the
Q, and R in the election. The ratio of votes P, Q, height of rooms of block C which is 25% more
and R get in the election is 6 : 4 : b. Q gets 1892 than the height of the rooms of block D. Ratio of
votes more than R. Find the value of b. the length of the rooms of blocks B, C, and D is 5
(Note: P is the winner of the election in village F : 6 :4. Height of rooms of block A is 50% more
and Q is the runner-up of the election in village than the breadth of rooms of block A. Breadth of
F.) rooms of block B is 3 meters more than the
a. 8 breadth of rooms of block A and 3 meters less
b. 4 than the breadth of rooms of block C. Height of
c. 7 rooms of block B is 9 meter and is same as the
d. 3 breadth of rooms of block D and is 3/4th to the
e. None of these height of the rooms of block A. Length of rooms
of block D is equal to the height of the rooms of
15. In village A, 70% of casted voters are literate block A. Length of the rooms of block B is 50%
and the rest are illiterate. The winning candidate more than the length of the rooms of block A.
gets 60% votes out of the total votes casted, of 16. If the height of the rooms of block C is
which 65% votes are literate voters. Find the increased by d% to fit a solid spherical ball of
number of illiterate voters who cast their votes to volume 4312/3 m3. Find the new volume of 15
runner-up candidate. rooms of block C.
(Note: There are only two candidates in the a. 58462 m3
election in village A and all the eligible voters b. 52920 m3
cast their vote and all votes are valid.) c. 51290 m3
a. 2557 d. 56460 m3
b. 2134 e. None of these

Click Here For Bundle PDF Course | support@guidely.in Page 5 of 14


Bank Po Mains PDF Course 2024
Quantitative Aptitude Day -19 (Eng)

oil of shopkeeper A is Rs. 60 more than the


17. A painter charges Rs. 12/m2 for painting a selling price of the hair oil of shopkeeper B. Find
house. Find the total amount painter gets for the selling price of the hair oil at which
painting the inner walls and ceiling of 42 rooms shopkeeper A sold it.
of the houses of block A. a. Rs. 212
a. Rs.258048 b. Rs. 204
b. Rs.248468 c. Rs. 180
c. Rs.254064 d. Rs. 192
d. Rs.263348 e. None of these
e. None of these
20. Station P is 900 km away from Station Q.
18. There are small wooden cubes of side 0.5 m Train A with a speed of M km/h and Train B with
which are fitted in the 13 rooms of block B such a speed of N km/h start at the same time from
that all 13 rooms are fully packed. Find the total Station P and Station Q respectively move
number of small wooden cubes which are fitted towards each other. After 4 hours from their
in 13 rooms of block B. starting journey when they are 252 km away
a. 136440 from each other, a technical glitch occurs in the
b. 145440 engine of Train A by which its speed becomes
c. 145880 4/5th of its original speed. After 1 more hour, both
d. 154440 trains are 18 km away from each other. Find the
e. 158480 speed of Train B.
a. 60 km/h
19. The cost price of hair oil is Rs. a. b. 80 km/h
Shopkeeper A marked it 50% above the cost c. 72 km/h
price and sell it after giving a discount of 15% on d. 90 km/h
the marked price. Shopkeeper B sold the article e. None of these
at a 12.5% profit. If the marked price of the hair

Click Here For Bundle PDF Course | support@guidely.in Page 6 of 14


Bank Po Mains PDF Course 2024
Quantitative Aptitude Day -19 (Eng)

Click Here to Get the Detailed Video Solution for the above given Questions
Or Scan the QR Code to Get the Detailed Video Solutions

Answer Key with Explanation


Direction (1 – 5): Marks obtained in Paper 1 by Marks obtained in Paper 1 by Kanak = 75
Rajat = 70 Weightage of obtained marks in Paper 1 by
Weightage of obtained marks in Paper 1 by Kanak = 75/100 *40
Rajat = 70/100 * 40 = 30
= 28 Marks obtained in Paper 2 by Kanak = 150
Weightage of obtained marks in Paper 2 by Weightage of marks obtained in Paper 2 by
Rajat = 48 Kanak = 150/200*60 = 45
Marks obtained in Paper 2 by Rajat = 48/60 * Final score of Kanak = 30 + 45
200 = 75
= 160 Weightage of marks obtained in Paper 2 by
Final score of Rajat = 28 + 48 = 76 Sachin = 55.5
Weightage of obtained marks in Paper 1 by Marks obtained in Paper 2 by Sachin = 55.5/60
Anshul = 24 *200 = 185
Marks obtained in Paper 1 by Anshul = 24/40 Weightage of marks obtained in Paper 1 by
*100 Sachin = 77.5 – 55.5
= 60 = 22
Weightage of marks obtained in Paper 2 by Marks obtained in Paper 1 by Sachin =
Anshul = 75 – 24= 51 22/40*100
Marks obtained in Paper 2 by Anshul = 51/60 = 55
*200 Final score of Sachin = 22 + 55.5 = 77.5
= 170 Weightage of marks obtained in Paper 1 by
The final score of Anshul = 75 Bhanu = 32

Click Here For Bundle PDF Course | support@guidely.in Page 7 of 14


Bank Po Mains PDF Course 2024
Quantitative Aptitude Day -19 (Eng)

Marks obtained in Paper 1 by Bhanu = Percentage increase in final score = ((83 –


32/40*100 = 80 75)/75) * 100
Marks obtained in Paper 2 by Bhanu = 165 = (8/75) * 100
Weightage of marks obtained in Paper 2 by = 10.66%
Bhanu = 165/200*60
= 49.5 3. Answer: A
Final score of Bhanu = 32 + 49.5 = 81.5 We know that sachin’s Total weightage Mark is
77.5
So,
Amount get by Sachin for 61% to 70 % = 500 *
10
= 5000
Amount get by Sachin for 71% to 75% = 1000 *
5
1. Answer: D = 5000
Naina’s final score = 75 *120/100 Amount got by Sachin for 76% to 77.5% = 2000
= 90 * 2.5
Marks obtained in Paper 2 by Naina = 180 = 5000
Weightage of marks obtained in Paper 2 by Total amount gets by Sachin = 5000 + 5000 +
Naina = 180/200*60 5000
= 54 = Rs.15000
Weightage of marks obtained in Paper 1 by
Naina = 90 – 54 4. Answer: D
= 36 New marks obtained in Paper 2 by Kanak = 150
Marks obtained in Paper 1 by Naina = 36/40*100 * (120/100)
= 90 = 180
2. Answer: C New weightage of obtained marks in Paper 2 by
New marks obtained in Paper 1 by Anshul = 60 Kanak = 180 * (3/10)
+ 20= 80 54
Weightage of marks obtained in Paper 1 by New weightage of obtained marks in Paper 1 by
Anshul = 80/100*40 = 32 Kanak = 78 – 54
Old final score of Anshul = 75 = 24
New final score of Anshul = 32 + 51= 83 Marks obtained in Paper 1 by Kanak = 24 * (5/2)

Click Here For Bundle PDF Course | support@guidely.in Page 8 of 14


Bank Po Mains PDF Course 2024
Quantitative Aptitude Day -19 (Eng)

= 60 Number of Coca-Cola bottles sold by A = 3 * 144


Percentage change in marks obtained in Paper 1 = 432
by Kanak = ((75 – 60)/75) * 100 Number of Thumbs Up bottles sold by A = 1340
= (15/75) * 100 – 720 = 620
= 20% Let the number of Pepsi bottles sold by B be 2b
and the number of Coca-Cola bottles sold by B
5. Answer: B be 3b.
Total weightage marks Sachin = 77.5 We get, (Pepsi + Thumbs Up) – (Thumbs Up –
Total weightage marks of Bhanu = 81.5 Coca-Cola)
The sum of the total weightage marks of Sachin = Pepsi + Coca Cola
and Bhanu together = 77.5 + 81.5 So, 2b + 3b = 1390 – 90
= 159 5b = 1300
Total weightage marks of Anshul = 75 b = 260
Total weightage marks of Kanak = 75 Number of Pepsi bottles sold by B = 2 * 260 =
The sum of the total weightage marks of Anshul 520
and Kanak together = 75 + 75 Number of Coca-Cola bottles sold by B = 3 * 260
= 150 =780
Reqd. percentage = ((159 – 150)/150) * 100 Number of Thumbs Up bottles sold by B = 1390
= (9/150) * 100 – 520 = 870
= 6% Let the number of Pepsi bottles sold by C be 3c
and the number of Coca-Cola bottles sold by C
Direction (6 – 10): Let the number of Pepsi be 4c.
bottles sold by A be 5a and the number of Coca- We get, (Pepsi + Thumbs Up) – (Thumbs Up –
Cola bottles sold by A be 3a. Coca-Cola)
We get, (Pepsi + Thumbs Up) – (Thumbs Up – = Pepsi + Coca Cola
Coca-Cola) 3c + 4c = 1591 - 72
= Pepsi + Coca Cola 7c = 1519
So, 5a + 3a = 1340 – 188 c = 217
= 8a = 1152 Number of Pepsi bottles sold by C = 3 * 217 =
a = 144 651
Number of Pepsi bottles sold by A = 5 * 144 = Number of Coca-Cola bottles sold by C = 4 * 217
720 = 868

Click Here For Bundle PDF Course | support@guidely.in Page 9 of 14


Bank Po Mains PDF Course 2024
Quantitative Aptitude Day -19 (Eng)

Number of Thumbs Up bottles sold by C = 1591


– 651 = 940
Let the number of Pepsi bottles sold by D be 5d
and the number of Coca-Cola bottles sold by D
be 2d.
We get, (Pepsi + Thumbs Up) – (Thumbs Up – 6. Answer: C
Coca-Cola) The sum of the number of Coca-Cola bottles
= Pepsi + Coca Cola sold by A and C together = 432 + 868
5d + 2d = 1475 – 124 = 1300
7d = 1351 The sum of the number of Thumbs Up bottles
d = 193 sold by D and E together = 510 + 990
Number of Pepsi bottles sold by D = 5 * 193 = = 1500
965 Then, 1300 * ((100 + d)/100) + 5 = 1500
Number of Coca-Cola bottles sold by D = 2 * 193 1300 * ((100 + d)/100) = 1495
= 386 ((100 + d)/100) = 1495/1300
Number of Thumbs Up bottles sold by D = 1475 100 + d = 115
– 965 = 510 d = 15
Let the number of Pepsi bottles sold by E be 2e
and the number of Coca-Cola bottles sold by E 7) Answer: D
be 5e. The number of 600 ml Pepsi bottles sold by B =
We get, (Pepsi + Thumbs Up) – (Thumbs Up – 520 * (5/26
Coca-Cola) = 100
= Pepsi + Coca Cola The number of 1 liter Pepsi bottles sold by B =
2e + 5e = 1332 – 135 520 * (13/26)
7e = 1197 = 260
e = 171 The number of 2-liter Pepsi bottles sold by B =
Number of Pepsi bottles sold by E = 2* 171 = 520 * (8/26)
342 = 160
Number of Coca-Cola bottles sold by E = 5 * 171 Total quantity of Pepsi sold by B = 100 * 0.6 +
= 855 260 * 1 + 160 * 2
Number of Thumbs Up bottles sold by E = 1332 = 60 + 260 + 320
– 342 = 990 = 640 liter

8) Answer: A

Click Here For Bundle PDF Course | support@guidely.in Page 10 of 14


Bank Po Mains PDF Course 2024
Quantitative Aptitude Day -19 (Eng)

Let the quantity of Soda in vessel A be 2x and Total amount E gets after selling Thumbs Up
the quantity of Soda in vessel B be 5y. bottles = 62 * 990
Then, the ratio of Pepsi, Thumbs Up, and Soda = 61380
in the container = 12 : 9 : 13 Total amount E gets after selling Coca-Cola
3x : 9y : 2x + 5y = 12 : 9 : 13 bottles = 60 * 855
From here we can see, 3x/9y = 12/9 = 51300
x/y = 4 Total amount E gets after selling all cold drinks =
Or, x = 4y 131490
Quantity of mixture in vessel A = 3x + 2x = 5x = Total cold drinks sold by E = 342 + 990 + 855
20y = 2187
Quantity of mixture in vessel B = 9y + 5y = 14 y The average selling price of 1 bottle of cold drink
Reqd. percentage = (14y/20y) * 100 sold by E = 131490/2187
= 70% = 60.12

9. Answer: B 11) Answer: D


Total number of Pepsi bottles B has = 520 * Let total votes casted be 100%
(100/52) Total valid casted votes = 100% – 15% = 85%
= 1000 Total casted votes runner up candidate get =
Total number of Thumbs Up bottles B has = 870 85% – 48% = 37%
* (100/58) The difference of votes between the winning
= 1500 candidate and runner-up candidate gets = 3476
Total number of Coca Cola bottles B has = 780 * 48% - 37% = 3476
(100/65) 11% = 3476
= 1200 1% = 316
Total number of all cold drinks bottles B has = 100% = 31600
1000 + 1500 + 1200 Total number of casted votes = 31600
= 3700 Population of village C = 31600 * (100/80)
= 39500
10. Answer: C
Total amount E gets after selling Pepsi bottles = 12) Answer: B
55 * 342 Total valid votes = 100% – 15%
= 18810 = 85%

Click Here For Bundle PDF Course | support@guidely.in Page 11 of 14


Bank Po Mains PDF Course 2024
Quantitative Aptitude Day -19 (Eng)

Valid votes runner-up candidate gets = 85% – The difference between the number of votes
52% winner candidate gets and the number of votes
= 33% runner up candidate gets = 3784
The difference between the number of votes 6x – 4x = 3784
winner candidate gets and the number of votes 2x = 3784
runner up candidate gets = 3268 x = 1892
52% - 33% = 3268 The number of votes P gets = 6 * 1892
19% = 3268 = 11352
1% = 172 The number of vote Q get = 4 * 1892
100% = 17200 = 7568
The total number of voters in village D is 17200. The number of vote R get = 7568 – 1892
= 5676
13) Answer: A The ratio of the number of votes get by P, Q, and
Let the number of votes the winner candidate of R = 11352 : 7568 : 5676
village E gets be x. =6:4:3
Number of votes got by winner candidate of The value of b is 3.
village B = x + 15900
Number of votes got by runner-up candidate of 15) Answer: C
village B = x + 15900 – 4392 Let total votes cast be 100%
= x + 11508 Percentage of votes winner candidate get = 60%
The ratio of the number of votes got by winner Percentage of votes runner-up candidate gets =
candidate of village E to the number of votes got 40%
by runner up candidate of village b = 2 : 3 The difference between the number of votes
x/(x + 11508) = 2 /3 winner candidate gets and the number of votes
3x = 2x + 23016 runner up candidate gets = 5180
x = 23016 60% - 40% = 5180
The number of votes got by runner up candidate 20% = 5180
of village E = 23016 – 2874 1% = 259
= 20142 100% = 25900
Total number of casted votes = 25900
14) Answer: D Number of literate voters = 25900 * (70/100)
Let the number of votes P gets be 6x and the = 18130
number of votes Q gets be 4x respectively. Number of illiterate voters = 25900-18130=7770

Click Here For Bundle PDF Course | support@guidely.in Page 12 of 14


Bank Po Mains PDF Course 2024
Quantitative Aptitude Day -19 (Eng)

Number of votes winner candidate get = 25900 *


(60/100)= 15540
Number of literate voters voting to winner
candidate = 15540*(65/100) =10101
Number of illiterate voters who votes to winner
candidate = 15540 – 10101= 5439
16) Answer: B
Number of illiterate voters voting for runner-up
The volume of the spherical ball = 4312/3 m2
candidate = 7770 – 5439
(4/3) ∏r3 = 4312/3
= 2331
(4/3) (22/7) r3 = 4312/3
r3 = 49 * 7
Direction (16 – 18): Height of rooms of block B =
r = 7 meter
9 meter
The diameter of the sphere is the minimum
The breadth of rooms of block D = 9 meter
dimension of the length of the room.
Height of rooms of block A = 9 * (4/3)
Diameter = 2 * 7
= 12 meter
= 14 m
Breadth of rooms of block A = 12 * (2/3)
The new height of the room = 14 m
= 8 meter
Volume of 15 rooms of block C = 18 * 14 * 14 *
Length of rooms of block D = 12 meter
15
Length of the rooms of block B = 9 * (5/3)
= 52920 m3
= 15 meter
The breadth of rooms of block B = 8 + 3
17) Answer: A
= 11 meter
Surface area of four walls of the room block A =
Length of rooms of block C = 15 * (6/5)
2 (10 * 12) + 2 (8 * 12)
= 18 meter
= 240 + 192
The breadth of rooms of block C = 11 + 3
= 432 m2
= 14 meter
The surface area of the ceiling of the room of
Length of the rooms of block A = 15 * (2/3)
block A = 10 * 8
= 10 meter
= 80 m2
Height of the rooms of block C = 10 meter
Total surface area to be painted of the room of
Height of the rooms of block D = 10 * (4/5)
block A = 512 m2
= 8 meter
Total cost to paint a room of block A = 512 * 12
6144

Click Here For Bundle PDF Course | support@guidely.in Page 13 of 14


Bank Po Mains PDF Course 2024
Quantitative Aptitude Day -19 (Eng)

Total cost to paint 42 rooms of block A = 6144 * 1.5a – 1.125a = 60


42 0.375a = 60
= Rs.258048 a = 160
The selling price of hair oil at which shopkeeper
18) Answer: D A sells = 1.275a
Volume of a room of block B = 15 * 11 * 9 = 1.275 * 160
= 1485 m3 = 204
Volume of a small wooden cube = 0.5 * 0.5 * 0.5
= 0.125 m3 20. Answer: C
Number of small wooden cubes fitted in a room Speed of Train A = M km/h
of block B = 1485/0.125 Speed of Train B = N km/h
= 11880 According to the question,
Number of small wooden cubes fitted in 13 (M + N) * 4 = 900 – 252
rooms of block B = 11880 * 13 (M + N) * 4 = 648
= 154440 Or, (M + N) = 162 …. (i)
And, ((4/5) M + N) * 1 = 144
19) Answer: B Or, 4M+5N = 720 …. (ii)
The cost price of Hair oil = a Subtracting eqn. ii from eqn. i
The marked price of hair oil marked by Multiplying eqn (i) with 4 both side, in order
shopkeeper A = 1.5a eliminate M.
Selling price of hair oil at which shopkeeper A 4M + 4N = 648
sell = 1.5a ((100 – 15)/100) (-)4M+(-)5N = (-)720
= 1.275a ------------------------
The selling price of hair oil at which shopkeeper -N = -72
B sells = a (112.5/100) N = 72 km/h
= 1.125a The speed of Train B is 72 km/h.
According to the question,

Click Here For Bundle PDF Course | support@guidely.in Page 14 of 14


Bank Po Mains PDF Course 2024
English Day - 19

English Language
Directions (1-5): Given below are a few 2. Which of the following option fits best in the
questions based on the paragraph given with a second blank?
few blanks to be filled from the appropriate A. convinced
options in order to make a meaningful sentence. B. perceived
If none of the options are correct then choose C. conceived
option E as your answer. D. treated
The Good Book is being (a)_____________ like a E. None of the above
bad book in Utah after a parent frustrated by
efforts to ban materials from schools 3. Which of the following option fits best in the
(b)___________ a suburban district that some third blank?
Bible verses were too vulgar or violent for A. secondary
younger children. And the Book of Mormon could B. senior secondary
be next. C. elementary
The 72,000-student Davis School District north of D. college
Salt Lake City removed the Bible from its E. None of the above
(c)___________ and middle schools while keeping
it in high schools after a committee reviewed the 4. Which of the following option fits best in the
(d)_____________ in response to a parental fourth blank?
complaint. The district has removed other titles, A. scripture
including Sherman Alexie’s “The Absolutely True B. sculpture
Diary of a Part-Time Indian” and John Green’s C. caricature
“Looking for Alaska,” following a 2022 state law D. despotic
requiring districts to include parents in decisions E. None of the above
over what (e)______________ “sensitive material.”
1. Which of the following option fits best in the 5. Which of the following option fits best in the
first blank? fifth blank?
A. fractioned A. meddled
B. termed B. constitutes
C. pasted C. dismantle
D. treated D. desecrate
E. None of the above E. None of the above

Click Here For Bundle PDF Course | support@guidely.in Page 1 of 7


Bank Po Mains PDF Course 2024
English Day - 19

Directions (6-10): Given below are a few White House nomination (e)/, papers filed with
questions with jumbled parts in order to the US Federal Election Commission (f).
rearrange them and make a contextually correct A. abcdf
sentence. One of the part is fixed and rest to be B. acdfb
rearranged if needed. If no rearrangement is C. bcdfa
required then choose option E as your answer. D. fadcb
6. The ties between India and Nepal (a)/ came E. No rearrangement is required
under severe strain after Kathmandu published
(b)/ a new political map in 2020 (c)/ Indian 9. Years of financial mismanagement (a)/ have
territories Limpiyadhura (d) that showed three pushed Pakistan's economy to the limit (b)/,
(e), Kalapani and Lipulekh as part of Nepal (f). exacerbated by a global energy crisis (c)/ that
A. abced submerged a third of the country (d)/ and
B. acdeb devastating floods (e)/ in 2022 (f).
C. bcdae A. abced
D. bedca B. acdeb
E. No rearrangement is required C. adebc
D. acedb
7. Under international law (a)/, ships are allowed E. No rearrangement is required
to sail through foreign EEZs (b)/, but
unauthorised surveys are not permitted and 10. Pakistan needs billions of dollars (a)/ in
China's operations (c)/ in the South China Sea financing to service (b)/ staggering levels of
have long been problematic (d)/ for countries in external debt (C)/, and foreign exchange
the region, as Beijing claims most of the energy- reserves (d)/ barely enough for a month of
rich sea (e)/, including foreign EEZs (f). imports (e)/ have dwindled to just $4.2 billion (f).
A. bcedf A. acdfe
B. cdefb B. adfec
C. bdcef C. cedfa
D. edcab D. cdeaf
E. No rearrangement is required E. No rearrangement is required

8. showed Monday (a)/ Republican former vice Directions (11-15): Given below are a few
president Mike Pence (b)/ has launched his questions with three highlighted words in each of
hotly-anticipated challenge (c)/ to his one-time them. You have to find the synonyms of the
boss Donald Trump (d)/ for the party's 2024 words highlighted in the same sequence as they

Click Here For Bundle PDF Course | support@guidely.in Page 2 of 7


Bank Po Mains PDF Course 2024
English Day - 19

are given in the question. If none of the options A. accord, compeer, bodacious
are correct then choose option E as your answer. B. compeer, accord, bodacious
11. If we mark spots around the globe where the C. compeer, bodacious, accord
militaries of the world’s superpowers could run D. bodacious, accord, compeer
into each other, both the South China Sea and E. None of the above
the Black Sea would top the chart. Earlier it was
rare that the world's most powerful nations would 14. In response to Russia's invasion of Ukraine,
engage in assertive manoeuvring over Western powers imposed a price cap on Russian
international waters, but now that is a common crude of $60 a barrel.
sight. While non-EU countries can import seaborne
A. involve, conspire, enterprising Russian crude, Western shipowners and insurers
B. enterprising, conspire, involve are prohibited from handling such cargoes
C. conspire, involve, enterprising unless they are sold at or below that price.
D. involve, enterprising, conspire A. count, levy, interdicted
E. None of the above B. interdicted, levy, count
C. levy, count, interdicted
12. While US-China tensions may have been D. interdicted, count, levy
given new impetus under the Trump E. None of the above
administration which levied tariffs broadly and
imposed sanctions on Huawei, the friction has 15. The aggressive push by Reliance to increase
continued unabated under President Joe Biden popular content on its streaming platform, in
as both countries vie for global tech pre- addition to premium sports content, could lead to
eminence. a massive disruption in the OTT streaming
A. goad, incessant, paramountcy space, which has been dominated mostly by
B. goad, paramountcy, incessant foreign players.
C. paramountcy, incessant, goad A. overpower, fierce, disrangement
D. incessant, goad, paramountcy B. overpower, disrangement, fierce
E. None of the above C. fierce, overpower, disrangement
D. fierce, disrangement, overpower
13. Reliance’s JioCinema has signed another E. None of the above
major streaming deal to take on rivals Netflix,
Disney+ Hotstar and Amazon in India after Directions (16-20): Given below are a few
striking a partnership with Warner Bros questions with blank in each of them in the last to
Discovery in April. be filled with an appropriate option in order to

Click Here For Bundle PDF Course | support@guidely.in Page 3 of 7


Bank Po Mains PDF Course 2024
English Day - 19

make the sentence meaningfully correct. If none India, fintech incubator Rainmatter, and
of the options are correct then choose option E Rainmatter Foundation,
as your answer. _________________________.
16. The BRICS brings together five of the largest A. which is still the smallest curated thing.
developing countries of the world, representing B. which is known for its sports adventure.
41 per cent of the global population, C. which supports climate-related non-profits
____________________________. D. which is known for its mouth-watering taste.
A. pertaining to the risk involved they just denied E. None of the above
doing that.
B. the following problem is the main cause of the 19. Musk's net worth dropped below $200 billion
rift. last year as investors dumped Tesla's shares on
C. also the BRICS is the one of the toughest worries the top executive and largest
ruled organisations. shareholder of the world's most valuable electric-
D. 24 per cent of the global GDP, and 16 per vehicle maker is more preoccupied with Twitter,
cent of the global trade. ____________________.
E. None of the above A. which he dumped at some billion dollars.
B. which he bought for $44 billion.
17. Zerodha co-founder Nikhil Kamath became C. which he took as a loan from Jake Dorsey.
the youngest and fourth Indian to join 'The Giving D. which is one of the most important factor in
Pledge', where the wealthiest families and deciding the future of the company.
individuals _________________________________. E. None of the above
A. doesn’t provide information to the needy and
use the cyber data to manipulate election of the 20. While the RBI acknowledged that the
developed nations. domestic economy faces some challenges due
B. doesn’t provide shelter to the needy. to the sluggish global economic outlook, it said
C. is country’s largest broking company and still strong macroeconomic factors, financial
bootstrapped. conditions and expected dividends
D. globally commit to giving the majority of their from______________________.
fortune to charitable causes. A. past reforms put the country in an
E. None of the above advantageous position
B. that the repo and reverse repo rates are rising
18. Kamath founded Zerodha in 2010, as well as from the last five monetary policy meetings.
Gruhas for private investments, hedge fund True C. that the last meeting was not fruitful due to
Beacon that manages wealth for ultra HNIs in less convincing environment.

Click Here For Bundle PDF Course | support@guidely.in Page 4 of 7


Bank Po Mains PDF Course 2024
English Day - 19

D. future prediction of inflation cannot be done as E. None of the above


situation is changing everyday.
Click Here to Get the Detailed Video Solution for the above given Questions
Or Scan the QR Code to Get the Detailed Video Solutions

Answer Key with Explanation


1. Answer: D 4. Answer: A
Here, treated fits best in the first blank as per the Here, scripture fits best in the fourth blanks as it
context of the sentence because here book is means the sacred writings of Christianity
being treated as bad and therefore it fits best. contained in the Bible and here the same is
So, option D is the correct answer. being talked about. So, option A is the correct
answer.
2. Answer: A Sculpture means a statue made out of hands.
Here, convinced fits best in the second blank as Caricature means a picture or description of
it means to be completely sure about something somebody that makes his or her appearance or
and here it fits perfectly as per the context of the behaviour funnier and more extreme than it
sentence. So, option A is the correct answer. really is.
Perceived means to notice or realize something.
Conceived means create by fertilizing an egg. 5. Answer: B
Here, constitutes fits best in the fifth blank as it
3. Answer: C means to contain and here it fits best as the
Here, elementary fits best as it is said in the content of book is being talked with respect to
context of a type of school and elementary the sensitivity or sensitive content. So, option B
schools are the first school for a child. So, option is the correct answer.
C is the correct answer. Desecrate means to damage a place of religious
importance or treat it without respect.

Click Here For Bundle PDF Course | support@guidely.in Page 5 of 7


Bank Po Mains PDF Course 2024
English Day - 19

6. Answer: A 10. Answer: A


Here, the first part after rearrangement will be a Here, the first part after rearrangement will be a
which is introductory part starting with the ties which gives introductory part as need of billions
between India and Nepal followed by b which of dollars for Pakistan followed by b which is
gives the reason for the stress mentioned fixed followed by c which gives reason for need
followed by c and then d and finally ends with e of money followed by d then f and finally ends
and f in which f is already fixed. So, option A is with e which shows how much amount is left with
the correct answer. them. So, option A is the correct answer.

7. Answer: E 11. Answer: D


Here, no rearrangement is required and Here, engage means to provide occupation for
therefore, option E is the correct answer. and here involve is the synonym, assertive
means disposed to or characterized by bold or
8. Answer: C confident statements and behaviour and here
Here, the first part after rearrangement will be b enterprising means same, manoeuvring means
as it gives introductory part as former vice to move to a different position using skill and
president followed by c which gives information here conspire means the same. So, option D is
about a challenge launched by him followed by d the correct answer.
and then e which is fix followed by f which gives
information about details submitted by him and 12. Answer: C
finally ends with a which is ending part. So, Here, impetus means a driving force and here
option C is the correct answer. goad is the synonym, unabated means being at
full strength or force and here incessant means
9. Answer: A the same, eminence means a position of
Here, the first part after rearrangement will be a prominence or superiority and here paramountcy
which is introductory part as it gives information means the same. So, option C is the correct
about some mismanagement from years answer.
followed by b which gives information about
whom it is referring followed by c then e which 13. Answer: A
gives information about the floods and then Here, deal means to take action with regard to
followed by d and finally ends with f which is someone or something and here accord, rivals
fixed. So, option A is the correct answer. means one of two or more striving to reach or
obtain something that only one can possess,

Click Here For Bundle PDF Course | support@guidely.in Page 6 of 7


Bank Po Mains PDF Course 2024
English Day - 19

striking means attracting attention or notice Here, the context of the sentence is about
through unusual or conspicuous qualities and BRICS and its importance and here option D
here bodacious means the same. So, option A is which is giving information about the contribution
the correct answer. of BRICS in global economy is the correct
option. So, option D is the correct answer.
14. Answer: E
Here, imposed means to establish or apply by 17. Answer: D
authority and here levy means the same, import Here, the context of the sentence is about the
means to bring from a foreign or external source giving pledge and option D explains the use of
and here count is somewhat near to the meaning fund donated under this pledge. So, option D is
but not exact synonym as per the context of the the correct answer.
sentence, prohibited means not permitted or
forbidden by the authority and here interdicted 18. Answer: C
means the same. So, option E is the correct Here, the blank should be filled with the
answer. information about rainmatter foundation and
option C gives the information correctly as per
15. Answer: D the context. So, option C is the correct answer.
Here, aggressive means marked by combative
readiness or tending toward or exhibiting and 19. Answer: B
here fierce means the same, disruption means a Here, the context of the sentence is about twitter
break or interruption in the normal course or and in the end as per the context it should end
continuation of some activity and here with the price at which it is bought and option B
derangement means the same, dominated explains it correctly. So, option B is the correct
means to exert the supreme determining or answer.
guiding influence on something and here
overpower means the same. So, option D is the 20. Answer: A
correct answer. Here, the context of the sentence is about
domestic economic situations as per the RBI and
16. Answer: D here option A is contextually fitting in the blank.
So, option A is the correct answer.

Click Here For Bundle PDF Course | support@guidely.in Page 7 of 7


Bank Po Mains PDF Course 2024
Reasoning Day -20 (Eng)

Reasoning Aptitude
Directions (1-5): Answer the questions based on are arranged in such a way to form a third
the information given below. meaningful word.
A nine-letter word (may or may not be 1) Which of the following letter is placed
meaningful) with no repeated letters is arranged immediate left of W in arrangement 2?
in three arrangements to form the series. The a) K
same letters are used in each of the b) The letter which is second to the right of F
arrangements. c) No letter
Arrangement 1: d) E
R is placed fourth to the left of W but none of the e) A
letters is placed at the end. Only two letters are
placed between R and I. The number of letters 2) What will be the first three letters meaningful
placed to the left of I is the same as to the right of word formed?
F.E is placed third to the left of T.T is placed a) RAT
adjacent to neither W nor R. As many letters b) ART
placed between F and T as between W and D.A c) TRA
is placed to the left of K.The number of letters d) TAR
placed between A and R is one more than the e) None of the above
number of letters placed between K and D.
Arrangement 2: 3) What is the position of I from the right end in
All the given letters from arrangement 1 are Arrangement 2?
arranged in alphabetical order from right to left. a) Fourth
Thus, forms the new arrangement of the series. b) Fifth
Arrangement 3: c) Second
The first letter and third letter from the left end in d) Third
arrangement 1 are taken along with the exact e) None of these
middle letter in arrangement 1 to form a first
meaningful word. R is placed to the right of A. 4) Which of the following combination of the
The exact middle letter of arrangement 1 is not letters are arranged adjacent to each other in
the first letter of any meaningful words. The arrangement 3?
fourth letter from the left end in arrangement 1 I. WK
and the third letter from the right end of both II. TF
arrangements 1 and 2 are taken to form a III. AT
second meaningful word. The remaining letters a) Only I

Click Here For Bundle PDF Course | support@guidely.in Page 1 of 10


Bank Po Mains PDF Course 2024
Reasoning Day -20 (Eng)

b) Only III immediately before S. R is not married to Q. At


c) Both I and II least two persons attend the event between S’s
d) All I, II, and III daughter and Q.
e) None of these 6) How V is related to P?
a) Mother-in-law
5) How many letters are there between T and K b) Son
in Arrangement 3? c) Brother
a) Four d) Niece
b) Same as between E and I in Arrangement 1 e) None of these
c) More than Five
d) Same as between R and F in Arrangement 2 7) Who among the following persons attend the
e) Five event between V and S’s son?
I. The one who attends the event on Friday
Directions (6-10) Study the following information II. The one who attends the event three persons
carefully and answer the below questions. after S’s husband
Seven persons from a three generations family – III. S
P, Q, R, S, T, U, and V attend the event during a a) Only II
week from Monday to Sunday. Two married b) Both I and III
couples are in the family. No single parent has a c) Both II and III
child. d) All I, II, and III
T’s daughter-in-law attends three persons after e) None
T. T neither attends on Monday nor Wednesday.
U is the only son of P’s brother. The number of 8) Who among the following person attends the
persons attending before U is one more than the event on Tuesday?
number of persons attending after V. Both U and a) R
V attend the event on neither Tuesday nor b) U’s grandfather
Thursday. S is the sister-in-law of P and vice- c) S
versa. Only two persons attend the events d) Q
between P and Q, who is the father of V. The e) None of these
number of persons attending the event between
U and Q is the same as the number of persons 9) Four of the following five are alike in a certain
attending the event between V and P. R, who is way as per the given arrangement and hence
the mother of P and attends the event three form a group. Find the one that doesn’t belong to
persons after the one who attends the event that group.

Click Here For Bundle PDF Course | support@guidely.in Page 2 of 10


Bank Po Mains PDF Course 2024
Reasoning Day -20 (Eng)

a) The one who attends the event on Monday Q, who sits to the right of P. Neither P nor S is
b) R facing F. D sits second to the right of B but none
c) The one who attends the event two persons of them neither sits adjacent chair of C nor at the
before R end. S sits third to the left of U. Only two persons
d) V sit between P and T. E sits adjacent to the one
e) The one who attends the event on Thursday who is facing T.
11) What is the position of D with respect to the
10) Which of the following information is not one who sits opposite to U?
true? a) Third to the left
a) P is the sister of the one who attends the b) Fourth to the right
event on Monday c) Immediate left
b) More than three persons attend the event d) Second to the right
before U’s sister e) None of these
c) R attends the event immediately after P’s
daughter 12) Four of the following five are alike in a certain
d) S attends the event on Friday way as per the given arrangement and thus form
e) All the above statements are true a group. Find the one that does not belong to
that group.
Directions (11-15) Study the following a) The one who sits immediate right of D
information carefully and answer the below b) C
questions. c) The one who sits third to the right of U
Fourteen chairs are placed in two parallel rows in d) S
such a way that in row 1 – persons A, B, C, D, E, e) The one who sits opposite to R
and F are sitting and all of them face south
whereas, in row2 – persons P, Q, R, S, T, and U 13) How many persons are sitting between F and
are sitting and all of them face north. Row 1 is E?
north of row 2. One chair in each row is vacant. a) Three
Neither the chairs at the end nor the chairs facing b) As many persons sitting to the left of U
each other are vacant. c) As many persons sitting between S and P
F sits third from the extreme right end. Only one d) One
person sits between F and C. The one who is e) Either B or C
facing C sits immediate right of Q. Only three
persons sit between P and R, and either of them 14) Who among the following person sits
sits at the end. Neither P nor R sits adjacent to adjacent chair of C?

Click Here For Bundle PDF Course | support@guidely.in Page 3 of 10


Bank Po Mains PDF Course 2024
Reasoning Day -20 (Eng)

a) Only B neither holds the first rank nor ranks before X. Y


b) Both A and E holds the rank after A but before E. Z holds the
c) Only E rank after B and is four ranks after C. Y is not
d) Both A and F secured the first rank.
e) None of these 16) In which direction is C standing with respect to
A?
15) Which of the following statement is not true? a) North-east
a) One person sits between B and E b) South-east
b) D is facing the one who sits immediate right of c) North-west
S d) South-west
c) T is facing C e) Cannot be determined
d) Two persons are sitting between Q and S
e) All the above statements are true 17) How many persons rank after E in the final
arrangement?
Directions (16-20): Study the following information a) 1
carefully and answer the questions given below. b) 2
Nine students viz., A, B, C, D, E, W, X, Y and Z c) 3
have cleared the class test and are ranked from 1 d) 4
to 9. They are standing in the ground at a certain e) None
distance. No two persons secured the same rank.
Note: The distance between any two students is 18) What is the approximate shortest distance
the twice of the lowest rank among them. For between D and A and who ranks according to that
example, if P is the first rank and Q is the second value?
rank, the distance between them is 4m. a) 5, Y
X is to the west of A and is the immediate previous b) 7, E
rank of D who is to the north of X. The number of c) 5, B
persons ranked between D and W is an odd prime d) 7, B
number where no one ranks after W who is to the e) 5, E
east of D. E ranks exactly between D and W and
is to the south of Y. C stands to the south of W 19) Find the odd one out according to their
and ranks before E. Z stands to the west of C and standing position in the ground?
ranks after Y. The direction of Y from E is same as a) D-Y
the direction of B from Z. B holds the immediate b) X-B
next rank to E and stands to the west of E. C c) A-E

Click Here For Bundle PDF Course | support@guidely.in Page 4 of 10


Bank Po Mains PDF Course 2024
Reasoning Day -20 (Eng)

d) Z-W a) A,B,C,E
e) C-D b) X,A,C,W
c) A,Y,E,B
20) Who among the following group of persons d) Z,W,A,B
hold the prime numbered ranks? e) X,D,Y,B

Click Here to Get the Detailed Video Solution for the above given Questions
Or Scan the QR Code to Get the Detailed Video Solutions

Answer Key with Explanation


Directions (1-5):
1) Answer: C
2) Answer: B
3) Answer: B
4) Answer: C Again we have,

5) Answer: B  The number of letters placed to the left of

Final Arrangement: I is the same as to the right of F.


 E is placed third to the left of T.
 T is placed adjacent to neither W nor R.
From the above condition, Case-1 and case 3
get eliminated.

We have,
 R is placed fourth to the left of W but none
of the letters is placed at the end.
 Only two letters are placed between R
Again we have,
and I.
 As many letters placed between F and T
From the above condition, there are four
as between W and D.
possibilities.

Click Here For Bundle PDF Course | support@guidely.in Page 5 of 10


Bank Po Mains PDF Course 2024
Reasoning Day -20 (Eng)

 A is placed to the left of K.  The fourth letter from the left end in
 The number of letters placed between A arrangement 1 and the third letter from
and R is one more than the number of the right end of both arrangements 1 and
letters placed between K and D. 2 are taken to form a second meaningful
From the above condition, Case-4 get word.
eliminated. Case-2 shows the final arrangement.

Again we have,
 The remaining letters are arranged in
such a way to form a third meaningful
word.

Arrangement 2:
Again we have,
 All the given letters from arrangement 1 Therefore the final arrangements of the given

are arranged in alphabetical order from series are as follows.

right to left. Thus, forms the new


arrangement of the series.

Arrangement 3: Directions (6-10):


Again we have, 6) Answer: D
 The first letter and third letter from the left 7) Answer: D
end in arrangement 1 are taken along 8) Answer: B
with the exact middle letter in 9) Answer: A(All the persons are of the same
arrangement 1 to form a first meaningful gender except option A)
word. 10) Answer: C
 R is placed to the right of A.
 The exact middle letter of arrangement 1
is not the first letter of any meaningful
words.

Again we have,

Click Here For Bundle PDF Course | support@guidely.in Page 6 of 10


Bank Po Mains PDF Course 2024
Reasoning Day -20 (Eng)

We have:
 T’s daughter-in-law attends three persons
after T. Again, we have:
 T neither attends on Monday nor  S is the sister-in-law of P and vice-versa.
Wednesday. That means, S must be the mother of U.
That means, in case (1) T visited on  Only two persons attend the event
Tuesday, in case (2) T visited on between P and Q, who is the father of V.
Thursday.  The number of persons attending the
 U is the only son of P’s brother. event between U and Q is the same as
 The number of persons attending before the number of persons attending the
U is one more than the number of event between V and P.
persons attending after V.  R, who is the mother of P and attends the
 Both U and V attend the event on neither event three persons after the one who
Tuesday nor Thursday. attends the event immediately before S.
That means, in case (1) & case (2) U  R is not married to Q.
attends the event on Wednesday, in case Since, only two married couples are in the
(1a) & case (2a) U attended the event on family, and Q is not married to R, thus Q
Saturday. must be married to S, and T must be
Based on the above given information we have: married to R.
That means, in case (1) Q attends the
event on Monday, in case (1a) Q attends
the event on Thursday, case (2) & case
(2a) are not valid.
Now, since, U is the only son of P’s
brother, and Q is the father of V, thus, V
must be the sister of U.

Click Here For Bundle PDF Course | support@guidely.in Page 7 of 10


Bank Po Mains PDF Course 2024
Reasoning Day -20 (Eng)

 At least two persons attend the event


between S’s daughter and Q.
That means, case (1a) is not valid.
Based on the above given information we have:

We have:
 F sits third from the extreme right end.
 Only one person sits between F and C.

Case (2) & case (2a) are not valid as R attends  The one who is facing C sits immediate

the event two persons after S, case (1a) At least right of Q.

two persons attend the event between V and Q. That means, in case (1) C sits third from

For Blood Relation: the left end of row1, in case (2) C sits
second from the left end of row1.
Based on the above given information we have:

Direction (11-15):
11) Answer: D Again, we have:
12) Answer: B  Only three persons sit between P and R,
13) Answer: B and either of them sits at the end.
14) Answer: C  Neither P nor R sits adjacent to Q, who
15) Answer: E sits to the right of P.
 Neither P nor S is facing F.
That means, in case (1) P sits at the left
end of row2, in case (1a) & case (2) P sits
second from the left end of row 2.
Based on the above given information we have:

Click Here For Bundle PDF Course | support@guidely.in Page 8 of 10


Bank Po Mains PDF Course 2024
Reasoning Day -20 (Eng)

Case (1) & case (1a) are not valid as D sits


second to the right of B.
Again, we have:
 S sits third to the left of U.
Since, S doesn’t sit facing F.
That means, S sits at the left end of row2.
 Only two persons sit between P and T.
 E sits adjacent to the one who is facing T.
That means, E sits immediate left of C.
Based on the above given information we have:

Again, we have:
 D sits second to the right of B but none of
them neither sits adjacent chair of C nor
at the end.
That means, in case (2) D sits second
from the right end of row1, case (1) &
case (1a) are not valid. Directions (16-20):
Based on the above given information we have: 16) Answer: B
17) Answer: C
18) Answer: D
19) Answer: E
20) Answer: E
Following the statements given above, the rank,
distance and the directions of each student are
obtained as follows.

Click Here For Bundle PDF Course | support@guidely.in Page 9 of 10


Bank Po Mains PDF Course 2024
Reasoning Day -20 (Eng)

Click Here For Bundle PDF Course | support@guidely.in Page 10 of 10


Bank Po Mains PDF Course 2024
Quantitative Aptitude Day -20 (Eng)

Quantitative Aptitude

Directions (1-4): Read the following information carefully and answer the questions based on it.
A shopkeeper sold five articles – P, Q, R, S and T. Further partial information in given in table below.

Note:
a) Value of ‘n’ is 4 times of larger root of given equation.
K2 – 32P + 252 = 0
b) Article T is marked up Rs. 1280 more than its cost price, which is 28% more than profit earned on
article S.
c) Discount given on article Q is 16.66%, and selling price of S is Rs. (5m + 7n + 4a + 296).
d) Value of ‘m’ is twice of missing value in the given sequence.
140, 136, 161, (?), 329, - 200
1) If cost price of P is 87.5% more than that of T, A.100/7
and P is sold after discount of (2L – 210) %, then B.40
find which of the following is/are is possible value C.250
of L? D.2/5
I. 1.5m E. None of these
II. (b – n – 8)
III. 2.4(b – a) 3) If Profit % given on article Q is thrice as that of
IV. 240 Discount%, then find difference between cost
A.I and III only price of Q and S.
B.II and IV only A.Rs. 160
C.I, II, and III only B.Rs. 80
D.I, II, III, and IV C.Rs. 120
E. None of these D.Rs. 240
E. None of these
2) If marked price of T is Rs. 80 less than cost
price of R, which is sold after discount of Rs. 400 4) If the discount given on article T is (b – a) %
or (5L/14)%. Find the value of L. and sold at profit of L%, while marked price of S

Click Here For Bundle PDF Course | support@guidely.in Page 1 of 14


Bank Po Mains PDF Course 2024
Quantitative Aptitude Day -20 (Eng)

is Rs. 400 less than that of P, and S sold after Statement I: Speed of train B is 72 km/hr and
giving discount of D%. Find (L% - D %). length of train A is 225m.
A.30% Statement II: Ratio of speed of train A and train B
B.20% is 5:4 and length of train B is 240m.
C.25% Statement III: Train A crosses 125 m long bridge
D.40% in 14 sec and cross a pole in 9 sec.
E. None of these A. Any one
B. Only I
Directions (5-7): Question consists of three C. Only II
statements, I, II and III. You’re required to decide D. Either III or I and II together
that data in which statement(s) is sufficient. E. Only II and III
5) 20% of total scores obtained in the first 4
matchesby a batsmanis 88. Find the average of 7) Find the time taken by B alone to complete the
all the scores obtained by him in five matches (I, work?
II, III, IV and V)? Statement I: B and C together can complete the
Statement I: Scores in match IV obtained by him work in 8 days and A alone takes half days than
are less than those in match III. D alone.
Statement II: Average of scores obtained in Statement II: Ratio of efficiency of A and B is 3:4
matches III, IV and V obtained by himis 120. and ratio of number of days taken by C alone
Statement III: Difference between the scores and D alone to complete the work is 3:4.
obtained in matches III and IV are 40.Ratio of Statement III: C and D can complete the work
scores obtained in matches I, II and V is 3 : 2 : 3, alone in 24 and 32 days respectively.
respectively. A. Any one
A. The data in statement I alone is sufficient. B. Only I and II together
B. The data in statements I, II and III together is C. Only II and III together
sufficient D. Only III and I together
C. The data in statement III alone are sufficient. E. Either I and II together or I and III together
D. The data in statement II alone are sufficient.
E. The data in statements III and II together are Directions (8-11): Read the following information
required. carefully and answer the questions based on it.
There are three companies TCS, Infosys, and
6) Find time taken by train A to cross a 240 long Wipro, each company have three departments –
train? P, Q, and R. The data given below gives

Click Here For Bundle PDF Course | support@guidely.in Page 2 of 14


Bank Po Mains PDF Course 2024
Quantitative Aptitude Day -20 (Eng)

information about the number of employees in 8) Male employees in department P and R


each department of a company. together of Wipro is ______% of male employees
 In TCS, 62.5% of employees in in department Q of Wipro.
department P are males, which is 2000 i. (N + 45) %
less than the number of male employees ii. (2M – 20) %
in Infosys in department R. 25% of the iii. [(M + L)/2 + (N – 5)] %
number of male employees in department iv. 2(L – N) %
R of Infosys and Wipro together is 6000. A.II, III, IV only
 Number of male employees in department B.I and IV only
P of Infosys is (M + 4) % more than that of C.I, III and IV only
TCS, while the number of employees in D.I, II, III, and IV
department P of Infosys is 20% less than E. None of these
the number of male employees in
department Q of Wipro. Male employees 9) Find the difference between 40% of total
in department R of TCS is (M + N) % the employees in department P of all companies
total employees in department R of TCS. together and 50% of male employees in
 Total employees in department P of department Q of Infosys and Wipro together.
Infosys are (3L + M/2) % more than the A.5000
male employees in department Q of the B.4000
same company. (M – N) % of total male C.6000
employees in Wipro is 51000, while the D.3000
female employees in department R of E. None of these
Wipro are 4000.
 (10L/9) % of total employees in 10) For TCS, the number of female employees in
department R of Infosys are males, while department R is [M x (N + L) + 200], then find the
6000 are female employees in department number of male employees in department P and
R of Infosys. Total number of employees R together.
in department P of Wipro is 15000, which A.22000
is [2(L – M) + N/2 + 10] % of total B.20800
employees in department P of Infosys. C.19600
Note: Difference between M and N is 15, while D.22600
the sum of M and L is 100. Sum of M and N is E. None of these
8.33% less than L

Click Here For Bundle PDF Course | support@guidely.in Page 3 of 14


Bank Po Mains PDF Course 2024
Quantitative Aptitude Day -20 (Eng)

11) If total number of female employees in A.40%


Infosys is 17600, then find the number of B.75%
employees in department P of TCS is how much C.80%
percentage more or less the total number D.25%
employees in department Q of Infosys. E. None of these

Directions (12-15): Read the following information carefully and answer the questions based on it.
The chart (1) given below shows the score of Indian cricket team at the fall of each wicket, and chart (2)
shows the runs scored by the 11 batsmen and the order in which they appeared in the following line up
(left to right).
Chart – 1

Note:

Click Here For Bundle PDF Course | support@guidely.in Page 4 of 14


Bank Po Mains PDF Course 2024
Quantitative Aptitude Day -20 (Eng)

a) At any point there are two batsmen on the field, till the fall of 10th wicket. Whenever the team loses a
wicket, the new batsman comes as per the batting order E.g., if one of the openers gets out, Dinesh
Kartik comes to bat.
b) A partnership between any two batsmen is the number of runs scored while both of them are batting.
c) 10th wicket partnership is between Siraj and Bishnoi
12) Which batsman is out, at fall of wicket 6? D.10
A. Shankar E.None of these
B. Shami
C. Pandya Directions (16-17): Read the following
D. Rohit Sharma information carefully and answer the questions
E. Can’t be determined based on it.
A survey conducted in East Delhi among N
13) Score of Rohit Sharma in partnership with people about their liking of songs from
Hardik Pandya, is how much percent more or blockbuster hit movie “DELHI BELLY”. Each
less than score of Hardik Pandya in partnership person likes at least one song among three given
with Ashwin? songs:
A.60% “Nakkadwaley Disco Udhaarwaley Khisko” – S1
B.40% “Jaa Chudail” – S2
C.62.5% “Saigal Blues ft. Delhi Belly” – S3
D.37.5% Number of people who like only S3 is 50% more
E. Can’t be determined than that of who like only S1. Number of people
who like both S2 and S3 but not S1 is 60% less
14) Which batsman isout, at fall of wicket 10? than those who like S1 and S3 but not S2.
A. Siraj People who like S1 are five times of those who
B. Bishnoi like all the three songs. People who like both S2
C. Ashwin and S3 are 72, while people who only like S3 are
D. Bumrah 162.5% more than that of those who like both S2
E. Can’t be determined and S3 but not S1. At least one person likes only
S3.
15) Find maximum score of Ashwin among his 16) If N = 540, then find the maximum possible
partnership with another batsman? value of people who like only S2.
A.7 A.246
B.8 B.235
C.5 C.224

Click Here For Bundle PDF Course | support@guidely.in Page 5 of 14


Bank Po Mains PDF Course 2024
Quantitative Aptitude Day -20 (Eng)

D.257 A.250
E.None of these B.380
C.360
17) If the number of people who like S2 only is D.370
not less than that of those who like S3 only, then E. None of these
find the minimum possible value of N.

Directions (18-19): The bar graph given below shows the total number of students in five schools – A, B,
C, D, and E, and difference between number of boys and girls in a particular school.

Note:
a) Number of boys in school D is half as that of girls and total number of boys in school A is 200.
b) Number of girls is more than boys in school B and D only, while in rest school number of girls are less
than that of boys.
18) Find the value of K, Where K = [(Difference E.N
between total number of boys and girls in school
C and D together) / (total number of girls in 19) Find which of the following statement(s)
school C and D together – 80)] x 100. is/are definitely true.
A.15% I. Total number of students in school E is 180%
B.20% as that of boys.
C.10% II. Difference between number of boys and girls
D.40% in school C is twice as that in school A

Click Here For Bundle PDF Course | support@guidely.in Page 6 of 14


Bank Po Mains PDF Course 2024
Quantitative Aptitude Day -20 (Eng)

III. Number of girls in school B is 56% of total C.II and III only
number of students in that school D.All are true
A.I only E.I and II only
B.I and III only
Click Here to Get the Detailed Video Solution for the above given Questions
Or Scan the QR Code to Get the Detailed Video Solutions

Answer Key with Explanation


Directions (1-4): (13a – 30) – (4a + 40) = 1280
First we need to calculate the value of m and n. 9a = 1350
K2 – 32P + 252 = 0 So, value of a = 1350/9 = 150
(K – 18)(K – 14) = 0 Profit earned on article S = 1280/128 x 100 = Rs.
K = 18, 14 1000
So, value of n = 4 x 18 = 72 Selling price of article S = (5 x 80 + 7 x 72 + 4 x
For calculating value of m, 150 + 296) = Rs.1800
140 – 22 = 136 So, cost price of article S = 1800 – 1000 = Rs.
136 + 52 = 161 800
161 – 112 = 40 So, (5b – m – n – 48) = 800
40 + 172 = 329 5b = 800 + 48 + 72 + 80
329 – 232 = - 200 Value of b = 1000/5 = 200
So, value of m = 2 x 40 = 80 Now, we have the following known values.
Marked price of T is Rs. 1280 more than its cost
price, so
Cost price of T = (4a + 80/2) = (4a + 40)
Marked price of T = (13a – 72/2 + 6) = (13a – 30)
Now,
1) Answer: C

Click Here For Bundle PDF Course | support@guidely.in Page 7 of 14


Bank Po Mains PDF Course 2024
Quantitative Aptitude Day -20 (Eng)

Cost price of P = 187.5% of 640 = Rs. 1200 Profit % on T (L1%) = (960 – 640)/640 x 100 =
Selling price of P = 1200 x 1.4 = 1680 50%
Discount % = (2400 – 1680)/2400 x 100 = 30% MRP of S = 2400 – 400 = Rs. 2000
So, (2L – 210) = 30 Selling price of S = Rs. 1800
Value of L = 240/2 = 120 Discount % on S = (2000 – 1800)/2000 x 100 =
I. 1.5m = 1.5 x 80 = 120 10%
II. (b – n – 8) = 200 – 72 – 8) = 120 Required value = (50% - 10%) = 40%
III. 2.4(b – a) = 2.4 x (200 – 150) = 120 Hence answer is option D
IV. 240
Only I, II, and III follows. 5) Answer: E
Hence answer is option C 1/5 x (scores obtained in matches I, II, III and IV)
= 88
2) Answer: B Total scores obtained in matches I, II, III and IV
Cost price of R = 80 + 1920 = Rs. 2000 = 440
MRP of R = 2000 x 1.2 + 400 = Rs. 2800 Statement I: Scores in match IV < scores in
So, discount % = 400/2800 x 100 = 100/7% match III
5L/14 = 100/7 Statement I alone is not sufficient
Value of L = 40 Statement II: 1/3 x (scores obtained in matches
Hence answer is option B III, IV and V) = 120
Total scores obtained in matches III, IV and V =
3) Answer: A 360
Discount % given on article Q = 50/3% Total scores obtained in matches III and IV =
So profit % on article Q = 3 x 50/3 = 50% 360 - scores in match V
Selling price of article Q = 5/6 x 1728 = 1440 Statement II alone is not sufficient
Cost price of article Q = 1440/1.5 = Rs. 960 Statement III: Scores obtained in match III -
Cost price of article S = Rs. 800 scores obtained in match IV = 40
Required difference = 960 – 800 = Rs. 160 (or) scores obtained in match IV - scores
Hence answer is option A obtained in match III= 40
Scores obtained in matches I, II and V =
4) Answer: D 3x:2x:3x
Discount % on T = (200 – 150) % = 50% Statement III alone is not sufficient
So, selling price of T = 1920 x 50% = Rs. 960 From II and III,

Click Here For Bundle PDF Course | support@guidely.in Page 8 of 14


Bank Po Mains PDF Course 2024
Quantitative Aptitude Day -20 (Eng)

Total scores obtained in matches I, II, III and IV Each statement alone is not sufficient to give an
= 440 answer.
Total scores obtained in matches III, IV and V = From I and II,
360 Let number days taken by C and D is 3x and 4x
Scores obtained in I + scores obtained in II - Number of days taken by A = 4x/2=2x
scores obtained in V = 80 Number of days taken by B = 2x*3/4=1.5x
So 3x + 2x - 3x = 80 => x = 40 So, 1/1.5x + 1/3x=1/8
Scores obtained in I = 3*40 = 120 From this we can calculate the answer.
Scores obtained in II = 2*40 = 80 From statements I and III,
Scores obtained in V = 3*40 = 120 1/B + 1/24 = 1/8
Hence total scores obtained in III and IV = 440 - From this, we can calculate the answer
120 - 80 = 240
Total scores obtained in all five matches = 440 + Directions (8-11):
120 = 560 First, we need to calculate the value of M, N, and
Required average = 560/5 = 112 L
6) Answer: D M – N = 15…………. (1)
From statement I, M + L = 100…………. (2)
Length of the train is 225 m. (M + N) = 11/12 x L…………... (3)
Cannot calculate the answer because speed of On adding (1) and (3)
train A is unknown. 2M = 11L/12 + 15
From statement II, Put this value in equation (2)
Cannot calculate the answer because length and 11L/12 + 15 + 2L = 200
speed of train A is unknown. 35L/12 = 185
Statement III: Value of L = 60
We can calculate required answer. Value of M = 40
(L+125)/14=Speed of train A and L/9=Speed of Value of N = 25
train A Total employees in P of Wipro = 15000
From this two equations, we can calculate the [2(L – M) + N/2 + 10] % of total employees in
required answer department P of Infosys = 15000
And, we can calculate the answer from the data 62.5% of total employees in department P of
of I and II together. Infosys = 15000
So, total employees in P of Infosys = 15000 x 8/5
7) Answer: E = 24000

Click Here For Bundle PDF Course | support@guidely.in Page 9 of 14


Bank Po Mains PDF Course 2024
Quantitative Aptitude Day -20 (Eng)

(10L/9) % of total employees in department R of 25% of number of male employees in


Infosys are males. department R of Infosys and Wipro together is
So, employees in R of Infosys = (10/9 x 60) = 6000.
66.66% of employees in R of Infosys So, males in R of (Infosys + TCS) = 4 x 6000 =
So, females in R of Infosys = 33.33% = 1/3 of 24000
total employees in R of Infosys So, males in R of Wipro = 24000 – 12000 =
So, total employees in R of Infosys = 3 x 6000 = 12000
18000 Total employees in R of Wipro = 12000 + 4000 =
Male employees in R of Infosys = 18000 – 6000 16000
= 12000 Number of male employees in department P of
Total employees in department P of Infosys are Infosys is (M + 4) % more than that of TCS
(3L + M/2) % more than male employees in Males in P of Infosys = (40 +4) % = 44% more
department Q of same company than males in P of TCS
So, total employees in P of Infosys = (3 x 60 + So, males in P of Infosys = 144% x 10000 =
40)/2 = 200% more than male employees in Q of 14400
Infosys So, females in P of Infosys = 24000 – 14400 =
So, male employees in Q of Infosys = 24000/3 = 9600
8000 Number of employees in department P of Infosys
(M – N) % of total male employees in Wipro is is 20% less than number of male employees in
7650, department Q of Wipro.
So, (40 – 25) % = 15% of total male employees So, males in Q of Wipro = 24000/80 x 1000 =
in Wipro = 7650 30000
Total male employees in Wipro = 7650/15 x 100 So, males in P of Wipro = 51000 – 12000 –
= 51000 30000 = 9000
Females in R of Wipro = 4000 So, females in P of Wipro = 15000 – 9000 =
Males in P of TCS = 2000 less than males in R 6000
of Infosys 8) Answer: C
So, males in P of TCS = 12000 – 2000 = 10000 Males in P of Wipro = 9000
62.5% of employees in P of TCS = 10000 Males in Q of Wipro = 30000
So, total employees in P of TCS = 8/5 x 10000 = Males in R of Wipro = 12000
16000 Required % = (9000 + 12000)/30000 x 100 =
Females in P of TCS = 16000 – 10000 = 6000 70%
I. (N + 45) % = (25 + 45) = 70%

Click Here For Bundle PDF Course | support@guidely.in Page 10 of 14


Bank Po Mains PDF Course 2024
Quantitative Aptitude Day -20 (Eng)

II. (2M – 20) % = (2 x 40 – 20) % = 60% So, female employees in department Q of


III. [(M + L)/2 + (N – 5)] % = (40 + 60)/2 + (25 – Infosys = 17600 – 9600 – 6000 = 2000
5) = 70% Total employees in department Q of Infosys =
IV. 2(L – N) % = 2 x (60 – 25) = 70% 8000 + 2000 = 10000
Only I, III, and IV follows Required % change = [(16000 – 10000)/10000] x
Hence answer is option C 100 = 60%
Hence answer is option E
9) Answer: D
40% of total employees in department P of all Directions (12-15):
companies together = 40% x (16000 + 24000 + Rohit Sharma scored 28 runs, and Virat Kohli
15000) = 22000 scored 7 runs.
50% of male employees in department Q of So, Fall of wicket 1 = Virat Kohli (7)
Infosys and Wipro together = 50% x (8000 + Partnership of Rohit Sharma (18) + Virat Kohli
30000) = 19000 (7) = 25
Required difference = 22000 – 19000 = 3000 Second wicket falls at 34, Still Rohit Sharma
Hence answer is option D need to score = 28 – 18 = 10 runs
Dines Kartik scored = 8 runs
10) Answer: B Difference between FOW – 2 and FOW – 1 = 34
For TCS, – 25 = 9
number of female employees in R = 40 x (60 + So, FOW – 2 = Dinesh Kartik (8)
25) + 200 = 3600 Partnership of Rohit Sharma (1) + Dinesh Kartik
number of male employees in R = (40 + 25) % = (8) = 9 runs
75% of total employees in R Difference between FOW – 3 and FOW – 2 = 42
So, 25% of total employees in R = 3600 – 34 = 8 runs
So, number of male employees in R = 75/25 x Score of Ravindra Jadeja = 4
3600 = 10800 Rohit Sharma (4) + Ravindra Jadeja (4) = 8 runs
So, male employees in department P and R FOW – 3 = Ravindra Jadeja (4)
together = 10000 + 10800 = 20800 Difference Between FOW – 4 and FOW – 3 = 57
Hence answer is option B – 42 = 15 runs
Runs scored by Hardik Pandya = 20
11) Answer: E Rohit Sharma (5) + Hardik Pandya (10) = 15
Total female employees in Infosys = 17600 runs
FOW – 4 = Rohit Sharma

Click Here For Bundle PDF Course | support@guidely.in Page 11 of 14


Bank Po Mains PDF Course 2024
Quantitative Aptitude Day -20 (Eng)

Difference Between FOW – 5 and FOW – 4 = 62


– 57 = 5 runs
Score of Vijay Shankar = 3
Hardik Pandya (2) + Vijay Shankar (3) = 5 runs
FOW – 5 = Vijay Shankar (3)
Difference Between FOW – 6 and FOW – 5 = 75
– 62 = 13 runs
Runs scored by R. Ashwin = 18
Hardik Pandya (8) + R. Ashwin (5) = 13 runs
FOW – 6 = Hardik Pandya
Difference between FOW – 7 and FOW – 6 = 82 12) Answer: C
– 75 = 7 runs According to question,
Shami scored = 5 Batsman out at fall of wicket 6 = Hardik Pandya
Ashwin (2) + Shami (5) = 7 runs Hence answer is option C
FOW – 7 = Shami
Difference between FOW – 8 and FOW – 7 = 86 13) Answer: D
– 82 = 4 runs Score of Rohit Sharma in partnership with Hardik
Bumrah scored = 0 Pandya = 5
R – Ashwin (4) + Bumrah (0) = 4 runs Score of Hardik Pandya in partnership with
FOW – 8 = Bumrah Ashwin = 8
Difference between FOW – 9 and FOW – 8 = 99 Required percentage change = (8 – 5)/8 x 100 =
– 86 = 13 runs 37.5%
Siraj Scored = 11 Hence answer is option D
10th wicket partnership is between Siraj and
Bishnoi 14) Answer: E
R – Ashwin (7) + Siraj (6) = 13
FOW – 9 = R – Ashwin 15) Answer: A
Difference between FOW – 10 and FOW – 9 = Maximum score of Ashwin among his
110 – 99 = 11 partnership with another batsman (Siraj) = 7
Bishnoi scored = 6 runs Hence answer is option A
Siraj (5) + Bishnoi (6) = 11 runs
We can’t decide FOW – 10 Directions (16-17):

Click Here For Bundle PDF Course | support@guidely.in Page 12 of 14


Bank Po Mains PDF Course 2024
Quantitative Aptitude Day -20 (Eng)

People who only like S3 are 162.5% more than If b = 40, then a = 4 (possible)
that of those who like both S2 and S3 but not S1. Further values are not possible for a and b
So, respective ratio of number of people those 16) Answer: C
likes only S3 to people those likes both S2 and According to question,
S3 but not S1 = 262.5%:100% = 21:8 [21a, 8a] Total number of people surveyed = 540
So, people like only S3 = 21a 5b + 21a + 8a + c = 540
People likes only S1 = 21a x 2/3 = 14a (5b + 29a) + c = 540
People likes both S1 and S3 but not S2 = 8a x We need to find the maximum possible value of
5/2 = 20a c, so the value of (5b + 29a) is the minimum
Let number of people likes all three songs = b possible.
So, number of people likes S1 = 5 x b = 5b Minimum possible value of (5b + 29a) = (5 x 40 +
Number of people likes both S1 and S2 but not 29 x 4) = 316
S3 = 5b – 14a – 20a – b = 4b – 34a So, maximum possible value of c = 540 – 316 =
Let number of people those likes only S2 = c 224
Hence answer is option C

17) Answer: D
Number of people who like S2 only is not less
than that of those who like S3 only.
We need to find the minimum number of people
surveyed, so the minimum possible value of
people who like only S2 is the same as that of
those who like S3 only.
So, N = (5b + 21a + 8a + 21a) = 5b + 50a
Put b = 64, and a = 1
Now,
So, minimum possible value of N = 5 x 64 + 50 x
8a + b = 72
1 = 370
Value of a = 9 – b/8………………... (1)
Hence answer is option D
5b > (14a + 20a + b)
4b > 34a
Directions (18-19):
So, value of b > 8.5a
Let number of boys in school D = M
If b = 64, then a = 1 (possible)
So, number of girls in School D = 13b – 4 + M
If b = 56, then a = 2 (possible)
Now,
If b = 48, then a = 3 (possible)

Click Here For Bundle PDF Course | support@guidely.in Page 13 of 14


Bank Po Mains PDF Course 2024
Quantitative Aptitude Day -20 (Eng)

2 x M = 13b – 4 + M 18) Answer: C


13b – M = 4………………….. (1) According to Question,
Total number of students in school D = 3.8a – 42 Value of K = [(280 – 260)/ (200 + 80 – 80)] x 100
So, M + 2M = 3.8a – 42 Value of K = 10%
3.8a – 3M = 42………………. (2) Hence answer is option C
From Equation 1 and 2,
3.8a – 39b = 30………………. (3) 19) Answer: D
Also, total number of boys school A = 200 Statement I.
Number of girls in school A = 200 – 5b Total number of students in E = 450
Total number of students in School A = 4a Number of boys in school E = 250
So, 200 + 200 – 5b = 4a Required percentage = 450/250 x 100 = 180%
4a + 5b = 400………………….. (4) This statement is true.
On solving equation (3) and Equation (4) Statement II.
175a = 15750 Difference between number of boys and girls in
So, value of a = 15750/175 = 90 school C = 80
Value of b = (400 – 360)/5 = 8 Difference between number of boys and girls in
Value of M = 13 x 8 – 4 = 100 school D = 40
Now we can calculate the data for all schools This statement is true
Statement III.
Required percentage = 280/500 x 100 = 56%
Hence answer is option D

Click Here For Bundle PDF Course | support@guidely.in Page 14 of 14


Bank Po Mains PDF Course 2024
English Day - 20

English Language
Directions (1-5): In the following questions, D. F
jumbled sentences are given. You have to E. B
rearrange the jumbled sentences to make a
coherent paragraph and one sentence is fixed 2. Which is the adjoining pair before the last
and highlighted in bold which is already arranged sentence after rearrangement?
in the question. On the basis of the A. C-A
rearrangement answer the following questions B. D-C
below. C. E-F
A. The 2018 Commonwealth Games champion, D. A-B
23, registered a best throw of 87.58m on E. B-D
Saturday.
B. He is the only second Indian(A)/ in the 10m 3. Sentence F is fixed in rearrangement as the
air rifle event at Beijing 2008 (B)/ to win an last sentence in the rearrangement so what will
individual gold (C)/ after Abhinav Bindra (D). be the penultimate sentence after
C. India has ____________broken out into rearrangement?
celebrations after Neeraj Chopra became the first A. C
Indian to win a historic _____________ gold medal B. E
in athletics(javelin throw) at the Tokyo 2020 C. A
Olympics. D. B
D. Prime Minister Narendra Modi congratulated E. D
Chopra on Twitter saying that history has been
created. 4. India has out into celebrations after
E. "It feels unbelievable", said Chopra. "It's a Neeraj Chopra became the first Indian to win a
proud moment for me and my country." historic gold medal in athletics (javelin
F. Chopra's gold was India's seventh medal - throw) at the Tokyo 2020 Olympics.
one gold, two silver and four bronze - in Tokyo, Fill in the blanks with suitable words from below.
their best ever Olympic haul having passed the A. Called, third
six they won at London 2012. B. proven, first
1. Which is the first sentence after C. created, Paralympic
rearrangement? D. broken, Olympic
A. C E. None of the above
B. D
C. E

Click Here For Bundle PDF Course | support@guidely.in Page 1 of 9


Bank Po Mains PDF Course 2024
English Day - 20

5. What will be the rearrangement of sentence B B. A-C; that


and if no interchange is needed mark 'no C. A-D; impending
interchange required' as your answer. D. B-C; identity
He is the only second Indian(A)/ in the 10m air E. No swapping needed; no replacement needed
rifle event at Beijing 2008 (B)/ to win an individual
gold (C)/ after Abhinav Bindra (D). 8. Union Ayush Minister Sarbananda Sonowal
A. ADCB (A) inaugurated the Yoga (B) Mahotsav 2022 to
B. DABC mark 100 days (C) countdown to the 8th (D)
C. BCAD International Day of Yoga.
D. ABDC A. A-B; set in motion
E. No rearrangement required B. C-D; target
C. B-C; celebration
Directions (6-10): The following questions given D. B-D; global
below have four words that have been E. No swapping and replacement needed
highlighted as they might’ve been placed at the
wrong positions. One of these words might also 9. The (A) epicentre of Uttarakhand to the (B)
be incorrect and need a replacement. Read the proximity of the farmers’ (C) movement and its
same carefully and mark the appropriate option (D) impact on some parts of the State make for
as the answer. an interesting case study.
6. The (A) underpass said the work is (B) A. A-D; efficacy
expected to begin in the next five to six months B. B-C; zone
and the (C) authority will be ready in two years C. A-B; no replacement needed
(D) before that. D. C-D; distance
A. B-D; assumed E. No swapping needed; effect
B. A-C; after
C. C-D; order 10. While the figure was even higher among the
D. A-B; over bridge poor and lower class (A) majority, at well over
E. No swapping needed; across 80%, what is (B) required and rather surprising is
that our survey also found that the facility of free
7. Actress Vidya Balan said that someone asked ration even reached a (C) households of the
her if her (A) upcoming movie 'Jalsa' was a (B) well-off households which may not have really
name on actor Amitabh Bachchan's house (C) (D) appalling it.
who goes by the same (D) biopic. A. A-C, B-D; interesting
A. B-D; which B. A-D, B-C; families

Click Here For Bundle PDF Course | support@guidely.in Page 2 of 9


Bank Po Mains PDF Course 2024
English Day - 20

C. A-B, C-D; essential capable of addressing differences ____________


D. Only C-D; No replacement needed without any foreign interference.
E. No swapping needed A. Paramount, mutual
B. Unequivocal, reciprocating
Directions (11-15): In each question given below C. Stupendous, mitigating
is a sentence with two blanks. Five options below D. Utmost, bilaterally
the same having words that can be filled in the E. Paramount, unilaterally
blanks have been given. Select the option that
can complete the sentences in the best possible 14. Bewilderingly, instead of __________
way as per the context: navigating past PM-CM dilemma, as is expected
11. __________in the story of Bombay or of a _________ civil servant like him, he ended up
Mumbai’s growth is the story of India’s post- choosing CM over PM that reflects serious
Independence __________. administrative incompetence.
A. Encapsulated, urbanisation A. Astutely, seasoned
B. Sheath, struggle B. Sagaciously, precocious
C. Engraved, set-up C. Inexplicably, revered
D. Surrounded, liberalisation D. Dedicatedly, senior
E. All of the above E. None of the pair fits

12. Car makers have lined up new launches to 15. The BCCI, during the ICC board meeting,
lift demand as the second wave __________ and has formally asked for a four-week window to
states lift curbs that had forced several take a final call but, internally, they have said that
companies to put plans on hold. There is they would like to keep the hosting rights and
__________ demand in the market with wouldn’t mind the tournament being held in UAE
consumers looking forward to new products. and Oman, a senior board official ___________ to
A. Remitting, staggering ICC board developments said. He said that
B. Hurts, help-up Muscat has been ________ in specifically for the
C. Abates, pent-up preliminary rounds.
D. Soaring, negligible A. Related, narrowed
E. Abated, volatile B. Privy, zeroed
C. Cognizant, selected
13. Both are very responsible leaders who treat D. Apprised, phenomenal
each other with ________ respect and who are E. Both (b) and (c)

Click Here For Bundle PDF Course | support@guidely.in Page 3 of 9


Bank Po Mains PDF Course 2024
English Day - 20

Directions (16-20): The given questions carry a community connectivity. Community radios have
paragraph each from which the last sentence proved their worth as agents of social behavior
has been deleted. There are options below each change during the spread of COVID-19. They
one of which can complete the given paragraphs provided a strong platform from which common
meaningfully. You must read the same carefully people can freely communicate their ideas. They
and choose the correct option as your answer. connect all the developmental factors, policies
16. Kejriwal has invited applications for such as poverty alleviation, employment
admission to Class IX from anyone who has generation, and agricultural, environmental, and
passed Class VIII examination from any allied activities. Every community radio has its
recognised school. There will be no fees and challenges depending upon ___________.
students from anywhere in India can enroll as A. its geographical area, rural-urban differences,
students subject to the condition that they will natural calamities, financial issues, tribes,
have to be physically present in a school in Delhi languages, and dialect.
to appear for the examinations. If the aim is to B. capacity and location which decides what
attract poor students, especially girl students should be played according to the tribes in the
whom the parents do not want to send to regular area
schools, _______________. C. the regulations and the permissions that have
A. it will be accomplished only if the parents of been formulated as every locality is different
these students can afford the tickets to Delhi D. the timings as per the weather as; some
B. are always under the gaze of the teachers places being too hot prefer evenings whereas the
who know their levels of learning and can advise others afternoons
them accordingly. E. None of the above
C. it is defeated by the condition that they will
have to come to Delhi for a few days every year. 18. So far, Beijing has not committed to joining
D. he chooses the wrong time to do this multilateral negotiations on the restructuring,
experiment. which Japan has offered to organize and initiate
E. Academics and educationists have revealed on behalf of Sri Lanka. As the world’s biggest
that virtual classes are not a substitute bilateral lender, China’s record on debt
restructuring suggests it prefers to do it alone.
17. Community radios are considered to be the For Sri Lanka’s sake, _______________.
cheapest and most effective grassroots medium A. China must understand that it is good to
of communication. They bridge the compromise sometimes for the sake of it.
communication gap between governments and
local authorities and act as vehicles of last-mile

Click Here For Bundle PDF Course | support@guidely.in Page 4 of 9


Bank Po Mains PDF Course 2024
English Day - 20

B. the creditors will have to come together so D. who made sure they win the match and
that they can pressurize Sri Lanka to recover stayed away from it
their money E. None of the above
C. it is to be hoped that all creditors will approach
these negotiations in a spirit of co-operation. 20. Following the death of Soviet leader Mikhail
D. this is why the Sri Lankan president has been Gorbachev earlier this week, tributes flew in
trying to form an all-party government towards the man who oversaw the collapse of
E. Any of the given options the USSR and facilitated the end of the Cold
War. However, Gorbachev was a polarizing
19. Intentionally or unintentionally, the bottom figure in his homeland as many Russians blamed
line is that Kohli had been faking intensity on the him for the collapse of the empire. Looking back
field recently. He has also been brash and at his foreign policy reveals a complex man
immature. This was a rare occasion when on the whose policies, intended or__________.
official broadcaster channel, king Kohli’s A. hinted at a global state of democracy which
imperfection was being underlined. Ironically, it his own country rejected brutally
was king Kohli himself ______________________. B. otherwise made him one of the most influential
A. who was being brutally honest in listing his leaders of our time
own weaknesses C. help him become the global leader Russia
B. which was telling us that the way he behaved needed but did not want
on the field is bad D. joined the cause for a peaceful reunification of
C. saying that a lot of improvement in public Germany
relations is needed E. wins him veneration internationally, but in
Russia, his legacy remains highly contested.
Click Here to Get the Detailed Video Solution for the above given Questions
Or Scan the QR Code to Get the Detailed Video Solutions

Answer Key with Explanation

Click Here For Bundle PDF Course | support@guidely.in Page 5 of 9


Bank Po Mains PDF Course 2024
English Day - 20

1. Answer: A 5. Answer: A
Option A is the correct answer. Sentence C Option A is the correct answer. Because, only if
states the first important topic of the following ADCB are placed chronologically it would carry a
sentences. The other sentences add details to valid meaning, and it will be grammatically
this Statement. It also acts like a headline. correct. We always need to understand the
Therefore it is the first in order. sequence or chronology in a sentence.
The correct sequence will be CAEBDF. Therefore option A is the correct answer.
2. Answer: E
Option E is the correct answer. Sentence B and 6. Answer: B
Sentence D are chronologically set after If the work is said to begin in the coming six
rearrangement. B gives information regarding months, how can it be completed two years
the medal and D follows with the information back? So, we can see that the word ‘before’ has
about the prime minister's appreciation which wrongly been used and should be replaced by
must take place after the description. ‘after’.
Also, what will be ready? The ‘underpass’ which
3. Answer: E is placed wrongly in A. So, the best is to
Option E is the correct answer. This Statement interchange A with C.
says that the prime minister has congratulated Hence, option (b) is the correct answer.
the player, which obviously should be placed
after covering other important details. 7. Answer: A
On one simple reading we can find that the word
4. Answer: D ‘who’ doesn’t fit anywhere in the sentence so this
In the first blank space, none other than the verb definitely needs to change. For house, the
'broken' fits, as do other verbs like 'created' or pronoun ‘which’ would be better.
'proven' or 'called'. These verbs don't make any The word ‘name’ still looks good in B but the
meaningful phrases, except 'broken' and word ‘biopic’ doesn’t go well in D and if these
'Olympic'. Likewise in the second blank space, two words are interchanged, the sentence
the word Olympic is required to fill, other words becomes absolutely correct.
do not match the context of Gold winning. Thus, marking (a) as the answer would be crisp!
Therefore in the first blank, we will fill in with
'broken' and the second blank space we will fill in 8. Answer: E
with 'Olympic'. So option D is the correct answer.

Click Here For Bundle PDF Course | support@guidely.in Page 6 of 9


Bank Po Mains PDF Course 2024
English Day - 20

There is no need of words to be interchanged or they wanted to live. Growth relates to the word
swapped as the given sentence is pretty easy urbanisation.
and understood. The word sheath stands for a cover for a knife or
Therefore, the correct answer is (e). other sharp weapon. This is unrelated as per the
context.
9. Answer: C Thus, the best answer is option (a).
The closeness of the state from the area where
the movement was taking place is being 12. Answer: C
mentioned. In this sentence, the states becoming a bit more
So, the words ‘epicentre’ and ‘proximity’ need to lenient are being discussed as the effects of the
be changed and the words given make complete second wave have started fading. With the limits
sense so there is no need of any replacement. being eased the car companies are now looking
Therefore, the correct answer should be option at launching new products which had long been
(c). in the pipeline. This hints at a demand that
10. Answer: A existed but was not visible in the market due to
The findings reflect that the ration availability the lack of new products. The word abates which
was done to majority of the people in need but means eased in the first form will be the best fit
some places it was seen that it had reached along with pent-up demand which means a
where it wasn’t even needed also. This is an suppressed demand. The first option goes out of
element of surprize for the observers and also context because of remitting, in option (b) help-
interesting as an observation. up doesn’t fit the second blank and is ruled out,
So, the word ‘appalling’ which is quite extreme in soaring in option (d) conveys an opposite
meaning and is used for horrifying and shocking meaning to the sentence as it means rising and
situations is a misfit in the sentence. option (e) is ruled out for the wrong usage of
Therefore, the correct answer should be option abate as a gerund
(a).
13. Answer: D
11. Answer: A For the first blank; ‘Utmost’ and ‘Paramount’ are
The word encapsulated in the first option means very good fits meaning (a great amount of). This
to sum up or concisely put something. The rules out ‘Unequivocal: without a doubt’ and
sentence in simple words means that Mumbai’s ‘Stupendous: amazing or fabulous.’ Option (a) is
growth can be summed up in the story of India’s ruled out as we need an adverb in the second
post-Independence choice of people as to where blank. So, mutually could have been better.

Click Here For Bundle PDF Course | support@guidely.in Page 7 of 9


Bank Po Mains PDF Course 2024
English Day - 20

Option (e) is ruled out as two parties would


resolve the issues bilaterally (involving two 16. Answer: C
sides) and not one (unilaterally). One aspect in the sentence with the blank is the
Hence, (d) is the best choice. poor students and the other is that the parents
do not want to send their girls out. Affordability of
14. Answer: A the tickets can solve the issue of poor parents
The sentence talks about an officer who is but, they don’t want to send the girls out will be
supposed to be sharp-witted and clever as he is opposite to going for exams once a year to
supposed to be making a choice diplomatically. Delhi. So, eventually choosing the first option
But, the reverse happens. So, for the first blank would make the sentence confusing and ironical.
such an adjective would be Astute (which means Options (a) and (d) are grammatically incorrect.
sharp-witted and intelligent). Sagacious (which And, the issue in the last line is no more
means having a good judgment) also fits but that between physical and virtual classes. So, the
is ruled out as for the second blank precocious fifth option can be eliminated as well.
(developing at an early age) is not the right fit. The best option that completes the idea is the
Seasoned (expert and unbiased in judgment) on third as; it says that the aim would be defeated
the other hand would be the best. Inexplicable because the people do not even want their
(unable to be understood) is not the right word daughters to step out.
for the first blank. So, the best would be to mark option (c) as the
Also, dedicatedly in option (d) is out of context. answer.

15. Answer: B 17. Answer: A


Options (c) and (d) are ruled out as cognizant The most logical option is the first one as it
(aware of) is not followed by ‘to’. Similarly, incorporates all the aspects as given in the other
apprised (informed) is followed by ‘with’. In options. Option (b) is grammatically
option (a) the second word, narrowed followed inappropriate and can be cancelled. (c) is quite
by down (means segregating the options) so vague as the paragraph doesn’t talk about any
even that is ruled out. Option (e) is automatically kind of norm or regulation. (d) is also absurd.
rule out. So the best is privy (aware of facts or So, the best would be to mark option (a) as the
information) and is rightly followed by ‘to’ and the answer.
second blank which says Muscat has been
‘zeroed in’ i.e.; finalized for the preliminary 18. Answer: C
rounds

Click Here For Bundle PDF Course | support@guidely.in Page 8 of 9


Bank Po Mains PDF Course 2024
English Day - 20

The given paragraph talks about restructuring The most appropriate option that should hence,
loans for Sri Lanka which has been volunteered be marked is (a).
by Japan. It has also been mentioned that China
has still not shown willingness to join. So, this 20. Answer: B
gives us a hint that, it is important for all the Option (a) is both extreme and redundant. It tells
countries to come together unlike what China us that he was rejected by his own people. The
believes should be done. All related parties must same idea has already been given in the
be involved. passage that says Russians blame him for the
So, the best would be to go with option (c) which fall of their empire. So, this option cannot be true
completes the given idea in the same fashion. and should be eliminated.
Options (c) and (e) are grammatically
19. Answer: A inappropriate as ‘helped him become’ or ‘won
Options (b) and (c) are grammatically him veneration’ would have made sense.
inappropriate as continuing the sentence with Option (d) brings in a totally new paradigm which
these options will not make sense. We do not makes the paragraph end in a confusing
use which for a person and directly continuing manner.
with ‘saying’ also makes the sentence incorrect. Therefore, the best would be to go with option
(d) is illogical as per the context. (b) as the answer.

Click Here For Bundle PDF Course | support@guidely.in Page 9 of 9

You might also like